{"question": "Breast milk is known to transmit", "exp": "CMV has been demonstrated in breast milk. Approximately 2/3 of CMV-seronegative breast-fed infants may become infected with CMV. The risk is more in a preterm infant. Thus the use of fresh donor milk for feeding a preterm infant is contraindicated unless the milk is known to be CMV-negative.Ref: Page 215; Nelson textbook of pediatrics; 18th edition", "cop": 2, "opa": "Tuberculosis", "opb": "CMV", "opc": "Varicella", "opd": "Rubella", "subject_name": "Pediatrics", "topic_name": "Nutrition", "id": "c9ff96fb-418b-4e60-acc3-a7ed51e12db9", "choice_type": "single"} {"question": "A child is below the third percentile for height. His growth velocity is normal, but chronologic age is more than skeletal age. The most likely diagnosis is", "exp": "sho stature: Defined as height below third centile or more than 2 standard detions below the median height for age and gender according to the population standard. Children whose stature s more than 3 SD below the population mean for age and gender are more likely to be suffering from pathological sho stature as compared to those with stature between -2 and -3SD , who are more likely to be affected by familial or constitutional sho stature. CAUSES:- 1. Most common cause is constitutional. 2. Familial 3. Pathological causes include: Undernutrition Chronic systemic illness Cerebral palsy Congenital hea disease Cystic fibrosis,asthma Malabsorption Acquired immunodeficiency syndrome 4. Endocrine causes include: Growth hormone deficiency Hypothyroidism Cushing syndrome Pseudohypoparathyroidism Precocious or delayed pubey 5. Psychosocial dwarfism 6. Children born small for gestational age. 7. Skeletal dysplasia; achondroplasia, rickets. 8. Genetic syndrome; Turner, Down syndrome. Constitutional growth delay:- These children are born with a normal length and weight and grow normally for first 6-12 months of life. Their growth then shows a decelration so that the height and weight fall below third centile. By 3 yr of age, normal height velocity is resumed abd they continue to grow just below and parallel to tge 3rd centile with a normal height velocity. Bone age is lower than chronological age and corresponds to the height age. History of delayed pubey and delayed height spu is usually present in one or both parents. Reference: GHAI Essential pediatrics, 8th edition", "cop": 1, "opa": "Constitutional delay in growth", "opb": "Genetic sho stature", "opc": "Primordial dwarfism", "opd": "Hypopituitarism", "subject_name": "Pediatrics", "topic_name": "Growth and development", "id": "4adf68de-a451-4172-98ba-d3ed23d1845c", "choice_type": "single"} {"question": "The most common cause of Renal scarring in a 3 year old child is", "exp": "vesicoureteral reflux (VUR) has been considered the most impoant risk factor for post-UTI renal scar formation in children. VUR predisposes children with UTI to pyelonephritis, and both are associated with renal scarring. Reference: GHAI Essential pediatrics, 8th edition", "cop": 3, "opa": "Trauma", "opb": "Tuberculosis", "opc": "Vesicoureteral reflux induced pyelonephritis", "opd": "Interstitial nephritis", "subject_name": "Pediatrics", "topic_name": "Urinary tract", "id": "3e752c67-258c-4cc6-8b6e-ee0ed45347c7", "choice_type": "single"} {"question": "The site of occurrence of Subependymal Giant Cell Astrocytoma is", "exp": "Subependymal Giant Cell Astrocytoma occurs in foramen of monro.", "cop": 3, "opa": "Cerebellum", "opb": "Pineal gland", "opc": "Foramen of monro", "opd": "Medulla Oblongata", "subject_name": "Pediatrics", "topic_name": null, "id": "1cd7b4c5-c783-4edf-ad49-2ddd32c26ce9", "choice_type": "single"} {"question": "H. influenzae has been isolated from the CSF of a 2 year old boy suffering from meningitis. The strain is beta lactamase producing and resistant to chloramphenicol, The most appropriate antimicrobial is", "exp": "HI Inlluenzae type b produces J3 lactamases and therefore are resistant to ampicillin. These 3 lactamase producing strains remain sensitive to the extended spectrum cephalosporins * Ceftriaxone or cefotaxime IV are used as a single agents * Infections caused by ampicillin - resistant strains can be treated with 1. Trimethoprim - sulfamethoxazole 2. Amoxicillin I clavulanic acid 3. Extended spectrum cephalosporins 4. Azithromycin and clarithromycin * F'loraquin lones are highly active against H. influenzae but are not currently recommended for the treatment of children or pregnailt women because of possible effects on aicular cailage", "cop": 3, "opa": "Trimethoprim -- suiphamethoxazole combination", "opb": "Ciprofloxacin", "opc": "Third generation cephalosporin", "opd": "Vancomycin", "subject_name": "Pediatrics", "topic_name": null, "id": "c3795397-f322-419d-ac18-e5f0131553c9", "choice_type": "single"} {"question": "Rose spot is associated with", "exp": "a. Typhoid fever(Ref: Nelson's 20/e p 1390)Skin rash in Enteric fever:In 25% of cases, a macular or maculopapular rash (rose spots) may be visible around the 7th-10th day of the illnessLesions may appear in crops of 10-15 on the lower chest and abdomen and last 2-3 daysThese lesions may be difficult to see in dark-skinned children.", "cop": 1, "opa": "Typhoid fever", "opb": "Malaria", "opc": "Scarlet fever", "opd": "Wilson disease", "subject_name": "Pediatrics", "topic_name": "General Considerations - Infectious Diseases", "id": "e6bbdc0e-4461-49c9-ad07-8a650c67e8c4", "choice_type": "single"} {"question": "In morbus caeruleus foramen ovole closes after", "exp": "In few cases, foramen ovale remains patent throughout life and gives rise to cyanosis, a condition called as Morbus caeruleus.(Refer: Nelson's Textbook of Pediatrics, SAE, 1st edition, pg no. 2162)", "cop": 4, "opa": "6 months", "opb": "2 years", "opc": "1 year", "opd": "Never", "subject_name": "Pediatrics", "topic_name": "All India exam", "id": "63b23189-cdae-419f-b78f-7dfdae00f74e", "choice_type": "single"} {"question": "Most common cause of abdominal mass in neonates is", "exp": "Multicystic dysplastic kidney(MCDK) is a condition that results from the malformation of the kidney during fetal development. The kidneyconsists of irregular cysts of varying sizes. Multicystic dysplastic kidney is a common type of renal cystic disease, and it is a cause of an abdominal mass in infants. Reference: GHAI Essential pediatrics, 8th edition", "cop": 4, "opa": "Neuroblastoma", "opb": "Wilm's tumor", "opc": "Distended bladder", "opd": "Multicystic dysplastic kidneys", "subject_name": "Pediatrics", "topic_name": "Urinary tract", "id": "77c2199b-2955-4a9f-9580-a0b9245f1fb0", "choice_type": "single"} {"question": "Most common cause of hydronephrosis is children", "exp": "In children Most common cause of hydrophosis is Pelvi Ureteric junction obstruction.", "cop": 1, "opa": "PUJ obstruction", "opb": "Ureterocele", "opc": "Posterior urethral valve", "opd": "Ectopic ureter", "subject_name": "Pediatrics", "topic_name": null, "id": "218d3b5c-95e3-4188-9995-d29e87a71ecb", "choice_type": "single"} {"question": "During neonatal resuscitation, the chest compression to ventilation ratio is", "exp": "It is important to ventilate in between chest compressions. A positive breath should follow every third chest compression(chest compression to ventilation ratio is 3:1). \nIn one minute, 90 chest compressions and 30 breaths are administered(120 events). \nTo obtain this, the chest should be compressed 3 times in 1.5 seconds leaving out approx. 0.5 second for ventilation.", "cop": 4, "opa": "15:01", "opb": "5:01", "opc": "10:01", "opd": "3:01", "subject_name": "Pediatrics", "topic_name": null, "id": "dda728fa-b94d-4c03-a625-8bdc411fead7", "choice_type": "single"} {"question": "Not a feature of Hartnup's disease", "exp": "In hartnup's disease, mental development is normal.", "cop": 3, "opa": "Cerebellar ataxia", "opb": "Cutaneous photosensitivity", "opc": "Mental retardation", "opd": "Psychological disturbance", "subject_name": "Pediatrics", "topic_name": null, "id": "c1cc8783-8efb-40c4-89b6-61ff7ee46232", "choice_type": "single"} {"question": "The most common anomaly of the upper urogenital tract is", "exp": "UPJ stenosis is the most common abnormality in childhood and is more frequent in male children, characterized by narrowing usually on the left sideOverall, position & shape of the kidney being most common oneHorseshoe kidney is the most common and most frequently found renal abnormality among men.(Refer: OP Ghai's Textbook of Pediatrics, 8th edition, pg no. 470)", "cop": 1, "opa": "Ureteropelvic junction stenosis", "opb": "Ectopic uretheral opening", "opc": "Ureterocele", "opd": "Ectopic ureter", "subject_name": "Pediatrics", "topic_name": "All India exam", "id": "1178fc34-0ce0-49ec-83e5-76bc0819ee5c", "choice_type": "single"} {"question": "The umbilical cord is shed off by", "exp": "The umbilical cord stump usually dries and shrivels up and is shed off by 1-2 weeksThe parents must look out for signs of redness, swelling, and edema with pus discharge around the stump. These might indicate infection-Omphalitis(Refer: Nelson's Textbook of Pediatrics, SAE, 1st edition, pg no. 890, 811)", "cop": 2, "opa": "2-4 days", "opb": "1-2 weeks", "opc": "2-4 weeks", "opd": "1-2 months", "subject_name": "Pediatrics", "topic_name": "All India exam", "id": "34d09699-8aed-4ab1-955c-5c987a0e4416", "choice_type": "single"} {"question": "Pediatric airway differs from adult by", "exp": "Airway: Pediatric Obligate nasal breatherLarge tongueLarynx & trachea are funnel-shapedThe narrowest pa is cricoid (glottis in adults)(Refer: Nelson's Textbook of Pediatrics, SAE, 1st edition, pg no. 2036 - 2037)", "cop": 1, "opa": "Large tongue", "opb": "Sho epiglottis", "opc": "Narrowest pa is glottis", "opd": "Larynx in lower position", "subject_name": "Pediatrics", "topic_name": "All India exam", "id": "c443b30d-d9cc-4e9c-b433-704feb50ca30", "choice_type": "single"} {"question": "Most common cause of congenital hypothyroidism in", "exp": "Congenital hypothyroidism:- The disorder encompasses a spectrum ranging from complete agenesis, paial agenesis to ectopic thyroid. Iodine deficiency is the commonest cause in endemic pas and thyroid dysgenesis is the most common etiology in non endemic areas. Clinical manifestations include hoarse cry, facial puffiness, umbilical hernia, hypotonia, mottling of skin and lethargy. Prolonged jaundice, constipation, and unexplained hypothermia may also indicate hypothyroidism. Open posterior fontanelle is an impoant indicator. History of maternal thyroid disease or ingestion of antithyroid medications enquired. Reference: GHAI essential Paediatrics", "cop": 2, "opa": "Antithyroid drugs", "opb": "Thyroid dysgenesis", "opc": "Defective hormone synthesis", "opd": "Excessive iodine consumption", "subject_name": "Pediatrics", "topic_name": "Nutrition", "id": "ca65e2d1-80dc-4f5a-843e-11619ded8173", "choice_type": "single"} {"question": "Nephrotic syndrome in children is caused by", "exp": "NEPHROTIC SYNDROME:- Characterised by massive proteinuria, hypoalbuminemua and edema, hyperlipidemia is often associated. Some patients show hematuria and hypeension. Heavy proteinuria( more than 1g/metre square/day) is the underlying abnormality leading to hypoalbuminemia ( serum albumin below 2.5g/dl). The resultant fall in plasma oncotic pressure leads to interstitial edema and hypovolemia. This stimulates the renin angiotensin aldosterone axis and ADH secretion that enhances sodium and water retention. Hypoalbuminemia also induces hepatic synthesis of beta lipoproteins resulting in hypercholesterolemia. Main cause of nephrotic syndrome in children is associated with minimal change disease. Other causes include amyloidosis, vasculitis, SLE, postinfectious glomerulonephritis, and hepatitis B nephropathy. Steroid sensitive nephrotic syndrome:- MCNS accounts for 80% cases of nephrotic syndrome. Electron microscopy shows non specific obliteration of epithelial foot processes. Immunofluorescence studies shows deposits of occassional mesangial IgM. Lab findings- urine examination shows heavy proteinuria. Hyaline and granular casts are present. Serum albumin is low. Hypercholesterolemia impa a milky appearance to plasma. Blood urea and creatinine values within normal range. Blood levels if IgG low and IgM elevated. Low serum calcium level. Steroid resistant nephrotic stndrome:- Homozygous or complete heterozygous mutations in genes encoding podocyte proteins, including podocin(NPHS2), nephrin(NPHS1) and wilms tumor(WT1) genes. Reference: GHAI essential Paediatrics", "cop": 1, "opa": "Minimal change disease", "opb": "RPGN", "opc": "MPGN", "opd": "FSGS", "subject_name": "Pediatrics", "topic_name": "Urinary tract", "id": "47228bcf-cf77-4a20-b1aa-7a491d26851d", "choice_type": "single"} {"question": "Not a criteria of J.R.A (Juvenile Rheumatoid Ahritis)", "exp": ". juvenile Rheumatoid ahritis : age should be below 16 yrs Ref : Ghai pediatrics eighth edition pg no 625", "cop": 1, "opa": "Age > 16 yrs", "opb": "Ahritis > 3 wks", "opc": "Ahritis > 6 months", "opd": "1 or more joint involvement", "subject_name": "Pediatrics", "topic_name": "Musculoskeletal disorders", "id": "1ad4153a-e3ce-4628-b857-68a3a18fee13", "choice_type": "single"} {"question": "Hereditary spherocytosis is best tretaed with", "exp": "Treatment of choice for hereditary spherocytosis is splenectomy especially in case of severe hemolysis,done after 6 years of age. The treatment of hereditary spherosytosis (HS) is dependent on the severity of the condition and recommendations vary a bit in the medical literature. In general, folate therapy (a type of vitamin B to suppo red blood cell production) is recommended for those with moderate to severe anemia, although some doctors may also recommend it for those with mild anemia. Red blood cell transfusions may be required in severe cases of anemia, paicularly in the first years of life or during infections and pregnancy. If red blood cell transfusions are needed repeatedly, iron chelating therapy may be required to reduce iron overload. Regular monitoring for anemia and gallstones is advised. Removal of the spleen (splenectomy) is usually only performed in severe HS or in moderate to severe cases with significant anemia and gallstone complications. Splenectomy is not recommended in cases of mild HS except in specific cases. The majority of medical researchers no longer recommend that the spleen be removed during gallbladder removal (cholecystectomy), unless there are other reasons to do so. In some cases only removing of pa of the spleen is advised. Expe evaluation is recommended in order to avoid unnecessary spleen removal. Reference: GHAI Essential pediatrics, 8th edition", "cop": 1, "opa": "Splenectomy", "opb": "Immunoglobulns", "opc": "Steroids", "opd": "Blood transfusion", "subject_name": "Pediatrics", "topic_name": "Hematology", "id": "cfa8807c-fcc7-4b6a-8314-781659ad1797", "choice_type": "single"} {"question": "The amount of calories required at 1 year of age are", "exp": "Ans. is 'c' i.e., 1200 K cal/day", "cop": 3, "opa": "900 K cal/day", "opb": "1000 K cal/day", "opc": "1200 K cal/day", "opd": "1400 K cal/day", "subject_name": "Pediatrics", "topic_name": null, "id": "3dcec6c8-3729-4b52-bcee-f451b397e3ad", "choice_type": "single"} {"question": "Following is not a cause of stridor in infants", "exp": null, "cop": 4, "opa": "Croup", "opb": "Laryngomalacia", "opc": "Retropharyngeal abscers", "opd": "Bronchiolitis", "subject_name": "Pediatrics", "topic_name": null, "id": "a87ec7b5-2a47-4d17-9547-1ede2753968a", "choice_type": "single"} {"question": "% of children with Simple Febrile seizure developing Epilepsy is", "exp": "Only 1-2% of children are developing simple febrile seizures The Risks For Developing Epilepsy : Problems with the child&;s development before the febrile seizure Having that last longer than 15 minutes, more than one seizure in 24 hours, or seizures in which only one side of the body is affected Seizures without fever in a parent or a brother or sister If the child has none of these risk factors, the chances of epilepsy developing later are only 1% to 2% (1 or 2 out of 100). This is very similar to the risk of developing epilepsy in any other child. Children with 1 of these risk factors have a 2.5% (1 in 40) chance of later epilepsy. For children with 2 or 3 risk factors, their chance of developing epilepsy later ranges from 5% (1 in 20) to over 10% (greater than 1 in 10). In rare cases, febrile seizures that last more than 30 minutes may cause scar tissue in the temporal lobe of the brain. In some of these children, chronic epilepsy develops. Reference: GHAI Essential pediatrics, 8th edition", "cop": 1, "opa": "1-2%", "opb": "2-5%", "opc": "5-10%", "opd": "10-15%", "subject_name": "Pediatrics", "topic_name": "Central Nervous system", "id": "635fb32b-489f-47dd-b50c-a8d62b702980", "choice_type": "single"} {"question": "Vaccine with maximum efficacy", "exp": "b. Measles(Ref: Nelson's 20/e p 1546, Ghai 8/e p 145; IAP guidebook of Immunization 2013-14, p 222-223)Seroconversion rates after measles vaccination are around 60% at the age of 6 months, 80-85% at the age of 9 months and beyond 95% at the age of 12-15 months.", "cop": 2, "opa": "OPV", "opb": "Measles", "opc": "BCG", "opd": "TT", "subject_name": "Pediatrics", "topic_name": "Immunization", "id": "535ded66-8217-4b1f-a5d1-411841dd3151", "choice_type": "single"} {"question": "Holt Oram syndrome is characterized by", "exp": "Holt Oram syndrome:Hypoplastic or absent radii1st-degree hea blockASD(Refer: OP Ghai's Textbook of Pediatrics, 8th edition, pg no. 401)", "cop": 1, "opa": "ASD", "opb": "VSD", "opc": "TGA", "opd": "TAPVC", "subject_name": "Pediatrics", "topic_name": "All India exam", "id": "db9636bb-4335-44a1-a2a7-090a3d2e0543", "choice_type": "single"} {"question": "Type 1 diabetes in children is most commonly associated with", "exp": "The estimated prevalence of celiac disease in patients with type 1 diabetes is approximately 6%, and about 1% in the general population. Type 2 diabetes does have genetic components, but they are not associated with celiac disease genes like type 1 diabetes&; . Reference: GHAI Essential pediatrics, 8th edition", "cop": 2, "opa": "Obesity", "opb": "Celiac disease", "opc": "Downs syndrome", "opd": "Hypothyroidism", "subject_name": "Pediatrics", "topic_name": "Endocrinology", "id": "7373f62c-e0b6-41a3-87bf-8a68d0e99bdc", "choice_type": "single"} {"question": "In Kawasaki disease, desquamation and denudation of skin from fingers and toes occurs in", "exp": "The cause of Kawasaki disease isn&;t known. The body&;s response to a virus or infection combined with genetic factors may cause the disease. However, no specific virus or infection has been found, and the role of genetics isn&;t known. Kawasaki disease can&;t be passed from one child to another. Swelling and redness in hands and bottoms of feet, followed by sloughing of skin of hands and feet Redness in the eyes Enlarged glands, especially in the neck Irritated throat, inner mouth, and lips Swollen, bright red \"strawberry tongue\" Stomach trouble, with diarrhea and vomiting Reference: GHAI Essential pediatrics, 8th edition", "cop": 2, "opa": "3-6 weeks", "opb": "2nd and 3rd week", "opc": "After 1st week", "opd": "After 6 weeks", "subject_name": "Pediatrics", "topic_name": "Miscellaneous", "id": "96022a64-9f7d-4e56-8958-dbcdcb3ebdb6", "choice_type": "single"} {"question": "Drug of choice for Infantile Spasm", "exp": "Drug of choice for infantile spasm is ACTH. Infantile spasms (also called West syndrome) can be caused by brain malformations, infections, brain injury, or abnormal blood vessels in the brain. IS also can happen in babies with ceain metabolic and genetic disorders. In rare cases, a baby&;s infantile spasms are caused by vitamin B6 deficiency. Infantile spasms (IS) is a seizure disorder in babies. The Spasms sta suddenly and last a second or two. They often come one after another in a cluster that lasts several minutes. They happen most often just after waking. They&;re often mistaken for A baby having a spasm might have: the head bent forward with arms flung out and the knees pulled into the body (described as \"jackknife\") the head bent back with the arms and legs straightened small movements in the neck or other pas of the body The most common drugs used to treat West syndrome are: Adrenocoicotropic hormone (ACTH) Prednisone Vigabatrin Pyridoxine Reference: GHAI Essential pediatrics, 8th edition", "cop": 2, "opa": "Vigabatrin", "opb": "ACTH", "opc": "Ethosuximide", "opd": "Valproate", "subject_name": "Pediatrics", "topic_name": "Central Nervous system", "id": "afb34c95-e9a6-4762-b658-6bd7f953b3b7", "choice_type": "single"} {"question": "Commonest type of cong. cyanotic hea disease is", "exp": "Most common cyanotic hea disease is tetrology of fallot encountered beyond the age of 1 yr constituting almost 75% of all blue patient. Ref : Ghai essential pediatrics,eighth edition, P.no:420", "cop": 3, "opa": "ASD", "opb": "VSD", "opc": "TOF", "opd": "PDA", "subject_name": "Pediatrics", "topic_name": "C.V.S", "id": "ab9e09e9-f33e-42b1-9a3b-0f972a4dc197", "choice_type": "single"} {"question": "4 years old child having palpable abdominal mass & hypeension with sweating & diarrhea is due to", "exp": "Neuroblastoma is a malignant tumor of autonomic nervous system.Clinical features are related to the localisation of the sympathetic nervous system and site of metastasis.The most common site is adrenal medulla.The patient can present with a localised abdominal mass ,sweating and hypeension due to the hyperstimulation of sympathetic nervous system.It is also characterised by watery diarrhea associated with VIP(vasoactive intestinal polypeptide) secretion by the tumor. Reference:Essential pediatrics-Ghai,8th edition,page no:616,617.", "cop": 1, "opa": "Neuroblastoma", "opb": "Nephroblastoma", "opc": "PCKD (Polycystic kidney disease)", "opd": "Medulloblastoma", "subject_name": "Pediatrics", "topic_name": "Childhood tumors", "id": "6e0ea27c-6ca4-4d6f-9906-8d3f85d7917e", "choice_type": "single"} {"question": "Anatomical closure of Ductus Arteriosus takes place at", "exp": "Functional closure happens 10-12 hours after birth. Anatomical closure takes place at 2-3weeks after birth.", "cop": 4, "opa": "At Birth", "opb": "10-12 hours after birth", "opc": "1 week after birth", "opd": "2-3 weeks after birth", "subject_name": "Pediatrics", "topic_name": null, "id": "c5b689f3-8701-4da3-b261-f0b897ea08c2", "choice_type": "single"} {"question": "11 hydroxylase deficiency leads to", "exp": "Seen in Newborns and infants, Virilization and the presence of both hypeension and hypokalemia. 11 -deoxycoisol, 17a-hydroxyprogesterone, urinary 17- ketosteroids, and 17-hydroxycoicosteroids are increased. Blood levels and production rates of coisol are usually within normal limits. Physiologic replacement doses of glucocoicoid, such as hydrocoisone or dexamethasone, restores blood pressure to normal levels, corrects K+ depletion, Addition of estrogen-progestogen combined cyclic therapy may be necessary in the adult patient with 17a- hydroxylase deficiency. Reference: GHAI Essential pediatrics, 8th edition", "cop": 1, "opa": "Hypeension", "opb": "Hypotension", "opc": "Hypoglycemia", "opd": "Normal pubey", "subject_name": "Pediatrics", "topic_name": "Genetic and genetic disorders", "id": "2bb3321f-2348-4162-89b8-c4029d47affc", "choice_type": "single"} {"question": "The ratio of chest compressions to ventilation in a newborn is", "exp": "If the hea rate does not improve after 30 sec with bag-and-mask (or endotracheal) ventilation and remains below 100 beats/min, ventilation is continued and chest compression should be initiated over the lower third of the sternum at a rate of 90 compressions/min. The ratio of compressions to ventilation is 3 : 1 (90 compressions:30 breaths). If the hea rate remains <60 beats/min despite effective compressions and ventilation, administration of epinephrine should be considered.Ref: Nelson; 20th edition; Page no: 845", "cop": 3, "opa": "1:01", "opb": "2:01", "opc": "3:01", "opd": "4:01", "subject_name": "Pediatrics", "topic_name": "New born infants", "id": "45c49f9b-1952-45c9-aaa5-b81cec4a1699", "choice_type": "single"} {"question": "In AML best prognosis is seen with", "exp": "- Acute Promyelocytic Leukemia (APML): characterized by a gene rearrangement involving retinoic acid receptor It is very responsive to all-trans retinoic acid (ATRA, tretinoin) combined with anthracyclines and cytarabine. The success of this therapy makes Bone marrow transplantation unnecessary in 1st remission. Arsenic trioxide is also an effective non-cytotoxic therapy for APML", "cop": 3, "opa": "Acute megakaryocytic leukemia", "opb": "Acute monocytic leukemia", "opc": "Acute promyelocytic lukemia (M3)", "opd": "Erythroleukemia", "subject_name": "Pediatrics", "topic_name": "Neoplastic disorders of WBCs", "id": "c1f77605-78ca-409e-9ed2-619cac79bd64", "choice_type": "single"} {"question": "A 3 day child vomits everything he feeds, has a distened abdomen & diarrhoea. The urine is positive for benedicts test for reducing substance. The substance in urine is", "exp": "Galactosemia caused by the deficiency of enzyme GALT(galactose-1-phosphate uridyl transferase).Patients appear normal at bih,but by 3-4 days of breast milk,/formula feeding,show life threatening disease with vomiting,diarrhoea,poor weight gain,hepatic and renal manifestations and cataract.In case if suspected galactosemia,the urine should be tested simultaneously with Benedict reagent and by glucose-oxidase method.A negative dipstick by glucose oxidase method with positive Benedict reaction indicates nonglucose reducing substances,like galactose. Reference:Essential pediatrics-Ghai,8th edition,page no:657.", "cop": 3, "opa": "Sucrose", "opb": "Glucose", "opc": "Galactose", "opd": "Fructose", "subject_name": "Pediatrics", "topic_name": "Metabolic disorders", "id": "f8167db5-c3a6-4c2c-8e24-2e5666c3bbf0", "choice_type": "single"} {"question": "For severe malnutrition MAC will be less than", "exp": "For severe malnutrition, MAC is < 11.5 cm Severe acute malnutrition In a child between 6 months to 5 years of age, presence of any1 or more of- Weight for height < -3 Z score or < 70% of expected Mid arm circumference - < 11.5 cm Symmetric bipedal edema of nutritional origin Visible severe wasting;", "cop": 4, "opa": "14.5", "opb": "13.5", "opc": "12.5", "opd": "11.5", "subject_name": "Pediatrics", "topic_name": "FMGE 2019", "id": "25db6d36-eda8-42de-aa97-5b8ffb7ea79f", "choice_type": "single"} {"question": "The recommended room temperature for maintaining warmth for neonates is", "exp": "The recommended room temperature for maintaining warmth for neonates: 25 degree C.", "cop": 3, "opa": "20 degree C", "opb": "34 degree C", "opc": "25 degree C", "opd": "30-34 degree C", "subject_name": "Pediatrics", "topic_name": "Neonatal hypothermia", "id": "0191055b-72b4-4978-a7f0-7a2f237cf1b2", "choice_type": "single"} {"question": "A child presented with cola coloured urine, proteinuria 2+ & h/o rash 2 week ago. Probable Dx", "exp": "Ans: B (HSP) Gross hematuria (cola coloured urine) after upper respiratory tract infection; Time of latencyIgA nephropathy 1-2 days (Nelson 18th/2171)PSGN (After Pharyngitis or impetigo): 7-21 days (Nelson 18th/ 2171)HSP: 1-3 wk upto 12 wks (Nelson 18th/2179); also presents with systemic feature like rashHenoch-Schonlein Purpura (HSP); Short reviewIt is a systemic vasculitis that causes the blood vessels in the skin to become inflamed, causing red spots. When the blood vessels in the skin get inflamed, they can bleed, causing a rash that is called Purpura. This rash is typically seen on the lower legs or arms. The specific skin lesion is characterized by the tissue deposition of an immune system product, called IgA immunoglobulin, which is also found in kidneysof patients with a renal disease, called IgA nephropathy.HSP occurs more often in children than in adults, and many cases follow an upper respiratory tract infection (infection in your sinuses and /or lungs). Halfof affected children are under age five, although kidney involvement is more likely to be severe in older children. Compared to children, adults had more severe and frequent kidney involvement.Symptoms occur over a period of days to several weeks: Skin rash, joint aches and pains, usually in knees and ankles, occasional swelling, abdominal pain and renal disease manifesting mostly as hematuria (blood in your urine), proteinuria (abnormal excretion of proteins in urine), edema (swelling) or alteration in the volume of urine. The hematuria may be noticed as red or tea-colored or cola-colored urine or the amount may be so small that it can only be seen under a microscope. The brain or the lung may also be involved in HSP.Even though the symptoms of HSP make it easier to diagnose in children, confirmation of the diagnosis of HSP requires evidence of tissue deposition in the skin or kidney of IgA immunoglobulin. Renal biopsy is another method to establish the diagnosis, but is reserved for patients in whom the diagnosis is uncertain or in whom there is evidence of more severe renal involvement.", "cop": 2, "opa": "IgA nephropathy", "opb": "HSP", "opc": "HUS", "opd": "Wegener Granulomatosis", "subject_name": "Pediatrics", "topic_name": "Urinary Tract", "id": "a1996dcd-655f-4440-b150-dfd4e39de9ca", "choice_type": "single"} {"question": "In congenital adrenal hyperplasia most common deficiency", "exp": "Ans) a (21- alpha ) Ref Nelson 18'h ed p 2360More than 90% of CAH cases are caused by 21 -hydroxylase deficiency.", "cop": 1, "opa": "21- alpha hydroxylase deficiency", "opb": "11- beta hydroxylase deficiency", "opc": "17 - hydroxylase deficiency", "opd": "3 beta hydroxylase deficiency", "subject_name": "Pediatrics", "topic_name": "Endocrinology", "id": "731eeeea-ca8c-4f60-b914-2285a78cf458", "choice_type": "single"} {"question": "Most common cause of Hemolytic Uremic Syndrome is", "exp": "Hemolytic-uremic syndrome (HUS) often occurs after a gastrointestinal infection with E coli bacteria (Escherichia coli O157:H7). However, the condition has also been linked to other gastrointestinal infections, including shigella and salmonella Causes Hemolytic-uremic syndrome (HUS) often occurs after a gastrointestinal infection with E coli bacteria (Escherichia coliO157:H7). However, the condition has also been linked to other gastrointestinal infections, including HUS is most common in children. It is the most common cause of acute kidney failure in children. Several large outbreaks have been linked to undercooked hamburger meat contaminated with E coli. E coli can be transmitted through: Contact from one person to another Consuming uncooked food, such as milk products or beef STEC-HUS is not to be confused with atypical HUS (aHUS) which is not infection-related. It is similar to another disease called (TTP). Reference: GHAI Essential pediatrics, 8th edition", "cop": 1, "opa": "E.coli", "opb": "Shigella", "opc": "Salmonella", "opd": "Pseudomonas", "subject_name": "Pediatrics", "topic_name": "Urinary tract", "id": "049ddf79-6627-468f-85f7-5e3eee605d5a", "choice_type": "single"} {"question": "Hutchinson's teeth are seen in", "exp": "It is a common pattern of presentation for congenital syphilis, and consists of three phenomena aka Hutchinson triad:\n\nInterstitial keratitis\nHutchinson incisors\nEighth nerve deafness.", "cop": 2, "opa": "Early congenital spirochete infection", "opb": "Late congenital syphilis", "opc": "Rickets", "opd": "Scurvy", "subject_name": "Pediatrics", "topic_name": null, "id": "e192b598-6797-4af1-8a80-098fb3e414e9", "choice_type": "single"} {"question": "In Glanzman's thrombasthenia, there is a defect of", "exp": "Glanzman's thrombasthenia\n\nPlatelet aggregation defect\nIIb / IIIa Gp deficient", "cop": 2, "opa": "Gp IIb / IIa", "opb": "Gp IIb / IIIa", "opc": "Gp Ib / IX", "opd": "Gp IIb / IX", "subject_name": "Pediatrics", "topic_name": null, "id": "bb1feff8-fdff-43ec-bd7f-b359e8cb9a21", "choice_type": "single"} {"question": "The most common paediatric cardiac tumour among the following is", "exp": "The vast majority of tumours originating from the hea are benign. Rhabdomyomas are the most common pediatric cardiac tumours and are associated with tuberous sclerosis in 70-95% of cases. Rhabdomyomas may occur at any age, from fetal life through late adolescence. They are often multiple, can occur in any cardiac chamber, and originate within the myocardium extending, often, into the atrial or ventricular cavities. Depending on their location and size, they can result in inflow or outflow obstruction leading to cyanosis or cardiac failure; many are asymptomatic. Atrial and ventricular arrhythmias have been repoed with rhabdomyomas, and on occasion, ventricular pre-excitation (Wolff-Parkinson-White) is present on electrocardiogram. Reference: Nelson; 20th edition; Page no: 2281", "cop": 2, "opa": "Fibromas", "opb": "Rhabdomyomas", "opc": "Myxomas", "opd": "Hemangiomas", "subject_name": "Pediatrics", "topic_name": "C.V.S", "id": "dda44456-0284-4f6c-8d6c-506f49c765b8", "choice_type": "single"} {"question": "Edema of hands & feet in infants is characterised", "exp": "Edema of hands and feet is associated with turner syndrome. Turner syndrome is identifiable at bih by presence of LYMPHEDEMA ,cystic hygroma and left sided obstructive cardiac lesions. Reference : Ghai TB of pediatrics 8th edition pg 537.", "cop": 3, "opa": "Klinefelter's syndrome", "opb": "Noonam syndrome", "opc": "Turners syndrome", "opd": "Fragile x syndrome", "subject_name": "Pediatrics", "topic_name": "Genetic and genetic disorders", "id": "3e4b09fb-81d9-4127-b203-5bda84937b49", "choice_type": "single"} {"question": "The given clinical condition may present with", "exp": "This is Vein of Galen Malfomation:\nAnamolous Vein of Galen ⇒ No Capillaries Blood flow directly from artery to vein\n↓\nHeart is subjected to hight pressure leading to congestive cardiac failure.", "cop": 4, "opa": "Microcephaly", "opb": "Intra cramial tumour", "opc": "Cerebral Palsy", "opd": "Congestive cardiac failure", "subject_name": "Pediatrics", "topic_name": null, "id": "f3f86344-4275-4aba-b44b-960d87209394", "choice_type": "single"} {"question": "Risk of Down's syndrome in Elderly (>35 years)", "exp": "Risk of Down's syndrome in Elderly (>35 Years) is 1% - 4%.", "cop": 2, "opa": "1% - 2%", "opb": "1% - 4%", "opc": "2% - 4%", "opd": "1% - 5%", "subject_name": "Pediatrics", "topic_name": null, "id": "f89991a4-b4d0-4891-9782-d9546a122006", "choice_type": "single"} {"question": "Risk of congenital heart disease in first degree relative is", "exp": "Ans. b (2 to 6%). (Ref. Nelson, Textbook of Paediatrics, 18th/pg. 1878)Congenital heart disease occurs in# 0.5 to 0.8% of live births.# 3 to 4 % of stillboms.# 10 to 25% of abortuses.# 2% of premature infants (except PDA)# 0.8% of normal population.# 2-6% after birth of a child with CHD or if parent affected.# 20-30% when 2 first-degree relatives have congenital heart disease.NOTE: Most congenital defects are well tolerated in the fetus because of the parallel nature of the fetal circulation. Even the most severe cardiac defects (hypoplastic left heart syndrome) can usually be well compensated for by the fetal circulation. It is only after birth when the fetal pathways (ductus arteriosus and foramen ovale) are closed that the full hemodynamic impact of an anatomic abnormality becomes apparent. One notable exception is the case of severe regurgitant lesions, most commonly of the tricuspid valve. In these lesions (Ebstein anomaly), the parallel fetal circulation cannot compensate for the volume load imposed on the right side of the heart. In utero heart failure, often with fetal pleural and pericardial effusions, and generalized ascites (nonimmune hydrops fetalis) may occur.", "cop": 2, "opa": "0.5 to 0.6%", "opb": "2 to 6%", "opc": "5 to 6%", "opd": "20 to 25%", "subject_name": "Pediatrics", "topic_name": "C.V.S.", "id": "b2b00e6c-8f8f-46d8-9cf9-b2cd25e8e0c1", "choice_type": "single"} {"question": "Least common valve involved in rheumatic fever is", "exp": "Endocarditis in rheumatic feverMitral valve involved almost all the cases of Acute Rheumatic FeverAlmost 25% of MR is associated with aoic regurgitationTricuspid regurgitation is seen in 10-30% of casesPulmonary valve involvement is never seen(Refer: Nelson's Textbook of Pediatrics, SAE, 1st edition, pg no. 1333 - 1335)", "cop": 4, "opa": "Aoic valve", "opb": "Tricuspid valve", "opc": "Mitral valve", "opd": "Pulmonary valve", "subject_name": "Pediatrics", "topic_name": "All India exam", "id": "4b50c4f9-bc8b-4e0b-a58e-87113888373a", "choice_type": "single"} {"question": "Coaaction of aoa is associated with", "exp": "Coarctation of aoa is association with bicuspid aoic valve. Ref : Ghai essential of pediatrics, eighth edition, p.no:432", "cop": 3, "opa": "VSD", "opb": "PDA", "opc": "Bicuspid aoic valve", "opd": "ASD", "subject_name": "Pediatrics", "topic_name": "C.V.S", "id": "a488dddf-354e-4553-8127-998b1d7da504", "choice_type": "single"} {"question": "In full term neonates, the ductus aeriosus closure is due to", "exp": "High pressure O2 can cause the closure of ductus aeriosus. Reference: GHAI Essential pediatrics, 8th edition", "cop": 4, "opa": "Cardiac output", "opb": "Prostaglandins", "opc": "Low pressure CO2", "opd": "High pressure O2", "subject_name": "Pediatrics", "topic_name": "New born infants", "id": "128701c2-65d1-47f4-bb1b-50b491e80bc9", "choice_type": "single"} {"question": "Most common vasculitis in children", "exp": "UPDATE- Most common vasculitis in children Kawasaki Disease.", "cop": 2, "opa": "Henoch Schonlein purpura", "opb": "Kawasaki disease", "opc": "Wegener's granulomatosis", "opd": "Polyarteritis nodosa", "subject_name": "Pediatrics", "topic_name": null, "id": "1dfc394e-ee21-4767-97eb-b8b48d35752d", "choice_type": "single"} {"question": "Traetment of choice in brochiolitis in", "exp": "Antibiotics have no role. Ribavirin an antiviral agent has no role in treatment of infants who were previously healthy.It shoens the course of illness in infants with underlying congenital hea disease, chronic lung diseases,and immunodeficiency.Ribavirin is delivered by a nebulizer 16 hours a day for 3-5 days in such cases. Ref : Essential Pediatrics,O.P.Ghai,7t edition,pg no:358", "cop": 1, "opa": "Ribavirin", "opb": "Amantadine", "opc": "Vidarabine", "opd": "Zidovudine", "subject_name": "Pediatrics", "topic_name": "Respiratory system", "id": "aa0a67ed-3e62-41c1-bd23-16395bf600ef", "choice_type": "single"} {"question": "A 5 year old child presents with perivascular IgA deposition and neutrophilic collection. There is erythematous rash on the lower limb and nonblanching purpura. Probable diagnosis is", "exp": "Henoch schonlein purpura : It is one of the most common vasculitic disorder of childhood. Characterised by the presence of nonthrombocytopenic , palpable purpura, transient ahralgia and abdominal symptoms. The illness begins with a rash more prominent over the extensor aspects of lower extremities and buttocks. It may be macular, maculopapular or even uicarial to begin with . Glomerulonephritis seen in one third of cases. Gastrointestinal manifestations usually occur in first 7-10 days if illness. Abdominal pain is intermittent, colicky and periumbilical.vomitingseen , whereas melena and hemetemesis are less common. Rare manifestations include CNS vasculitis, coma, Guillain Barre syndrome, pulmonary hemorrhage, carditis and orchitis. Criteria for childhood HSP:- Palpable purpura in the presence of at least one of the following 4 features: 1. Diffuse abdominal pain. 2. Any deposit showing IgA deposition. 3. Ahritis/Ahralgia. 4. Renal involvement. Investigation: Nonspecific rise in total serum IgA levels. Skin biopsy shows leukocytoclastic vasculitis. On indirect immunofluorescence there are mesangial deposits of IgA andC3 in skin and renal biopsy. Reference: GHAI essential Paediatrics", "cop": 1, "opa": "Henoch-Schonlein purpura", "opb": "Wegener's granulomatosis", "opc": "Vasculitis", "opd": "Kawasaki's disease", "subject_name": "Pediatrics", "topic_name": "Urinary tract", "id": "66fc37f0-6717-48a9-8471-b07743986196", "choice_type": "single"} {"question": "The following rashes seen in a previously well 5 month old child can be treated by administration of", "exp": "Deficiency of zinc leads to Acrodermatitis enteropathica -rashes seen especially around the orifices. Acrodermatitis enteropathica is a genetic AR disorder Characterized by periorificial dermatitis, alopecia, and diarrhea. Caused by mutations in the gene that encodes a membrane protein that binds zinc in the intestine. Treatment - Oral zinc supplementation", "cop": 4, "opa": "Calcium", "opb": "Pyridoxine", "opc": "Niacin", "opd": "Zinc", "subject_name": "Pediatrics", "topic_name": "Malnutrition", "id": "4ac4c6d5-b64b-4f46-810e-6aa104c2c3f3", "choice_type": "single"} {"question": "Best indicator of growth monitoring in children", "exp": "Rate of increase in height or weight as age advances- Best indicator of growth monitoring in children Uses of growth cha:- 1) For growth monitoring 2) Diagnostic tool for identifying \"high risk\" children 3) Planning and policy making 4) Education tool 5) Tool for action Mid arm circumference, Body weight and chest circumference are one time point estimates therefore they are not adequate to provide information about long term nutritional status of the child.", "cop": 3, "opa": "Weight", "opb": "Mid-arm circumference", "opc": "Rate of increase in height & weight", "opd": "Head circumference", "subject_name": "Pediatrics", "topic_name": "Growth", "id": "e3004549-5512-4eb9-8985-81b825b839ac", "choice_type": "single"} {"question": "Neonate with retrolental fibroplasia is associated with", "exp": "Retrolental fibroplasia- Abnormal proliferation of fibrous tissue immediately behind the lens of the eye, leading to blindness. This is mostly due to excessive administration of oxygen in premature babies. Image : Retrolental fibroplasia or Retinopathy of prematurity Reference: GHAI Essential pediatrics, 8th edition", "cop": 4, "opa": "Hypoxia", "opb": "Hypocapnea", "opc": "Hypoxaemia", "opd": "Hyperoxemia", "subject_name": "Pediatrics", "topic_name": "Miscellaneous", "id": "45d9417f-b7cb-4daf-b5dc-f4893c64bcb5", "choice_type": "single"} {"question": "Most common cause of cushings syndrome is", "exp": "Most common cause of Cushing's syndrome is prolonged administration of glucocorticoid harmones.", "cop": 4, "opa": "Pituitary adenoma", "opb": "Adrenal adenoma", "opc": "Ectopic ACTH", "opd": "Iatrogenic steroids", "subject_name": "Pediatrics", "topic_name": null, "id": "86ee4636-cfaf-48e3-8505-b95fa2b951d6", "choice_type": "single"} {"question": "A 10 year old boy presents with mucosal bleeding of 1 week duration. The investigation of choice that will be most useful in him is", "exp": "In case of platelate disorder site of bleeding is usually skin and mucous membrane. where as in coagulation disorder bleeding will be from soft tissue,muscle,joint and deep sites. Platelet count is the investigation of choice in this case . Reference: GHAI Essential pediatrics, 8th edition", "cop": 4, "opa": "Prothrombin time", "opb": "Clotting time", "opc": "Paial thromboplastin time", "opd": "Platelet count", "subject_name": "Pediatrics", "topic_name": "Hematology", "id": "0498b70e-da93-430a-b1e7-2cbe80ac8920", "choice_type": "single"} {"question": "The most sensitive Indicator of Depletion of intravascular volume in infant is", "exp": "The most sensitive indicator of depletion of intravascular volume in infant is Hea rate. Reference: GHAI Essential pediatrics, 8th edition", "cop": 3, "opa": "Cardiac output", "opb": "Stroke volume", "opc": "Hea rate", "opd": "Blood pressure", "subject_name": "Pediatrics", "topic_name": "Fluid and electrolytes", "id": "6aa2019b-4200-4e8c-9946-5479dff8c7b8", "choice_type": "single"} {"question": "A child is hepatitis B immunized the marker for seroconversion is", "exp": "i.e. (Anti HBs Ag): (193- Ghai 7th)Common sero pattern of hepatitis B infectionHBsAgAnti HBsAnti HBcHBeAgAnti HBeInterpretation+-IgM+-Acute infection--IgM+/-+/-Acute infection anti HBC window+-IgG+/--Chronic infection; high infectivity+-IgG-+Chronic infection; low infectivity+-+ /---Precore/ Cor mutant infection-+IgG-+/-Recovery from infection-+ --Immunization; false positive; infection in remote past", "cop": 3, "opa": "HBs Ag", "opb": "Anti HBc IgM", "opc": "Anti HBs Ag", "opd": "Anti HBc IgG", "subject_name": "Pediatrics", "topic_name": "Infection", "id": "99c2b170-65cc-43b1-8a26-7dfff792334f", "choice_type": "single"} {"question": "The most common congenital cardiac lesion identified at bih is", "exp": "Ventricular Septal Defect (VSD) This is the most common congenital cardiac lesion identified at bih accounting for one-quaer of all CHD. VSD is a communication between the two ventricles. 90% are located in the membranous pa of the ventricular septum with variable extension into the muscular septum. Others are located in the muscular septum and can be multiple. Reference: Essential Paediatrics; O.P. Ghai; Page no: 414", "cop": 2, "opa": "ASD", "opb": "VSD", "opc": "PDA", "opd": "Coarctation of Aoa", "subject_name": "Pediatrics", "topic_name": "C.V.S", "id": "2cb62211-c6b7-4b90-ac2c-525a33dcda96", "choice_type": "single"} {"question": "Most common type of AML in Down's syndrome", "exp": "- Megakaryocytic maturation AML - Most common AML in Down's syndrome. - Blast cells in these patients are highly sensitive to medications - show better outcome than AML in those without Trisomy 21.", "cop": 4, "opa": "M2", "opb": "M3", "opc": "M6", "opd": "M7", "subject_name": "Pediatrics", "topic_name": "Neoplastic disorders of WBCs", "id": "b1643cf0-f23d-4520-88e4-96f9053909ec", "choice_type": "single"} {"question": "The features of Hemolytic Uremic Syndrome in children include", "exp": "Hemolytic-uremic syndrome (HUS) is a group of blood disorders characterized by low red blood cells, acute kidney failure, and low platelets. Initial symptoms typically include bloody diarrhea, fever, vomiting, and weakness.Kidney problems and low platelets then occur as the diarrhea is improving.While children are more commonly affected, adults may have worse outcomes.Complications may include neurological problems and hea failure. Reference: GHAI Essential pediatrics, 8th edition", "cop": 2, "opa": "\"Helmet cells\" in peripheral blood smear", "opb": "Thrombocytopenia", "opc": "Positive coomb's test", "opd": "Hypothyroidism", "subject_name": "Pediatrics", "topic_name": "Urinary tract", "id": "5d06de86-56af-4908-9b0a-b8dc8309ff49", "choice_type": "single"} {"question": "Hutchinson teeth are seen in", "exp": "(Congenital syphilis) (1265- Nelson 18th)HUTCHINSON TEETH - which arc the peg or barrel shaped upper central incisors that erupt during the 6th year of life: abnormal enamel, which results in a notch along the biting surface, seen in Late manifestation of congenital syphilisEarly manifestations of congenital syphilis* Approximately 66% of infected ifants are asymptomatic* Hepatosplenomegaly jaundice, elevated liver enzymes are common* Lymphadenopathy to be diffuse, shotty nodes may persist* Coombs negative hemolytic anemia, Thrombocytopenia* Osteochondritis, periostitis* Limbs are held immobile due to excruciating pain (pseudo paralysis of parrot)*** Mucous patches, rhinitis (snuffles)* and condylomatous lesions are highly characteristic* CNS abnormalities, failure to thrive, chorioretinitis, nephritis and nephritic syndrome* Renal involvement include - hypertension, hematuria, proteinuria, hypoproteinemia, hypercholesterolemia, and hypocomplementemia* Less common clinical manifestations - gastroenteritis, peritonitis, pancreatitis, pneumonia, eye involvement glaucoma and chorioretinitis) nonimmune hydrops and testicular masses.Late manifestations of congenital syphilis* Frontal bossing, a bony prominence of the forehead (Olympian brow), unilateral or bilateral thickening of the sternoclavicular third of clavicle (Clavicular or Higoumenaki'sign)* Saber shins, scaphoid scapula, Hutchinson teeth, mulbeny molars* Saddle nose, Rhagedes, Juvenile paresis - Juvenile tables* Hypersensitivity phenomenon - interstitial keratitis, blindness less common ocular manifestations include - choroiditis, retinitis, vascular occlusion and optic atrophy, eighth nerve deafness* Clutton joint - U/L or B/L synovitis usually in the knee painless, soft tissue gammas, paroxysmal cold hemoglobinuria (rare)Hunterian chancre single painless, regular indurated reddish brown plaque seen in primary syphilis* Congenital varicella syndrome is characteristically associated with limb reduction defects and scarring of skin* Classical Tetrad of congenital Toxoplasmosis* Hydrocephalus or Microcephaly*** Chorioretinitis* Convulsions (seizures)* Cerebral calcifcations*RUBELLA - is a teratogenic virus* can be transmitted from fetus to mother. The rate of transmission and congenital defects maximum in first trimester infections (hearing loss) * Under eradication of congenital rubella syndrome programme the first priorty group of rubella vaccination is- All non pregnant women of age 15 to 34 years*** Most common congenital defects are deafness, cardiac malformations and cataractsOther resulting from defects include - glaucoma, retinopathy microcephalus, cerebral palsy, IUGR, hepatosplenomegaly mental and motor retardation\"Chancre- redux\" is a clinical feature of early relapsing syphilis at the site of the healed lesionTerm \"Viable not cultivable\" (VNC) is used for T.pallidum. M. leprae", "cop": 2, "opa": "Toxoplasma", "opb": "Congenital syphilis", "opc": "Rubella", "opd": "CMV- nfections", "subject_name": "Pediatrics", "topic_name": "Infection", "id": "5625245a-ed6d-45b6-88d4-46f7bd9ef00f", "choice_type": "single"} {"question": "Most common cause of seizure in newborn is", "exp": "Most common cause of seizure in first day of new born baby is anoxia ,hypoxia induced ischemic Symptomatic neonatal seizures may be caused by: lack of oxygen before or during bih because of problems such as placental abruption (premature detachment of the placenta from the uterus), a difficult or prolonged labour, or compression of the umbilical cord infection acquired before or after bih, such as bacterial (a blood clot in the brain) bleeding in the brain congenital brain abnormalities, either genetic or acquired during fetal development, such as tuberous sclerosis blood sugar or electrolyte imbalances, including hypoglycemia (low blood sugar), hypocalcemia (low calcium), hyponatremia (low sodium), or hypernatremia (high sodium) metabolic problems, such as maple syrup urine disease, pyridoxine dependency, or drug withdrawal, which may be seen in infants born to mothers addicted to barbiturates, alcohol, heroin, cocaine or methadone Reference: GHAI Essential pediatrics, 8th edition", "cop": 1, "opa": "Hypoxia induced ischemic encephalopathy", "opb": "Hypocalcemia", "opc": "Metabolic abnormality", "opd": "Sepsis", "subject_name": "Pediatrics", "topic_name": "Central Nervous system", "id": "112a0759-42f3-4cdd-84be-6483f2e53804", "choice_type": "single"} {"question": "Drug dosage best decided in child by", "exp": "Ans. is 'a' i.e., Weight", "cop": 1, "opa": "Weight", "opb": "Age", "opc": "Height", "opd": "Investigation", "subject_name": "Pediatrics", "topic_name": null, "id": "c572e92e-744c-4300-b9aa-3c9f86eba2fd", "choice_type": "single"} {"question": "Fluid of choice in child with burn < 24 hour is", "exp": "Ans. is 'c' i.e., Ringer lactate Fluid resuscitation in burn injury Parkland formulaa a. Initial 24 hours: Ringer's lactated (RL) solution 4 ml/kg/% burn for adults and 3 ml/kg/% burn for children. RL solution is added for maintenance for children: 4 ml/kg/hour for children weighing 0-10 kg 40 ml/hour +2 ml/hour for children weighing 10-20 kg 60 ml/hour + 1 ml/kg/hour for children weighing 20 kg or higher This formula recommends no colloid in the initial 24 hours. b Next 24 hours: Colloids given as 20-60% of calculated plasma volume. No crystalloids. Glucose in water is added in amounts required to maintain a urinary output of 0.5-1 ml/hour in adults and 1 ml/hour in children.", "cop": 3, "opa": "Fresh frozen plasma", "opb": "Isolye-P", "opc": "Ringer lactate", "opd": "Platlet tranfusion", "subject_name": "Pediatrics", "topic_name": null, "id": "37e17cfc-501f-4e17-ae9b-ce6c0efba113", "choice_type": "single"} {"question": "A 3 year old boy with normal developmental milestones with delayed speech and difficulty in communication and concentration.He is not making friends. Most probable diagnosis", "exp": "Autistic disorder:- Pervasive developmental disorders. Also called autistic spectrum disorders, cluster of syndromes that share marked abnormalities in the development of social and communicative skills. Symptoms: Qualitative impairment in social interaction as manifested by two of the following- impairment in use of multiple non verbal behaviors( eye gaze, facial expression, body postures) , failure to develop peer relationships, lack of sharing of enjoyment, lack of social or emotional reciprocity. Qualitative impairment in communication in at least one of the following areas- delay or total lack of spoken languages, marked impairment in the ability to initiate or sustain a conversation with others, stereotyped or repetitive use if languages and lack of varied spontaneous play. Restricted repetitive and stereotyped patterns of behaviors, interests, and activities as manifested by preoccupation with one or more restricted patterns of interests; inflexible adherence to nonfunctional routines or rituals ; repetitive motor mannerisms( such as rocking, hand flipping, finger flicking) and preoccupation with pas of objects. Intelligence is variable. Reference: GHAI Essential pediatrics, 8th edition", "cop": 1, "opa": "Autism", "opb": "ADHD", "opc": "Mental retardation", "opd": "Specific learning disability", "subject_name": "Pediatrics", "topic_name": "Growth and development", "id": "f5aa1a96-04cb-4556-b453-539b06b9e6e0", "choice_type": "single"} {"question": "Hypoglycemia is defined as a blood glucose value of less than", "exp": "Hypoglycemia is defined as a blood glucose value of less than 40 mg/ dl (plasma glucose of less than 45 mg/ dl). These babies should be screened for hypoglycemia at 2, 6, 12, 24, 48 and 72 hr after bih with reagent strips (dextrostix).Babies showing blood sugar value of less than 40 mg/ dl on reagent strip should be treated for hypoglycemia but should have confirmation of hypoglycemia by a lab test Appropriate for gestational age babies who are breastfeeding adequately do not require any screening for hypoglycemia.Ref: Paediatrics; O.P. Ghai; 8th edition; Page no: 179", "cop": 3, "opa": "60 mg/dl", "opb": "50 mg/dl", "opc": "40 mg/dl", "opd": "30 mg/dl", "subject_name": "Pediatrics", "topic_name": "New born infants", "id": "15391077-e62b-4cad-affd-936cd7d07ed5", "choice_type": "single"} {"question": "A child , Names 4 colours by the age of", "exp": "Emerging patterns for language behaviour from 1-5years of age 15months Jargon Follows simple commands May name a familiar object (e.g., ball) Responds to his/her name 18months 10 words (average) Names pictures Identifies 1 or more pas of the body 24months Puts 3 words together (subject, verb, object) 30months Refers to self by the pronoun \"I\" Knows full name 36months Knows age and sex Counts 3 objects correctly Repeats 3 numbers or a sentence of 6 syllables Most of the speech intelligible to strangers 48months Counts 4 pennies accurately Tells story 60months Names 4 colours Repeats sentence of 10 syllables Counts 10 pennies correctly Ref: Nelson paediatrics; Table 11-1", "cop": 4, "opa": "24 months", "opb": "36 months", "opc": "48 months", "opd": "60 months", "subject_name": "Pediatrics", "topic_name": "Growth and development", "id": "f1ba21a1-d33c-41c4-a8ef-1fb0e1bf8930", "choice_type": "single"} {"question": "Neonatal period extends upto", "exp": "Neonatal period extends from bih to <28 days (under 4 wks) of age. First week of life(<7days or <168 hrs) is known as early neonatal period. Late neonatal period extends from 7th day to <28th day. Reference : page 124 Ghai Essential Pediatrics 8th edition", "cop": 3, "opa": "21 days of life", "opb": "30 days of life", "opc": "28 days of life", "opd": "35 days of life", "subject_name": "Pediatrics", "topic_name": "New born infants", "id": "3e70f1f9-24c5-4c9f-b5b0-b082ae0947cb", "choice_type": "single"} {"question": "Aneurysm of anterior Cerebral circulation is associated with", "exp": "Saccular Aneurysm of anterior Cerebral circulation, Valvular abnormalities are seen in Autosomal dominant PCKD.", "cop": 1, "opa": "Autosomal dominant PCKD", "opb": "Autosomal recessive PCKD", "opc": "Infantile polycystic kidney disease", "opd": "Medullary cystic kidney disease", "subject_name": "Pediatrics", "topic_name": null, "id": "38fd478c-b9e0-4512-bd33-54b37150a5c5", "choice_type": "single"} {"question": "Pastia's lines are seen in", "exp": "Pastia's lines :\n\nRash gets concentration in axilla and grain.\nIt is seen in Scarlet fever.", "cop": 4, "opa": "Measles", "opb": "Varicella", "opc": "Syphilis", "opd": "Scarlet fever", "subject_name": "Pediatrics", "topic_name": null, "id": "b49b3bbb-ef01-4ab7-a1e7-63d0474edb9a", "choice_type": "single"} {"question": "Treatment of TOF", "exp": "Ans. is 'a' i.e., Modified BT shunt Tetrology of fallot Cyanonic hea disease (MC). Defect in infundibular septum leads to : Pulmonary stenosis. VSD (Not ASD) Dextroposition and overriding of aoa. Right ventricular hyperophy (NOT LVH). Pink TOF (Acyanotic TOF) when PS is mild to moderate, balanced shunt across the VSD, pt does not have cyanosis. The modified Blalock-Taussig shunt is currently the most common aoopulmonary shunt procedure and consists of a Gore-Tex conduit anastomosed side to side from the subclan aery to the homolateral branch of the pulmonary aery Fontan procedure - Pulmonary ateria with intact uentricular septum. Glenn shunt - Anastomosis between SVC and pulmonary aery (tricuspid atresia) Rastelli operation - Double outlet right ventricle with pulmonary stenosis", "cop": 1, "opa": "Modified BT shunt", "opb": "Fontan procedure", "opc": "Glenn shunt", "opd": "Rastelli operation", "subject_name": "Pediatrics", "topic_name": null, "id": "c5914536-ebb7-4c5e-80ed-d07227f28b9f", "choice_type": "single"} {"question": "Measles virus is", "exp": "Measles (rubeola) is caused by an RNA virus belonging to the Paromyxovirus family. The virus is transmitted by droplet spread from the secretions of the nose and throat usually 4 days before and 5days after the rash. Virus multiplies in the respiratory epithelium. Reference: Essential pediatrics,op ghai,9 th edition, pg no:209", "cop": 1, "opa": "Paramyxovirus", "opb": "Ohomyxovirus", "opc": "Poxvirus", "opd": "Picornavirus", "subject_name": "Pediatrics", "topic_name": "Infectious disease", "id": "de31f009-10c6-4fce-a965-d1da12e98253", "choice_type": "single"} {"question": "Frequent relapse in Nephrotic syndrome implies", "exp": "Frequent relapse : 2 or more relapse in 6 months of initial response  , 4 or more relapse in any 12 month period.", "cop": 3, "opa": "3 or more relapse in 1yr", "opb": "Relapse every month", "opc": "2 or more relapse in 6 months of initial response", "opd": "Relapse every 3 months", "subject_name": "Pediatrics", "topic_name": null, "id": "7cf97a47-4458-4e0d-8391-4402a7101e97", "choice_type": "single"} {"question": "5 years old child presented with generalised edema and ascites there was no hypertension or hematuria Renal function tests were normal . Urinalysis revealed massive proteinuria . Most probable diagnosis is", "exp": "All the above features suggest Nephrotic syndrome Most common cause is Minimal Change disease.", "cop": 4, "opa": "Membranous glomerulonephritis", "opb": "Post - streptococcal glomerulonephritis", "opc": "IgA nephropathy", "opd": "Minimal Change disease", "subject_name": "Pediatrics", "topic_name": null, "id": "0333b0e4-9c8b-4502-aaa8-0b9c0e8e4502", "choice_type": "single"} {"question": "In TGA, position of aoa is", "exp": "Ans. is 'd' i.e., Anterior and right to pulmonary aery o In TGA aoa lies anterior and to the right of pulmonary aery.", "cop": 4, "opa": "Posterior and left to pulmonary aery", "opb": "Posterior and right to pulmonary aery", "opc": "Anterior and left to pulmonary aery", "opd": "Anterior and right to pulmonary aeryTA PVC", "subject_name": "Pediatrics", "topic_name": null, "id": "12e1ffbe-8e21-4865-9108-bfee6af77e99", "choice_type": "single"} {"question": "Hyalin emembrane disease presents as", "exp": "In hyaline membrane disease the basic pathology is surfactant defeciency.surfactant is a lipoprotein containing phospholipids like phosphatydylcholine and phosphatydylglycerol. Surfactant is produced by type ll alveolar cells of lungs which helps to reduce surface tension in alveoli,in its absence surface tension increases and alveoli tend to collapse during expiration.During inspiration more negative pressure is needed to keep alveoli patent.There is inadequate oxygenation and increased work for breathing. Ref : ESSENTIAL PEDIATRICS,O.P.GHAI,PG NO:143, 7th edition", "cop": 3, "opa": "Pulmonary edema", "opb": "Hea failure", "opc": "Acute respiratory distress syndrome", "opd": "Pulmonary embolism", "subject_name": "Pediatrics", "topic_name": "Respiratory system", "id": "c7678a67-e905-42c6-ad81-0152b1d05398", "choice_type": "single"} {"question": "Most dangerous Osteogenesis Imperfecta (OI) in utero foetus", "exp": "(C) Type 3 OI# Osteogenesis Imperfecta (OI): 4 different genes responsible for collagen production, and any combination, or all, of them can be affected. These combinations produce eight types of brittle bone disease, labeled as type 1 OI through type 8 OI. The first four types are the most common. Last 4 types are extremely rare and most are subtypes of type 4 OI.> Type 1 OI is the mildest and most common form of brittle bone disease your baby can have. With this type, your child's body produces normal quality collagen, but just not enough of it.> Type 2 OV, the most severe form of brittle bone disease, is fatal. In type 2 OI, your child's body either produces poor-quality collagen or not enough of it. Type 2 OV can produce bone deformities.> Type 3 OI is a severe form of brittle bone disease that also causes bones to break easily. In Type 3 OI, your child's body procuces enough collagen, but it is of poor quality. A baby's bones can even begin to break before birth.> Type 4 OI is the most variable form of brittle bone disease because its symptoms range from mild to severe", "cop": 3, "opa": "Type 1 OI", "opb": "Type 2 OI", "opc": "Type 3 OI", "opd": "Type 4 OI", "subject_name": "Pediatrics", "topic_name": "Miscellaneous", "id": "0ec8042e-973a-4d15-8fda-08cb20df17c8", "choice_type": "single"} {"question": "In wood lamp examination,Tinea versicolor looks like", "exp": "Tinea versicolor is caused by Malassezia furfur. examination shows a golden yellow fluorescence under ultraviolet light of Wood&;s lamp. Reference : Anathanarayan & paniker&;s 9th edition, pg no:615", "cop": 3, "opa": "Green", "opb": "Red", "opc": "Yellow", "opd": "Blue", "subject_name": "Pediatrics", "topic_name": "Infectious disease", "id": "e928210e-eb19-40f5-ab22-2bcfdac5ffa7", "choice_type": "single"} {"question": "Hindmilk is richer in", "exp": "Hindmilk comes later towards the end of the feed and is richer in fat that provides more energy and gives a sense of satiety.Ref: Page 153; Ghai Essential Pediatrics; 8th edition", "cop": 3, "opa": "Carbohydrate", "opb": "Protein", "opc": "Fat", "opd": "Minerals", "subject_name": "Pediatrics", "topic_name": "Nutrition", "id": "dccf8838-68cd-468e-ab69-b7159320bb1a", "choice_type": "single"} {"question": "Most common extracranial solid tumor in children is", "exp": "Neuroblastoma is the most common extracranial solid tumor in children and the most commonly diagnosed malignancy in infants. Reference: Nelson; Neuroblastoma; Page no: 2461", "cop": 2, "opa": "Wilm's tumor", "opb": "Neuroblastoma", "opc": "Hydronephrosis", "opd": "Rhabdomyosarcoma", "subject_name": "Pediatrics", "topic_name": "Childhood tumors", "id": "dbdcd23c-ca69-4b38-967d-92b4f7e7e6b9", "choice_type": "single"} {"question": "Perifascicular Atrophy is seen in", "exp": ".It acts mostly like an autoimmune disorder. That means your body mistakes its own tissues as the enemy and attacks itself. When you have dermatomyositis, yourimmune system goes after the blood vessels inside your muscles and the connective tissues in your skin. The pathology of dermatomyositisincludes inflammation, vasculitis, and perifascicular atrophy. ... A distinctive feature ofdermatomyositis is atrophy and degeneration of myofibers at the periphery of fascicles (perifascicular atrophy-PFA), which occurs even in absence of inflammation", "cop": 4, "opa": "Duchenne Muscular Atrophy", "opb": "Wilson disease", "opc": "Beckers's dystrophy", "opd": "Dermatomyositis", "subject_name": "Pediatrics", "topic_name": "Musculoskeletal disorders", "id": "55e8549e-f485-4042-b2df-806698286142", "choice_type": "single"} {"question": "Meningomyelocele with progressive hydrocephalus is commonly seen in", "exp": "Hydrocephalus + Meningomyelocele ⇒ Seen in Arnold chiari II", "cop": 1, "opa": "Arnold chiari II", "opb": "Choroid plexus papilloma", "opc": "Dandy walker malformation", "opd": "Vein of Galen malformation", "subject_name": "Pediatrics", "topic_name": null, "id": "fe5ef759-244d-43b9-9270-f98ed1c1f872", "choice_type": "single"} {"question": "Klinefelter syndrome is diagnosed by", "exp": "Persons with Klinefelter syndrome are identified with the help of karyotyping. 80% of children with klinefelter syndrome have a male karyotype of X-47,XXY,while the remaining 20% have multiple sex mosaicism or structurally abnormal X chromosome Reference: (Nelson TB of pediatrics pg 410 edition 19.", "cop": 1, "opa": "Karyotyping", "opb": "USG abdomen", "opc": "Triple test", "opd": "Echocardiography", "subject_name": "Pediatrics", "topic_name": "Genetic and genetic disorders", "id": "25bd6148-644a-4bc1-882d-9424499e232c", "choice_type": "single"} {"question": "24 hour normal fluid requirement for a one year old child weighing 8 kg is", "exp": "Ans: (b) 800 mlRef : OP Ghai 8th ed./73f 293The fluid requirement of children less than 10 kg is 100ml per kg of body weight. The preffered fluid shall be N/5 in 5% dextrose with added potassium for daily requirements.", "cop": 2, "opa": "700 ml", "opb": "800 ml", "opc": "900 ml", "opd": "1000 ml", "subject_name": "Pediatrics", "topic_name": "Maintenance and Replacement Therapy", "id": "1cfa3666-23f0-461a-b187-2d2a2784a96d", "choice_type": "single"} {"question": "APGAR score include A/E", "exp": "Apgar score is an objective method of evaluating the newborn&;s condition. It includes Hea rate, Respiratory effo, Muscle tone, Reflex irritability and Color. It is generally performed at 1min and again at 5 min after bih. Reference : page 126 Ghai Essential Pediatrics 8th edition", "cop": 3, "opa": "Respiratory effo", "opb": "Hea rate", "opc": "Respiratory rate", "opd": "Colour", "subject_name": "Pediatrics", "topic_name": "New born infants", "id": "4fa29ea9-bf76-4f1d-a099-a176bb6d76f0", "choice_type": "single"} {"question": "The most common presentation of hypoparathyroidism beyond the neonatal period is", "exp": "In postnatal period hypoparathyroidism presents with tetany of hands and feet,resulting in carpopedal spasm. Hyperreflexia due to hypocalcemia is common. Trousseau sign is a carpopedal spasm that occurs after a blood pressure cuff around the arm is inflated above the systolic blood pressure for several minutes. Chvostek sign (ie, twitching of ipsilateral facial muscles with tapping on the facial nerve in front of the ear and just below the zygomatic bone) is a manifestation of neuromuscular excitability. Chvostek sign is present in 25% of healthy adults and in even higher rates in children. Thus, its presence or absence should be documented prior to thyroidectomy. Reference: Ghai essential of pediatrics, eighth edition, p.no:521.", "cop": 2, "opa": "Syncope secondary to prolonged QT intervals", "opb": "Tingling of extremities", "opc": "Seizure", "opd": "Bronchospasm", "subject_name": "Pediatrics", "topic_name": "Endocrinology", "id": "34b84cfb-cee1-4336-81b9-fa3b60911f53", "choice_type": "single"} {"question": "Most common organism associated with cystic fibrosis", "exp": "Pseudomonas may colonise s of patients with cystic fibrosis and causes recurrent pulmonary exacerbations. Ref : Essential pediatrics,O.P.Ghai,7 th edition,pg no:355", "cop": 1, "opa": "Pseudomonas aeruginosa (non mucoid)", "opb": "Burkholderia cepacia", "opc": "Pleisomonas", "opd": "Aeromonas", "subject_name": "Pediatrics", "topic_name": "Respiratory system", "id": "512a6c8b-1e13-4a5a-bf65-61873e522931", "choice_type": "single"} {"question": "Down&;s syndrome most commonly occurs due to", "exp": "Down syndrome occurs due to trisomy 21. In 97% of cases it occurs as result of non dysfunction of chromosome during maternal meiosis. Reference : Nelson TB of pediatrics 19th edition pg 402.", "cop": 2, "opa": "Reciprocal translocation", "opb": "Nondysjunction in maternal meiosis", "opc": "Translocation defect", "opd": "Nondysjunction in paternal meiosis", "subject_name": "Pediatrics", "topic_name": "Genetic and genetic disorders", "id": "95e1aa0c-8b7d-45ae-bdfa-77c2cb47191b", "choice_type": "single"} {"question": "Most commonly associated disorder with Down's syndrome among the following is", "exp": "Among the following options, Early onset Alzheimer's disease is most appropriate.\n\nAML & ALL have equal incidence\nDuodenal atresia is most Common GI disorder associated with Down's syndrome.\nEarly onset Alzheimer's disease (Pre-senile Alzheimer's in associated with Down's syndrome.", "cop": 4, "opa": "Hirschsprung disease", "opb": "CML", "opc": "Late onset Alzheimer's disease", "opd": "Early onset Alzheimer's disease", "subject_name": "Pediatrics", "topic_name": null, "id": "4d8eee37-4ddf-4651-a434-271016508767", "choice_type": "single"} {"question": "Most common inherited cause of Intellectual disability", "exp": "Most common inherited caused of Intellectual disability is Fragile X syndrome.", "cop": 2, "opa": "Down's syndrome", "opb": "Fragile X syndrome", "opc": "Huntington's chorea", "opd": "Ataxia telangiectasia", "subject_name": "Pediatrics", "topic_name": null, "id": "97012fe1-51d8-441d-91b7-4e1c13ba97e4", "choice_type": "single"} {"question": "Congenital dislocations", "exp": "Ans. is 'a' i.e., Scurvy", "cop": 1, "opa": "Scurvy", "opb": "Rickets", "opc": "Polio", "opd": "Osteomalacia", "subject_name": "Pediatrics", "topic_name": null, "id": "46afe3c5-898d-4f5b-9ccf-a357852c3798", "choice_type": "single"} {"question": "Most common cause of convulsion on the First day of life in a newborn is", "exp": "Most common cause of seizure in first day of new born baby is anoxia ,hypoxia induced ischemic. Symptomatic neonatal seizures may be caused by: lack of oxygen before or during bih because of problems such as placental abruption (premature detachment of the placenta from the uterus), a difficult or prolonged labour, or compression of the umbilical cord infection acquired before or after bih, such as bacterial (a blood clot in the brain) bleeding in the brain congenital brain abnormalities, either genetic or acquired during fetal development, such as tuberous sclerosis blood sugar or electrolyte imbalances, including hypoglycemia (low blood sugar), hypocalcemia (low calcium), hyponatremia (low sodium), or hypernatremia (high sodium) metabolic problems, such as maple syrup urine disease, pyridoxine dependency, or drug withdrawal, which may be seen in infants born to mothers addicted to barbiturates, alcohol, heroin, cocaine or methadone Reference: GHAI Essential pediatrics, 8th edition", "cop": 1, "opa": "Anoxia", "opb": "Head injury", "opc": "Hypoglycemia", "opd": "Hypocalcemia", "subject_name": "Pediatrics", "topic_name": "Central Nervous system", "id": "c85b78e6-6cfd-43e9-813e-1c1fa343d339", "choice_type": "single"} {"question": "Holt oram syndrome is characterised by", "exp": "Holt or am syndrome characterized by ASD. Ghai essential of pediatrics, eighth edition, p.no:401,table 15.7", "cop": 1, "opa": "ASD", "opb": "Pulmonary stenosis", "opc": "TGA", "opd": "TAPVC", "subject_name": "Pediatrics", "topic_name": "C.V.S", "id": "83898b29-bcd9-41dc-a4ce-81a5b699c021", "choice_type": "single"} {"question": "Pneumatoceles often develop in children after pneumonia due to the following organism", "exp": "Radiological signs are extensive in a form of massive consolidation.E.coli,Klebsiella ,staphylococcus pneumonia have pneumatoceles. Ref : Essential pediatrics,O.P.Ghai,7 th edition,pgno:355", "cop": 3, "opa": "Klebsiella", "opb": "Streptococcus", "opc": "Staphylococcus aureus", "opd": "Haemophilus influenzae", "subject_name": "Pediatrics", "topic_name": "Respiratory system", "id": "4f07ac95-23f6-422b-a9ce-6611c5175be9", "choice_type": "single"} {"question": "1st Visible sign of puberty in males is", "exp": "Tanner's staging in Males\nStage I :\nTesticular size < 4ml\nStage II : \nSparse hair growth\nScrotum and testis (>4ml) enlarge\nStage III : \nHair growth darkens\nPenis grows in lenght\nStage IV :\nHair growth becomes dense and coarse\nScrotum and penis continues to grow\nGrowth spurt occurs\nStage V :\nHair growth extend to inner thigh\nGenitalia reaches adult size", "cop": 2, "opa": "Testicular enlargement > 3ml in Stage III", "opb": "Testicular enlargement > 4ml in Stage II", "opc": "Growth of penis", "opd": "Testicular enlargement > 3ml in Stage I", "subject_name": "Pediatrics", "topic_name": null, "id": "dae398cf-2600-440a-aef3-88bb55c44649", "choice_type": "single"} {"question": "5 year old child with pseudomembrane in throat, fever, sore throat, the organism causing is", "exp": "Corynebacterium diphtheriaNon-motile, non-capsulated, club-shaped gram-positive bacillusInfect nasopharynx & release exotoxin which cause local inflammation of epithelial surface, the formation of a membrane (pseudomembrane) and toxemia.(Refer: Nelson's Textbook of Pediatrics, SAE, 1st edition, pg no. 1345 - 1348)", "cop": 2, "opa": "Comma shaped gram negative bacilli", "opb": "Club shaped gram positive bacilli", "opc": "Spore forming gram positive bacilli", "opd": "Filamentous gram positive bacilli", "subject_name": "Pediatrics", "topic_name": "All India exam", "id": "92d0f201-03f1-4425-ac15-753b362328b7", "choice_type": "single"} {"question": "Wimberger sign of Scurvy is defined as", "exp": null, "cop": 2, "opa": "Calcified cartilage seen at metaphysis", "opb": "Sclerotic rim around epiphyseal centre of Ossification", "opc": "Zone of rarefaction at metaphysis", "opd": "Ground glass appearance", "subject_name": "Pediatrics", "topic_name": null, "id": "256a870e-266f-4bd5-a7cc-deca22f4fdd0", "choice_type": "single"} {"question": "Calorie requirement per a day of a child weighing 15 kg would be", "exp": "We draw the energy from our food: The nutrients are transpoed to the body cells in the digestive tract after splitting up and are broken down there. This releases the required energy. They are measured in kilojoule (kJ) or kilocalories (kcal). One kilocalorie corresponds to 4,184 kilojoules. The calorie though is an outdated unit of measurement and was replaced in scientific use by the joule. In common parlance, calories are still often used. Reference: GHAI Essential pediatrics, 8th edition", "cop": 2, "opa": "1150 kcal", "opb": "1250 kcal", "opc": "1450 kcal", "opd": "1550 kcal", "subject_name": "Pediatrics", "topic_name": "Nutrition", "id": "2511881b-10b7-4e0f-a132-28589d6d9112", "choice_type": "single"} {"question": "Most common cause of Pyogenic Meningitis in 6 month to 2 years of age is", "exp": "Bacterial meningitis is caused by several different types of bacteria, including: Streptococcus pneumoniae, also called pneumococcus. Neisseria meningitidis, also called meningococcus. Haemophilus influenzae, also called Hib. Listeria monocytogenes. group B strep E. coli. Reference: GHAI Essential pediatrics, 8th edition", "cop": 3, "opa": "Staphylococcus aureus", "opb": "Pneumococcus", "opc": "Streptococcus pneumonia", "opd": "H. influeuenzae", "subject_name": "Pediatrics", "topic_name": "Central Nervous system", "id": "e8beec15-740e-4ecf-ad01-1a52fdc45668", "choice_type": "single"} {"question": "Treatment of Rolandic epilepsy is", "exp": "Rolandic epilepsy otherwise called as benign childhood epilepsy with centrotemporal spikes.It typically stas during childhood and is outgrown in adolescence.The child typically wakes up at night due to focal seizures causing throat tingling and tonic/clonic contractions of one side of the face with preserved consciousness.Seizure syndromesDrug of choiceROLANDIC EPILEPSYCARBAMAZEPINEWEST SYNDROME/ INFANTILE SPASMSACTH(FIRST CHOICE)VIGABATRIN(SECOND CHOICE)VALPROATE/CLONAZEPAMDRAVET SYNDROMEVALPROATE/CLONAZEPAM", "cop": 3, "opa": "Phenytoin", "opb": "Lamotrigine", "opc": "Carbamazepine", "opd": "ACTH", "subject_name": "Pediatrics", "topic_name": "All India exam", "id": "b67961e9-233c-4b0d-879e-302663efd6a9", "choice_type": "single"} {"question": "Treatment in days for meningitis caused by Streptococcus pneumonia in children is", "exp": "The child with bacterial meningitis shows improvement in 10 days and rarely necessary beyond 14 daysStaph meningitis and gram-negative infection require treatment for 21-28 daysN.meningitis- Penicillin for 5-7 daysH. Influenza- for 7-10 days(Refer: Nelson's Textbook of Pediatrics, SAE, 1st edition, pg no. 1317)", "cop": 3, "opa": "5 days", "opb": "7 days", "opc": "10 days", "opd": "21 days", "subject_name": "Pediatrics", "topic_name": "All India exam", "id": "ce7b163e-1e34-4697-8d50-3976b352eb56", "choice_type": "single"} {"question": "In Kawasaki disease, desquamation and denudation of skin from fingers and toes occur at", "exp": "Kawasaki disease can be divided into 3 clinical phases:Clinical phasedurationClinical featuresAcute febrile phase1-2 weeksfeverSubacute2-3 weeksDesquamation and peeling of fingers and toesCoronary aery aneurysmHigh risk of sudden deathConvalescence6-8weeksrecoveryKawasaki disease- Clinical Criteria1. Fever persisting at least for 5 days2. Presence of at least 4 of the following features. a) Changes in extremities:Acute: erythema of palms, soles, edema of hands and feetsSubacute: periungual peeling of fingers, toesb) Polymorphous exanthemac) Bilateral nonexudative bulbar conjunctival injectiond) Changes in lips and oral cavity(erythema, strawberry tongue)e) Cervical lymphadenopathy( unilateral and >1.5cm diameter)Other Features:GIANT CORONARY AERY ANEURYSMS (>8mm internal diameter)Thrombocytosis after 1st weekInduration of BCG inoculation site", "cop": 2, "opa": "3rd - 6th weeks", "opb": "2nd - 3rd week", "opc": "After 1st week", "opd": "After 6 weeks", "subject_name": "Pediatrics", "topic_name": "All India exam", "id": "512df70d-d91a-469e-a756-62427d8fbcec", "choice_type": "single"} {"question": "In developmental milestones of children, first draws", "exp": "(A) Zero > Scribbling on paper with a crayon or pencil. The child scribbles spontaneously by 12-24 months, copies arid draws a horizontal or vertical line at 2 years; a circle by 3 years, a cross (plus sign) by 4 years and tilted cross (multiplication sign) by 5 years. He can draw a rectangle by the age of 4 years and a triangle by 5 years.", "cop": 1, "opa": "Zero", "opb": "Triangle", "opc": "Quadrangle", "opd": "Rectangle", "subject_name": "Pediatrics", "topic_name": "Miscellaneous", "id": "89767cab-d25d-4e6d-b6f0-ee9325ffbc09", "choice_type": "single"} {"question": "Deep white matter lesion with bilateral deep bright thalamic appearance is suggestive of", "exp": "Ans. is 'c' i.e. Krabbe's disease Deep white matter lesion along with the involvement of thalamus suggests Krabbe's disease.Krabbe's disease is a leukodystrophy.LeukodystrophyIt is a term commonly used to refer to demyelination disorders.They are mainly due to genetic defects in the formation and maintenance of myelin.They usually present in infants and children with increasing development delay, progressive dementia and neurological deficit.These disorders are inherited as autosomal recessive trait and are associated with lysosomal enzyme deficiencies.They include the following disorders:-Metachromatic leukodystrophyGloboid leukodystrophy (Krabbe's disease)Spongiform degeneration (Canavan's disease)T. scan of leukodystrophies:-C. T. scan of the leukodystrophies shows low density areas in the white matter which are bilateral but not necessary symmetric and becomes more extensive as the disease process progresses.Krabbe's diseaseIt occurs due to deficiency of the enzyme P-galactocerebroside.It presents in infancy with retardation and spasticity.CT scan appearance of Krabbe's diseaseB/L symmetrical hypodensities in both frontoparietal and occipital regions in the white matter zone.Both thalamus and basai ganglia are hyperdenseCerebellum unaffected, ventricles normal, septum midlineMetachromatic leukodystrophyIt is a common leukodystrophy which commences in infancy.It occurs due to deficiency of the enzyme Arylsulphatase A.C. T. shows diffuse symmetric attenuation of the cerebellar and cerebral white matter.Alexander's diseaseIt is one of the few leukodystrophies that occurs sporadically and is characterized histopathologically by the abundant presence of Rosenthal fibres in the affected brain.Definitive diagnosis thus depends on brain biopsy.It occurs due to mutation in glial fibrillary acidic protein (GFAP).C. T. scan demonstrates degeneration of the white matter most prominent in the frontal lobes.Canavan's diseaseIt occurs due to deficiency of the enzyme aspart acylase leading to accumulation of N-Acetyl aspartic acid in the brain.C. T. scan shows diffuse white matter degeneration primarily in the cerebral hemispheres with less involvement in the cerebellum and brainstem.", "cop": 3, "opa": "Alexander disease", "opb": "Canavan's diseases", "opc": "Krabbe's disease", "opd": "Metachromatic leukodystrophy", "subject_name": "Pediatrics", "topic_name": "Endocrinology", "id": "ba81aeb9-d666-470d-aadc-17913046da14", "choice_type": "single"} {"question": "Only congenital cyanotic heart disease with Left Ventricular hypertrophy and Left atrial enlargement", "exp": "Tricuspid atresia→blood from Right atrium to Left atrium→more blood to Left atrium leading to Left atrial enlargement→more blood to left Ventricle leads to left ventricular hypertrophy.", "cop": 2, "opa": "Coarctation of Aorta", "opb": "Tricuspid atresia", "opc": "Ebstein's anomaly", "opd": "Transposition of great arteries", "subject_name": "Pediatrics", "topic_name": null, "id": "a35f60a8-1b0b-4f0f-b45e-ab32aee048e0", "choice_type": "single"} {"question": "Liver and spleen produce RBC's for", "exp": "Ans. a (Prenatal life). (Ref. Nelson, Paediatrics, 17th/pg.l599; Dutta, Obst., 4th/pg.42)DEVELOPMENTAL HEMATOPOIESIS OCCURS IN ANATOMICAL STAGESMesoblasticHepaticMyeloidYolk sacLiverBone marrow10-14th day to 10-12 weeks6-8 weeks to throughout the gestation5th month to throughout life long", "cop": 1, "opa": "Prenatal life", "opb": "First year of life", "opc": "Upto five years of life", "opd": "Lifelong", "subject_name": "Pediatrics", "topic_name": "Blood", "id": "6c1583ed-dcf0-4958-933f-f7105fdf8b1e", "choice_type": "single"} {"question": "Patau syndrome feature include A/E", "exp": "Patau syndrome is due to trisomy 13. It is characterised by holoprosencephaly,hypotelorism and cleft lip. Rocker bottom feet is a feature of Edward syndrome or trisomy 18. Reference: Nelson TB of pediatrics pg 404 19th edition.", "cop": 4, "opa": "Cleft lip", "opb": "Hypotelorism", "opc": "Holoprosencephaly", "opd": "Rocker bottom foot", "subject_name": "Pediatrics", "topic_name": "Genetic and genetic disorders", "id": "f986a3d3-3448-4714-9d21-e28ca0f60331", "choice_type": "single"} {"question": "NOT a feature of Digeorge Syndrome", "exp": "Features of Digeorage Syndrome include parathyroid gland hypoplasia leading to Hypocalcemia, not Hypercalcaemia.", "cop": 4, "opa": "Caused due to deletion of long arm of Chromosome 22", "opb": "Featured with Cleft palate", "opc": "Associated with cardiac septal defects", "opd": "Features includes Hypercalcaemia", "subject_name": "Pediatrics", "topic_name": null, "id": "481b03fe-ecd2-4923-85c3-9de1cf5d4cc7", "choice_type": "single"} {"question": "Breast milk is deficient in the following vitamins", "exp": "Breast milk is rich in all Vitamins except Vitamin D, K and B12 (in strict vegetarain Mother)", "cop": 3, "opa": "Vitamin C", "opb": "Vitamin A", "opc": "Vitamin D", "opd": "Vitamin E", "subject_name": "Pediatrics", "topic_name": null, "id": "4cba7672-036d-48e3-b615-b2d83c2f1c8b", "choice_type": "single"} {"question": "Pulmonary hypoplasia with urinary problems is associated with", "exp": "Potter syndromePotter syndrome refers to the typical physical appearance and associated pulmonary hypoplasia of a neonate as a direct result of oligohydramnios and compression while in utero.In Potter syndrome, the primary problem is kidney failure mostly due to renal agenesis. The kidneys fail to develop properly as the baby is growing in the womb.(Refer: Nelson's Textbook of Pediatrics, SAE, 1st edition, pg no. 2554)", "cop": 2, "opa": "Mobius syndrome", "opb": "Potter syndrome", "opc": "Patau Syndorme", "opd": "WAGR syndrome", "subject_name": "Pediatrics", "topic_name": "All India exam", "id": "e48321de-6eac-4c37-8094-12e301ea3588", "choice_type": "single"} {"question": "The most appropriate treatment for Homocystinuria is", "exp": "Treatment with high doses of vitamin B6 (200-1,000 mg/24 hr) causes a dramatic improvement in most patients who are responsive to this therapy. The degree of response to vitamin B6 treatment may be different in different families. Some patients may not respond because of folate depletion; a patient should not be considered unresponsive to vitamin B6 until folic acid (1-5 mg/24 hr) has been added to the treatment regimen. Restriction of methionine intake in conjunction with cysteine supplementation is recommended for patients who are unresponsive to vitamin B6.Reference: Nelson Textbook of Paediatrics; 20th edition; Chapter 85; Defects in Metabolism of Amino Acids", "cop": 4, "opa": "Betaine", "opb": "Folic acid", "opc": "Vitamin C", "opd": "Vitamin B6", "subject_name": "Pediatrics", "topic_name": "Metabolic disorders", "id": "4787b64c-b430-4b1a-84d8-00f7024291b4", "choice_type": "single"} {"question": "Drug of choice for the treatment of Kawasaki disease is", "exp": "Drug of choice for the treatment of Kawasaki disease is lintravenous Immunoglobulin.\nAsprin is also used as anti-inflammatory drug.", "cop": 4, "opa": "Aspirin", "opb": "Low molecular weight Heparin", "opc": "Steroids", "opd": "lintravenous Immunoglobulin", "subject_name": "Pediatrics", "topic_name": null, "id": "7bb8e4bf-03a5-489c-a429-5d47489e5728", "choice_type": "single"} {"question": "A patient of moderate VSD in chronic CCF develops clubbing with no cyanosis. Diagnosis", "exp": "Mild to moderate VSD there are minimal symptoms . VSD is the commonest congenital lesion complicated by infective endocarditis. Ref :pg 417 Ghai pediatrics. Eighth edition", "cop": 3, "opa": "Shunt reversal", "opb": "Long standing pulmonary edema", "opc": "SABE", "opd": "Pulmonary aery hypeension", "subject_name": "Pediatrics", "topic_name": "C.V.S", "id": "622c7c0b-64f3-43b2-93ba-ea26b42c6674", "choice_type": "single"} {"question": "Most common antigen involved in erythroblastosis fetalis is", "exp": "Erythroblastosis fetalis (Hemolytic Disease of the newborn)\nErythroblastosis fetalis is caused by the transplacental passage of maternal antibody active against paternal RBC antigens of the infant and is characterized by an increased rate of RBC destruction  Although more than 60 different RBC antigens are capable of eliciting an antibody response, the significant disease is associated primarily D antigen of Rh group and with ABO incompatibility.\n\nOther rare antigens involved are - \n\nC or E antigen of Rh group o RBC antigens - Cw, Cx, Cu, K (kell), M, Duffy, S, P, MNS, Xg, Lutheran, Diego and Kidd\nAnti-Lewis antibodies do not cause disease.", "cop": 4, "opa": "C antigen in Rh group", "opb": "E antigen in Rh group", "opc": "Duffy antigen", "opd": "D antigen in Rh group", "subject_name": "Pediatrics", "topic_name": null, "id": "abe766ce-4f8f-4e81-b163-ae32c770d279", "choice_type": "single"} {"question": "Conjugated hyperbiliubinemia in infancy is seen", "exp": "Dubin Johnson syndrome is an autosomal recessive disorder resulting from impaired hepatic excretion of bilirubin and thus causes conjugated hyperbilirubinemia(2-6 mg/dl). Gilbe syndrome, Crigler- Najjar syndrome and breast milk jaundice are causes of unconjugated hyperbilirubinemia. Reference : page 312 Ghai Essential Pediatrics 8th edition", "cop": 3, "opa": "Gilbe syndrome", "opb": "Criggler najjar syndrome", "opc": "Dubin Johnson syndrome", "opd": "Breast milk jaundice", "subject_name": "Pediatrics", "topic_name": "New born infants", "id": "25a18a56-173d-4a52-85ff-d3a3db8311ca", "choice_type": "single"} {"question": "A 5 year old child presents with mental retardation, seizures, with lighter skin complexion. The diagnosis is", "exp": "(C) Phenylketonuria# Phenylketonuria: Phenylketonuria (PKU) is a disorder that causes a buildup of the amino acid phenylalanine, which is an essential amino acid that cannot be synthesized in the body but is present in food.> Most affected newborns are detected during routine screening tests. Newborns with PKU rarely have symptoms right away, although sometimes an infant is sleepy or eats poorly.> If not treated, affected infants progressively develop mental retardation over the first few years of life, which eventually becomes severe.> Other symptoms include seizures, nausea and vomiting, an eczema-like rash, lighter skin and hair than their family members, aggressive or self-injurious behavior, hyperactivity, and sometimes psychiatric symptoms.> Untreated children often give off a \"mousy\" body and urine odor as a result of a by-product of phenylalanine (phenylacetic acid) in their urine and sweat.", "cop": 3, "opa": "Maple syrup urine disease", "opb": "Albinism", "opc": "Phenylketonuria", "opd": "Alkaptonuria", "subject_name": "Pediatrics", "topic_name": "Miscellaneous", "id": "5c4ed426-14aa-47a6-b08d-de3c3178cca1", "choice_type": "single"} {"question": "Newborn infant prevents hypothermia by", "exp": "Normally, Oxidation occurs\n↓\nEnergy generated in used for ATP production by phosphorylation\nThis is known as Coupling of oxidation with phosphorylation.\nIn Newborn: Due to Brown fat,\nEnergy generated from oxidation is used for heat production not ATP production to prevent hypothermia.\nThis Mechanism is Non - shivering thermogenesis or Uncoupling of oxidation with phosphorylation.", "cop": 3, "opa": "Coupling of oxidation with phosphorylration", "opb": "Uncoupling of oxygenation with phosphorylation", "opc": "Non - shivering theromgenesis", "opd": "Shivering thermogenesis", "subject_name": "Pediatrics", "topic_name": null, "id": "c371a2f0-ea23-4811-949c-7ff24ab10d68", "choice_type": "single"} {"question": "Developmental milestone attained at 9 months of age is", "exp": "Developmental milestones:-\nGROSS MOTOR DEVELOPMENT:\n2 months: Holds head in the plane of rest of the body when held in ventral suspension. In the prone position in bed, the chin lifts momentarily.\n3 months: lift head above the plane of the body. Head control starts by 3 months and fully developed by 5 months.\n4 months: Remain on forearm support if put in prone position, lifting the upper part of the body off the bed.\n5 months: Rolls over.\n6 months: sit in a tripod fashion.\n8 months: sits without support., crawling\n9 months: Takes a few steps with one hand held. Pulls to standing and cruises holding on to furniture by 10 months. \n10 months: creeps\n12 months: creeps well, walk but falls, stand without support.\n15 months: walks well, walks backward/ sideways pulling a toy. May crawl upstairs.\n18 months: Runs, walks upstairs with one hand held. Explores drawers.\n2 years: walk up and downstairs, jumps.\n3 years: rides tricycle, alternate feet going upstairs.\n4 years: hops on one foot, alternate feet going downstairs.\n5 years: skips\nFINE MOTOR DEVELOPMENT:-\n2 months- eyes follow objects to 180 deg.\n3 months-Grasp reflex disappears and hand is open most of the time.\n4 months- Bidextrous approach( reaching out for objects with both hands).\n6 months- Unidextrous approach( Reach for an object with one hand).\n8 months- radial grasp start to develop. Turns to sound above the level of ear.\n9 months- immature pincer grasp, probes with the forefinger.\n12 months- Unassisted pincer grasp. Releases object on request.Uses objects predominantly for playing, not for mouthing. Holds block on each hand and bang them together.\n15 months- imitate scribbling, tower of two blocks.\n18 months- scribbles, tower of 3 blocks. turn pages of a book, 2-3 at a time.\n2 years- tower of 6 blocks, vertical and circular stroke.\n3 years- tower of 9 blocks, dressing and undressing with some help, can do buttoning.\n4 years- copies cross, bridge with blocks.\n5 years- copies triangle, gate with blocks.\nSOCIAL AND ADAPTIVE MILESTONES:\n2 months: social smile(smile after being talked to).watches mother when spoken to and may smile.\n3 months: recognizes mother, anticipates feeds.\n4 months: holds rattle when placed in hand and regards it. Laughs aloud. Excited at the sight of food.\n6 months: recognizes strangers, stranger anxiety. Enjoy watching own image in the mirror, shows displeasure when toy pulled off.\n9 months: waves bye bye.\n12 months: comes when called, plays a simple ball game. kisses the parent on request. Makes postural adjustments for dressing.\n15 months: jargon, starts imitating mother. \n18 months: copies parents in tasking, dry by day, calls mother when he wants potty, points to three parts of the body on request.\n2 years: ask for food, drink, toilet, pulls people to show toys.\n3 years: shares toys, know full name and gender, dry by night.\n4 years: Plays cooperatively in a group, goes to the toilet alone, washes face, brushes teeth. Role play.\n5 years helps in the household task, dresses and undresses.\nLANGUAGE MILESTONES:\n1 month: alerts to sound.\n2 month: respond to sound by startle or quietening to a smooth voice.\n3 months: babbles when spoken to. Makes sounds (ahh, coos, ) laughs.\n4 months: laughs aloud.\n6 months: monosyllables\n9 months: understands spoken words, bisyllables.\n12 months: 1-2 words with meaning.\n18 months: a vocabulary of 10 words. Can name one part of the body.\n2 years: 3-word simple sentences.\n3 years: asks questions, knows full name and gender.\n4 years: says songs or poem, tells a story, knows three colors.\n5 years: ask the meaning of words.", "cop": 4, "opa": "Knows full name and gender", "opb": "Monosyllables", "opc": "Builds tower of 2 blocks", "opd": "Stands with support", "subject_name": "Pediatrics", "topic_name": null, "id": "c43c90d9-1a6c-43f8-a0e7-6ae7ec9322ab", "choice_type": "single"} {"question": "Most common presentation of Wilm&;s tumor is", "exp": "The most common initial clinical presentation for Wilm's tumor is the incidental discovery of an asymptomatic abdominal mass by parents while bathing or clothing an affected child or by a physician during a routine physical examination. Hypeension is present in approximately 25% of tumors at presentation and has been attributed to increased renin activity. Abdominal pain, gross painless hematuria, and fever are other frequent findings in diagnosis. Occasionally, rapid abdominal enlargement and anemia occur as a result of bleeding into the renal parenchyma or pelvis. WT thrombus extends into the inferior vena cava in 4-10% of patients, and rarely into the right atrium. Patients might also have microcytic anemia from iron deficiency or anemia of chronic disease, polycythemia, elevated platelet count, and acquired deficiency of von Willebrand factor or factor VII deficiency. Reference: Nelson; Neuroblastoma; Page no: 2466", "cop": 1, "opa": "Asymptomatic abdominal mass", "opb": "Hypeension", "opc": "Hematuria", "opd": "Fever", "subject_name": "Pediatrics", "topic_name": "Childhood tumors", "id": "aa73e321-b01b-4c5b-836c-a2dd7c957d62", "choice_type": "single"} {"question": "The risk of recurrence of congenital hea disease", "exp": "Only 2-8 % of congenital hea diseases recur Ref : page no 412 ghai pediatrics eighth edition", "cop": 3, "opa": "0.80%", "opb": "1%", "opc": "2-6%", "opd": "10-12%", "subject_name": "Pediatrics", "topic_name": "C.V.S", "id": "6d94e1de-9195-4024-92ba-e71790870287", "choice_type": "single"} {"question": "Early closure of sagittal suture leads to", "exp": "Scaphalophegaly : Early closure of saggital suture. In this compensatory skull grows parallel to the affected suture in order to accommodate the growing brain Ref Ghai pediatrics eighth edition pg no 41", "cop": 1, "opa": "Scaphocephaly", "opb": "Oxycephaly", "opc": "Trigonocephaly", "opd": "Plagiocephaly", "subject_name": "Pediatrics", "topic_name": "Musculoskeletal disorders", "id": "12ff1908-d62e-49f3-a5c2-ffeb4c1a5331", "choice_type": "single"} {"question": "Most common mode of treatment of a 1 year old child with asthma is", "exp": "The pharmacological therapy of bronchial asthma involves use of drugs that relax smooth muscle and dilates the airways and drugs that decrease inflammation and there by prevent exacerbations. The commonly used bronchodilators are adrenaline,terbutaline and salbutamol.Terbutaline and salbutamol are specific beta 2 agonists and therefore less cardiac side effects. Ref : Essential paediatrics,O.P.Ghai,7 th edition ,pg no:361", "cop": 1, "opa": "Inhaled sho acting beta 2 agonist", "opb": "Oral sho acting theophylline", "opc": "Oral ketotifen", "opd": "Leukotriene agonist", "subject_name": "Pediatrics", "topic_name": "Respiratory system", "id": "ee7b75d5-cd1b-4a9e-b291-e0e45d2e126d", "choice_type": "single"} {"question": "Neonatal sepsis is not caused by", "exp": ".Neonatal sepsis can be caused by bacteria such as Escherichia coli (E coli), Listeria, and some strains of streptococcus. Group B streptococcus (GBS) has been a major cause of neonatal sepsis. However, this problem has become less common because women are screened during pregnancy. Ref ghai 9e", "cop": 4, "opa": "Staphylococcus aureus", "opb": "E. coli", "opc": "Group B streptococci", "opd": "Acinobacter", "subject_name": "Pediatrics", "topic_name": "New born infants", "id": "0b46274b-f539-4438-a589-0a72c8eb2af1", "choice_type": "single"} {"question": "Ideal time gap between 2 live vaccinations is about", "exp": "(B) 4 weeksGUIDELINES FOR SPACING OF LIVE & INACTIVATED ANTIGENSAntigen CombinationRecommended minimum interval between doses>= InactivatedNone; can be administered simultaneously or at any interval between dosesInactivated & LiveNone; can be administered simultaneously or at any interval between doses>=2 live parenteral*4-week minimum interval, if not administered simultaneously * Live oral vaccines (E.g., Ty21a typhoid vaccine, oral polio vaccine) can be administered simultaneously or at any interval before after inactivated or live parenteral vaccines.If live parenteral (injected) vaccines (MMR, MMRV, Varicella zoster, and yellow fever) and live intranasal influenza vaccine (LAIV) are not administered at the same visit, they should be separated by at least 4 weeks.VACCINATION CHART FOR BABIES IN INDIABCGNewborn*. Bacilli Calmette-Guerin (BCG):*. Vaccination is given to combat Tuberculosis & prevent TB Meningitis in child.*. Vaccine contains live culture of the bacterium that causes TB in humans, but in a weakened form, which triggers the immune system of the child, preventing TB to attack the body. It is given at birth.Hep BNew born 1st6th week 2nd6th month 3rd*. Hepatitis B vaccination:*. It is given to prevent liver diseases caused by the Hepatitis B virus, which can cause liver cancer, liver failure and in some cases, death.*. The first dose of vaccination is given at birth.*. RECOMBIVAX HB Hepatitis B vaccine is non- infectious and is created out of a genetic recombination.*. It is made with the help of yeast.OPV6th month 1st9th month 2nd4-6 years 3rd*. Oral Polio Vaccine (OPV):*. It is given to prevent the polio disease. It can be given orally (OPV) or via injection (IPV). OPV is a polio vaccine, It is made up of an inactivated non-wild strain of the virus which helps produce antibodies within the body.IPV6th week 1st10th week 2nd14th week 3rd16th-18th month B1*. Inactivated Polio Vaccine (IPV):*. It is given to prevent polio disease in child. It is a mandatory vaccine for all children.*. It contains a wild strain of inactivated polio virus.*. It is usually given along with the DTP vaccine.DTP6th Week 1st10th week 2nd14th week 3rd16th-18th month B1 4-6 years B2*. Diphtheria, Tetanus, & Pertussis (DTP):*. Commonly known as a whooping cough, this vaccination is mandatory.*. Vaccine is a combination of three antigens-Diphtheria toxoid, Tetanus toxoid, and Pertussis.*. These antigens are inactivated to create the DTP vaccine.*. On receiving the vaccine, antibodies are produced within the body to fight occurrence of the diseases.Rota V6th week 1st 10th week 2nd 14th week 3rd*. This vaccination is essential to protect a child against Rotavirus infection which can cause severe diarrhea in children.*. Rota virus vaccine is composed of 5 strains of inactivated rotaviruses.*. It is developed using DNA technology to build immunity in the infants against rotavirus infection.PCV6th week 1st 10th week 2nd 14th week 3rd 13th-15th month Booster*. Pneumococcal Conjugate Vaccine (PCV):*. It is given to prevent Pneumococcal infections in child.*. Pneumococcal bacteria causes ear infections, pneumonia of the lungs, bacteremia of the blood and brain meningitis.*. PCV vaccine is inactivated & does not contain any live Pneumococcal bacteria.Influenza type B (HiB)6th week 1st10th week 2nd 14th week 3rd 16th-18th month B1*. This vaccination is necessary to protect a child against Haemophilus Influenza type B (HiB) - a bacteria that causes diseases like Meningitis, Pneumonia and Epiglottitis.*. HiB is a conjugate vaccine.*. It consists of inactivated bacterial components which on being introduced to the body helps create immunity against the bacteria.MMR9th month 1st 13th-15th month 2nd 4-6 years 3rd*. Measles, Mumps & Rubella (MMR):*. Vaccination is mandatory to prevent from diseases like Measles, Mumps, and Rubella (MMR).*. Vaccine contains live, attenuated measles, mumps and rubella virus with Moraten & Edmonston- Zagreb strain in Genotype A.Typhoid9th-12th month 24th month*. Typhoid Conjugate Vaccine (TCV):*. It is given to prevent typhoid in child. It is made up of Purified Vi polysaccharide from the TY2 S or Typhi Strain.Hep A12th month 1st 18th month 2nd*. Hepatitis A vaccination is a mandatory one to prevent chronic liver infections, protection from Hepatitis A Virus.*. Hepatitis A is made from sterile suspension of inactivated virus (Strain HM175) propagated in MRC-5.Varicella13rd-15th month 1st 4-6 years 2nd*. Varicella vaccine:*. It is a vaccine that protects against chickenpox.*. One dose of vaccine prevents 95% of moderate disease & 100% of severe disease.*. Two doses of vaccine are more effective than one.", "cop": 2, "opa": "2 weeks", "opb": "4 weeks", "opc": "8 weeks", "opd": "12 weeks", "subject_name": "Pediatrics", "topic_name": "Miscellaneous", "id": "fcbea85a-49ed-44ec-8a01-657aef89fcf9", "choice_type": "single"} {"question": "Most specific but late Radiographic feature of Scurvy is", "exp": "Laboratory Findings and Diagnosis of ScurvyThe diagnosis of vitamin C deficiency is usually based on the characteristic clinical picture, the radiographic appearance of the long bones, and a history of poor vitamin C intake. The typical radiographic changes occur at the distal ends of the long bones and are paicularly common at the knees. The shafts of the long bones have a ground-glass appearance because of trabecular atrophy. The coex is thin and dense, giving the appearance of pencil outlining of the diaphysis and epiphysis. The white line of Frankel, an irregular but thickened white line at the metaphysis, represents the zone of well-calcified cailage. The epiphyseal centers of ossification also have a ground-glass appearance and are surrounded by a sclerotic ring. The more specific but late radiologic feature of scurvy is a zone of rarefaction under the white line at the metaphysis. This zone of rarefaction (Trummerfeld zone), a linear break in the bone that is proximal and parallel to the white line, represents the area of debris of broken-down bone trabeculae and connective tissue. A Pelkan spur is a lateral prolongation of the white line and may be present at coical ends.Reference: Nelson Textbook of Paediatrics; 20th edition; Chapter 50; Vitamin C (Ascorbic Acid)", "cop": 4, "opa": "Ground Glass appearance of Shaft of long bones", "opb": "Pencil Outlining of Epiphysis", "opc": "White line of Frankel", "opd": "Trummerfeld Zone at the Metaphysis", "subject_name": "Pediatrics", "topic_name": "Musculoskeletal disorders", "id": "2f2ddde3-e13c-43db-bd6f-bd8530000149", "choice_type": "single"} {"question": "A neonate is having jaundice involving palms and soles. The approximate serum bilirubin is", "exp": "Serum levels of total bilirubin are approximately 4-6 mg/dl (Face), 6-8 mg/dl (Chest and upper abdomen), 8-12 mg/dl (Lower abdomen and thighs), 12-14 mg/dl (Arms and legs) and >15 mg/dl (Palms and soles). Yellow staining of palms and soles is a danger sign and requires urgent serum bilirubin estimation and fuher management.In general, the estimation of bilirubin levels by dermal zones is unreliable paicularly at higher TSB levels, after phototherapy and when it is carried out by an inexperienced observer. Total serum bilirubin can be assessed non invasively by a transcutaneous handheld device.Ref: Paediatrics; O.P. Ghai; 8th edition; Page no: 173", "cop": 4, "opa": "4-6 mg/dl", "opb": "6-8 mg/dl", "opc": "8-12 mg/dl", "opd": ">15 mg/dl", "subject_name": "Pediatrics", "topic_name": "New born infants", "id": "a8679550-7a1c-4174-bb3c-4f892113a7c4", "choice_type": "single"} {"question": "In children renal failure is defined as", "exp": null, "cop": 3, "opa": "Urine output less than 0.5ml/kg/hr for 8hr", "opb": "estimated creatinine clearance decrease by 50%", "opc": "Urine output less than 0.3ml/kg/hr for 24hr", "opd": "Loss of renal function > 3 months", "subject_name": "Pediatrics", "topic_name": null, "id": "897da6e5-cb94-4d6f-a5b9-e81f52160af6", "choice_type": "single"} {"question": "The most common type of total anomalous pulmonary venous connection is", "exp": "Most common type of TAPVC is supracardiac type,where 2 pulmonary vein join to form common pulmonary vein that may drains to left innominate vein or right superior venacava. Classification of TAPVC : Darling&;s classification (1957) is the most popular. 1. Type I-Supracardiac connections : 45% of TAPVC patients.The common venous confluence joins SVC. 2. Type II-Cardiac TAPVC It accounts for 25% of cases.CVC drains into coronary sinus or directly into RA. 3. Type III-Infracardiac TAPVC Approx.21% of cases.CVC drains into hepatic vein,ductus venosus,poal vein or IVC.The common pulmonary vein penetrates the diaphragm through the esophageal hiatus. 4. Type IV-Mixed Type It accounts for <10% of cases. Ref : Ghai essential of pediatrics, eighth edition, p.no:426", "cop": 1, "opa": "Supracardiac", "opb": "Infracardiac", "opc": "Mixed", "opd": "Cardiac", "subject_name": "Pediatrics", "topic_name": "C.V.S", "id": "38fd710f-c0b5-457e-84aa-9809848ad3f1", "choice_type": "single"} {"question": "A newborn baby presented with profuse bleeding from umbilical stump after bih. Probable diagnosis is", "exp": "Factor XIII deficiency Bleeding from umbilical stump suggests Factor XIII deficiency. Neonates with a positive bleeding history, paicularly with features such as delayed bleeding, umbilical stump bleeding or miscarriages and in whom the initial panel of screening test is negative should be tested for factor XIII deficiency. The normal screening tests such as PT and PTT are normal in factor XIII deficiency because factor XIII is needed to stabilize the clot after a fibrin clot has formed, so tests used for evaluating the extrinsic or intrinsic pathways are not affected. (Note, that factor XIII is used, once the clot has formed.) Test used for evaluation of factor XIII Test used in qualitative evaluation of factor XIII is evaluation of clot stability in 5M urea. Clots formed from normal individuals remain stable in 5M urea, while clots from factor XIII deficient patient dissolve in urea. This assay detects only the most severely affected patients with 1% to 2% factor XIII deficieincy or tests. A quantitative chromogenic assay has been developed but is not yet readily available. In addition to hemorrhages these patients may have poor wound healing a high incidence of infeility among males and aboion among affected females and a high incidence of intracerebral hemorrhage. About other options : Von Willebrand disease Patients with Von-Willebrand disease rarely presents in the neonatal period because plasma concentration of Von Willebrand factor is high in neonatal period. Platelet function disorders Genetic disorders that result in deficient platelet function are rare causes of bleeding in infants. Most patients with these platelet disorders do not present in neonatal period but rather later in life. They are not recognized as a problem during the neonatal period. (1) Glanzman's thromboasthenia This is an extremely rare disorder of the blood in which the platelets lack the glycoprotein Hence platelet aggregation cannot occur and bleeding time is significantly prolonged and there is increased bleeding. (ii) Bernard soulier syndrome It is an autosomal recessive disorder characterized by thrombocytopenia and large platelets (giant platelets) There is absence of the platelet membrane Gplb/IX complex. Symptoms are consistent with low or dysfunctional platelets and include easy bruising, nosebleeds, mucosal bleeding, menorrhagia and occasionally G.1. bleeding Factor XII deficiency It is not associated with clinical bleeding.", "cop": 1, "opa": "Factor XIII deficiency", "opb": "VWF deficiency", "opc": "Factor XII deficiency", "opd": "Glanzmann thrombosthenia", "subject_name": "Pediatrics", "topic_name": null, "id": "ae3d923c-c04c-4b98-b091-aba9f13d0c6e", "choice_type": "single"} {"question": "The risk of recurrence of congenital hea disease is", "exp": "The incidence of congenital hea disease in the normal population is 0.8%.The risk increases to 2-6% for a second child if the sibling is having congenital hea disease(recurrence) or if the parent is affected.When two 1st degree relatives have congenital hea disease, the risk for the subsequent child may reach 20-30%.VSD30-35%ASD(SECUNDUM)6-8%PDA6-8%TOF5-7%", "cop": 3, "opa": "0.80%", "opb": "1%", "opc": "2-6%", "opd": "10-12%", "subject_name": "Pediatrics", "topic_name": "All India exam", "id": "ebe1a13a-79b5-4179-8a1e-7366b4983c60", "choice_type": "single"} {"question": "First sign of puberty in boys is", "exp": "(Testes enlargement) (68-G) (498-Ghai 7th)First sign of sexual maturation is enlargement of breast in female and testes in boy (68-G)* Pubertal onset before he age of 8 years in girl and 9.5 years in boys is suggestive of precocious pubertyAdolescentWHO/UN definitionsStages of adolescence* Adolescent = 10-19 years* Youth = 15-24 years* Young people = 10-24 years* Early adolescence = 9-13 years* Mid adolescence = 14-15 years* Late adolescence = 16-19 years* Menarche usually occurs at breast and pubic hairs stage 4", "cop": 1, "opa": "Testes enlargement", "opb": "Breast enlargement", "opc": "Penis enlargement", "opd": "Scrotum enlargement", "subject_name": "Pediatrics", "topic_name": "Adolescence", "id": "20a242b3-0e36-4859-bedc-3f2af90851c0", "choice_type": "single"} {"question": "Maternal disomy of chromosome 15 is seen in", "exp": "Uniparental disomy occurs when both chromosomes of a pair or areas from 2 chromosome in any individual have been inherited from a single parent. Maternal uniparental disomy is seen in Leader -Willi syndrome. Paternal uniparental disomy is seen in Angelman syndrome . Reference: Nelson TB of pediatrics pg 412 19th edition.", "cop": 1, "opa": "Prader - Willi syndrome", "opb": "Klinefelter's syndrome", "opc": "Angelman syndrome", "opd": "Turner's syndrome", "subject_name": "Pediatrics", "topic_name": "Genetic and genetic disorders", "id": "6806e3f4-88b1-4014-ab2c-bb77759b2faa", "choice_type": "single"} {"question": "Commonest cause of head banging in children are", "exp": "Ans. b (Mental retardation). (Ref. Nelson, 18th/116)HABIT DISORDERS##Head bangingNail biting# Body rocking# Hair pulling (trichotillomania)# Tnumb sucking# Teeth grinding (bruxism)#Hitting/biting own body# Body manipulations# Repetitive vocalizations#Aerophagia (air swallowing)# Tics Certain habit disorders are more common in children with developmental delays, particularly those with pervasive devel- opmental disorders. Self-injurious habits, such as self-biting or head banging, can occur in up to 25% of normally developing toddlers, but are almost invariably associated with developmental delays in children older than 5 yr.NOTE: Pediatric autoimmune neuropsychiatric disorder associated with streptococcal infection (PANDAS) is a condition in which antibodies to group A streptococcus cross react with basal ganglia tissue.", "cop": 2, "opa": "Raised intracranial tension", "opb": "Mental retardation", "opc": "Headache", "opd": "Meningeal irritation", "subject_name": "Pediatrics", "topic_name": "Growth, Development, and Behavior", "id": "90628e0e-1663-43e9-b2e3-16efad99f01f", "choice_type": "single"} {"question": "Antidote of paracetamol poisoning", "exp": "i.e. (N- acetyl cysteine): (680 - Ghai 7th)Overdosage of PCM is treated with N-acetyl cysteine used orally within 16 hour after ingestion at a loading dose of 140 mg/ Kg diluted to 5% solution orally. Followed by 70 mg /Kg q 4hr for another 2 daysPCM is the most common analgesic, antipyretic used in childrenParacetamol produces a hghly reactive toxic minor metabolites - N-acetyl benzoquinone amine. Normally this metabolite is detoxified by conjugation with glutathione - when a very large doses of paracetamol is taken glucuronidation capacity is saturated and this metabolite is not detoxified (Hepatic glutathione is depleted)|Excess metabolite N acetyl benzoquinone** binds to protein in liver cells (and renal tubules)|Centrilobular hepatic necrosis**This single most important laboratory test in patient who present after acetaminophen is the acetaminophen level in the serum.Antidotes for common poisoningPoison Antidote* Acetaminophen (PCM)N- acetyl cysteine* AnticholinergicsPhysostigmine* BenzodiazepinesFlumazenil* DigoxinDigoxin immune antibody* MethemoglobinemiaFragnent* OpioidsMethylene blue* OrganophosphatesNaloxone* SalicylatesAtropine* Ethylene glycolSodium bicarbonates* MethanolFomepizole", "cop": 2, "opa": "Flumazenil", "opb": "N-acetyl cysteine", "opc": "Nalexone", "opd": "Sodium bicarbonate", "subject_name": "Pediatrics", "topic_name": "Miscellaneous", "id": "e47dffb5-4b43-4458-96cd-8f10b4604fda", "choice_type": "single"} {"question": "Drug used in congenital hea disease to keep PDA patent", "exp": "Ans. is 'a' i.e., PGE, Prostaglandin El (PGE1) infusion usually effective in keeping the ductus aeriosus open before surgical intervention to reduce hypoxemia and acidemia before surgery in ductus dependent lesion like. Pulmonary atresia TOF with severe PS TOF with pulmonary atresia Transposition of great aeries with VSD and PS Indomethacin is used for ductal closure", "cop": 1, "opa": "PGEi", "opb": "PGE2", "opc": "PGI2", "opd": "Indomethacin", "subject_name": "Pediatrics", "topic_name": null, "id": "5a73342f-9b6b-4996-b6ee-9b63df131695", "choice_type": "single"} {"question": "In phenylketonuria FeCI test with urine gives .............. color", "exp": "In ferric chloride test used for detecting the presence of phenols in phenylketonuria,a drop of ferric chloride is added to the urine.Phenols present in urine,will form a complex with Fe(III) to give a transient green colour. Reference:Essential pediatrics-Ghai,8th edition,page no:652", "cop": 1, "opa": "Green", "opb": "Blue", "opc": "Red", "opd": "Purple", "subject_name": "Pediatrics", "topic_name": "Metabolic disorders", "id": "60070840-753e-4662-9bbf-ee4017349398", "choice_type": "single"} {"question": "Salmonellosis is most common in", "exp": "child with sickle cell disease are more prone to infection by H.influenzae, S.pneumoniae, salmonella,M.pneumoniae, E.coli,S.aureus. Reference : Ghai essential of pediatrics, eighth edition, p.no:345", "cop": 1, "opa": "Sickle cell anemia", "opb": "Thalassemia", "opc": "Hemophilia", "opd": "Cystic fibrosis", "subject_name": "Pediatrics", "topic_name": "Hematology", "id": "fd65a3fa-b2e1-4ce6-8118-ffd1ed5d63c5", "choice_type": "single"} {"question": "Laron dwarfism is caused by", "exp": "Laron syndromeDwarfDevelopment of end-organ resistance to growth hormone(Refer: Nelson's Textbook of Pediatrics, SAE, 1st edition, pg no. 2640-2641)", "cop": 2, "opa": "Deficiency of GH", "opb": "GH receptor defect", "opc": "Deficiency of thyroxin", "opd": "Thyroxin receptor defect", "subject_name": "Pediatrics", "topic_name": "All India exam", "id": "a500c9a4-4bcd-4ae1-ab7d-0911cd88d088", "choice_type": "single"} {"question": "Most common manifestation of congenital Rubella syndrome is", "exp": "Triad of congenital Rubella syndrome :\n\nSalt pepper retiopathy > cataract.\nDeafness.\nPDA.\nOf which deafness is the most common finding.", "cop": 3, "opa": "Cataract", "opb": "Chorioretinitis", "opc": "Deafness", "opd": "Patent ductus arteriosus", "subject_name": "Pediatrics", "topic_name": null, "id": "39830cd9-041f-440e-872c-020d876fc558", "choice_type": "single"} {"question": "Most common infection in a child Nephrotic Syndrome", "exp": "Most common infection in children nephrotic syndrome is spontaneous bacterial peritonitis . Reference: GHAI Essential pediatrics, 8th edition", "cop": 1, "opa": "Spontaneous bacterial peritonitis", "opb": "Pneumonia", "opc": "UTI", "opd": "Cellulitis", "subject_name": "Pediatrics", "topic_name": "Urinary tract", "id": "41526ede-e3ea-4d06-a397-1aace3e8f237", "choice_type": "single"} {"question": "The time taken for air to reach the descending colon in a normal infant is", "exp": "The time taken to reach the descending colon in an infant is 7-9 hrs approximately Reference: GHAI Essential pediatrics, 8th edition", "cop": 4, "opa": "1-2 hours", "opb": "3-4 hours", "opc": "5-6 hours", "opd": "8-9 hours", "subject_name": "Pediatrics", "topic_name": "New born infants", "id": "55a98bb0-6b41-42c6-8894-8b28e67c3e85", "choice_type": "single"} {"question": "Eteplirsen has been recently FDA approved for the treatment of", "exp": "Duchenne muscular dystrophy: MC hereditary neuromuscular disease in children. X-linked recessive deletion of dystrophin gene ( Xp 21)- one of the largest gene C/F: More common in males. Family history present Pseudohyperophy of calf muscles Cardiac involvement Recurrent respiratory infection Proximal muscle weakness in limbs Intellectual disability 'GOWER SIGN'is positive- Gowers' sign indicates weakness of the proximal muscles, esp of the lower limb. Mx- Phase 3 trials have shown benefits from ETEPLIRSEN - FDA APPROVED", "cop": 1, "opa": "Duchenne muscular dystrophy", "opb": "Spinal muscular atrophy", "opc": "SSPE", "opd": "AML", "subject_name": "Pediatrics", "topic_name": "Other disorders of bone", "id": "fbaa0afd-56ec-4432-bc25-93d637f238a3", "choice_type": "single"} {"question": "Infant with cystic fibrosis (CF) are likely to develop", "exp": "The common clinical presentations include meconium ileus in neonatal period,recurrent bronchiolitis in infancy and early childhod,recurrent respiratory tract infections ,chronic lung disease , bronchiectasis, steatorrhoea,with incresing age pancreatitis and azoospermia. Ref : Essential pediatrics,O.P.Ghai,7 th edition ,pg no:36", "cop": 1, "opa": "Meconium ileus", "opb": "Loose motions", "opc": "Vomiting", "opd": "Constpation", "subject_name": "Pediatrics", "topic_name": "Respiratory system", "id": "68ccfc5f-7932-44f2-8402-98c4013f9b41", "choice_type": "single"} {"question": "Keshan disease in due to deficiency of", "exp": "Ans. is 'a' i.e., Selenium Keshan disease Cardiomyopathy associated with selenium deficiency. Wilson disease Autosomal recessive. Defective metabolism of copper leads to copper toxicity. Deficiency of ceruloplasmin. Presence of KF ring in cornea. Acrodermatitis enteropathica Deficiency of zinc. Dermatitis of extremities & around orifices.", "cop": 1, "opa": "Selenium", "opb": "Copper", "opc": "Zinc", "opd": "Iron", "subject_name": "Pediatrics", "topic_name": null, "id": "9ccffe7c-9028-40c6-ac33-d1009486c1f9", "choice_type": "single"} {"question": "A renal mass seen on prenatal checkup is most probably", "exp": "Prenatal renal tumors:Enlargement of a kidney on prenatal imaging is usually due to hydronephrosis or a cystic renal enlargement, such as multicystic dysplastic kidney or polycystic diseaseSolid tumors of the kidneys are rarely seen in the antenatal period but can be diagnosed by ultrasound. Most develop properly as the baby is growing in the womb.(Refer: Nelson's Textbook of Pediatrics, SAE, 1st edition, pg no. 2467)", "cop": 2, "opa": "Wilm's tumor", "opb": "Mesoblastic nephroma", "opc": "Neuroblastoma", "opd": "Renal sarcoma", "subject_name": "Pediatrics", "topic_name": "All India exam", "id": "b2bbef36-9018-4f6a-a507-d9624fc85e3a", "choice_type": "single"} {"question": "Most common situation associated with transient tachypnea of newborn is", "exp": "Ans. is 'c & d' i.e., Elective caesarean section & Normal vaginal delivery [Ref O.P. Ghai 8th/e p. 171 & 7th/e p. 146; Various articles of Obs & GynaTransient tachypnea of Newborn* Transient tachypnea of the newborn is a benign self-limiting disease occuring usually in term neonates and is d to delayed clearance of lung fluid. It is also called respiratory distress syndrome type II.* TTN follows -i) Uneventful normal preterm or term vaginal deliveryii) Cesarean delivery* TTN is believed to be secondary to slow absorption of fetal lung fluid resulting in decreased pulmona compliance and tidal volume and increased dead space therefore also known as wet lung.* Clinical manifestations# Early onset of tachypnea# Sometimes retraction or expiratory grunting# Occasionally cyanosis# Patients usually recover rapidly within 3 days.# Hypoxemia, hypercapnia and acidosis are uncommon.* The lungs are generally clear without rales or rhonchi and chest roentgenogram shows -# Prominent pulmonary vascular markings# Overaeration# Fluid lines in the fissure# Flat diaphragms# Prominent inter-labar fissure# Occasionally pleural fluid.* Distinguishig this disease from hyaline membrane disease is difficult. The distinctive feature of transient tachyapnea are -# Mild symptomology# Sudden recovery of the infant# Absence of roentgenographic reticulogranular pattern or air bronchogram.", "cop": 3, "opa": "Term delivery requiring forceps", "opb": "Term requiring ventouse", "opc": "Elective caesarean section", "opd": "Normal vaginal delivery", "subject_name": "Pediatrics", "topic_name": "New Born Infants", "id": "7210c823-e36b-4d39-9ade-a4c94b1ae640", "choice_type": "single"} {"question": "In a child, one should be worried if", "exp": ".ge 2 months 3 months 6 months 9 months 12 months 15 months 18 months 2 years 3 years 4 years 5 years Milestone . . ( mile after being talked to) social smile s ther* anticipates feeds Recognizes mo * . t angers stranger anxiety Recognizes s r ' waves \"bye bye\" . h Called* plays simple ball game comes w en ' Jargon . Copies parents in task (~.g. sweeping) f food drink toilet; pulls people to Asks or * ' show toys Shares toys; knows full name and gender Plays cooperatively in a group; goes to toilet alone Helps in household tasks, dresses and undress", "cop": 4, "opa": "Stammering occurs at 3 years", "opb": "Lack of Toilet control at 2.5 years", "opc": "Teeth do not erupt by 11 months", "opd": "Social smile absent by 10 weeks", "subject_name": "Pediatrics", "topic_name": "Growth and development", "id": "b837c0aa-c645-4701-bd62-cc48ad4a72e9", "choice_type": "single"} {"question": "Requirement of Potassium in child is", "exp": "Requirement of potassium in child is 1-2 mEq/kg . Reference: GHAI Essential pediatrics, 8th edition", "cop": 1, "opa": "1-2 mEq/kg", "opb": "4-7 mEq/kg", "opc": "10-12 mEq/kg", "opd": "13-14 mEq/kg", "subject_name": "Pediatrics", "topic_name": "Fluid and electrolytes", "id": "ef21f5cb-63db-4534-ac68-b251471a45d7", "choice_type": "single"} {"question": "A newborn has hydrocephalus, chorioretinitis, intracranial calcifications, and anemia. For the mentioned clinical scenario, select the most likely diagnosis", "exp": "Toxoplasmosis can cause symptoms similar to other congenital infections, but the combination of hydrocephalus, chorioretinitis, and intracranial calcifications is considered the \"classic triad\" of toxoplasma infection in a neonate. Infection usually occurs during primary infection of the mother or as a reactivation of infection in an immune-compromised host. These infants may also display symptoms similar to other congenital infections, such as anemia, a petechial rash, organomegaly, jaundice, and seizures.", "cop": 1, "opa": "Congenital toxoplasmosis", "opb": "Congenital syphilis", "opc": "Congenital rubella", "opd": "Congenital CMV", "subject_name": "Pediatrics", "topic_name": "New Born Infants", "id": "35c6e862-1c38-4d9d-a094-18a0dec28c5f", "choice_type": "single"} {"question": "Inheritence of Familial hypophosphatemic Rickets is", "exp": null, "cop": 2, "opa": "Autosomal recessive", "opb": "X-linked dominant", "opc": "X-linked recessive", "opd": "Autosomal dominant", "subject_name": "Pediatrics", "topic_name": null, "id": "6d516b44-b9c1-4dbb-bd4b-2b8eb0d936e1", "choice_type": "single"} {"question": "Acute malnutrition is manifested by", "exp": "Acute malnutrition is manifested by weight for height which is a result of more recent food deficit or illnessRef: Ghai Essential Pediatrics 8th edition", "cop": 2, "opa": "Weight for age", "opb": "Weight for height", "opc": "Age for height", "opd": "Brocas index", "subject_name": "Pediatrics", "topic_name": "Nutrition", "id": "63579113-dd9c-4d79-b371-054572c2972d", "choice_type": "single"} {"question": "Duchenne&;s Muscular Dystrophy is a disease of", "exp": "Duchenne and Becker muscular dystrophy are caused by loss-of-function mutations in the dystrophin gene on the X chromosome. The encoded protein, dystrophin, is a key component of the dystrophin-glycoprotein complex.Dystrophin is thought to provide mechanical stability to the myofiber and its cell membrane during muscle contraction.Robbins 9e 1242", "cop": 3, "opa": "Neuromuscular junction", "opb": "Sarcolemmal proteins", "opc": "Muscle contractile proteins", "opd": "Disuse atrophy due to muscle weakness", "subject_name": "Pediatrics", "topic_name": "Musculoskeletal disorders", "id": "446d3985-c357-45bc-abd2-bb874a60531d", "choice_type": "single"} {"question": "Infantile polycythemia is seen in", "exp": "Other clinical features include lethargy, tachypnea, cyanosis, jitteriness, poor feeding, feeding intolerance, hypoglycemia. Most common presentation: The majority of newborn infants withpolycythemia are asymptomatic. They can only be identified by performance of a hematocrit. Reference: GHAI Essential pediatrics, 8th edition", "cop": 1, "opa": "Cerebellar hemangioma", "opb": "Retinoblastoma", "opc": "Hepatoblastoma", "opd": "Nesidioblastosis", "subject_name": "Pediatrics", "topic_name": "Hematology", "id": "cb20523a-89d1-467d-aac6-7ed26779fde0", "choice_type": "single"} {"question": "Most common cause of Acute Aseptic Meningitis in children is", "exp": "(C) Enterovirus # VIRAL CAUSES INCLUDE THE FOLLOWING: Enteroviruses - coxsackievirus, echovirus, poliovirus. Herpes simplex virus (HSV) types 1 and 2 (HSV-1, HSV-2) Varicella-zoster virus. Arboviruses, Epstein-Barr virus, HIV, Influenza virus types A and B, Mumps virus.> Many different viruses can cause meningitis.> About 90% of cases of viral meningitis are caused by members of a group of viruses known as Enteroviruses, such as Coxsacki-eviruses and Echoviruses.> These viruses are more common during summer and fall months.>Herpesviruses and the mumps virus can also cause viral meningitis.", "cop": 3, "opa": "Arbovirus", "opb": "Respiratory syncytial virus", "opc": "Enterovirus", "opd": "Herpes family", "subject_name": "Pediatrics", "topic_name": "Miscellaneous", "id": "0b14eff2-2d49-4880-b141-d6e9e96f27aa", "choice_type": "single"} {"question": "Fanconi&;s anaemia is a", "exp": "Fanconi anemia is a rare disease passed down through families (inherited) that mainly affects the bone marrow. It results in decreased production of all types of blood cells. This is the most common inherited form of , a rare kidney disorder. Causes Fanconi anemia is due to an abnormal gene that damages cells, which keeps them from repairing damaged DNA. To inherit Fanconi anemia, a person must get one copy of the abnormal gene from each parent. The condition is most often diagnosed in children between 3 and 14 years old. Symptoms People with Fanconi anemia have lower-than-normal numbers of white blood cells, red blood cells, and platelets (cells that help the blood clot). Not enough white blood cells can lead to infections. A lack of red blood cells may result in fatigue (Changes in the color of the skin, such as darkened areas of the skin, called cafe au lait spots, and Reference: GHAI Essential pediatrics, 8th edition", "cop": 1, "opa": "Constitutional anaemia", "opb": "Hemolytic anaemia", "opc": "Iron deficiency anaemia", "opd": "Auto-immune anaemia", "subject_name": "Pediatrics", "topic_name": "Genetic and genetic disorders", "id": "ec930619-cb8e-4069-8662-4724a7dd0e55", "choice_type": "single"} {"question": "Pseudoparalysis in an infant is suggestive of", "exp": "In Vitamin C deficiency, bones are tender & the infant is reluctant to move his limbs, which are kept in a frog-like position.This may be mistaken for paralysis(pseudoparalysis).Page 127, Ghai essential pediatrics; 6th edition", "cop": 4, "opa": "Acute Rheumatic fever", "opb": "Vitamin B6 deficiency", "opc": "Vitamin E deficiency", "opd": "Vitamin C deficiency", "subject_name": "Pediatrics", "topic_name": "Nutrition", "id": "73c38c72-8f98-4592-b58b-54db9ed8e350", "choice_type": "single"} {"question": "The antidote of paracetamol poisoning", "exp": "Overdosage of PCM is treated with N-acetyl cysteine used orally within 16 hours after ingestion at a loading dose of 140 mg/ Kg diluted to 5% solution orally.\nFollowed by 70 mg /Kg q 4hr for another 2 days PCM is the most common analgesic, antipyretic used in children Paracetamol produces a highly reactive toxic minor metabolites -\nN-acetyl benzoquinone amine. Normally this metabolite is detoxified by conjugation with glutathione - when a very large dose of paracetamol is taken glucuronidation capacity is saturated and this metabolite is not detoxified (Hepatic glutathione is depleted) ↓\nExcess metabolite N acetyl benzoquinone** binds to the protein in liver cells (and renal tubules) ↓ Centrilobular hepatic necrosis** This single most important laboratory test in a patient who present after acetaminophen is the acetaminophen level in the serum.", "cop": 3, "opa": "Flumazenil", "opb": "Nalexone", "opc": "N-acetyl cysteine", "opd": "Sodium bicarbonate", "subject_name": "Pediatrics", "topic_name": null, "id": "ba51939e-ec8c-4026-bdd8-567d900eb2a4", "choice_type": "single"} {"question": "The percentage of fetal haemoglobin at bih is", "exp": "At bih, fetal hemoglobin comprises 50-95% of the infant&;s hemoglobin Reference: GHAI Essential pediatrics, 8th edition", "cop": 3, "opa": "20%", "opb": "50%", "opc": "70%", "opd": "90%", "subject_name": "Pediatrics", "topic_name": "New born infants", "id": "2c8bb6d1-5287-4a25-91ea-abb10f6027e9", "choice_type": "single"} {"question": "In RBC development, Haemoglobin first appears in", "exp": "Polychromatic/ Intermediate erythroblast is the stage where haemoglobin first appears in RBC development.", "cop": 2, "opa": "Basophilic erythroblast", "opb": "Polychromatic erythroblast", "opc": "Orthochromatic erythroblast", "opd": "Reticulocyte", "subject_name": "Pediatrics", "topic_name": null, "id": "c723ae69-aee4-4529-9b43-8fff4f195d73", "choice_type": "single"} {"question": "New born babies are able to breathe and suck at the same time due to", "exp": "Infant&;s larynx is positioned high in the neck, the level of glottis being opposite to C3 or C4 at rest and reaches C1 or C2 during swallowing. This high position allows the epiglottis to meet soft palate and make a nasopharyngeal channel for nasal breathing during suckling. The milk feed passes separately over the dorsum of tongue and the sides of epiglottis, thus allowing breathing and feeding to go on simultaneously. Reference: GHAI Essential pediatrics, 8th edition", "cop": 3, "opa": "Wide sho tongue", "opb": "Sho soft palate", "opc": "High larynx", "opd": "Sho pharynx", "subject_name": "Pediatrics", "topic_name": "New born infants", "id": "cd906451-4aaa-4b45-b7f6-f67dbaea9df8", "choice_type": "single"} {"question": "5 month old child with hepatomegaly, ketosis, hyperuricemia and malaise diagnosis", "exp": "Mucopolysaccharidosis Acid mucopolysaccharides are abnormally deposited in the tissues,and excreted in the urine. clinical features Mental retardation Cloudy cornea Bone changes:thickening of skull,broad spatula like ribs,beak shaped veebrae facies:coarse,lips are thick,teeth are peg like,bridge of nose depressed hepatosplenomegaly hyperurecemia Ref : ESSENTIAL PEDIATRICS,O.P.GHAI,PG NO:636, 7th edition", "cop": 4, "opa": "Diabetes", "opb": "Urea cycle defect", "opc": "Glycogen storage disease", "opd": "Mucopolysacchridosis", "subject_name": "Pediatrics", "topic_name": "Gastrointestinal tract", "id": "db6550da-3ca1-4fc5-9bf0-fc44dc8f613c", "choice_type": "single"} {"question": "Most common cause of stridor sholy after bih", "exp": "Laryngomalacia is the most common congenital anomaly of larynx.Inspiratory stridor is the hallmark of this condition.Symptoms aggrevated when child is in supine position or crying.Condition is generally benign and self limited and most cases resolve by 18 months of age. Ref : Essential Pediatrics,O.P.Ghai,7th edition,Pg No:340", "cop": 3, "opa": "Laryngeal papilloma", "opb": "Laryngeal web", "opc": "Laryngomalacia", "opd": "Vocal cord palsy", "subject_name": "Pediatrics", "topic_name": "Respiratory system", "id": "d8d44f5e-3afc-4a64-a455-e83feb3047e2", "choice_type": "single"} {"question": "A normal child develops the ability to use 10 words with meaning at the age of", "exp": "

. Developmental milestones:- GROSS MOTOR DEVELOPMENT: 2 months: Holds head in plane of rest of the body when held in ventral suspension. In prone position in bed, the chin lifts momentarily. 3 months:lift head above the plane of the body. Head control stas by 3 months and fully developed by 5 months. 4 months:Remain on forearm suppo if put in prone position, lifting the upper pa of the body off the bed. 5 months: Rolls over. 6 months:sit in tripod fashion. 8 months: sits without suppo., crawling 9 months: Takes a few steps with one hand held. Pulls to standing and cruises holding on to furniture by 10 months. 10 months: creeps 12 months:creeps well, walk but falls, stand without suppo. 15 months: walks well, walks backward/ sideways pulling a toy. May crawl upstairs. 18 months: Runs, walks upstair with one hand held. Explores drawers 2 years: walk up and downstairs, jumps. 3 years : rides tricycle, alternate feet going upstairs. 4 years: hops on one foot, alternate feet going downstairs. 5 years:skips FINE MOTOR DEVELOPMENT:- 2 months- eyes follow objects to 180 deg. 3 months-Grasp reflex disappears and hand is open most of the time. 4 months- Bidextrous approach( reaching out for objects with both hands). 6 months- Unidextrous approach( Reach for an object with one hand). 8 months- radial grasp sta to develop. Turns to sound above the level of ear. 9 months- immature pincer grasp, probes with forefinger. 12 months-Unassisted pincer grasp. Releases object on request.Uses objects predominantly for playing, not for mouthing. Holds block on each hand and bang them together. 15 months- imitate scribbling , tower of two blocks 18 months- scribbles, tower of 3 blocks.turn pages of a book, 2-3 at a time. 2 years- tower of 6 blocks, veical and circular stroke. 3 years-Tower of 9 blocks, dressing and undressing with some help, can do buttoning. 4 years- copies cross, bridge with blocks 5 years- copies triangle, gate with blocks. SOCIAL AND ADAPTIVE MILESTONES: 2 months: social smile(smile after being talked to).watches mother when spoken to and may smile. 3 months:Recognizes mother, anticipates feeds. 4 months: Holds rattle when placed in hand and regards it . Laughs aloud. Excited at the sight of food. 6 months:recognizes strangers, stranger anxiety . Enjoy watching own image in mirror, shows displeasure when toy pulled off. 9 months:waves bye bye 12 months:comes when called, plays simple ball game.kisses the parent on request. Makes postural adjustments for dressing. 15 months:jargon, stas imitating mother. 18 months: copies parents in tasking, dry by day, calls mother when he wants potty, points to three pas of body on request. 2 years: ask for food, drink, toilet, pulls people to show toys. 3 years:shares toys, know fullname and gender, dry by night. 4 years:Plays cooperatively in a group, goes to toilet alone, washes face, brushes teeth. Role play . 5 years:helps in household task , dresses and undresses. LANGUAGE MILESTONES: 1 month: Ales to sound. 2 month:respond to sound by stale or quitening to a smooth voice. 3 months: babbles when spoken to. Makes sounds (ahh,coos, ) laughs. 4 months: laughs aloud. 6 months: monosyllables 9 months: understands spoken words, bisyllables. 12 months: 1-2 words with meaning. 18 months: vocabulary of 10 words. Can name one pa of body. 2 years: 3 word simple sentences 3 years:asks questions, knows full name and gender. 4 years: says songs or poem, tells story, knows three colours. 5 years: ask meaning of words. {Reference: GHAI Essential pediatrics, eighth edition}", "cop": 3, "opa": "12 months", "opb": "15 months", "opc": "18 months", "opd": "24 months", "subject_name": "Pediatrics", "topic_name": "Growth and development", "id": "75b19c75-01aa-48c2-9ad1-88e773e69ecd", "choice_type": "single"} {"question": "The Most common cause of meningitis in children aged 5 years is", "exp": "Several kinds of bacteria can cause bacterial meningitis. The most common ones in the U.S. are: problems) A bacteria called Haemophilus influenzaetype b (Hib) was a common cause of meningitis in babies and young children until the Hib Neisseria meningitidis and Streptococcus pneumoniae. Reference: GHAI Essential pediatrics, 8th edition", "cop": 2, "opa": "H-influenzae", "opb": "N-meningitides", "opc": "Strep pneumonia", "opd": "Staph. aureus", "subject_name": "Pediatrics", "topic_name": "Central Nervous system", "id": "62cf391a-92ff-46ec-bd58-ca3460210c02", "choice_type": "single"} {"question": "Breastfeeding is contraindicated in", "exp": "Contraindications of breastfeedingGalactosemiaActive untreated tuberculosis-only in the initial periodHIV positive mother-especially in a developed country; In India its not contraindicatedSome medication(Refer: Nelson's Textbook of Pediatrics, SAE, 1st edition, pg no. 287)", "cop": 1, "opa": "Galactosemia", "opb": "Tuberculosis history", "opc": "Hepatitis B positive mother", "opd": "Low bih weight infant", "subject_name": "Pediatrics", "topic_name": "All India exam", "id": "714a6fa3-f1ec-4cd4-881d-28b22e1f2d63", "choice_type": "single"} {"question": "Most common cause of fresh bleeding per rectum in a 5 years old child is", "exp": "(D) (Juvenile rectal polyp)* Fresh bleeding indicates bleeding from anal canal or rectum.* Whenever a child comes with bleeding per anum, a diagnosis of rectal polyp should be made until it is excludes by rectal examination.* According to C.P.P.T. common causes of bright red bleeding in a children 2-12 year old in decreasing order frequency are-- Anal fissure- Solitary rectal ulcer- Juvenile polyp- Lymphonodular hyperplasia.Differential diagnosis of bright red bleeding from rectum InfantChild (2-12 yrs)Adolescent >12Hematochezia with diarrhea and crampy abdominal painInfectious colitisPseudomembranous colitisEosinophilic coliisHirschsprung's entrocolitisInfectious colitis pseudomembranous colitisHemolytic uremic syndromeH.S. purpuraLymphonodular hyperplasiaInfectious colitisPseudomembranouscolitisGranulomatous colitisH.U.SH.S.P.Hematochezia without diarrhoea and abdominal painAnal fissure Eosinophilic colitis Rectal gastric mucosa, heterotropiaColonic hemangiomasAnal fissureSolitary rectal ulcerJuvenile polypLymphonodular hyperplasiaAnal fissure HemorrhoidSolitary rectal ulcerColonic A-V malformationThe aetiology and man and management of rectal bleeding depends on the age of the child, the type and quantity of bleeding and the associated symptoms. Unlike adults malignancy is exceptionally rare. In infants an anal fissure, necrotizing enterocolitis, intussusceptions and allergic enterocolitis are possible causes. In older children, more common causes includes an anal fissure, a juvenile polyp and certain gastroenteritides (e.g. Campylobacter infection); Meckel's diverticulum, duplication cyst and inflammatory bowel disease are less common. - Bailey and Love.Juvenile polyps are the most common type of childhood polyp, occurring in up to 1% of preschool children. The solitary polyps found in the rectum which most commonly present with rectal bleeding.These are hamartomatous polyps, which are usually pedunculated but can be sessile. The typical child with a juvenile polyp is 4 to 6 years of age, presents with intermittent painless rectal bleeding with bowel movements. - Rudolph's PediatricsJuvenile polyps are usually are not premalignant. Because the gross appearance of these polyps is identical to adenomatous polyps, these lesions should also be treated by polypectomy.Juvenile polyposis is a term used when more 5 to 10 juvenile polyps develop.Juvenile polyposis coli applies if the polyps are limited to the colon, whereas generalzed juvenile polyposis describes the presence of polyps throughout the GI tract, both have a significant malignant potential.Mentioned options are rare in childrenFamilial adenomatous polyposis (FAP) patients present in 2nd to 3rd decade.Rectal ulcer (solitary rectal ulcer) is commonly seen in 20 to 40 yrs age group.Piles are rare in children. (Ref: Rudolp'spediatrics, 21/e chapter 17.22)* MC solid tumor in childhood- Brain tumors* MC extracranial solid tumor in childhood - Neuroblastoma* MC intraabdominal solid tumor in childhood - NeuroblastomaCauses of Recurrent Gross Haematuria (2169-Nelson '8th)1. IgA Nephropathy - (MC cause)2. Alport's syndrome3. Thin glomerular basement membrane disease4. Hypercalciuria5. Urolithiasis* MCC of vasculitis in children is Henoch- schonlein prupura (HSP)* MC tumors in children are Leukaemia* MC solid tumors in children are CNS tumors* MC site of brain tumors in children is Infratentorial (cerebellar)* MC site of brain tumors in neonates is supratentorialMC posterior fossa tumour in children is cerebellar Astrocytoma* The Leukemias (ALL) are the most common malignant neoplasms in childhood", "cop": 4, "opa": "Volvulus", "opb": "Trauma", "opc": "Worm infestation", "opd": "Juvenile rectal polyp", "subject_name": "Pediatrics", "topic_name": "Gastro Intestinal System", "id": "abcbd7de-01ac-497a-a83a-c4b760a1075f", "choice_type": "single"} {"question": "Drug of choice for Absence attacks is", "exp": "- DOC for Absent seizures: Typical - Ethosuximide Atypical & in India - Sodium valproate If nothing is mentioned in question then DOC is Ethosuximide Drug of choice for: Focal seizures : oxcarbamazepine and carbamazepine; Juvenile myoclonic epilepsy: valproate and lamotrigine; Lennox-Gastaut syndrome: clobazam, valproate, topiramate, lamotrigine, and, most recently, as add on, rufinamide Infantile spasms: adrenocoicotropic hormone (ACTH).", "cop": 3, "opa": "Phenytoin", "opb": "Primidone", "opc": "Ethosuximide", "opd": "Phenobarbitone", "subject_name": "Pediatrics", "topic_name": "Epilepsy", "id": "a9a04416-32c3-4966-8ef0-2ca5f7818126", "choice_type": "single"} {"question": "A neonate is suspected to be suffering from necrotizing enterocolitis (NEC). On fuher examination and investigation, he is diagnosed to be Bell's stage I NEC. The management of choice would be", "exp": "Ans. c. Conservative management with IV fluids and antibiotics The management of choice for Bell's Stage I necrotizing enterocolitis is conservative management with IV fluids and antibiotics. Treatment of Bell's Stage I Necrotizing Enterocolitis: NPO, II/fluids and Antibiotics for 3 days", "cop": 3, "opa": "Laparotomy and proceed", "opb": "Inseion of bilateral pelvic drains", "opc": "Conservative management with IV fluids and antibiotics", "opd": "Initial conservative management and laparotomy after 24 hours", "subject_name": "Pediatrics", "topic_name": null, "id": "cba8d965-79e8-4577-b3ad-9d178cfcdfc1", "choice_type": "single"} {"question": "Commonest cause of obstructive hydrocephalus in children", "exp": "Aqueducts stenosis is most common cause of obstructive hydrocephalus in children. Reference: GHAI Essential pediatrics, 8th edition", "cop": 1, "opa": "Acqueductal stenosis", "opb": "Aquaductal gliosis", "opc": "Subarachnoid hemorrhage", "opd": "Tubercular meningitis", "subject_name": "Pediatrics", "topic_name": "Central Nervous system", "id": "ff4fa443-2059-4f45-8528-52294e33e624", "choice_type": "single"} {"question": "Commonest type of seizure in newborn", "exp": "Hypoxic-ischemic encephalopathy : It is an impoant cause of neonatal seizures. Seizures resulting from hypoxic-ischemic encephalopathy may be seen in term and prematureinfants. They frequently present within the first 72 hours of life.Seizures may include subtle, clonic, or generalized seizures. These are subtle seizure . There are 4 major types of seizures in neonates: Subtle seizures are relatively common in the neonatal period and are more often encountered in the preterm than full term infant. Such seizures include oral-buccal-lingual movements, ceain ocular phenomena, peculiar limb movements, autonomic alterations and apnea. Clonic seizures include focal and multifocal varieties which may migrate to another pa of the body in a non-ordered fashion. Tonic seizures include focal episodes (less common) and generalized episodes (more common). Generalized tonic seizures may mimic decerebrate and decoicate posturing. Myclonic seizures may be focal, multifocal or generalized and are the least common of the four varieties during the neonatal period. Reference: GHAI Essential pediatrics, 8th edition", "cop": 3, "opa": "Clonic", "opb": "Tonic", "opc": "Subtle", "opd": "Myoclonic", "subject_name": "Pediatrics", "topic_name": "Central Nervous system", "id": "37abb69d-6306-484d-b7a2-a22c830e2f48", "choice_type": "single"} {"question": "Manifestations of vitamin C deficiency are", "exp": "In Vitamin C deficiency, bones are tender and the infant is reluctant to move his limbs, which are kept in a frog-like position. This may be mistaken for paralysis. Page 127, Ghai essential pediatrics; 6th edition", "cop": 1, "opa": "Pseudoparalysis", "opb": "Sabre tibia", "opc": "Epistaxis", "opd": "Craniotabes", "subject_name": "Pediatrics", "topic_name": "Nutrition", "id": "0162fdf3-b7a4-4a23-87a6-121a84c48fd5", "choice_type": "single"} {"question": "A premature infant born at 34 weeks presented with acute onset lethargy and cessation of feeding. The imaging modality of choice to screen for intraventricular haemorrhage is", "exp": "Intraventricular hemorrhage (IVH) is a major complication of prematurity. IVH typically initiates in the germinal matrix, which is a richly vascularized collection of neuronal-glial precursor cells in the developing brain. The etiology of IVH is multifactorial and is primarily attributed to the intrinsic fragility of the germinal matrix vasculature and the disturbance in the cerebral blood flow. While this review broadly describes the mechanism underlying IVH, the main focus is on the recent development in molecular mechanisms that elucidates the fragility of the germinal matrix vasculature. The microvasculature of the germinal matrix is frail because of an abundance of angiogenic blood vessels that exhibit paucity of pericytes, immaturity of basal lamina and deficiency of glial fibrillary acidic protein (GFAP) in the ensheathing astrocytes-endfeet. High VEGF and angiopoietin-2 levels activate a rapid angiogenesis in the germinal matrix. The elevation of these growth factors may be ascribed to a relative hypoxia of the germinal matrix perhaps resulting from high metabolic activity and oxygen consumption of the neural-progenitor cells. Hence, the rapid stabilization of the angiogenic vessels and the restoration of normal cerebral blood flow on the first day of life are potential strategies to prevent IVH in premature infants. Reference: GHAI Essential pediatrics, 8th edition", "cop": 2, "opa": "NCCT", "opb": "Transcranial USG", "opc": "MRI", "opd": "Plain radiograph", "subject_name": "Pediatrics", "topic_name": "All India exam", "id": "47465d28-195f-4050-a6d5-a9f24bc73000", "choice_type": "single"} {"question": "Male pseudohermaphroditism is seen in", "exp": "5 alpha reductase conve testosterone to dihydrotestosterone,which is required for external genital devolopment in male.deficiency of it leads to development of mullerian duct into fallopian tube,Uterus and upper third of vagina. Reference : Ghai essential of pediatrics, eighth edition, p.no:538", "cop": 1, "opa": "5- a redutase deficiency", "opb": "21 hydroxylase deficiency", "opc": "17 hydroxylase deficiency", "opd": "Gonadal dysgenesis", "subject_name": "Pediatrics", "topic_name": "Endocrinology", "id": "0a8c2b79-6524-44b4-928a-978fa18b5fbe", "choice_type": "single"} {"question": "Most common soft tissue sarcoma in children is", "exp": "Rhabdomyosarcoma is the Most common Pediatric soft tissue sarcoma.", "cop": 3, "opa": "Liposarcoma", "opb": "Myosarcoma", "opc": "Rhabdomyosarcoma", "opd": "Sacro - occygeal teratoma", "subject_name": "Pediatrics", "topic_name": null, "id": "1b627fb1-54f4-46c9-93b5-0d08d9ca15c3", "choice_type": "single"} {"question": "Maple syrup urine is due to defective decarboxylation of", "exp": null, "cop": 1, "opa": "Branched chain amino acids", "opb": "Sulfur containing amino acids", "opc": "Hydroxyl amino acids", "opd": "Unbranched chain amino acids", "subject_name": "Pediatrics", "topic_name": null, "id": "f7d79536-a071-4731-bb6c-78dfbba59db2", "choice_type": "single"} {"question": "A 10 months old baby boy presented with inability to feed since 1 day. He had a prior history of cough, fever and hurried breathing since 3 days. On examination the child had a Respiratory rate of 56/min. The line of management is", "exp": "Inability to feed is one of the danger signs of pneumonia.\nHence the child can be categorised as Severe pneumonia and needs hospital admission and treatment with injectable antibiotics.", "cop": 3, "opa": "Home management with Paracetamol", "opb": "Home management with Amoxycillin", "opc": "Hospital management with antibiotics", "opd": "Reassurance", "subject_name": "Pediatrics", "topic_name": null, "id": "2e6fd178-f93e-4dba-a2cb-c382e2d7f01f", "choice_type": "single"} {"question": "A 2 year old child presents with scattered lesions in the skull. Biopsy revealed Langerhans giant cells. The most commonly associated is marker with this candition will be", "exp": "In Langerhans cell histiocytosis,the hallmark feature is the presence of Birbeck granules on electron microscopy and positivity for S-100 protein and CD 1a. Reference:Essential pediatrics-Ghai,8th edition,page no:620.", "cop": 1, "opa": "CD 1a", "opb": "CD 57", "opc": "CD 3", "opd": "CD 68", "subject_name": "Pediatrics", "topic_name": "Childhood tumors", "id": "63ba2249-5630-48af-bd50-cb30fcccc7e1", "choice_type": "single"} {"question": "Bart hemoglobin consist of", "exp": "Bart hemoglobin is tetramer of four γ chains.", "cop": 1, "opa": "Four γ chains", "opb": "Four β chains", "opc": "Two γ chains + Two β chains", "opd": "Two α chains + Two γ chains", "subject_name": "Pediatrics", "topic_name": null, "id": "cf22cc8e-b242-4e07-87d2-f0a21139394d", "choice_type": "single"} {"question": "The most characteristic radiographic sign in a child with leukemia is", "exp": "The most characteristic radiological signs in a child with leukemia are metaphyseal lucent band,reduced bone density,lytic bone lesions,metaphyseal coical bone erosions,collapsed veebrae,widening of sutures,and periosteal reactions. Reference: GHAI Essential pediatrics, 8th edition", "cop": 2, "opa": "Osteoclerosis of the metaphysic", "opb": "Metaphseal translucencieas", "opc": "Perios", "opd": "Osteolytic lesion", "subject_name": "Pediatrics", "topic_name": "Childhood tumors", "id": "34ba634e-b20e-43ff-9d5d-40fa56386fc4", "choice_type": "single"} {"question": "Malformations of the following organ system of the fetus are found to be most commonly associated with Single umbilical aery .", "exp": "Malformation of the genitourinary system is generally associated with the single umbilical aery. Paediatric complications associated with single umbilical aery include: An isolated SUA is associated with renal malformations in 7-9 per cent of infants. Vesicoureteric reflux (VUR) is the most common abnormality occurring in 4 per cent of infants with an isolated SUA. In a meta analysis of 26 studies the incidence of any renal abnormality associated with isolated SUA was 8 per cent, with only 4 per cent requiring treatment of the abnormality. From this meta analysis the investigation of 14 cases of isolated SUA would yield one patient requiring treatment. In another study VUR was also the most common abnormality requiring treatment and three out of five of their patients had a urinary tract infection (UTI) before the age of three months. Reference: GHAI Essential pediatrics, 8th edition", "cop": 3, "opa": "Central nervous system", "opb": "Cardiovascular", "opc": "Genitourinary", "opd": "Skeletal", "subject_name": "Pediatrics", "topic_name": "Urinary tract", "id": "7f17ac2e-67f7-4615-a179-363c44caf307", "choice_type": "single"} {"question": "The gold standard lab test for diagnosis of paroxysmal nocturnal Hemoglobinuria is", "exp": "Flow cytometry\n\nIt is gold standard test for diagnosis of PNW\nIt shows absence of CD55 and CD59 on RBC", "cop": 4, "opa": "Ham test", "opb": "Nestroft test", "opc": "Sucrose lysis test", "opd": "Flow cytometry", "subject_name": "Pediatrics", "topic_name": null, "id": "3a565cfe-b9a9-4799-a0d4-1a8bdb30286c", "choice_type": "single"} {"question": "The normal resting hea rate for a newborn is", "exp": "The normal resting hea rate in the newborn is 110-140bpm but may vary from 90 beats /min in a relaxed state to 180 bpm during activity.", "cop": 3, "opa": "75 -115 bpm", "opb": "85-125 bpm", "opc": "110-150 bpm", "opd": "140-200 bpm", "subject_name": "Pediatrics", "topic_name": "All India exam", "id": "4b4c4d6d-dfd3-4d5e-8d9b-826e60b83106", "choice_type": "single"} {"question": "Early onset sepsis in a neonate occurs within", "exp": "Early-onset sepsis (EOS) (less than 72 hr) infections are caused by organisms prevalent in the maternal genital tract or in the delivery area.Late-onset sepsis (LOS) (72 hr or later) infections are caused by the organisms thriving in the external environment of the home or the hospital. The infection is often transmitted through the hands of the care providers.Ref: Paediatrics; O.P. Ghai; 8th edition; Page no: 163", "cop": 4, "opa": "12 hours", "opb": "24 hours", "opc": "48 hours", "opd": "72 hours", "subject_name": "Pediatrics", "topic_name": "New born infants", "id": "e5be868c-90fe-47d4-a312-7e46e81deb3e", "choice_type": "single"} {"question": "Best indicator of acute malnutrition", "exp": "Ans. is 'a' i.e., Weight for height Severe acute malnutrition is defined by a very low weight for height (below -3z scores of the median WHO growth standards), by visible severe wasting, or by the presence of nutritional oedema.", "cop": 1, "opa": "Weight for height", "opb": "Weight for age", "opc": "Height for age", "opd": "BMI < 2 SD", "subject_name": "Pediatrics", "topic_name": null, "id": "3c6d4127-efca-49ff-86e2-a3f323820e9b", "choice_type": "single"} {"question": "Differential cyanosis occurs in", "exp": "Differential cyanosis is present when the preductal (right upper extremity) oxygen saturation is higher than the postductal(lower extremity) saturation.ETIOLOGY:- PDA with reversal of shunt.- Persistent pulmonary hypeension of the newborn.Reverse differential cyanosis:In this condition only the upper pa of the body is cyanosed and the lower pa is pink.ETIOLOGY:PDA +TGV+severe pulmonary hypeensionPDA+TGV+preductal coarctation of the aoa", "cop": 1, "opa": "PDA", "opb": "TGA", "opc": "Tricuspid stenosis", "opd": "VSD", "subject_name": "Pediatrics", "topic_name": "All India exam", "id": "74a93338-7182-493f-86ad-95c12881cac1", "choice_type": "single"} {"question": "Iron supplementation in a healthy term breastfed baby should be staed at the age of", "exp": ".However, human milk contains littleiron, so infants who are exclusively breastfed are at increased risk ofiron deficiency after 4 months of age. The clinical repo recommends giving breastfed infants 1 mg/kg/day of a liquid iron supplement until solid foods, such as iron-foified cereals, are introduced.", "cop": 4, "opa": "2 weeks", "opb": "4 weeks", "opc": "6 weeks", "opd": "8 weeks", "subject_name": "Pediatrics", "topic_name": "Nutrition", "id": "ccb11749-a559-4e76-8e86-50be75aeb294", "choice_type": "single"} {"question": "Edema in nephrotic syndrome is due to", "exp": "NEPHROTIC SYNDROME:- Characterised by massive proteinuria, hypoalbuminemua and edema, hyperlipidemia is often associated. Some patients show hematuria and hypeension. Heavy proteinuria( more than 1g/metre square/day) is the underlying abnormality leading to hypoalbuminemia ( serum albumin below 2.5g/dl). The resultant fall in plasma oncotic pressure leads to interstitial edema and hypovolemia. This stimulates the renin angiotensin aldosterone axis and ADH secretion that enhances sodium and water retention. Hypoalbuminemia also induces hepatic synthesis of beta lipoproteins resulting in hypercholesterolemia. Main cause of nephrotic syndrome in children is associated with minimal change disease. Other causes include amyloidosis, vasculitis, SLE, postinfectious glomerulonephritis, and hepatitis B nephropathy. Steroid sensitive nephrotic syndrome:- MCNS accounts for 80% cases of nephrotic syndrome. Electron microscopy shows non specific obliteration of epithelial foot processes. Immunofluorescence studies shows deposits of occassional mesangial IgM. Lab findings- urine examination shows heavy proteinuria. Hyaline and granular casts are present. Serum albumin is low. Hypercholesterolemia impa a milky appearance to plasma. Blood urea and creatinine values within normal range. Blood levels if IgG low and IgM elevated. Low serum calcium level. Steroid resistant nephrotic syndrome:- Homozygous or complete heterozygous mutations in genes encoding podocyte proteins, including podocin(NPHS2), nephrin(NPHS1) and wilms tumor(WT1) genes. Reference: GHAI essential Paediatrics", "cop": 3, "opa": "Sodium & water retention", "opb": "Increased venous pressure", "opc": "Hypoalbuminemia", "opd": "Hyperlipidemia", "subject_name": "Pediatrics", "topic_name": "Urinary tract", "id": "6e00ef20-15f3-4de7-9573-7a61de2cbc65", "choice_type": "single"} {"question": "ACTH secretion is highest during", "exp": "High levels of ACTH are detected by the adrenal gland receptors which stimulate the secretion of coisol, causing blood levels of coisol to rise. As the coisol levels rise, they sta to slow down the release of coicotrophin-releasing hormone from the hypothalamus and ACTH from the pituitary gland. Ref : Nelson", "cop": 3, "opa": "Noon", "opb": "Evening", "opc": "Morning", "opd": "Night", "subject_name": "Pediatrics", "topic_name": "Growth and development", "id": "0dffe44f-2574-4138-87f6-b6d902e69964", "choice_type": "single"} {"question": "The best indicator of the nutritional status of a child is", "exp": "The best indicator of the nutritional status of a child is the rate of increase in height & weight. Rest of them are age independent indices.", "cop": 2, "opa": "Mid arm circumference", "opb": "Rate of increase in height and weight", "opc": "Head circumference", "opd": "Chest circumference", "subject_name": "Pediatrics", "topic_name": null, "id": "75c73381-eecf-447b-a014-41dd8ee9936b", "choice_type": "single"} {"question": "Amount of ORS given in 5kg child with diarrhoea with some dehydration", "exp": "For some dehydration, amount of ORS given is 75ml/kg over 4 hours, hence 75×5 = 375 ml over 4 hours.", "cop": 3, "opa": "300ml over 2 hours", "opb": "350ml over 1 hour", "opc": "375ml over 4 hours", "opd": "400ml over 6 hours", "subject_name": "Pediatrics", "topic_name": null, "id": "aae5e837-b93d-4915-b536-aee969831fb0", "choice_type": "single"} {"question": "In newborn, cold stress implies", "exp": "Classification of Neonatal hypothermia\nCold stress 36 - 36.4o C\nModerate hypothermia 32 - 35.9o C\nSevere hypothermia < 32o C", "cop": 2, "opa": "35 - 35.4 oC", "opb": "36 - 36.4 oC", "opc": "34 - 34.6 oC", "opd": "37 - 37.6 oC", "subject_name": "Pediatrics", "topic_name": null, "id": "36e9db6d-2a5b-4219-805e-9fff69fdd234", "choice_type": "single"} {"question": "Chemotherapy agents used for Retinoblastoma includes", "exp": "Vincristine, Etoposide, Carboplatin are used to treat Retinoblastoma.", "cop": 1, "opa": "Vincristine, Etoposide, Carboplatin", "opb": "Vincristine, Doxorubicin, Carboplatin", "opc": "Doxorubicin, Etoposide, Carboplatin", "opd": "Actinomycin, Doxorubicin, Carboplatin", "subject_name": "Pediatrics", "topic_name": null, "id": "82d724fb-22f8-4fd2-b01b-57d683f0f411", "choice_type": "single"} {"question": "A child admitted to emergency with urinary incontinence, miosis, sweating, salivation.... poisoning likely by", "exp": "(A) Carbamates > Diagnosis of organic insecticide (Carbamate) poisoning was made on the basis of definite clinical picture such as bradycardia, rhinorrhea, vomiting, diarrhoea, urinary incontinence, miosis, sweating, muscle fasciculations, altered neurological status and a typical smell of insecticide.", "cop": 1, "opa": "Carbamates", "opb": "Arsenic", "opc": "Lead", "opd": "Opioid", "subject_name": "Pediatrics", "topic_name": "Miscellaneous", "id": "72d44358-6822-4641-b026-1f96c2387d76", "choice_type": "single"} {"question": "Febrile convulsions", "exp": "(B) Show an increase in familial incidence # Characteristics of simple benign febrile convulsions:> Fits occurs within 24 hour of the onset of fever, last less than 10 minutes, usually single per febrile episode.> Convulsions are generalized, a small proportion (4-18%) of case may show focal convulsions.> There is no postictal neurological deficit and electroencephalogram a few days after the seizure is normal.> There may be a family history of febrile convulsions in the sibling.> Febrile convulsions are unusual before the age of six months and after 5 years.> Vast majority of patients do not require treatment for either their acute presentation with a seizure or for recurrences.>Best way to manage is to control the temperature with acetaminophen (Paracetamol) or by sponging. When anticonvulsant therapy is judged by a doctor to be indicated, anticonvulsants can be prescribed.> Sodium valproate or clonazepam are active against febrile seizures, with sodium valproate showing superiority over clonazepam.", "cop": 2, "opa": "Are best treated with Sodium Valproate", "opb": "Show an increase in familial incidence", "opc": "May associates with low fever", "opd": "Occur in 2-5% of children", "subject_name": "Pediatrics", "topic_name": "Miscellaneous", "id": "47b6eb7c-1469-4a43-a7fc-9d18f04e23fe", "choice_type": "single"} {"question": "The Sodium content of ReSoMal (rehydration solution for malnourished children) is", "exp": "The sodium content in ResOMal is 45 mmol/l. Therapeutic action - Oral rehydration salts with high potassium and low sodium contents Indications - Prevention and treatment of dehydration, in patients suffering from complicated acute malnutrition only Forms and strengths - Sachet containing 84 g of powder, to be diluted in 2 litres of clean, boiled and cooled water. Dosage and duration - Prevention of dehydration Child under 2 years: 50 to 100 ml after each loose stool as long as diarrhoea persists Child over 2 years: 100 to 200 ml after each loose stool as long as diarrhoea persists Adult: 200 to 400 ml after each loose stool as long as diarrhoea persists - Treatment of dehydration Child and adult: 5 ml/kg every 30 minutes over the first 2 hours, then 5 to 10 ml/kg/hour for the next 4 to 10 hours, until dehydration is corrected. Contra-indications, adverse effects, precautions - Do not administer to patients with cholera or uncomplicated acute malnutrition: use standard ORS instead. - May cause: hea failure when administered too rapidly. During treatment, closely monitor the rate of administration in order to avoid overhydration. Increase in respiratory and pulse rates and appearance or increase of oedema are signs of over rapid rehydration. In this event, stop ReSoMal for one hour then reassess the patient's condition. Reference: GHAI Essential pediatrics, 8th edition", "cop": 3, "opa": "90 mmol/L", "opb": "60 mmol/L", "opc": "45 mmol/L", "opd": "30 mmol/L", "subject_name": "Pediatrics", "topic_name": "Fluid and electrolytes", "id": "70862d9a-1e2e-4534-ab3a-3ad5d36842cf", "choice_type": "single"} {"question": "Amount of ORS to be given in the first 4 hours to a child with some dehydration is", "exp": "Treatment plan B: in case of some dehydration 75 ml/kg of ORS in the first 4 hours.use child&;s age when you donot know the weight. The signs of dehydration disappears in 4 hours.after this maintainance fluid therapy can be done. Ref : ESSENTIAL PEDIATRICS,O.P.GHAI,PG NO:265,7 th edition", "cop": 2, "opa": "50 ml/kg body wt.", "opb": "75 ml/kg body wt.", "opc": "100 ml/kg body wt.", "opd": "200 ml/kg body wt.", "subject_name": "Pediatrics", "topic_name": "Gastrointestinal tract", "id": "e6377bbb-bf34-46c0-a1a5-1355c467782b", "choice_type": "single"} {"question": "After premature delivery, mother&;s milk is low in", "exp": "Preterm milk contains more energy, protein, fat, sodium, zinc, anti-infective factors, and macrophages and has a lower content of lactose, calcium, and phosphorus. Ref: Page 98; Ghai essential pediatrics; 6th edition", "cop": 1, "opa": "Lactose", "opb": "Fat", "opc": "Protein", "opd": "Sodium", "subject_name": "Pediatrics", "topic_name": "Nutrition", "id": "0d2d7202-502d-4ad0-a16f-cd92d6181830", "choice_type": "single"} {"question": "Congenital hea lesion (s) in rubella are", "exp": "Cardiac abnormalities occur in half of the children infected during the 1st 8 wk of gestation. Patent ductus aeriosus is the most frequently repoed cardiac defect followed by lesions of the pulmonary aeries and valvular disease. Reference: Nelson textbook of pediatrics 19th edition Page 1077", "cop": 1, "opa": "VSD", "opb": "ASD", "opc": "Aoic stenosis", "opd": "Coarctation of aoa", "subject_name": "Pediatrics", "topic_name": "Infectious disease", "id": "7cf4a876-5e1f-435e-87d3-9f249ffd7649", "choice_type": "single"} {"question": "Primary cause of hyaline membrane disease is", "exp": "Ans. d (Surfactant deficiency). (Ref. Nelson, Textbook of Paediatrics, 18th/731)HYALINE MEMBRANE DISEASE# Common in premature, IDM and infants delivered by caesarean section.# Clinically manifests shortly after birth (dyspnoea and cyanotic attacks).# X-ray chest shows \"ground glass\" mottling.# As HMD progresses, influx of plasma renders the lungs more radio-opaque, reticulogranular shadowing becomes more confluent & ground glass haziness is seen with loss of clarity of diaphragm & heart.# Mortality 33%# Hyaline membrane is a homogenous eosinophilic membrane plastering the alveolar ducts and terminal bronchiole in HMD.# Rx .-Timely steroids, exogenous surfactant therapy and oxygen (not 100 %) are useful in treatment.", "cop": 4, "opa": "Prematurity", "opb": "Oxygen toxicity", "opc": "Alveolar inflammation", "opd": "Surfactant deficiency", "subject_name": "Pediatrics", "topic_name": "New Born Infants", "id": "2b8a3961-bacc-4d6d-bd77-14258285bb24", "choice_type": "single"} {"question": "A 2 year old male baby presents with cough, high grade fever, fast breathing since 7 days. On examination respiratory rate is 50/min and visible chest indrawing present. Next line of management is", "exp": "Diagnosis can be made as Pneumonia. Pneumonia without any danger sign can be treated at home with Oral antibiotic. In case of presence of any danger signs, the patient has to admitted to the hospital or to be referred to higher center.", "cop": 2, "opa": "Home management with Paracetamol", "opb": "Home management with Amoxycillin", "opc": "Hospital management with antibiotics", "opd": "Reassurance", "subject_name": "Pediatrics", "topic_name": null, "id": "a3236da0-5e1a-4d97-a56c-9fa348da18aa", "choice_type": "single"} {"question": "Second most common neoplasm seen in children is", "exp": "Also called low-grade gliomas, these are the most common brain tumors in children. Medulloblastomas are the most common type of childhood brain cancer. Most medulloblastomas occur before age 10. Ependymomas are a type of childhood brain tumor that can be benign (noncancerous) or malignant (cancerous). The most common cancers of children are: Leukemia. Brain and spinal cord tumors. Neuroblastoma. Wilms tumor. Lymphoma (including both Hodgkin and non-Hodgkin) Rhabdomyosarcoma. Retinoblastoma. Bone cancer (including osteosarcoma and Ewing sarcoma) Reference: GHAI Essential pediatrics, 8th edition", "cop": 2, "opa": "Lyphoma", "opb": "Brain tumor", "opc": "Wilm's tumor", "opd": "Neuroblastoma", "subject_name": "Pediatrics", "topic_name": "Central Nervous system", "id": "f670b943-4493-4004-88d5-52a74e841c2c", "choice_type": "single"} {"question": "For the prevention of parent to child transmission of HIV, the NACO's recommendation is to give", "exp": "Single dose nevirapine,given once to the mother in labor and once to the newborn infant during 48-72 hrs of life reduces perinatal transmission by 50%. A sho term regime consisting of 300 mg BD zidovudine from 36 weeks of pregnancy and 300 mg every 3 hourly during delivery also offers a 50% reduction in transmission. Reference: Ghai essential pediatrics 9 th edition Page 230", "cop": 3, "opa": "Nevirapine 200 mg in active labour to mother", "opb": "Nevirapine 200 mg, four hours after rupture to membranes, to mother", "opc": "Nevirapine 200 mg in active about to mother and syrup nevirapine 2 mg/kg body weight to newborn with 72 hours of delivery", "opd": "Syrup nevirapine 2 mg/kg body weight to newborn within 72 hours of bih", "subject_name": "Pediatrics", "topic_name": "Infectious disease", "id": "e57a25e8-5412-467f-b4fa-aa1f634cce1f", "choice_type": "single"} {"question": "Sweaty feet odor is seen in", "exp": "Sweaty feat odor - Isovaleric AcidemiaSmoky sweat - MSUDMousy or Musty - PhenylketonuriaBoiled cabbage - TyrosinemiaIn alkaptonuria - urine becomes darkish brown when exposed to air while purplish brown in porphyria(Refer: Nelson's Textbook of Pediatrics, SAE, 1st edition, pg no. 651)", "cop": 3, "opa": "Maple syrup urine disease", "opb": "Gauchers disease", "opc": "Isovaleric acidemia", "opd": "Phenylketonuria", "subject_name": "Pediatrics", "topic_name": "All India exam", "id": "a73b56db-a712-42b9-a8ef-037ef08423e1", "choice_type": "single"} {"question": "Bronchitis in children is caused by", "exp": "Breathing tubes (bronchi) in the lungs. In children, the most common cause of acutebronchitis is a virus. A cough, fever, runny nose, and body aches are common symptoms. Reference: GHAI Essential pediatrics, 8th edition", "cop": 1, "opa": "H. influenzae", "opb": "R.S.V", "opc": "Myocoplasma", "opd": "E B V", "subject_name": "Pediatrics", "topic_name": "Respiratory system", "id": "2291f182-0579-4686-94a5-66a5d71c081f", "choice_type": "single"} {"question": "Commonest feature of hypothyroidism in children is", "exp": "Ans. is 'c' i.e., Cold extremities o Amongst the given options cold extremities is the best answer. o Prolongation of physiological jaundice is the earliest sign.", "cop": 3, "opa": "Cataract", "opb": "Recurrent seizures", "opc": "Cold extremities", "opd": "Laryngospasms", "subject_name": "Pediatrics", "topic_name": null, "id": "d17a2408-d240-411b-9193-c7ef57a14cd8", "choice_type": "single"} {"question": "Breast milk at room temperature is stored for", "exp": "Expressed Breast milk (EBM) can be stored at room temperature for 6-8 hrs. Ref: Page 155, Ghai essential pediatrics; 8th edition", "cop": 1, "opa": "4 hrs", "opb": "8 hrs", "opc": "12 hrs", "opd": "24 hrs", "subject_name": "Pediatrics", "topic_name": "Nutrition", "id": "5812b6e3-30a5-4bc4-8f9f-81e88c7f3ef8", "choice_type": "single"} {"question": "Breast feeding is contraindicated in", "exp": "Contraindications to breastfeeding are those conditions that could compromise the health of the infant if breast milk from their mother is consumed. One example of this is galactosemia. This is. If the mother has HIVor antiviral therapy, untreated active tuberculosis, Human T-lymphotropic virus 1or II, uses illicit drugs, or mothers undergoing chemotherapy or radiation treatment. Breastfeeding contraindication are situations where the mother has conditions such as an addiction or disease that would make it harmful to the baby, should the baby be breastfed. Breast milk contains many nutrients that formulas in store shelves do not have which makes breast feeding a healthier and ideal way to feed an infant. Reference: GHAI Essential pediatrics, 8th edition", "cop": 3, "opa": "Mother in failure", "opb": "Mother with Hep B", "opc": "Mother with TB", "opd": "Mother with Leukemia", "subject_name": "Pediatrics", "topic_name": "New born infants", "id": "f6f17cd9-699a-44bd-97ef-ae71ecc773be", "choice_type": "single"} {"question": "Most common cause of Seizures during Neonatal Period includes", "exp": "(A) Hypoxic Ischemic Encephalopathy # HYPOXIC ISCHEMIC ENCEPHALOPATHY (HIE) It is a type of brain damage that occurs when an infant's brain doesn't receive enough oxygen and blood. It is a dangerous condition that requires immediate medical intervention. HIE affects 20 out of every 1000 full term births; the incidence rate in premature babies is 60% of all live births.# Causes of Neonatal Seizures:> Seizures resulting from hypoxic-ischemic encephalopathy Seen in both term and premature infants. Present within the first 72 hours of life. Seizures may include subtle, clonic, or generalized seizures.> Intracranial hemorrhage: Occurs more frequently in premature than in term infants. Distinguishing infants with pure hypoxic-ischemic encephalopathy from those with intracranial hemorrhage often is difficult.> Subarachnoid hemorrhage: More common in term infants. Type of hemorrhage occurs frequently and is not clinically significant. Typically, infants with subarachnoid hemorrhage appear remarkably well.> Germinal matrix-intraventricular hemorrhage: Is seen more frequently in premature than in term infants, particularly in infants born prior to 34 weeks' gestation. Subtle seizures are seen frequently with this type of hemorrhage.> Subdural hemorrhage: Is seen in association with cerebral contusion. It is more common in term infants.> Metabolic disturbances: Include Hypoglycemia, Hypocalcemia, & Hypomagnesemia. Less frequent metabolic disorders, such as inborn errors of metabolism, are seen more commonly in infants who are older than 72 hours. Typically, they may be seen after the infant starts feeding.> Intracranial infections (which should be ruled out vigorously) that are important causes of neonatal seizures include Meningitis, Encephalitis (including herpes encephalitis), Toxoplasmosis, and Cytomegalovirus (CMV) infections. Most common bacterial pathogens include Escherichia coli and Streptococcus pneumoniae.> Most cerebral malformations present with seizures at a later age, major malformation syndromes are important to consider.> Lissencephaly, pachygyria, polymicrogyria, and linear sebaceous nevus syndrome can present with seizures in the neonatal period.", "cop": 1, "opa": "Hypoxic Ischemic Encephalopathy", "opb": "Brain haemorrhage", "opc": "Tetanus", "opd": "Nutrient deficiency", "subject_name": "Pediatrics", "topic_name": "Miscellaneous", "id": "9f873efc-cdd7-4fb6-9e3d-3c15ddb30e4d", "choice_type": "single"} {"question": "Handedness develops by age of", "exp": "Handedness is usually established by the 3rd year. Frustration may result from attempts to change children's hand preference, Variations in fine motor development reflect both individual proclivities and different oppounities for learning. Children who are seldom allowed to use crayons, for example, develop a mature pencil grasp later.(Refer: Nelson's Textbook of Pediatrics, SAE, 1st edition, pg no. 77)", "cop": 2, "opa": "2 years", "opb": "3 years", "opc": "4 years", "opd": "5 years", "subject_name": "Pediatrics", "topic_name": "All India exam", "id": "71c81302-ca5e-4abe-9dea-29d7c3da41ce", "choice_type": "single"} {"question": "Mean head circumference at birth is", "exp": "Head circumference:-\n\n Measured with a nonstretchable tape passing through the maximum point of occipital protuberance posteriorly and at a point just above the glabella anteriorly.\n\nHead circumference must be compared with that of mother to know any familial variations.\nIf the head circumference is more than expected, always measure the both parents' head circumference, for benign familial megalencephaly is common.\nAt birth , it is around 34 cm.HC increases 2cm/ month for the first 3 months, 1cm/month for next three months and 0.5 cm per month for next 6 months.", "cop": 4, "opa": "48-51 cm", "opb": "38-40 cm", "opc": "42-45 cm", "opd": "33-35 cm", "subject_name": "Pediatrics", "topic_name": null, "id": "cf780ade-30c3-4c43-b56f-0608bdf588f1", "choice_type": "single"} {"question": "Subluxated lens, thin elongated, hyperextensible extremities and aortic aneurysm are the characteristic clinical features of", "exp": "(A) Marfan's syndrome # MARFAN'S SYNDROME is a disorder of connective tissue (elastin). The gene called fibrillin is on chromosome 15. Inheritance is autosomal ominant. Sporadic cases may occur. The patients are tall and slender. Extremities especially the fingers and toes are long and thin. The muscles are markedly hypotonic and joints are hyperextendible. At times the disorder may be associated with arthrogryposis (contraction of joints in flexion due to fibrous ankylosis). There may be subluxation of the lens and other ocular abnormalities e.g., cataract, coloboma, squint, nystagmus and megalocornea. Deformities of the chest and spine may be present. Other associated anomalies include aortic cystic medionecrosis leading to aortic dilatation and other valvular deformities. Intelligence is normal. It should be distinguished from cases of homocystinuria, which also may present with ectopia lentis and similar bone changes, but has associated mental retardation. The basic manifestations are joint hypermobility, skin hyperextensibility, dystrophic scarring of the skin and easy bruising, and connective tissue fragility. Wound healing is delayed and there are freely movable subcutaneous nodules. Molecular genetics has helped to characterize 11 different types of Ehlers-Danlos syndrome. Homocystinuria is a relatively common metabolic error with autosomal recessive inheritance. In the most common situation, Type 1, cystathionine is not synthesized from homocysteine and serine because the enzyme cystathionine synthetase in the liver is deficient. Although the precursor, homocysteine accumulates in the tissues, it is rapidly oxidized to homocystine. The usual age of presentation is 3-4 years. The patients develop subluxation of the lens, recurrent thromboembolic episodes and have marfanoid features. Klinefelter syndrome is characterized by tall thin stature, small testes and small phallus, with the chromosomal karyotype of 47 XXY. Behavioral and psychiatric disorders and mental retardation are common.", "cop": 1, "opa": "Marfan's syndrome", "opb": "Ehler Danlos syndrome", "opc": "Homocystienuria", "opd": "Klienfelter's syndrome.", "subject_name": "Pediatrics", "topic_name": "Miscellaneous", "id": "d0d12b8b-bae6-4394-99ac-35ca1a6082a6", "choice_type": "single"} {"question": "Penile growth occurs during SMR stage", "exp": "In males, the first visible sign of pubey and the hallmark of SMR2 is the testicular enlargement. Penile growth occurs during stage 3.Reference: Nelson Textbook of Paediatrics; 20th edition", "cop": 3, "opa": "1", "opb": "2", "opc": "3", "opd": "4", "subject_name": "Pediatrics", "topic_name": "Adolescence", "id": "6be7b0a9-a170-49d8-8cf4-2bd76e05c9c9", "choice_type": "single"} {"question": "The most common site of brain metastasis is", "exp": "MC site of brain metastases (85%) CEREBRUM2nd MC site of brain metastases- CEREBELLUM LUNG, BREAST, MELANOMA (SKIN CANCER), COLON AND KIDNEY cancers commonly spread to the brain.(Refer: Nelson's Textbook of Pediatrics, SAE, 1st edition, pg no. 2460)", "cop": 1, "opa": "Cerebrum", "opb": "Cerebellum", "opc": "Medulla oblongata", "opd": "Pons", "subject_name": "Pediatrics", "topic_name": "All India exam", "id": "c6417cf2-291a-4eb1-ae5a-daf3b7c7d7c6", "choice_type": "single"} {"question": "Major criteria for jones classification is", "exp": "i.e. (Chorea): (380 - Ghai 7th)JONES CRITERIA FOR RHEUMATIC FEVERMajor Criteria Minor Criteria* Carditis* Arthritis* Subcutaneous nodule* Chorea* Erythema marginatumA. Clinicali. Feverii. Arthralgiaiii. Previous rheumatic fever rheumatic heart diseaseB. Laboratoryi. Acute phase reactants leukocytosis, elevated ESR and CRPii. Prolonged PR interval on ECGEssential criteriaEvidence for recent streptococcal infection as evidenced bya. Increased antistreptolycin 'O' titerb. Positive throat culturec. Recent scarlet feverTwo major qr one major and two minor criteria are required in the presence of essential criteria to diagnose acute rheumatic feverAssessment for Presence of Heart DiseaseNadas' CriteriaMajor Minor* Systolic murmur* Systolic murmur less than grade III* Grade III or more* Abnormal second sound* Diastolic murmur* Abnormal ECG* Cyanosis* Abnormal X-ray* CHF* Abnormal blood pressurePresence of one major or two minor criteria indicate heart disease (399- Ghai 7th)", "cop": 3, "opa": "Fever", "opb": "Arthralgia", "opc": "Chorea", "opd": "TPR Interval", "subject_name": "Pediatrics", "topic_name": "C.V.S.", "id": "493d426b-ae0c-45e0-a116-fb9bba4e325a", "choice_type": "single"} {"question": "Common presentations of juvenile Hypothyroidism", "exp": "Most common presentation of juvenile hypothyroidism is growth retardation. others include delayed skeletal maturation,delayed dental development,delayed pubey, myopathy and pseudohyperophy,enlarged sella,pseudomotor cerebri. Usually due to hormonal synthesis defect such as TPO or to c -erb A mutation PE : goiter, delayed maturation, testicular enlargement/precocious menarche NOT usually MR- recovery is general rule with thyroxine Reference: OP Ghai ,essential paediatrics ,8 th edition.page no 517", "cop": 1, "opa": "Growth retardation", "opb": "Mental retardation within 2 years", "opc": "Delayed pubey", "opd": "Umbilical Hernia", "subject_name": "Pediatrics", "topic_name": "Endocrinology", "id": "1012a8f1-bb16-4953-bf52-77612fc24af0", "choice_type": "single"} {"question": "An infant should be exclusively breastfed till", "exp": "An infant should be exclusively breastfed till six months of age Cardinal principles of breastfeeding practice Initiation of breastfeeding within an hour after bih Exclusive breastfeeding up to 6 months of age Continuation of breastfeeding along with complementary feeding for up to 2 yr of age Reference: Paediatrics; O.P Ghai; 8th edition; Page no: 90", "cop": 2, "opa": "4 months", "opb": "6 months", "opc": "12 months", "opd": "24 months", "subject_name": "Pediatrics", "topic_name": "Nutrition", "id": "69014cd2-d43f-4d21-a856-8894a265aec1", "choice_type": "single"} {"question": "In a 3 year old most common cause of Hepatitis B is", "exp": "*HBV is transmitted through contact with blood and body fluids of an infected person . *In highly endemic areas , HBV infections occur mainly due to perinatal transmission resulting from carrier mothers to their infants during first 5 years of life. * It may also occur through horizontal transmission from casual contact . * HBV is not transmitted by breastfeeding Reference : OP Ghai pgno 217 9th edition.", "cop": 3, "opa": "Pin prick", "opb": "Saliva exchange", "opc": "Perinatal", "opd": "Blood transfusion", "subject_name": "Pediatrics", "topic_name": "Infectious disease", "id": "f24f5a9f-0033-47a0-9ce7-fbaeb3d85ef0", "choice_type": "single"} {"question": "Most common cause of Nephrotic Syndrome in Children", "exp": "NEPHROTIC SYNDROME:- Characterised by massive proteinuria, hypoalbuminemua and edema, hyperlipidemia is often associated. Some patients show hematuria and hypeension. Heavy proteinuria( more than 1g/metre square/day) is the underlying abnormality leading to hypoalbuminemia ( serum albumin below 2.5g/dl). The resultant fall in plasma oncotic pressure leads to interstitial edema and hypovolemia. This stimulates the renin angiotensin aldosterone axis and ADH secretion that enhances sodium and water retention. Hypoalbuminemia also induces hepatic synthesis of beta lipoproteins resulting in hypercholesterolemia. Main cause of nephrotic syndrome in children is associated with minimal change disease. Other causes include amyloidosis, vasculitis, SLE, postinfectious glomerulonephritis, and hepatitis B nephropathy. Steroid sensitive nephrotic syndrome:- MCNS accounts for 80% cases of nephrotic syndrome. Electron microscopy shows non specific obliteration of epithelial foot processes. Immunofluorescence studies shows deposits of occassional mesangial IgM. Lab findings- urine examination shows heavy proteinuria. Hyaline and granular casts are present. Serum albumin is low. Hypercholesterolemia impa a milky appearance to plasma. Blood urea and creatinine values within normal range. Blood levels if IgG low and IgM elevated. Low serum calcium level. Steroid resistant nephrotic stndrome:- Homozygous or complete heterozygous mutations in genes encoding podocyte proteins, including podocin(NPHS2), nephrin(NPHS1) and wilms tumor(WT1) genes. Reference: GHAI essential Paediatrics", "cop": 1, "opa": "Minimal change disease", "opb": "IgA nephropathy", "opc": "Mesangial GN", "opd": "FSGN", "subject_name": "Pediatrics", "topic_name": "Urinary tract", "id": "235d5da2-efb5-41b6-9093-0f852d1e9cba", "choice_type": "single"} {"question": "In the management of severe acute malnutrition (SAM) child, iron should be started", "exp": "Do not given Iron is Stabilization Phase. If given Iron releases free radical, causing injury to GIT.\nIron should be started in Rehabilitation Phase (after day 7)", "cop": 3, "opa": "Immediately", "opb": "On day 2 start of stabilization phase", "opc": "In Rehabilitation Phase, after fever subside", "opd": "Iron should never be given", "subject_name": "Pediatrics", "topic_name": null, "id": "8c07d78b-fb08-4341-ac6b-6b7fb489cddf", "choice_type": "single"} {"question": "A solid renal mass detected during prenatal ultrasound is most likely a", "exp": "Mesoblastic nephroma is the most common solid renal tumor identified in the neonatal period and the most frequent benign renal tumor in childhood. It represents 3-10% of all pediatric renal tumors. Many cases are diagnosed with prenatal ultrasound and can manifest as polyhydramnios, hydrops, and premature delivery. Most of the patients are diagnosed before 3 mo of age, whereas WT is rarely diagnosed before 6 mo of age. Radical nephrectomy is the treatment of choice and may be sufficient by itself. Local recurrence is uncommon. Reference: Nelson; Neuroblastoma; Page no: 2467", "cop": 4, "opa": "Wilm's tumor", "opb": "Neuroblastoma", "opc": "Clear cell sarcoma", "opd": "Mesoblastic nephroma", "subject_name": "Pediatrics", "topic_name": "Childhood tumors", "id": "ceadecae-8317-4425-a667-403532c99629", "choice_type": "single"} {"question": "In breastfeeding, hindmilk poion is richer in", "exp": "Hind milk is rich in fat and provides more energy and satisfies the hunger of the baby. It comes towards the end of the feed.", "cop": 1, "opa": "Fats", "opb": "Lactose", "opc": "Vitamins", "opd": "Proteins", "subject_name": "Pediatrics", "topic_name": "New born infants", "id": "9b74df3d-e629-4548-abcb-17db0e8c4be9", "choice_type": "single"} {"question": "Drug of choice for pertussis in children is", "exp": null, "cop": 4, "opa": "Penicillin", "opb": "Erythromycin", "opc": "Ampicillin", "opd": "Azithromycin", "subject_name": "Pediatrics", "topic_name": null, "id": "e7be6379-64dd-4e55-beca-9ea41f6c9d43", "choice_type": "single"} {"question": "The most common etiology of short stature is", "exp": "Short Stature\nDefined as height below the third centile or more than 2 standard deviations below the median height for age and gender according to the population standard. Children whose statures more than 3 SD below the population mean for age and gender are more likely to be suffering from pathological short stature as compared to those with stature between -2 and -3SD, who are more likely to be affected by familial or constitutional short stature.\nCAUSES:-\n\nThe most common cause is constitutional.\nFamilial\nPathological causes include:\n\tUndernutrition\n\tChronic systemic illness\n\tCerebral palsy\n\tCongenital heart disease\n\tCystic fibrosis, asthma\n\tMalabsorption\n\tAcquired immunodeficiency syndrome\nEndocrine causes include:\n Psychosocial dwarfism.\nChildren born small for gestational age.\nSkeletal dysplasia; achondroplasia, rickets.\nGenetic syndrome; Turner, Down syndrome.", "cop": 3, "opa": "Thyroxine deficiency", "opb": "Growth hormone deficiency", "opc": "Constitutional growth delay", "opd": "Systemic diseases", "subject_name": "Pediatrics", "topic_name": null, "id": "764d540d-38c4-4682-9e3d-cee67fd26345", "choice_type": "single"} {"question": "Most common urinary tumour in children is", "exp": "(Wilm's tumour) (2143 -Nelson 18th) (591-Ghai 7th)1. Rhabdomyosarcoma is the most common soft tissue sarcoma in children 15 years (MC site head & neck region)2. Wilm s tumor or nephroblastoma is the most common malignant tumour of the kidney (Under 5years of age)3. Neuroblastoma is the most common intra- abdominal solid tumor in childhood (Before 6 years of age)4. Retinoblastoma is the most common primary ocular tumor of childhood (White reflex is MC sign)* Wilms tumour associated with syndrome - WAGR, Denys- Drash, Backwith - Wiedemann syndrome others hemihypertrophy sporadic aniridia, neurofibromatosis (Von Recklinghausen disease) and Von Wille brand diseaseGenitourinary anomalies most commonly associated Wilms tumour are hypoplasia, fusion and ectopia of the kidney, duplications of collecting systems, hypospadias and cryptorchildism", "cop": 3, "opa": "Rhabdomyosarcoma", "opb": "Leomyoma", "opc": "Wilm's tumour", "opd": "Nephrogenic rests", "subject_name": "Pediatrics", "topic_name": "Childhood Tumors", "id": "da31af72-1e60-44eb-b3c3-8b2eaa167415", "choice_type": "single"} {"question": "Ba&;s hydrops fetalis is lethal because", "exp": "Ba hydrops fetalis is a lethal complication of alpha thalassemia trait where oxygen can&;t release from hemoglobin. Hb Bas is formed of 4 gamma chains. The oxygen affinity is so high that it releases very little oxygen into the fetal tissues. Reference: Ghai essential of pediatrics, eighth edition, p.no:342", "cop": 3, "opa": "Hb Ba's cannot bind oxygen", "opb": "The excess a - globin form insoluble precipitates", "opc": "Hb Ba's cannot release oxygen to fetal tissues", "opd": "Microcytic red cells become trapped in the placental", "subject_name": "Pediatrics", "topic_name": "Hematology", "id": "878c57c1-e00e-4fa1-a8ae-dd4db8bfb121", "choice_type": "single"} {"question": "A patient with VSD develops pulmonary hypeension, the characterstic feature will be", "exp": "Pulmonary hypeension in VSD lead to cyanosis. Ref : Ghai essential of pediatrics, eighth edition, p.no: 417", "cop": 2, "opa": "ESM in pulmonary area", "opb": "Cyanosis", "opc": "Inveed 'T' wave in ECG", "opd": "Clubbing", "subject_name": "Pediatrics", "topic_name": "C.V.S", "id": "f1264d97-e7b5-41da-9f5f-e20ef7c9c705", "choice_type": "single"} {"question": "Anterior fontanelle is closed at", "exp": "[Cl (12-18 months) Ref: 525, 1974-Nelson 17,h1. Anterior fontanelle: Formed by joining of four sutures in the midline, Diamond shape, 2 by 2 cm measurements* The floor is formed by membranous and it becomes ossified 18 month after birth. It becomes pathological, if it fails to ossify even after 24 months (84 - Dutta 6th)* The average time of closure is 18 months but the fontanel may normally close as early as 9 months. The fontanele is normally slightly depressed and pulsatile and is best evaluated when an infant is held upright while asleep or feeding (1974-Nelson 17th)Disorders associated with a large anterior fontanel* Achondroplasia* IUGR* Apert syndrome* Kenny syndrome* Athyrotic hypothyroidism* Osteogenesis imperfecta* Cleidocranial dysostosis* Prematurity* Congenital rubella syndrome* Pyknodysostosis* Hallermann-Streiff syndrome* Russell - Silver syndrome* Hydrocephaly* 13 - 18, 21 trisomies* Hypophosphatasia* Vitamin D deficiency ricketsPersistently small fontanels suggest microcephaly, cranio synostosis congenital hyperthyroidism or wormian bones2. Posterior fontanelle: between the intersection of the occipital and parietal bones that may be closed at birth or at the most admit the tip of finger. The posterior fontanele is usually closed and nonpalpable after the first 6- 8 weeks of life.Its persistence suggests underlying hydrocephalus or possibility of congenital hypothyroidism", "cop": 3, "opa": "< 3 months", "opb": "12 months", "opc": "12-18 months", "opd": "> 3 years", "subject_name": "Pediatrics", "topic_name": "Growth, Development, and Behavior", "id": "dcce700e-f96d-4357-a08d-e5d209afdd06", "choice_type": "single"} {"question": "A 1 & 1/2 yr child can do", "exp": "Developmental milestones:- GROSS MOTOR DEVELOPMENT: 2 months: Holds head in plane of rest of the body when held in ventral suspension. In prone position in bed, the chin lifts momentarily. 3 months:lift head above the plane of the body. Head control stas by 3 months and fully developed by 5 months. 4 months:Remain on forearm suppo if put in prone position, lifting the upper pa of the body off the bed. 5 months: Rolls over. 6 months:sit in tripod fashion. 8 months: sits without suppo., crawling 9 months: Takes a few steps with one hand held. Pulls to standing and cruises holding on to furniture by 10 months. 10 months: creeps 12 months:creeps well, walk but falls, stand without suppo. 15 months: walks well, walks backward/ sideways pulling a toy. May crawl upstairs. 18 months: Runs, walks upstair with one hand held. Explores drawers 2 years: walk up and downstairs, jumps. 3 years : rides tricycle, alternate feet going upstairs. 4 years: hops on one foot, alternate feet going downstairs. 5 years:skips FINE MOTOR DEVELOPMENT:- 2 months- eyes follow objects to 180 deg. 3 months-Grasp reflex disappears and hand is open most of the time. 4 months- Bidextrous approach( reaching out for objects with both hands). 6 months- Unidextrous approach( Reach for an object with one hand). 8 months- radial grasp sta to develop. Turns to sound above the level of ear. 9 months- immature pincer grasp, probes with forefinger. 12 months-Unassisted pincer grasp. Releases object on request.Uses objects predominantly for playing, not for mouthing. Holds block on each hand and bang them together. 15 months- imitate scribbling , tower of two blocks 18 months- scribbles, tower of 3 blocks.turn pages of a book, 2-3 at a time. 2 years- tower of 6 blocks, veical and circular stroke. 3 years-Tower of 9 blocks, dressing and undressing with some help, can do buttoning. 4 years- copies cross, bridge with blocks 5 years- copies triangle, gate with blocks. SOCIAL AND ADAPTIVE MILESTONES: 2 months: social smile(smile after being talked to).watches mother when spoken to and may smile. 3 months:Recognizes mother, anticipates feeds. 4 months: Holds rattle when placed in hand and regards it . Laughs aloud. Excited at the sight of food. 6 months:recognizes strangers, stranger anxiety . Enjoy watching own image in mirror, shows displeasure when toy pulled off. 9 months:waves bye bye 12 months:comes when called, plays simple ball game.kisses the parent on request. Makes postural adjustments for dressing. 15 months:jargon, stas imitating mother. 18 months: copies parents in tasking, dry by day, calls mother when he wants potty, points to three pas of body on request. 2 years: ask for food, drink, toilet, pulls people to show toys. 3 years:shares toys, know fullname and gender, dry by night. 4 years:Plays cooperatively in a group, goes to toilet alone, washes face, brushes teeth. Role play . 5 years:helps in household task , dresses and undresses. LANGUAGE MILESTONES: 1 month: Ales to sound. 2 month:respond to sound by stale or quitening to a smooth voice. 3 months: babbles when spoken to. Makes sounds (ahh,coos, ) laughs. 4 months: laughs aloud. 6 months: monosyllables 9 months: understands spoken words, bisyllables. 12 months: 1-2 words with meaning. 15months : vocabulary of 6 words 18 months: vocabulary of 10 words. Can name one pa of body. 2 years: 3 word simple sentences 3 years:asks questions, knows full name and gender. 4 years: says songs or poem, tells story, knows three colours. 5 years: ask meaning of words. Reference: GHAI Essential pediatrics, 8th edition", "cop": 3, "opa": "Build tower of 6 blocks", "opb": "Speak of 10-25 meaningful word", "opc": "Scribble", "opd": "points 3-4 body pas", "subject_name": "Pediatrics", "topic_name": "Growth and development", "id": "f486c6fd-f8f8-4302-947f-7c159d5c6a5b", "choice_type": "single"} {"question": "Profuse watery diarrhea in an immune compromised child is due to", "exp": ".Giardiasis caused by Giardia lamblia is a major cause of diarrhoea in children and travellers Individuals with malnutrition,humoral immunodefeciencies,and cystic fibrosis are paicularly susceptible. Features:acute diarrhoea with sudden onset of explosive,watery foul smelling stools, along with nausea and anorexia,abdominal distension,flatulence,epigastric camps and mild fever. ESSENTIAL PEDIATRICS,O.P.GHAI,PG NO:244,7th edition", "cop": 3, "opa": "Cryptococcus", "opb": "Amoeba", "opc": "Giardia", "opd": "Lactose intolerance", "subject_name": "Pediatrics", "topic_name": "Gastrointestinal tract", "id": "2d93a6c5-aad2-4309-8eae-fa7fa59418ca", "choice_type": "single"} {"question": "A child presents with sho episodes of vacant stare several times a day. The vacant episode begins abruptly and the child remains unresponsive during the episode. There is no associated history of aura or postictal confusion and the child is otherwise normal. The likely diagnosis is", "exp": "Answer is B (Absence seizures): Presence of multiple sho episodes of vacant stare (absence) in an otherwise normal child with no history of aura or postictal confusion suggests the diagnosis of typical absence seizures. Absence seizures versus Dav dreaming Absence seizures can easily be confused with episodes day dreaming however there are ceain clues that can help differentiate these two conditions Day dreaming is usually not associated with automatism while automatism is common in seizures. Day dreaming can often be 'broken' with stimulation while seizures can usually not be 'broken'. Day dreaming usually occurs when child is tired /bored or involved in a monotonous activity but seizures can sta abruptly at any time (for example in the middle of a sentence). Abrupt onset of vacant episodes and the fact that the child remains unresponsive during the episode (cannot be broken) ours a diagnosis of absence seizures in this child. Absence seizures versus complex paial seizures Complex paial seizures are frequently associated with aura and postictal confusion. Absence of these features ours a diagnosis of 'Absence seizures' Absence seizures must primarily be differentiated from complex paial seizures and Day dreaming Clinical data Absence Complex paial Daydreaming Frequency/day Multiple Rarely over 1-2 Mutiple; situation-dependent Duration Frequently < 10 sec (Rarely longer than 30 sec) Average duration ober 1 min, 10 sec Seconds to minutes; rarely more rarely less Aura Never Frequently' No Eye blinking Common Occasionally No Automatism Common Frequently No Postictal impairment None Frequently No Seizures activated by 1-11p er yen ti la tio n Very frequently Occassionallv No Photic Frequently Rarely No EEG ktul Generalized spike and wave Usually unilateral or bilateral temporal frontal discharges Normal Interictal Usually normal Variable; may be spikes or sharp waves in frontal or temporal lobes Normal", "cop": 2, "opa": "Grandmal seizures", "opb": "Absence seizures", "opc": "Complex paial seizures", "opd": "Day dreaming", "subject_name": "Pediatrics", "topic_name": null, "id": "f6dea44b-fd89-4db3-84c2-d2500a2a59b0", "choice_type": "single"} {"question": "Features of cystic fibrosis", "exp": "The sweat chloride is high(>60 mEq/L) Defect in long arm of chromosome 7 Autosomal recessive condition. Ref : Essential pediatrics,O.P.Ghai,7 th edition,pg no:369", "cop": 1, "opa": "Lung normal at bih", "opb": "Low sweat choride tests", "opc": "Autosomal dominant", "opd": "Defect in Chromosome 11", "subject_name": "Pediatrics", "topic_name": "Respiratory system", "id": "e9af0380-c45e-4739-8d71-b106645cac08", "choice_type": "single"} {"question": "In a fetus highest O2 concentration found in", "exp": "Highest o2 concentration umblical vien > ivc > svc> =ra=rv > umblical aery Reference: GHAI Essential pediatrics, 8th edition", "cop": 2, "opa": "svc", "opb": "ivc", "opc": "right ventricle", "opd": "aoa", "subject_name": "Pediatrics", "topic_name": "All India exam", "id": "a77222e6-83a9-4a87-a01e-a99fef985e21", "choice_type": "single"} {"question": "6 months old baby brought to OPD with complaitns of loud, noisy breathing since birth after feeding on supine position. Baby birth history is insignificant. Treatment for the condition is", "exp": "The given case scenario is laryngomalacia. It is the most common anamoly of Larynx. The baby presents with stridor in supine position and relieved on prone position. The condition is self resolving by 1 year.", "cop": 3, "opa": "3rd generation cephalosporins", "opb": "Heimlich's maneuver", "opc": "Reassurance", "opd": "Single dose of Dexamethasome", "subject_name": "Pediatrics", "topic_name": null, "id": "816f52cb-2f9c-4440-82b7-5ed58fa180f4", "choice_type": "single"} {"question": "6 years old drowsy child is brought to emergency with a history of vomitings and loose motion for 3 days. On examination, he had sunken eyes, hypothermia, skin on pinching taken very long time to reve. Diagnosis is", "exp": "A child with severe dehydration will have at least two of the following four signs: sensorium is abnormally sleepy or lethargic, sunken eyes, drinking poorly or not at all, and a very slow skin pinchA child with some signs of dehydration will have two of the following: restlessness or irritability, sunken eyes, drinking eagerly or slow skin pinchA child with either one or none of these signs is classified as having dehydration.(Refer: OP Ghai's Textbook of Pediatrics, 8th edition, pg no. 265)", "cop": 4, "opa": "No dehydration", "opb": "Mild dehydration", "opc": "Some dehydration", "opd": "Severe dehydration", "subject_name": "Pediatrics", "topic_name": "All India exam", "id": "b718234c-249c-4446-96f3-82ca011cc85c", "choice_type": "single"} {"question": "Best method of estimation of amount of proteinuria in a 2 year old child with nephrotic syndrome is", "exp": null, "cop": 4, "opa": "Dipstick test", "opb": "24 hr urine protein", "opc": "Microalbuminuria", "opd": "Spot urine sample for Protein/ Creatinine ratio", "subject_name": "Pediatrics", "topic_name": null, "id": "b8c9d80e-b612-4e50-ab84-c928825336db", "choice_type": "single"} {"question": "10 days old baby develops swelling left in posterior cervical area, bluish in appearance, increase in size on crying and brilliently transilluminant", "exp": "Ans. is 'c' i.e., Cystic hygroma Cystic hygroma Usually present in neonate or early infancy. Filled with lymph Commonly present in posterior cervical area, axilla, mediastinum, parotid, floor of mouth. Soft & paially compressible Size increase during crying, coughing Brilliently transilluminant Sometimes size increase with age or during infection. Treatment is complete excision.", "cop": 3, "opa": "Parotiditis", "opb": "Hemangioma", "opc": "Cystic hygroma", "opd": "Lymph node", "subject_name": "Pediatrics", "topic_name": null, "id": "6b4f0681-8559-4b40-b702-21083db35b52", "choice_type": "single"} {"question": "Chest radiographs of children with foreign body aspiration", "exp": "In foreign body aspiration a radiograph is vital as the object may be radio opaque. often its not radio opaque or is obscured by the cardiac shadow or the inflammatory response. over inflation of affectedlung due to inflammatory response. Ref : BAILEY AND LOVE&;S SHO PRACTICE OF SURGERY,24 th edition ,PG NO:852", "cop": 2, "opa": "Are always abnormal", "opb": "May show over inflation of the involved lung", "opc": "Always show the foreign body", "opd": "Commonly show pneumothorax", "subject_name": "Pediatrics", "topic_name": "Respiratory system", "id": "6912d946-d0b3-4ffd-b3c0-b58ff597adca", "choice_type": "single"} {"question": "A child is having nocturnal asthmatic attacks 2 times in a week and day time attacks 3 times or more. This child is categorized to be having", "exp": ".intermittent :<1 time a week ,asymptomatic,normal PEFR between attack,nocturnal attack less than or equal to 2 per month mild persistant :>1 time a week but less than 1 time a day,noctunal attack>2 /month moderate persistant :daily use beta 2 agonist, attack affect activity,nocturnal attack>1/week severe persistant :continuous, limited physical activity,nocturnal attack frequent Ref : ESSENTIAL PEDIATRICS,O.P.GHAI,PG NO:363, 7th edition", "cop": 4, "opa": "Mild persistent asthma", "opb": "Mild intermittent asthma", "opc": "Severe persistent asthma", "opd": "Moderate persistent asthma", "subject_name": "Pediatrics", "topic_name": "Respiratory system", "id": "60a05266-2365-4174-815b-650b4016c56e", "choice_type": "single"} {"question": "Feature of Down syndrome is", "exp": "Ans) c (Trisomy 21) Ref Nelson 18th ed p 508Down syndrome is the most common chromosome disorder and the single most common genetic cause of moderate mental retardation. The incidence of Down syndrome in live births is approximately 1 in 750. The occurrence of trisomy 21 as well as other autosomal trisomies increases with advanced maternal age (> 35 yr)In approximately 95% of the cases of Down syndrome there are 3 copies of chromosome 21. The parental origin of the supernumerary chromosome 21 is maternal in 97% of the cases. Approximately 1 % of individuals are mosaics with some cells having 46 chromosomes, while another 4% of individuals have a translocation that involves chromosome 21.The majority of translocations in Down syndrome are fusions at the centromere between chromosomes 13, 14, 15, or 21, known as Robertsonian translocations. The translocations can be de novo or inherited.Down syndrome patients without a visible chromosome abnonnality are the least common.It is not possible to distinguish the phenotypes of individuals with full trisomy 21 and those with a translocation.Patients who are mosaic tend to have a milder phenotype.", "cop": 3, "opa": "18 trisomy", "opb": "14/21 translocation", "opc": "21 trisomy", "opd": "Trisomy 13", "subject_name": "Pediatrics", "topic_name": "Genetics And Genetic Disorders", "id": "d211a20c-5291-4cb0-a11b-d969f9f9fc65", "choice_type": "single"} {"question": "Commonest cardiac lesion in tuberous sclerosis is", "exp": "Tuberous sclerosis complex (TSC) is an autosomal dominant neurocutaneous syndrome with a high incidence of sporadic cases and variable clinical expression. It has an estimated frequency of 1/6000. Major manifestations of TSC include skin lesions in more than 95%, autism and seizures in 85%, kidney disease in 60%, mental retardation in 50%, and cardiac rhabdomyoma in 50%.Mental retardation and autism are more in TSC patients who presents with generalized seizures including infantile spasms in the first 2 years of life. Reference: GHAI Essential pediatrics, 8th edition", "cop": 4, "opa": "SD", "opb": "VSD", "opc": "Mitral stenosis", "opd": "Rhabdomyoma", "subject_name": "Pediatrics", "topic_name": "Genetic and genetic disorders", "id": "5f5d9279-8c85-42ad-8275-da8883f1eed5", "choice_type": "single"} {"question": "Most commonly, ductus aeriosus is", "exp": "Most commonly, the ductus aeriosus is just distal to the subclan, and the ductus enters the pulmonary aery at its bifurcation. Ref: Nelson textbook of pediatrics 21st edition Pgno: 2344", "cop": 2, "opa": "Just proximal to the left common carotid", "opb": "Just distal to the left subclan", "opc": "Just distal to the left common carotid", "opd": "Just proximal to the left subclan", "subject_name": "Pediatrics", "topic_name": "C.V.S", "id": "2947c9f6-0da0-4282-bef3-4c0c204f29ae", "choice_type": "single"} {"question": "Congenital wilson&;s disease is characterized by", "exp": "Wilson disease is an inborn error of metabolism due to toxic accumulation of copper in liver,brain,cornea and other tissues.Manifestations are more likely to be hepatic in early childhood & neurological in adolescents or adults.The spectrum of hepatic manifestations include all forms of acute or chronic liver disease,i.e.,&;viral hepatitis&; like illness,acute liver failure,chronic hepatitis,poal hypeension,cirrhosis or asymptomatic hepatomegaly.KF (Kayser-Fleischer)/rings are most common in children with neurological(96%) than hepatic(60%) Wilson disease. Wilson disease is characterised by decreased serum ceruloplasmin & increased urinary copper excretion with increased hepatic copper stores. Reference:Essential pediatrics-Ghai,8th edition,page no:320,321", "cop": 2, "opa": "KF ring is present at bih", "opb": "May present as acute hepatitis", "opc": "Decreased Urinary copper excretion", "opd": "Decreased hepatic copper concentration", "subject_name": "Pediatrics", "topic_name": "Metabolic disorders", "id": "c5213958-6c4b-43a5-ae5f-1d10123787e2", "choice_type": "single"} {"question": "Single gene defect causing multiple unrelated problems", "exp": "Pleiotropism : A gene that has multiple phenotypic effects It is a special interest because it helps in understanding the relationships between different organisms of the same individuals Examples: I. Drosophila bar eyed individuals may be significantly altered by the wing nature II. Seed coat colour gene of sweet pea controls flower colour and also red spot in leaf axils II. Sickle cell anemia in human . Reference: GHAI Essential pediatrics, 8th edition", "cop": 1, "opa": "Pleiotropism", "opb": "Pseudodominance", "opc": "Penetrance", "opd": "Anticiaption", "subject_name": "Pediatrics", "topic_name": "Genetic and genetic disorders", "id": "791f904b-6d99-4052-913a-ea93f94cda02", "choice_type": "single"} {"question": "A child has presented with a BP of 190/110, pedal edema++, facial edema, gross hematuria and absent ascites, The most probable diagnosis is", "exp": "Glomerulonephritis signs and symptoms include: Pink or cola-colored urine from red blood cells in your urine (hematuria) Foamy urine due to excess protein (proteinuria) High blood pressure (hypeension) Fluid retention (edema) with swelling evident in your face, hands, feet and abdomen. Causes of Acute Glomerulonephritis 1.Non-infectious 2.Infectious # Numerous infectious organisms, include fungi, bacteria, rickettsia, virus, especially influenza, and parasites could induce PAGN. # However, APSGN accounts for 80 to 90% of such cases and is used as the prototype for this group of disorder. # APSGN is a classic example of the acute nephritic syndrome Reference: GHAI Essential pediatrics, 8th edition", "cop": 1, "opa": "Acute glomerulonephritis", "opb": "Minimal change nephritis syndrome", "opc": "Renal vein thrombosis", "opd": "Renal amyloidosis", "subject_name": "Pediatrics", "topic_name": "Urinary tract", "id": "4e058de7-f4ee-47a4-a4ae-821ce65cc213", "choice_type": "single"} {"question": "Periventricular Leukomalacia most commonly causes", "exp": "Pathology\nCerebral Palsy subtype\n\n\n\n\nPeriventricular\n\t\t\t\tLeukomalacia\n\n\nSpastic diplegia\n\n\n\n\nParasaggital brain injury\n\n\nSpastic quadriparesis\n\n\n\n\nMiddle cerebral artery (MCA) teritory infarction\n\n\nSpastic hemiplegia\n\n\n\n\nBasal ganglia\n\n\nDyskinetic cerebral palsy", "cop": 3, "opa": "Spastic quadriplegia", "opb": "Hypotonic Cerebral palsy", "opc": "Spastic diplegia", "opd": "Dyskinetic cerebral palsy", "subject_name": "Pediatrics", "topic_name": null, "id": "4794d606-0acc-489b-96b7-3beb081002ee", "choice_type": "single"} {"question": "Concentration of epinephrine used in neonatal resuscitation is", "exp": "Medications: indication, dosage and effects MedicationIndicationEffectsConcentration administeredDose of prepared solutionRouteEpinephrine (1:1000)HR <60 / min after 30 sec of effective PPV and chest compressions Inotropic; chronotropic; peripheral vasoconstrictor 1:100000.1-0.3 ml/kgIV; through umbilical vein (endotracheal route if no IV access) Normal salineRinger lactateAcute bleeding with hypovolemia Increased intravascular volume improves perfusion 10 ml/kgUmbilical vein Naloxone (0.4 mg/ml) Respiratory depression with maternal history of narcotic use within 4 hr of bihNarcotic antagonist 0.4 mg/ml 0.25 ml/kgIV preferred; delayed onset of action with intramuscular use; administer only after restoring ventilationSodium bicarbonate is administered only if prolonged asphyxia is associated with metabolic acidosis despite use of epinephrine and volume expanders.IV intravenous; PPV positive pressure ventilation Ref: Paediatrics; O.P. Ghai; 8th edition; Page no: 132", "cop": 3, "opa": "0.111111111", "opb": "0.736111111", "opc": "1 : 10000", "opd": "1 : 100000", "subject_name": "Pediatrics", "topic_name": "New born infants", "id": "a74e5da4-efa5-48e7-9551-eee5ceef4bb1", "choice_type": "single"} {"question": "Split skin grafts in children should be harvested from", "exp": "A split-thickness skin graft(STSG) is a skin graft including the epidermis and pa of the dermis. Its thickness depends on the donor site and the needs of the person receiving the graft.A full-thickness skin graft consists of the epidermis and the entire thickness of the dermis. Split skin grafts in children should be harvested from Buttocks. Reference: GHAI Essential pediatrics, 8th edition", "cop": 1, "opa": "Buttocks", "opb": "Thigh", "opc": "Trunk", "opd": "Upper limb", "subject_name": "Pediatrics", "topic_name": "Miscellaneous", "id": "d1c3b1f3-3463-4fc9-98e9-52a4a4a6f86b", "choice_type": "single"} {"question": "Number of primary teeth;", "exp": "there are total of 20 primary teeth PRIMARY/ MILK/ TEMPORARY TEETH SECONDARY/ PERMANENT TEETH BEGIN AT 6-7 months 6 years 1st tooth to erupt Lower central incisor 1st molar Last tooth Second molar 3rd molar or wisdom tooth Completes at 2 and half - 3 years 12 years except the 3rd molar Total no. of teeth 20 28-32", "cop": 1, "opa": "20", "opb": "24", "opc": "16", "opd": "32", "subject_name": "Pediatrics", "topic_name": "FMGE 2019", "id": "b7695889-6913-44bb-9ef2-9d68d13494cb", "choice_type": "single"} {"question": "Screening of neonatal thyroid disease is done by", "exp": "Since the neonates are asymptomatic at bih all newborns are screened for neonatal hypothyroidism. TSH is estimated either in cord blood at the time of bih (preferred) or blood obtained from heel prick after 2 days of bih.(Refer: Nelson's Textbook of Pediatrics, SAE, 1st edition, pg no. 2669 - 2773)", "cop": 3, "opa": "T4", "opb": "T3", "opc": "TSH", "opd": "TPO antibodies", "subject_name": "Pediatrics", "topic_name": "All India exam", "id": "453400f4-8772-45e6-9e35-63e9e32f11f1", "choice_type": "single"} {"question": "Sho stature, secondary to growth hormone deficiency is associated with", "exp": "sho stature : Defined as height below third centile or more than 2 standard detions below the median height for age and gender according to the population standard. Children whose stature s more than 3 SD below the population mean for age and gender are more likely to be suffering from pathological sho stature as compared to those with stature between -2 and -3SD , who are more likely to be affected by familial or constitutional sho stature. CAUSES:- 1. Most common cause is constitutional. 2. Familial 3. Pathological causes include: Undernutrition Chronic systemic illness Cerebral palsy Congenital hea disease Cystic fibrosis,asthma Malabsorption Acquired immunodeficiency syndrome 4. Endocrine causes include: Growth hormone deficiency Hypothyroidism Cushing syndrome Pseudohypoparathyroidism Precocious or delayed pubey 5. Psychosocial dwarfism 6. Children born small for gestational age. 7. Skeletal dysplasia; achondroplasia, rickets. 8. Genetic syndrome; Turner, Down syndrome. Growth hormone deficiency:- Growth hormone deficiency may be caused by congenital CNS malformations, genetic defects or acquired neurological insults. These children have normal growth at bih. Growth retardation become apparent around 1 year of age. Midfacial crowding, round facies, mild obesity,depressed nasal bridge,single central incisor tooth and micropenis are common. Body propoions are normal. Development of tooth delayed.Facial appearance is doll like. These children look much younger than their actual age. Bone age is delayed. Newborns may present with severe hypoglycemic seizures due to concomitant ACTH deficiency. Reference: GHAI Essential pediatrics, 8th edition", "cop": 1, "opa": "Normal body propoion", "opb": "Low bih weight", "opc": "Normal epiphyseal development", "opd": "Height age equal to skeletal age", "subject_name": "Pediatrics", "topic_name": "Growth and development", "id": "f53ca41d-6e30-426d-adb9-eac0b7dd7e72", "choice_type": "single"} {"question": "Most common cause of Neonatal Hypothyroidism is", "exp": "Iodine deficiency is the commonest cause of congenital hypothyroidism in ceain pas of India, while thyroid dysgenesis is the most common aetiology in non-endemic areas (75% of all cases). Thyroid scan pictures showing different types of thyroid dysgenesis. At upper panel hypoplasia (A), hemiagenesis (B), ectopia (C), double ectopia (D), and athyreosis (E) compared with normal thyroid (G) shown at lower panel. Panel F showing scintigraphy performed at the time of diagnosis (F1 and F2) repoed as hemiagenesis while repeated scintigraphy at the time of the study (at 12 years age) showed ectopia (F3). Radiopharmaceutical uptake by thyroid tissue is shown by black arrows and salivary glands by empty arrows. Reference: Ghai essential of paediatrics, eighth edition, p.no:516", "cop": 1, "opa": "Throid dysgenesis", "opb": "Transplacental maternal thyroid antibodies", "opc": "Inherited genetic disorders", "opd": "Decreased thyroglubin", "subject_name": "Pediatrics", "topic_name": "Endocrinology", "id": "8fabfd5f-9c07-4a4c-9e7d-1bb042a3ba2a", "choice_type": "single"} {"question": "Transient myleoproliferative disorder of the newborn is seen in association with", "exp": "Approximately 10% of children with Down syndrome develop a preleukemic clone,transient myeloproliferative disorder,with somatic mutations in hematopoietic transcription factor GATA1.These children have a high leukocyte count,circulating blasts in peripheral blood,hepatosplenomegaly,effusions,anemia and thrombocytopenia in the neonatal period,which resolves by 3 months.About 20% of children with transient myeloproliferative disorder develop AML. Reference:Essential pediatrics-Ghai,8th edition,page no:605.", "cop": 2, "opa": "Turnur's syndrome", "opb": "Down's syndrome", "opc": "Neurofibromatosis", "opd": "Ataxia telangiectasia", "subject_name": "Pediatrics", "topic_name": "Childhood tumors", "id": "d24dfd87-6573-4dc4-8f75-f04051a74482", "choice_type": "single"} {"question": "Chromosome involved in Wilm's tumor is", "exp": "WTl is the best characterized Wilms tumor gene. It is located at chromosome llp13 and encodes for a transcription factor that is critical for normal development of kidneys and gonads. WT2 is localized to a cluster of genes at llpl5. Reference: Essential paediatrics; O.P.Ghai; Childhood Malignancies; Page no: 617", "cop": 1, "opa": "11", "opb": "13", "opc": "18", "opd": "22", "subject_name": "Pediatrics", "topic_name": "Childhood tumors", "id": "9efce15d-8682-4927-b158-6927e4fd276e", "choice_type": "single"} {"question": "Maximum growth spu is seen in girls at time of", "exp": "Adolescence: Stage of transmission from childhood to adulthood. During this stage, there will be an appearance of sexual characters with changes in cognition and psychology. Adolescence refers to this entire process, pubey refers to the physical aspect. Age 10-19 year is considered a period of adolescence and pubey marks the early half of adolescence. Pubey in girls stas with breast development( thelarche) anytime between 8-13 years. This is followed by an appearance of pubic hair ( pubarche) and subsequently, menstruation ( menarche), occurring at an average of 12.6 years. Menarche occurs after 2-21/2 years of thelarche. In boys, the earliest change is an increase in testicular size( testicular volume reaching 4 ml) and this occurs between 9-14 years. This is followed by an appearance of pubic hair and lengthening of the penis. Spermarche( sperm production) stas during mid-adolescence. Laryngeal growth, manifesting as cracking of voice, begins in boys in mid-pubey, deepening of voice is complete by end of pubey. During pubey, boys gain 20-30 cm and girls 16-28 cm of height. Peak growth velocity in girls occurs before attainment of menarche. Boys have their peak growth in later stages of pubey. Reference: GHAI Essential pediatrics, 8th edition", "cop": 3, "opa": "Pubarche", "opb": "Thelarche", "opc": "Menarche", "opd": "Adrenarche", "subject_name": "Pediatrics", "topic_name": "Adolescence", "id": "eb7ddba8-137e-482a-9c7f-625b381c67cc", "choice_type": "single"} {"question": "The commonest cardiac lesion in Tuberous sclerosis is", "exp": "MAJOR FEATURES OF TUBEROUS SCLEROSIS COMPLEX:Coical tubersSubependymal noduleSubependymal giant cell astrocytomaFacial angiofibroma Periungual fibromaHypomelanotic macules (>3) Shagreen patchMultiple retinal hamaomasCardiac rhabdomyomaRenal angiomyolipomaPulmonary lymphangioleiomyomatosisApproximately 50% of children with tuberous sclerosis have cardiac rhabdomyoma.Most common cardiac lesion: cardiac rhabdomyomaMost common renal lesion: angiomyolipomaMost common skin lesion: hypomelanotic maculeMost common CNS lesion: coical tubersMost common pulmonary lesion: lymphangiomyomatosis", "cop": 4, "opa": "ASD", "opb": "VSD", "opc": "Mitral stenosis", "opd": "Rhabdomyoma", "subject_name": "Pediatrics", "topic_name": "All India exam", "id": "61b7a908-a477-4e4a-9ec4-1842a689a6cd", "choice_type": "single"} {"question": "In newborn, the aery used for palpation of the pulse is", "exp": "Radial, brachial and carotid aeries are difficult to palpate in a newborn baby.The femoral aery is easily palpable in the newborn.Bilateral femoral pulse should be palpated and indicates the degree of volume status in newborn.In coarctation of the aoa, the femoral pulse will be absent.", "cop": 4, "opa": "Carotid", "opb": "Brachial", "opc": "Radial", "opd": "Femoral", "subject_name": "Pediatrics", "topic_name": "All India exam", "id": "e672ac60-1a3a-4528-bde3-e3ebf45188a6", "choice_type": "single"} {"question": "Edward syndrome is", "exp": "Edward syndrome is trisomy 18 . It is due to non dysfunction of chromosomes during meiosis Trisomy 21 is down symdrome and trisomy 13 is patau syndrome. Reference: Nelson TB of pediatrics pg 404 edi 19.", "cop": 2, "opa": "Trisomy 13", "opb": "Trisomy 18", "opc": "Trisomy 21", "opd": "Trisomy 20", "subject_name": "Pediatrics", "topic_name": "Genetic and genetic disorders", "id": "5631586c-28c3-4691-bd12-b8bc06a26080", "choice_type": "single"} {"question": "Conjugated hyperbilirubinemia", "exp": "Ans. is 'a' i.e., Dubin johnson syndrome Breast milk jaundice - Decrease bilirubin uptake across hepathocyte membrane. Entero-hepatic recirculation. Leads to indirect hyperbilirubinemia. Crigler naj jar & Gilbe syndrome (deficiency of glucuronyl transferase) =20 wks", "opb": ">= 24 wks", "opc": ">=28 wks", "opd": ">= 32 wks", "subject_name": "Pediatrics", "topic_name": "New born infants", "id": "2c999931-4eb7-4617-8cd4-aae2c40ee6c1", "choice_type": "single"} {"question": "Most common site of metastases in neuroblastoma", "exp": "metastases in neuroblastoma :\n Most commonly involved organs are lymph nodes, long bones, skull, bone marrow, liver.", "cop": 3, "opa": "Lung", "opb": "Liver", "opc": "Skull", "opd": "Vertebrae", "subject_name": "Pediatrics", "topic_name": null, "id": "649bc41f-166f-4f92-8a5e-5e107bf38ba5", "choice_type": "single"} {"question": "The Finnish type of Congenital Nephritic syndrome occurs due to Gene mutation affecting the following protein", "exp": "It occurs due to gene mutation NPHS 1 which encodes nephron and the protein is affected and results in massive protein loss , edema. Congenital nephrotic syndrome Finnish type is a genetic condition of the kidney that begins early in development during pregnancy or within the first three months of life. The syndrome is characterized by a group of symptoms, including protein in the urine (proteinuria), low blood protein levels, high cholesterol levels, and swelling (nephrotic syndrome), which progresses rapidly to end-stage kidney disease. Infants with congenital nephrotic syndrome may have failure to thrive and frequent infections. Although more commonly seen in individuals of Finnish descent, congenital nephrotic syndrome Finnish type has been repoed worldwide. Congenital nephrotic syndrome Finnish type is caused by mutations in the NPHS1 gene and is inherited in an autosomal recessive manner. At this time, kidney transplantation seems to be the only treatment available for this condition. Reference: GHAI Essential pediatrics, 8th edition", "cop": 3, "opa": "Podocin", "opb": "Alpha-actinin", "opc": "Nephrin", "opd": "CD2 activated protein", "subject_name": "Pediatrics", "topic_name": "Urinary tract", "id": "ef59b5dc-91ba-450d-9c64-0ddfad952e37", "choice_type": "single"} {"question": "Hilar dance on fluoroscopy is seen in", "exp": ". Ghai essential of pediatrics, eighth edition p.no:413 hilar dance on fluoroscopy due to vigorous pulmonary aery pulsations seen in ASD", "cop": 1, "opa": "ASD", "opb": "VSD", "opc": "PS", "opd": "TR", "subject_name": "Pediatrics", "topic_name": "C.V.S", "id": "8985a7cd-5e34-4d08-adb0-f0362d317024", "choice_type": "single"} {"question": "First line ATT in children", "exp": "*The first line drugs in ATT are effective with minimal toxicity and include isoniazid,rifampicin,ethambutol,streptomycin and pyrazinamide.Ethionamide is a second line drug. * In a view of increasing INH resistance, use of three drugs (INH, Rifamipicin , and ethambutol) instead of two (INH and rifampicin) is suggested during the continuous phase . Reference : Ghai essential pediatrics 9 th edition Page 249", "cop": 4, "opa": "Streptomycin", "opb": "Pyrazinamide", "opc": "Ethionamide", "opd": "1 & 2", "subject_name": "Pediatrics", "topic_name": "Infectious disease", "id": "f3a06469-eaa1-4e72-a0a3-bd92befeb675", "choice_type": "single"} {"question": "Vaccine not given in Adolescents", "exp": "Adolescence ranges from 10 to 19 yrs Early ( 10-13yrs); Mid(. 14-16yrs); Late( 17-19yrs) DPT vaccine is given at 10-12 yrs, once every 10 yrs thereafter. HPV vaccine - 2 doses( if 9-14 yrs). 3 doses( if beyond 15 yrs) Influenza vaccine- Annually Rota virus vaccine presently is given in 4 states (AP, HP, Haryana, Odisha).It is given to prevent diarrhoea caused by rota virus. Three kinds of live attenuated oral vaccines ; Rotatrix( monovalent ) 2 doses:10 &14 wks Rotateq( pentavalent)3doses:6,10,14 wks Rotovac( monovalent recombinant )3doses Immunization of rotavirus vaccine should be completed by 8 months of age. Adolescents affected with rotavirus show mild symptoms in comparison to infants and toddlers Ref: Ghai, 9th edition, Chap 5 & 10.", "cop": 3, "opa": "MMR", "opb": "Tetanus", "opc": "Rotavirus", "opd": "HPV", "subject_name": "Pediatrics", "topic_name": "Adolescence", "id": "bc58e16a-4e2a-48ed-bede-4dd7aae802a3", "choice_type": "single"} {"question": "NOT a feature of Fragile X syndrome", "exp": "Fragile X syndrome follows X-linked dominant inheritance Clinical features are\n\nElongated face\nIncreased risk of ADHD and Autism\nLarge gonads\nIntellectual disability\nLarge ears (not low set ears)", "cop": 1, "opa": "Low set ears", "opb": "X-linked dominant inheritance", "opc": "Large gonads", "opd": "Intellectual disability", "subject_name": "Pediatrics", "topic_name": null, "id": "4423c872-8ef8-4d1b-9fd4-cb85253ef6df", "choice_type": "single"} {"question": "Commonest cause of congenital adrenal hyperplasia is", "exp": "(21 hydroxylase) (489-90-G) (492-Ghai 7th)* 21 hydroxylase & 11-b hydroxylase deficiency are common varieties21 hydroxylase11-b hydroxylase deficiency* May present as salt-wasting form of CAH, due to the associated aldosterone deficiency resulting in life threatening emergencies in the early postnatal life* These may present with severe vomiting and vascular collapse* Boys with CAH do not have anomaly of the external genitals at birth* Results in accumulation of deoxycortisol (DOC) because enzymatic block is beyond this stage* These children develop hypertension of varying severity due to excessive DOC", "cop": 1, "opa": "21 hydroxylase deficiency", "opb": "11 B-hydroxylase deficiency", "opc": "17 alpha-hydroxylase deficiency", "opd": "18 hydroxylase deficiency", "subject_name": "Pediatrics", "topic_name": "Endocrinology", "id": "6e82a34f-5e1b-4b1b-8ee6-5eb93b618025", "choice_type": "single"} {"question": "A foreign body is commonly responsible for vaginal bleeding in pediatric patients.", "exp": "Ans. is 'a' i.e., Congenital malformation o Smoking during pregnancy causes --> o IUGR (Low bih weight) Fetal death Sudden infant death syndrome May horm the child's mental development and behavior.", "cop": 1, "opa": "Congenital malformation", "opb": "Intrauterine growth retardation", "opc": "Poor weight gain", "opd": "Foetal death", "subject_name": "Pediatrics", "topic_name": null, "id": "db485740-88f0-475d-b7e5-459e47d189ba", "choice_type": "single"} {"question": "Neptrotic syndrome is claracterised by", "exp": "Ans. is 'b' i.e., Hematuria", "cop": 2, "opa": "Ea) Massine protenuria", "opb": "Hematuria", "opc": "Edema", "opd": "Hyperlipidemia", "subject_name": "Pediatrics", "topic_name": null, "id": "b7c1f3b0-bf3d-4383-8c2d-9b85da65e188", "choice_type": "single"} {"question": "For karyotyping, the dividing cells are arrested by the addition of colchicines in the following mitotic phase", "exp": "b. Metaphase(Ref: Nelson's 20/e p 605-606, Ghai 8/e p 645)Chromosomes are examined after arresting dividing cells in metaphase with mitotic spindle inhibitors (e.g., N-diacetyl- N-methylcolchicine), followed by staining.", "cop": 2, "opa": "Prophase", "opb": "Metaphase", "opc": "Anaphase", "opd": "Telophase", "subject_name": "Pediatrics", "topic_name": "Genetics And Genetic Disorders", "id": "295dd159-ef26-4755-8e22-ed98d6ea5d33", "choice_type": "single"} {"question": "The floor of the nasal cavity in children is made of", "exp": "\"The floor of the nasal cavities, which also form the roof of the mouth, is made up by the bones of the hard palate: the horizontal plate of the palatine bone posteriorly and the palatine process of the maxilla anteriorly\".(Refer: Nelson's Textbook of Pediatrics, SAE, 1st edition, pg no. 2007)", "cop": 4, "opa": "Palatine bone and vomer", "opb": "Sphenoid and ethmoid", "opc": "Nasal bone and maxilla", "opd": "Palatine bone and maxilla", "subject_name": "Pediatrics", "topic_name": "All India exam", "id": "ca6bbb70-bdb8-4106-984a-39c129c6d0f7", "choice_type": "single"} {"question": "Dosage of drug in child is calculated based on March 2013", "exp": "Ans. B i.e. Weight Drug dosage for children is calculated based on age and weight of the child.", "cop": 2, "opa": "Race", "opb": "Weight", "opc": "Sex", "opd": "Height", "subject_name": "Pediatrics", "topic_name": null, "id": "ccb972cc-c6c7-41e3-890d-07e5421ea473", "choice_type": "single"} {"question": "The metabolic dearrangement in congenital pyloric stenosis is", "exp": "The classical presentation is non bilious vomiting begining at 3-6 weeks of age Severity and frequency of vomiting increases gradually Persistant vomiting leads to malnutrition,dehydration and hypochloremic alkalosis. constipation is usual Diagnosis is based on history and palpability of mass. Ref : ESSENTIAL PEDIATRICS,O.P.GHAI,PG NO:253,7 th edition", "cop": 1, "opa": "Hypochloremic alkalosis", "opb": "Hyperchloremic alkalosis", "opc": "Hyperchloremic acidosis", "opd": "Hypochloremic acidosis", "subject_name": "Pediatrics", "topic_name": "Gastrointestinal tract", "id": "8c4ec18e-dfd5-44a1-80d3-68ab07e52775", "choice_type": "single"} {"question": "Select the most common clinical sign or symptom for diagnosis PHACE syndrome", "exp": "The child with a unilateral facial lesion and blindness suggests either Sturge-Webber or PHACE syndrome (posterior fossa malformations, hemangiomas, arterial anomalies, coarctation of the aorta or other cardiac defects, and eye abnormalities). These patients can have seizures, hemiparesis, intracranial calcifications, and mental retardation.", "cop": 1, "opa": "A 6-month-old child with blindness on the same side as a large facial lesion", "opb": "An infant with infantile spasms, a hypsarrhythmic EEG pattern, and ash-leaf depigmentation on her back", "opc": "An 18-year-old patient with a history of fractures and optic gliomas, who now has developed a malignant schwannoma", "opd": "A 2-year-old child with multiple episodes of skin infection and failure to thrive", "subject_name": "Pediatrics", "topic_name": "Musculo Skeletal Disorders", "id": "2287bf08-9ffb-4d1e-87a3-557a10b8bea9", "choice_type": "single"} {"question": "A patient of moderate VSD in chronic CCF develops clubbing with no cyanosis. Diagnosis is", "exp": "SHUNT REVERSAL / EISENMENGER SYNDROME: Refers to patients with the septal defect in which blood is shunted paially or completely from right to left as a result of the development of pulmonary vascular disease resulting in apparent cyanosis.Long-standing pulmonary edema is characterized by cyanosis, dyspnea.New onset clubbing without cyanosis in a patient with congenital hea defects indicates the development of SABE .Pulmonary aerial hypeension commonly occurs in large non-restrictive VSD and is unlikely in moderate VSD.", "cop": 3, "opa": "Shunt reversal", "opb": "Long standing pulmonary edema", "opc": "SABE", "opd": "Pulmonary aery hypeension", "subject_name": "Pediatrics", "topic_name": "All India exam", "id": "6901fb5f-ae4a-4807-bf79-912d141459a2", "choice_type": "single"} {"question": "Most common Cardiac anomoly associated with Turner's syndrome", "exp": "Most common Cardiac anomoly associated with Turner's syndrome is Bicuspid Aortic Valve > Coarctation of Aorta.", "cop": 2, "opa": "Ventricular septal defect", "opb": "Bicuspid Aortic Valve", "opc": "Coarctation of Aorta", "opd": "Pulmonary stenosis", "subject_name": "Pediatrics", "topic_name": null, "id": "8c60a907-0d5a-4ecb-a484-aedb56fb3363", "choice_type": "single"} {"question": "Perinatal period corresponds to", "exp": "Peaining to the period immediately before and after bih. The perinatal period is defined in diverse ways. Depending on the definition, it stas at the 20th to 28th week of gestation and ends 1 to 4 weeks after bih. Ref : Nelson paediatrics chapter : Growth and develop", "cop": 1, "opa": "28 weeks of gestation to 7 days after bih", "opb": "Period of labour to newborn period", "opc": "Third trimester of pregnancy to newborn period", "opd": "36 weeks of gestation to 3 days after bih", "subject_name": "Pediatrics", "topic_name": "Growth and development", "id": "6721e0e3-00e8-45c9-a2f6-f457fe0b979b", "choice_type": "single"} {"question": "Kleiner syndrome is diagnosed by", "exp": "Klinefelter syndrome\n\nExtra x chromosome (47 XXY) karyotype\nSmall testes with hyalinised seminiferous tubules\nDelayed puberty\nDelayed secondary sexual characters Small penis\nTall & underweight\nMental retardation\nGynaecomastia", "cop": 1, "opa": "Karvo typing", "opb": "USG abdomen", "opc": "Triple test", "opd": "Echocadiography", "subject_name": "Pediatrics", "topic_name": null, "id": "5f487a75-f6cb-4238-81ef-0e0d34a6a9ca", "choice_type": "single"} {"question": "A child is brought to the paediatric OPD with fever of 24 hours furation. History reveals 3 episodes of chest infection and passage of foul smelling stools. The most probable diagnosis is", "exp": "The symptoms are suggestive of cystic fibrosis. Ref : Essential paediatrics,O.P.Ghai,7 th edition,pg no:369", "cop": 1, "opa": "Cystic Fibrosis", "opb": "Maple syrup urine Disease", "opc": "Bilirubin Congugation Defect", "opd": "Criggler Najar Syndrome", "subject_name": "Pediatrics", "topic_name": "Respiratory system", "id": "967ea5ae-cc33-4571-b9aa-1e6e16c4b540", "choice_type": "single"} {"question": "Most common presentation of Down syndrome is", "exp": "Down syndrome:- Trisomy 21 Patients with down syndrome have mental and physical retardation, flat facial profile, an upward slant of eyes and epicanthic folds. Oblique palpebral fissure seen. Nose is small with flat nasal bridge. Mouth shows narrow sho palate with small teeth and furrowed protruding tongue. There is significant hypotonia. Skull appears small and brachycephalic with flat occiput . Ears are small and dysplastic. Characteristic facial grimace on crying. Hands are sho and broad. Clinodactyly( hypoplasia of middle phalanx of fifth finger) and simian crease are usual. There is wide gap between the first and second toe( Sandle gap). Associated anomalies:- 1. Congenital hea disease- ventricular septal defect. 2. Gastrointestinal - atresia, annular pancreas and Hirschsprung disease. 3. Ophthalmic- cararact, nystagmus, squint. 4. Thyroid dysfunction 5. Conductive hearing loss Best serological marker for down syndrome is Beta HCG. Reference: GHAI Essential pediatrics, eighth edition", "cop": 1, "opa": "Congnitive impairment", "opb": "Delayed dentition", "opc": "Recurrent chest infection", "opd": "Constipation", "subject_name": "Pediatrics", "topic_name": "Genetic and genetic disorders", "id": "a7f2ea4f-13cd-4d37-9226-f2e4d8002ee6", "choice_type": "single"} {"question": "Breast milk at room temperature stored for", "exp": "Breast milkCan be stored at room temperature for 8-10 hoursIn a refrigerator for 24 hoursIn a freezer for 3 months(Refer: Nelson's Textbook of Pediatrics, SAE, 1st edition, pg no. 286 - 290)", "cop": 2, "opa": "4 hrs", "opb": "8 hrs", "opc": "12 hrs", "opd": "24 hrs", "subject_name": "Pediatrics", "topic_name": "All India exam", "id": "f01e7ee2-e9b0-429a-b81f-cc46ddd9ce5a", "choice_type": "single"} {"question": "The drug of choice in treatment of inflantile spasms is", "exp": "Infantile spasms:-\n\nWest syndrome\nOnset : 3-8 months of life\nCharacterised by combination of salaam spells( sudden dropping of the head and flexion of arms), developmental retardation, hypsarrythmia on ECG.\n\nCauses-\n\nHypoxic ischemic encephalopathy\nNeurocutaneous syndromes\nPerinatal infections\nHemorrhage\nInjury\nMetabolic disorders\nLocalized structural malformations \nThe spasms occur in clusters usually on waking\n\nDrug therapy-\n\nIntramuscular ACTH 40-60 unit per day may be given 4-6 weeks and tgen tapered off.\nOn the other hand oral prednisolone at 2 mg/ kg / day in divided doses may also be used.\nThese agents abolish spasms and may result in resolution of hypsarrythmia.", "cop": 4, "opa": "Phenytoin", "opb": "Phenobarbitone", "opc": "Carbamazepin", "opd": "ACTH", "subject_name": "Pediatrics", "topic_name": null, "id": "d4f1b7b4-d096-409d-98f2-abbba360b7ac", "choice_type": "single"} {"question": "Residual auditory defect is a common complication in meningitis caused by", "exp": "Residual auditory defect is caused by Haemophilus meningitis .so BERA is the investigation of choice done in them. Reference: GHAI Essential pediatrics, 8th edition", "cop": 4, "opa": "Staphylococcus", "opb": "Meningococcus", "opc": "Escherichia coli", "opd": "Haemophilus influenzae", "subject_name": "Pediatrics", "topic_name": "Central Nervous system", "id": "5a527f39-cb8f-43f5-a3c4-3515289c4346", "choice_type": "single"} {"question": "The definition of hypeension in children is", "exp": "The definition of hypeension in adults is BP >=140/90 mm Hg, regardless of body size, sex, or age. This is a functional definition that relates level of BP elevation with the likelihood of subsequent cardiovascular events. Because hypeension-associated cardiovascular events, such as myocardial infarction or stroke, usually do not occur in childhood, the definition of hypeension in children is statistical rather than functional. The National High Blood Pressure Education Program Working Group on High Blood Pressure in Children and Adolescents published the Fouh Repo on the Diagnosis, Evaluation, and Treatment of High Blood Pressure in Children and Adolescents (Fouh Repo) in 2004. This repo established normal values based on the normative distribution of BP in healthy children and included tables with systolic and diastolic values for the 50th, 90th, 95th, and 99th percentile by age, sex, and height percentile. The Fouh Repo defined hypeension as average systolic blood pressure (SBP) and/or diastolic BP that is >=95th percentile for age, sex, and height on >=3 occasions. Prehypeension was defined as average SBP or diastolic BP that is >=90th percentile but <95th percentile. In adolescents beginning at age 12 yr, prehypeension is defined as BP between 120/80 mm Hg and the 95th percentile. A child with BP levels >=95th percentile in a medical setting but normal BP outside of the office has white coat hypeension. The Fouh Repo fuher recommended that if BP is >=95th percentile, then hypeension should be staged. Children with BP between the 95th and 99th percentile plus 5 mm Hg are categorized as stage 1 hypeension, and children with BP above the 99th percentile plus 5 mm Hg have stage 2 hypeension. Stage 1 hypeension, if asymptomatic and without target organ damage, allows time for evaluation before staing treatment, whereas stage 2 hypeension calls for more prompt evaluation and pharmacologic therapy. Reference: Nelson; 20th edition; Page no: 2294", "cop": 2, "opa": "Average systolic blood pressure (SBP) and/or diastolic BP that is >=95th percentile for age, sex, and height on >=2 occasions", "opb": "Average systolic blood pressure (SBP) and/or diastolic BP that is >=95th percentile for age, sex, and height on >=3 occasions", "opc": "Average systolic blood pressure (SBP) and/or diastolic BP that is >=90th percentile for age, sex, and height on >=2 occasions", "opd": "Average systolic blood pressure (SBP) and/or diastolic BP that is >=90th percentile for age, sex, and height on >=3 occasions", "subject_name": "Pediatrics", "topic_name": "C.V.S", "id": "127f7d7a-3385-4932-8f50-146933549024", "choice_type": "single"} {"question": "Most common malignant tumour in childhood", "exp": "Leukemia is the most common malignancy of childhood.There are two main subtypes,the commoner acute lymohoblastic leukemia(ALL) & acute myeloid leukemia(AML).A small propoion may have chronic myeloid leukemia(CML) and juvenile myelomonocytic leukemia(JMML).ALL is the most common childhood malignancy accounting for one-fouh of all childhood cancers. Reference:Essential pediatrics-Ghai,8th edition,page no:599.", "cop": 2, "opa": "Rhabdomyosarcoma", "opb": "Leukemia", "opc": "Lymphangioma", "opd": "Neuroblastoma", "subject_name": "Pediatrics", "topic_name": "Childhood tumors", "id": "e53287f1-2c28-4f7d-9baf-48c02931b917", "choice_type": "single"} {"question": "Definitive treatment of choice for Transposition of great vessels (TGA) is", "exp": "The aerial switch operation is now established as the treatment of choice for TGA and most centers endeavour to offer this procedure for all infants with TGA. In this operation, the pulmonary aery and aoa are transected. The distal aoa is anastomosed to the proximal pulmonary stump (neo-aoic root) and the pulmonary aery to the proximal aoic stump (neopulmonary aery). The coronary aeries are moved along to the neo-aoic root along with a cuff of aoic tissue to allow suturing without compromise of coronary blood flow. There is a limited window of time for performing the aerial switch for TGA. Infants with TGA and intact septum should ideally undergo this procedure within the first 2-4 weeks of life. As pulmonary vascular resistance falls after bih the left ventricle regresses rapidly. In 1-2 months the left ventricle has the ability to adjust to the elevated systemic vascular resistance after the aerial switch through hyperplasia of the available muscle. After this, it is difficult for the left ventricle to adapt to an aerial switch. Later in infancy, the atrial switch operation (Senning operation) is the only option for TGA with intact ventricular septum. This is not an ideal long-term option because the right ventricle remains as the systemic ventricle for life. Over time, right ventricle dysfunction and severe tricuspid regurgitation sets in. Additionally, extensive restructuring of atria predisposes to atrial rhythm disturbances. Reference: Essential Paediatrics; O.P. Ghai; Page no: 426", "cop": 2, "opa": "Atrial switch", "opb": "Aerial switch", "opc": "Atrial septostomy", "opd": "Medical management", "subject_name": "Pediatrics", "topic_name": "C.V.S", "id": "2742767b-cba8-450c-ab3c-cc0f848d7611", "choice_type": "single"} {"question": "Juene syndrome is", "exp": "Juene syndrome : Autosomal recessive disorder. Characterised by typical skeletal dysplasia such as narrow thorax and micromelia with respiratory and renal manifestations. Classic manifestations in infancy include dwarfism with sho ribs, sho limbs and characteristic radiographic changes in limbs. All patients have small chest but presentation varies. Patients who survive newborn period may later develop pancreatic and renal insufficiency. FIGURE 1 Characteristics of Jeune syndrome. A. Neonate with a bell-shaped chest, protuberant appearing abdomen, and mild shoening of the limbs. B. Radiograph of a neonate poraying the characteristic sho, horizontal ribs, costochondral flaring, and handlebar-shaped clavicles associated with Jeune syndrome. Reference: GHAI Essential pediatrics, 8th edition", "cop": 1, "opa": "Autosomal Recessive", "opb": "Autosomal Dominant", "opc": "X Linked Dominant", "opd": "X Linked Recessive", "subject_name": "Pediatrics", "topic_name": "Genetic and genetic disorders", "id": "378d31a8-3610-4a7d-944a-2f90cd9f724b", "choice_type": "single"} {"question": "The initial fluid of choice of the treatment of hypernatremic dehydration is", "exp": "The initial resuscitation of hypernatremic dehydration requires restoration of the intravascular volume with NS. RL and other fluids should not be used because they are hypotonic fluids.Seizures are the most common manifestation of cerebral edema from an overly rapid decrease of the serum sodium concentration during correction of hypernatremic dehydration. Signs of increased intracranial pressure or impending herniation may develop quite rapidly (see Chapter 68). Acutely, increasing the serum concentration an infusion of 3% sodium chloride can reverse the cerebral edema. Each 1 mL/kg of 3% sodium chloride increases the serum sodium concentration by approximately 1 mEq/L. An infusion of 4 mL/kg often results in resolution of the symptoms. This strategy is similar to that used for treating symptomatic hyponatremia Ref.nelson 20th edition 390 page", "cop": 1, "opa": "Normal saline", "opb": "N/2 saline", "opc": "N/4 saline", "opd": "D/4 saline + 5% Dextrose", "subject_name": "Pediatrics", "topic_name": "Nutrition", "id": "061900ac-f668-4a58-b3e9-7b0dc64e6d74", "choice_type": "single"} {"question": "Caorctation of aoa is most commonly seen with", "exp": "Coarctation of aoa is associated with bicuspid aoic valve. Ref : Ghai essential of pediatrics, eighth edition, p.no:431", "cop": 4, "opa": "ASD", "opb": "VSD", "opc": "PDA", "opd": "Bicuspid aoic valve", "subject_name": "Pediatrics", "topic_name": "C.V.S", "id": "77498013-697f-41a3-9604-a281482d56b4", "choice_type": "single"} {"question": "Bovine cough is characteristic of", "exp": "In bovine cough glottis cant be completely closed. Aphonic cough seen usually in vocal cord paralysis. it is one which doesn&;t have an explosive nature. Ref : PAEDIATRIC CLINICAL EXAMINATION BY A.SANTHOSH KUMAR,4 TH EDITION, PG NO:141", "cop": 3, "opa": "Acute epiglotitis", "opb": "Tracheitis", "opc": "Laryngeal adenoidectomy", "opd": "Antibiotics", "subject_name": "Pediatrics", "topic_name": "Respiratory system", "id": "a2a83c4e-5b0d-477e-958a-b325f383999b", "choice_type": "single"} {"question": "The mode of inheritance of Duchenne muscular dystrophy is", "exp": "Duchene muscular dystrophy is a X linked recessive disorder. Gait disturbances is commonly seen and calf muscle pseudo hyperophy is also seen", "cop": 4, "opa": "Autosomal dominant", "opb": "Autosomal recessive", "opc": "X-linked dominant", "opd": "X-linked recessive", "subject_name": "Pediatrics", "topic_name": "Musculoskeletal disorders", "id": "9020445a-69a9-4ea2-b3bd-27b1130bd4b9", "choice_type": "single"} {"question": "Risk of endocarditis among the following is highest in", "exp": "Risk of endocarditis - Ventricular septal defect > patent ductus arteriosus > Atrial septal defect.", "cop": 1, "opa": "Ventricular septal defect", "opb": "Patent ductus arteriosus", "opc": "Atrial septal defect", "opd": "Mitral stenosis", "subject_name": "Pediatrics", "topic_name": null, "id": "b827d3fe-58c7-41e0-8d00-0bd5e374352f", "choice_type": "single"} {"question": "A 3 year old boy with normal developmental milestones with delayed speech and difficulty in communication and concentration. He is not making friends. Most probable diagnosis is", "exp": "Ans. is 'a' Autism \"Difficulty in concentration and communication, delayed speech and problems in forming social relationship in early childhood suggests the diagnosis of Autism\"Essentials of Diagnosis and Typical Features of AutismOnset in infancy or early childhood (before age of 3 years)Abnormal language and speech development (Abnormal communication)Inability to develop normal social skill (lack of eye contact gestures and facial expression)Understand little or no language (therefore fail to acquire speech)Deficient comprehension and communicative use of speech and gesture.Behavioural peculiarities such as ritualized repetitive or stereotyped behaviour and rigidity.Difficulty in concentrationSevere deficit in social responsiveness and interpersonal skillsIntrusive stereotypes (Repetitive behaviour) together with inability to concentrate may prevent children from engaging in meaningful activity or social interaction (difficulty in making friends)Do not engage in pretended play (which starts before age of 2 in normal children)Also knowAbout 75% of children with autism are mentally retardedEpilepsy develops in one fifth to one third of autistic individuals.EEG abnormalities are noted in half of the patients.", "cop": 1, "opa": "Autism", "opb": "ADHD", "opc": "Mental retardation", "opd": "Specific learning disability", "subject_name": "Pediatrics", "topic_name": "Developmental-Behavioral Screening and Surveillance", "id": "e2ff2fab-fe59-4c36-9b83-df821ae85586", "choice_type": "single"} {"question": "An indicator of inadequate breast feeding in an infant is", "exp": "Weight loss of >7-10% of bih weight is an indicator of inadequate breastfeeding in an infant. Indicators of inadequate breastfeeding in an infant: Weight loss >7-10% of bih weight Lethargy (sleeping longer than 2-3 hours) Delayed stooling (Less than 4 times/day) Decreased urine output (urinating less than 5-7 times/day) Hypernatremic dehydration Inconsolable crying and increased hunger Insufficient milk intake may be caused by insufficient milk production, failure of established breastfeeding, and health conditions in the infant that prevent proper breast stimulation. Breastfed neonates usually feed 8-12 times a day, with a minimum of 8 times per day. However, the neonate should be breastfed on demand. Ref: Nelson textbook of pediatrics 21st edition Chapter : 56", "cop": 2, "opa": "Baby sleeping for 2-3 hours after feed", "opb": "Weight loss >7% of bih weight", "opc": "Urine 5-7 times/day", "opd": "Stool 4 times/day", "subject_name": "Pediatrics", "topic_name": "Nutrition", "id": "19d419ee-551d-40f4-baeb-d64631c9eb10", "choice_type": "single"} {"question": "NOT a feature of Holt Oram Syndrome is", "exp": "Holt Oram Syndrome Patient have normal blood picture.", "cop": 3, "opa": "Absent Radius", "opb": "Atrial septal defect", "opc": "Thrombocytopenia", "opd": "Autosomal dominant inheritance", "subject_name": "Pediatrics", "topic_name": null, "id": "205aa6ee-1223-47f3-a1ff-7690eef22da4", "choice_type": "single"} {"question": "Bilabial consonants are", "exp": "Pronounced or aiculated with both lips, as the consonants b, p, m, and wBaby stas producing bilabial words by the 4th month of age(Refer: OP Ghai's Textbook of Pediatrics, 8th edition, pg no. 32)", "cop": 2, "opa": "P, B, 9", "opb": "P, B, M, W", "opc": "M, L, W", "opd": "T, L, M, W", "subject_name": "Pediatrics", "topic_name": "All India exam", "id": "17b3a139-ad5c-4f1c-ba64-5a5db312e809", "choice_type": "single"} {"question": "Maternal Antibody responsible for heartblock in a baby born to mother suffering from SLE", "exp": "Anti-Ro antibody crosses placenta and affects conducting system of fetal heart leading to Heart block.", "cop": 3, "opa": "Anti-La antibody", "opb": "Anti-dsDNA antibody", "opc": "Anti-Ro antibody", "opd": "Anti-La antibody", "subject_name": "Pediatrics", "topic_name": null, "id": "598fe872-0fd6-4936-9ba1-96b848d1db1b", "choice_type": "single"} {"question": "Pseudohermaphroditism in a female child is most commonly due to", "exp": "Excess androgen production is the most common cause of disorder of sexual differentiation (pseudohermaphroditism) mainly 21- hydroxylase deficiency. Others are 11- beta-hydroxylase and 3 beta-hydroxysteroid dehydrogenase deficiency Image : Female pseudohermaphroditism with normal uterus. A, Sonographic view of the external genitalia; note the enlarged clitoris (C) with fused prominent labia (L). B, The corresponding postnatal disposition verifying enlarged clitoris and fused labia. Reference: Ghai essential of paediatrics, eighth edition, p.no:538", "cop": 1, "opa": "21- hydroxylase deficiency", "opb": "17-hydroxylase deficiency", "opc": "11-hydroxylase deficiency", "opd": "3-hydroxylase deficiency", "subject_name": "Pediatrics", "topic_name": "Endocrinology", "id": "ea6e3cda-69ff-4e23-b8d3-549be71c1fb3", "choice_type": "single"} {"question": "A 14 year old boy has difficulty in expressing himself in writing and making frequent spelling mistakes, does not follow instruction and cannot wait for his turn while playing a game. He is likely to be suffering from", "exp": "The Child may suffer from Specific learning disability If one find difficulty in expressing himself in writing and making frequent spelling mistakes, does not follow instruction and cannot wait for his turn while playing a game. Reference: GHAI Essential pediatrics, 8th edition", "cop": 3, "opa": "Mental retardation", "opb": "Lack of interest in studies", "opc": "Specific learning disability", "opd": "Examination anxiety", "subject_name": "Pediatrics", "topic_name": "Growth and development", "id": "dbe108b8-75d7-4800-a479-9be1e7ba0860", "choice_type": "single"} {"question": "The most impoant determinant of prognosis in Wilms tumor", "exp": "The most impoant prognostic factors in Wilms tumor is its histopathology.3% of tumors show ourable pathology(focal anaplasia)for which only surgical excision is needed.Another 6% have unourable pathology(diffuse analplasia);these tumors are pleomorphic and ruptured and show early metastasis to bones. Other prognostic factors are Stage of the disease,Ploidy(diploid tumors have better prognosis than hyperdiploid tumors).Young age,low stage and low tumor weight(<550 g) are ourable prognostic factors.Presence of anaplasia and loss of heterozygosity of 1p or 16q increase risk recurrence. Reference:Essential pediatrics-Ghai,8th edition,page no:618", "cop": 3, "opa": "Stage of disease", "opb": "Loss of heterozygosity of chromosome I p", "opc": "Histology", "opd": "Age less than one year at presentation", "subject_name": "Pediatrics", "topic_name": "Childhood tumors", "id": "abad8f18-2ede-4ca6-99e7-4fd678158a6e", "choice_type": "single"} {"question": "Bone age is advanced in", "exp": "An advanced or delayed bone age does not always indicate disease or \"pathologic\" growth. Conversely, the bone age may be normal in some conditions of abnormal growth. Children do not mature at exactly the same time. Just as there is wide variation among the normal population in age of losing teeth or experiencing the first menstrual period, the bone age of a healthy child may be a year or two advanced or delayed. Those with an advanced bone age typically hit a growth spu early on but stop growing at an earlier age. Consequently, when a naturally sho child has an advanced bone age, it stunts their growth at an early age leaving them even shoer than they would have been. Because of this, those who are sho with an advanced bone age, need medical attention before their bones fully fuse. An advanced bone age is common when a child has had prolonged elevation of sex steroid levels, as in precocious pubey or congenital adrenal hyperplasia. The bone age is often marginally advanced with premature adrenarche, when a child is overweight from a young age or when a child has lipodystrophy. Those with an advanced bone age typically hit a growth spu early on but stop growing at an earlier age. Bone age may be significantly advanced in genetic overgrowth syndromes, such as Sotos syndrome, Beckwith-Wiedemann syndrome and Marshall-Smith syndrome. Bone maturation is delayed with the variation of normal development termed Constitutional delay of growth and pubey, but delay also accompanies growth failure due to growth hormone deficiency and hypothyroidism. Reference: GHAI Essential pediatrics, 8th edition", "cop": 1, "opa": "Congenital adrenal hyperplasia", "opb": "Hypothyroidism", "opc": "Hypopituitarism", "opd": "Constitutional delay in growth", "subject_name": "Pediatrics", "topic_name": "Musculoskeletal disorders", "id": "9f6b11aa-1bd7-4444-9736-a271914dcf03", "choice_type": "single"} {"question": "Child with PDA will not have aEUR'", "exp": "CO. washout During intrauterine life the fetus does not breathe. The .fetus does not require .functioning lung to oxygenate its blood. The fetus receives oxygenated blood from the mother through the umbilical vein. The lungs remains collapsed during the fetal life and the pulmonary circulation is a high pressure circuit. So, the fetal blood does not pass through the lungs. Instead, it requires a duct that can bypass the lung and can carry the blood straight way to systemic circulation i.e., Aoa. This function is carried out by ductus aeriosus. During intrauterine life only 10% of the cardiac output passes through the lungs the remaining 90% is shunted through the ductus aeriosus to the Aoa and the systemic circulation. After bih, the umbilical vein is cut off The blood must now pass through the lungs for oxygenation.Soon, after bih fetus stas breathing, the lung is filled with air and it expands. The pulmonary vasculature changes from high pressure circuit to low pressure circuit.Now, the fetal blood must pass through the lungs to facilitate proper gas exchange. In order to make this possible, the ductus aeriosus undergoes constriction, and functional closure occur soon after bih in term neonates. Eighty percent of the ductus aeriosus in term infants close by 48 hours and nearly 100% by 96 hours. Oxygen and endothelins are very strong vasoconstrictors and pro- staglandins E2 and 12 are strong vasodilator of the ductus aeriosus. During fetal life, the 02 concentration is Low and PGE2 and PGI2 level in high. This keeps the ductus aeriosus patent. Soon after bih there is sudden elevation in circulating oxygen tension and fall in prostaglandin levels. This results in strong vasoconstriction and functional closure of the ductus aeriosus soon after the delivery. The functional closure is followed by anatomic closure in the next 1-3 months.But in ceain infants the ductus aeriosus does not close after the bih it remains open and is called patent ductus aeriosus. Since after bih the systemic circulation has higher pressure than the pulmonary circulation the blood flows from Aoa to the pulmonary aery through the patent ductus aeriosus. Hemodynamic consequences of the PDA Shunting of blood from the systemic circulation to the pulmonary circulation results in congestive cardiac failure, which manifests clinically \"with wide pulse pressure\" and \"bounding pulses\". Overloading of the pulmonary vasculature leads to pulmonary edema/hemorrhage which predisposes the neonate to \"chronic lung\" disease (CO2 retention may occur). Blood flow to the kidney and gastrointestinal tract is compromised due to shunting of blood front systemic circulation, predisposing to acute renal failure (ARE) and \"necrotizing enterocolitis (NEC) and metabolic acidosis\". Hypoperfusion followed by reperfusion increases the risk of \"Intraventricular hemorrhage\" (1VH). Clinical features :- Hyperdynamic circulation A wide pulse pressure (> 25 mm Hg), prominent precordial pulsations and \"bounding pulses\"Q. An ejection systolic murmurQ heard best at the 2nd left parasternal area. Metabolic acidosis not attributable to hypoperfusion and sepsis. Deteriorating respiratory status on day 3-4 after a period of relative stability. Increasing ventilator requirements and recurrent apneas. Unexplained CO2 retention, fluctuating F102 requirements.", "cop": 4, "opa": "Bounding pulses", "opb": "Pulmonary hemorrhage", "opc": "Necrotising enterocolitis", "opd": "CO, washout", "subject_name": "Pediatrics", "topic_name": null, "id": "682733bd-e6e4-4133-920f-28444e7e1cf3", "choice_type": "single"} {"question": "Phocomelia is", "exp": "Phocomelia is absence of long bones It occurs due to thalidomide consumption during pregnancy", "cop": 1, "opa": "Absence of long bones", "opb": "Absence of brain", "opc": "Reduplication of bones", "opd": "Absence of hea", "subject_name": "Pediatrics", "topic_name": "Musculoskeletal disorders", "id": "f06ba070-7920-4cfc-813a-f4b4cb36515a", "choice_type": "single"} {"question": "Nephrotic range of proteinuria is", "exp": "Nephrotic range of proteinuria is > 40 mg/m2/hr or  > 1 gm/m2/24hrs.", "cop": 2, "opa": "> 30 mg/m2/hr", "opb": "> 40 mg/m2/hr", "opc": "> 2 gm/m2/24hrs", "opd": "> 4 gm/m2/24hrs", "subject_name": "Pediatrics", "topic_name": null, "id": "2adfbaf1-6779-4f97-a11f-a35f82ab5157", "choice_type": "single"} {"question": "Grisel&;s syndrome is", "exp": "Grisel's syndromeSpontaneous subluxation of the atlantoaxial joint following peri-pharyngeal inflammation is known as Grisel's syndrome Children are most frequently affected and classically have toicollis associated with neck stiffness or pain upon neck movement(Refer: Nelson's Textbook of Pediatrics, SAE, 1st edition, pg no. 3297, 3298)", "cop": 2, "opa": "Glucosaminidase deficiency", "opb": "Atlanto-axial joint dislocation", "opc": "Hexosaminidase deficiency", "opd": "L4-L5 displacement", "subject_name": "Pediatrics", "topic_name": "All India exam", "id": "d40e323f-8d5b-4930-8406-a5a00516f513", "choice_type": "single"} {"question": "Very low birth weight child weights", "exp": "Ans. (b) <1500 gm", "cop": 2, "opa": "1000 gm", "opb": "<1500 gm", "opc": "<2500 gm", "opd": "<750 gm", "subject_name": "Pediatrics", "topic_name": "Newborn Infant - Physical Examination", "id": "8c6a5d3f-155a-409e-94e9-9ca8e2450224", "choice_type": "single"} {"question": "Not a feature of Cystic fibrosis among the following is", "exp": "Azoospermia is seen in Cystic fibrosis, not Oligospermia.", "cop": 3, "opa": "Delayed passage of Meconium", "opb": "Steatorrhea", "opc": "Oligospermia", "opd": "Late onset Diabetes", "subject_name": "Pediatrics", "topic_name": null, "id": "7ed8650d-7e7b-4448-a6e6-53d590c38ed8", "choice_type": "single"} {"question": "Treatment of choice in 11 years old children with hepatitic C infections", "exp": "(Interferon) (216- Ghai 6th) (574- CMDT-8) (195-Ghai 7th)* Combined therapy with interferon a2 and ribavirin have yielded better results* Treatment of acute hepatitis C with peg interferon or interferon a for 6-24 weeks appreciably decreases the risk of chronic hepatitis* Ribavirin may be added if HCV DNA fails to clear after 3months of peg interferon or interferone a", "cop": 2, "opa": "Vaccine", "opb": "Interferon", "opc": "Gamma - globulin", "opd": "Corticosteroids", "subject_name": "Pediatrics", "topic_name": "Infection", "id": "eda21737-b5fe-4fb7-8bef-f15ac1af31a3", "choice_type": "single"} {"question": "MCV (fl) in infant of 1 month of age is", "exp": "MCV at bih will be about 100-110 will reduce to 80-100 by 5 years. MCV. 0-1 month 1-3 months 3-6 months 6 months-1 year. 88-123 91-112 74-108 70-85. femtoliters Reference: GHAI Essential pediatrics, 8th edition", "cop": 2, "opa": "76-80", "opb": "80-100", "opc": "90-100", "opd": "101-125", "subject_name": "Pediatrics", "topic_name": "Hematology", "id": "fb29edd6-31eb-41e1-873d-7359d3623919", "choice_type": "single"} {"question": "Sapropterin is used in the treatment of", "exp": "Sapropterin, a synthetic form of tetrahydrobiopterin, which acts as a cofactor in patients with residual PAH activity, is approved by FDA to reduce phenylalanine levels in PKU. Galactosemia- Avoid milk & milk products. Hereditary Fructose Intolerance- Complete elimination of sucrose & fructose drom diet. Gaucher's disease- Enzyme replacement therapy.", "cop": 1, "opa": "Phenylketonuria", "opb": "Hereditary Fructose Intolerance", "opc": "Galactosemia", "opd": "Gaucher's disease", "subject_name": "Pediatrics", "topic_name": "Disorders of Amino acid Metabolism", "id": "3274a89d-1580-4759-a97d-57b2740b0f30", "choice_type": "single"} {"question": "Treatment for childhood hypothyroidism is with", "exp": "Levothyroxine is used for the treatment of childhood hypothyroidism.In case of congenital hypothyroidism,treatment should be staed immediately after diagnosis.The initiation dose is 10-15 microgram/kg/day.T4 & TSH are expected to normalise over one week and one month,respectively,with this treatment.Lifelong thyroid replacement is required for most cases. In acquired hypothyroidism,treatment should be gradual.A dose of 100 microgram/m2/day is recommended.Levothyroxine should be given empty stomach in the morning.Followup should be done every 3 months during the first 2 years of therapy and six monthly thereafter.Most children require lifelong therapy. Reference:Essential pediatrics-Ghai,8th edition,page no:517,518.", "cop": 3, "opa": "T4", "opb": "T3", "opc": "Levothyroxine", "opd": "TSH", "subject_name": "Pediatrics", "topic_name": "Childhood tumors", "id": "61f486f8-1661-4948-a889-c9abaf6137e7", "choice_type": "single"} {"question": "Commonest location for Craniopharyngioma is", "exp": "Commonest site of craniopharyngioma is Supra sellar region . Craniopharyngiomas are primarily suprasellar tumors (75%) while a small intrasellar component is present in 20-25% of cases . Purely intrasellar location is quite uncommon (<5%), and may be associated with the expansion of the pituitary fossa . Larger tumors can extend in all directions, frequently distoing the or compressing the . Occasionally, craniopharyngiomas appear as intraventricular, homogeneous, soft-tissue masses without calcification (papillary subtype). The third ventricle is a paicularly common location. Reference: GHAI Essential pediatrics, 8th edition", "cop": 2, "opa": "Intrasellar", "opb": "Suprasellar", "opc": "Intraventricular", "opd": "Intracerebral", "subject_name": "Pediatrics", "topic_name": "Central Nervous system", "id": "9b9ae42c-391d-40da-b95f-4c7f842fe180", "choice_type": "single"} {"question": "A child draws circle at", "exp": "

. Developmental milestones:- GROSS MOTOR DEVELOPMENT: 2 months: Holds head in plane of rest of the body when held in ventral suspension. In prone position in bed, the chin lifts momentarily. 3 months:lift head above the plane of the body. Head control stas by 3 months and fully developed by 5 months. 4 months:Remain on forearm suppo if put in prone position, lifting the upper pa of the body off the bed. 5 months: Rolls over. 6 months:sit in tripod fashion. 8 months: sits without suppo., crawling 9 months: Takes a few steps with one hand held. Pulls to standing and cruises holding on to furniture by 10 months. 10 months: creeps 12 months:creeps well, walk but falls, stand without suppo. 15 months: walks well, walks backward/ sideways pulling a toy. May crawl upstairs. 18 months: Runs, walks upstair with one hand held. Explores drawers 2 years: walk up and downstairs, jumps. 3 years : rides tricycle, alternate feet going upstairs. 4 years: hops on one foot, alternate feet going downstairs. 5 years:skips FINE MOTOR DEVELOPMENT:- 2 months- eyes follow objects to 180 deg. 3 months-Grasp reflex disappears and hand is open most of the time. 4 months- Bidextrous approach( reaching out for objects with both hands). 6 months- Unidextrous approach( Reach for an object with one hand). 8 months- radial grasp sta to develop. Turns to sound above the level of ear. 9 months- immature pincer grasp, probes with forefinger. 12 months-Unassisted pincer grasp. Releases object on request.Uses objects predominantly for playing, not for mouthing. Holds block on each hand and bang them together. 15 months- imitate scribbling , tower of two blocks 18 months- scribbles, tower of 3 blocks.turn pages of a book, 2-3 at a time. 2 years- tower of 6 blocks, veical and circular stroke. 3 years-Tower of 9 blocks, dressing and undressing with some help, can do buttoning.copies circle. 4 years- copies cross, bridge with blocks 5 years- copies triangle, gate with blocks. SOCIAL AND ADAPTIVE MILESTONES: 2 months: social smile(smile after being talked to).watches mother when spoken to and may smile. 3 months:Recognizes mother, anticipates feeds. 4 months: Holds rattle when placed in hand and regards it . Laughs aloud. Excited at the sight of food. 6 months:recognizes strangers, stranger anxiety . Enjoy watching own image in mirror, shows displeasure when toy pulled off. 9 months:waves bye bye 12 months:comes when called, plays simple ball game.kisses the parent on request. Makes postural adjustments for dressing. 15 months:jargon, stas imitating mother. 18 months: copies parents in tasking, dry by day, calls mother when he wants potty, points to three pas of body on request. 2 years: ask for food, drink, toilet, pulls people to show toys. 3 years:shares toys, know fullname and gender, dry by night. 4 years:Plays cooperatively in a group, goes to toilet alone, washes face, brushes teeth. Role play . 5 years:helps in household task , dresses and undresses. LANGUAGE MILESTONES: 1 month: Ales to sound. 2 month:respond to sound by stale or quitening to a smooth voice. 3 months: babbles when spoken to. Makes sounds (ahh,coos, ) laughs. 4 months: laughs aloud. 6 months: monosyllables 9 months: understands spoken words, bisyllables. 12 months: 1-2 words with meaning. 18 months: vocabulary of 10 words. Can name one pa of body. 2 years: 3 word simple sentences 3 years:asks questions, knows full name and gender. 4 years: says songs or poem, tells story, knows three colours. 5 years: ask meaning of words. {Reference: GHAI Essential pediatrics, eighth edition}", "cop": 4, "opa": "12 months", "opb": "24 months", "opc": "30 months", "opd": "36 months", "subject_name": "Pediatrics", "topic_name": "Growth and development", "id": "37905b25-3a39-4662-b3f1-5cd5cd46991f", "choice_type": "single"} {"question": "Birth length is double at the age of", "exp": "D. i.e. (4 years) (4 Ghai 6th) (6 - Ghai 7th)WeightLength or heightHead circumference*birth - about =3 kg*birth50 cm*Birth34 cm5 month(Doubles)6 kg1 years75 cm1 years45 cm1 years9 kg2years85 cm2years47 cm2 years12 kg3 years95 cm 3 years14 kg4 years(doubles)100 cm3 years49 cm4 years16 kg 4 years50 cm* A useful guide for normal range is target height + 7 cm for girls and target height + 8.5 cm for boys", "cop": 4, "opa": "1 years", "opb": "2 years", "opc": "3 years", "opd": "4 years", "subject_name": "Pediatrics", "topic_name": "Growth, Development, and Behavior", "id": "cedc5f48-d74b-4848-bbf6-5772f264b4f2", "choice_type": "single"} {"question": "Most common indication of liver transplant in children is", "exp": "Biliary atresia is the most common indication for liver transplantation in India, followed by metabolic and inborn disorders.", "cop": 2, "opa": "Chronic Viral hepatitis", "opb": "Biliary atresia", "opc": "Tyrosinemia", "opd": "Von Gierce disease", "subject_name": "Pediatrics", "topic_name": null, "id": "acf56383-97de-400e-85a5-e0bc9a1e6e3c", "choice_type": "single"} {"question": "Most common gene involved in Noonan syndrome", "exp": "NOONAN SYNDROME: - Autosomal dominant, Normal karyotype - MC gene involved --> PTPN 11 - Seen in both boys & girls --> FEILE - FEATURES: Sho stature webbed neck Anti-mongoloid slant eyes (Opposite of Down syndrome) Cubitus valgus Clinodactyly, Cryptorchidism cardiac anomalies: Supra valvular Pulmonary Stenosis -->, HOCM, ASD Presence of Intellectual disability", "cop": 2, "opa": "PTCN 11", "opb": "PTPN 11", "opc": "PTPN 22", "opd": "PTCN 22", "subject_name": "Pediatrics", "topic_name": "Other hea diseases in children", "id": "5041af6f-2dee-4e57-ad4a-f8291de7f3d9", "choice_type": "single"} {"question": "The dose of betamethasone in prenatal to prevent respiratory distress syndrome is", "exp": "Betamethasone is the antenatal coicosteroid of choice and the recommendation is to give 2 doses of 12mg intramuscularly 24 hrs apa to all pregnant women in preterm labour between 24 and 35 wks irrespective of whether the membranes are intact or not. This is to reduce the rate of respiratory distress syndrome and intraventricular hemorrhage. The other option is dexamethasone 6mg every 12hrs for 4 doses. Reference : page 194 Textbook of Obstetrics Sheila Balakrishnan 2nd edition", "cop": 2, "opa": "6 mg", "opb": "12 mg two doses 24 hours apa", "opc": "6 mg every 12 hours", "opd": "4 mg stat", "subject_name": "Pediatrics", "topic_name": "New born infants", "id": "b2d6c8f2-7f5d-47a6-80a2-22590dd29f32", "choice_type": "single"} {"question": "Neonatal sepsis is commonly caused by", "exp": "(E. coli) (161-G) (136-Ghai 7th)* Neonatal sepsis is the single most important cause of neonatal deaths in the community accounting for over half of them.* Most cases of neonatal sepsis in the community are caused by Escherichia-coli and staphylococcus aureus. In hospitals, Klebsiella pneumonia is also a common organism", "cop": 2, "opa": "b-haemolytic streptococci", "opb": "E. coli", "opc": "H. influenzae", "opd": "Listeria", "subject_name": "Pediatrics", "topic_name": "New Born Infants", "id": "26789ae1-170e-4155-9a0f-271735fb6c2b", "choice_type": "single"} {"question": "Severe Acute Malnutrition is defined as", "exp": "Severe acute malnutition in children of 6 months - 5 year is defined as \na) Weight / Height < - 3SD\nb) Mid - upper arm circumference < 11.5 cm\nc) Presence of bipedal edema", "cop": 3, "opa": "Weight / Height < - 2SD", "opb": "Mid - Upper arm circumference < 12.5 cm", "opc": "Presence of bipedal edema", "opd": "Weight / age < - 3SD", "subject_name": "Pediatrics", "topic_name": null, "id": "4c970812-44c7-4d12-a5b7-4d8816f93633", "choice_type": "single"} {"question": "Ω Omega shaped epiglottis is seen in", "exp": "On Laryngoscopy, Omega shaped epiglottis is seen in laryngomalacia.", "cop": 3, "opa": "Acute epiglottitis", "opb": "Croup", "opc": "Laryngomalacia", "opd": "Bronchiolitis", "subject_name": "Pediatrics", "topic_name": null, "id": "50b81105-2713-4fec-85b9-c149959b1601", "choice_type": "single"} {"question": "Auer rods are specific for", "exp": "-Auer rods are Needle-like azurophilic fusiform inclusions in cytoplasm of myeloblasts -Auer rods are most reliable morphological feature of AML -Myeloblasts Stain +ve with MPO and Sudan Black B", "cop": 1, "opa": "Acute myeloid leukemia", "opb": "Acute lymphocytic leukemia", "opc": "Chronic lymphocytic leukemia", "opd": "Hodgkin's lymphoma", "subject_name": "Pediatrics", "topic_name": "Neoplastic disorders of WBCs", "id": "d695d0d3-78db-4fba-b896-bb268da062a5", "choice_type": "single"} {"question": "EMG is least useful for diagnosis of", "exp": "EMG is leastly used in cerebral palsy since the cause is in the brain and not in the muscles", "cop": 1, "opa": "Cerebral palsy", "opb": "Spinal muscular atrophy", "opc": "Charot-Marie-Tooth disease", "opd": "Myasthenia gravis", "subject_name": "Pediatrics", "topic_name": "Musculoskeletal disorders", "id": "c84e4ac4-6ecb-4c9f-b69e-4427e9cc404d", "choice_type": "single"} {"question": "The most common cause of severe obstructive uropathy in children is", "exp": "Posterior Urethral ValvesThe most common cause of severe obstructive uropathy in children is posterior urethral valves, affecting 1 in 8,000 boys. The urethral valves are tissue leaflets fanning distally from the prostatic urethra to the external urinary sphincter.Valves are of unclear embryologic origin and cause varying degrees of obstruction. Ref: Nelson Textbook of Pediatrics; 20th edition; Page no: 2574", "cop": 1, "opa": "Posterior urethral valves", "opb": "Phimosis", "opc": "Urethral atresia", "opd": "Bladder neck obstruction", "subject_name": "Pediatrics", "topic_name": "Miscellaneous", "id": "36fd3187-0ae3-47e8-80e8-f0836db3fe90", "choice_type": "single"} {"question": "Preventable causes of Mental Retardation are", "exp": "Cretinism is congenital iron deficiency syndrome which can be preventable by intake in required amounts. Reference: GHAI Essential pediatrics, 8th edition", "cop": 3, "opa": "Downs", "opb": "Phenylketonuria", "opc": "Cretinism", "opd": "Cerebral palsy", "subject_name": "Pediatrics", "topic_name": "Central Nervous system", "id": "ef15f52d-dc57-45ca-8c4a-6b53211213a8", "choice_type": "single"} {"question": "NOT a feature of Atrial Septal defect among the\nfollowing", "exp": "Left Atrial hypertrophy is never seen in ASD.", "cop": 1, "opa": "Complications include Left Atrial hypertrophy", "opb": "Complications due to ASD are of late onset", "opc": "Most Common Congenital heart disease presenting in Adult age", "opd": "Wide fixed Split in 2nd heart sound", "subject_name": "Pediatrics", "topic_name": null, "id": "2684e3a2-908b-4611-8d66-f83b02aefb7a", "choice_type": "single"} {"question": "Treatment of multiple carboxylase deficiency", "exp": "Treatment of multiple carboxylase or biotinidase deficiency is biotin.Oral dose is 10-40 mg/kg/day Reference:Essential pediatrics-Ghai,8th edition,page no:650", "cop": 1, "opa": "Biotin", "opb": "Pyridoxine", "opc": "Thiamine", "opd": "Folic acid", "subject_name": "Pediatrics", "topic_name": "Metabolic disorders", "id": "048df25c-98b5-41a1-9c6d-3e714ce4ef73", "choice_type": "single"} {"question": "The best indicator of acute malnutrition is", "exp": "Severe acute malnutrition is defined by a very low weight for height (below-3z scores of the median WHO growth standards), by visible severe wasting, or by the presence of nutritional edema.(Refer: Nelson's Textbook of Pediatrics, SAE, 1st edition, pg no. 301)", "cop": 1, "opa": "Weight for height", "opb": "Weight for age", "opc": "Height for age", "opd": "BMI <2SD", "subject_name": "Pediatrics", "topic_name": "All India exam", "id": "a72d3868-e530-413a-b2db-561d92980c76", "choice_type": "single"} {"question": "Skin change in newborn that disappear spontaneously", "exp": ".Harlequin color change is a cutaneous condition seen in newborn babies characterized by momentary red color changes of half the child, sharply demarcated at the body&;s midline. This transient change occurs in approximately 10% of healthy newborns. It is seen usually between two and five days of bih. Reference: GHAI Essential pediatrics, 8th edition", "cop": 1, "opa": "Harlequins skin change", "opb": "Mangolian spots", "opc": "Erytherma toxicum", "opd": "Lymphoma", "subject_name": "Pediatrics", "topic_name": "New born infants", "id": "ea9ffee5-8a0e-49d1-8b97-df9863fd8ed8", "choice_type": "single"} {"question": "Hutchisons traid is seen", "exp": "* Hutchinson&;s triad is seen in congenital syphilis. *it includes interstitial keratitis, deafness and Hutchinson&;s teeth ( central incissors with peg shaped, notched appearance ). Reference : IADVL&;s concise textbook of dermatology, first edition, pg no: 126", "cop": 2, "opa": "Primary syphilis", "opb": "Congenital syphilis", "opc": "Secondary syphilis", "opd": "Teiary syphilis", "subject_name": "Pediatrics", "topic_name": "Infectious disease", "id": "f351aa69-dbc3-4fca-a98a-1be920e89b24", "choice_type": "single"} {"question": "Enzyme deficient is Von Gierke disease is", "exp": "Enzyme deficient in Von Gierke disease is Glucose 6 - phosphatase.", "cop": 4, "opa": "Acid maltase", "opb": "Alpha 1, 4 glucosidase", "opc": "Glucose phosphorylase", "opd": "Glucose 6 - phosphatase", "subject_name": "Pediatrics", "topic_name": null, "id": "79cf0a8f-0acd-4925-adbb-bafebb7d9525", "choice_type": "single"} {"question": "Recent marker for Downs syndrome is", "exp": "Inhibin A is a protein secreted by the ovary, and is designed to inhibit the production of the hormone FSH by the pituitary gland. The level of inhibin A is increased in the blood of mothers of fetuses with Down syndrome. Reference: GHAI Essential pediatrics, 8th edition", "cop": 3, "opa": "HCG", "opb": "Alpha fetoprotein", "opc": "Inhibin", "opd": "Estriol", "subject_name": "Pediatrics", "topic_name": "Genetic and genetic disorders", "id": "1c9503ba-14d8-459f-8509-d468f171815a", "choice_type": "single"} {"question": "Between 3 to 4months of age, Infant weight increases approximately around", "exp": "Between 3 and 4 months of age, the rate of growth slows to approximately 20 g/day.By age 4 months, bih weight is doubled.(Refer: Nelson; Chapter 10; pg no. 69)", "cop": 1, "opa": "20 g/d", "opb": "40 g/d", "opc": "50 g/d", "opd": "60 g/d", "subject_name": "Pediatrics", "topic_name": "All India exam", "id": "d0243de2-7811-4783-86c3-304b50405a16", "choice_type": "single"} {"question": "Most common cardiac anomoly associated with Turner's syndrome among the following is", "exp": "Most common cardiac anomoly associated with Turner's syndrome is Bicuspid Aortic valve > Coarctation of Aorta.", "cop": 3, "opa": "Aortic stenosis", "opb": "Ventricular septal defect", "opc": "Coarctation of Aorta", "opd": "Pulmonary stenosis", "subject_name": "Pediatrics", "topic_name": null, "id": "f77d5a89-9fc9-459c-a584-a38fe00d59b6", "choice_type": "single"} {"question": "A most common cause of Neonatal Hypothyroidism is", "exp": "Iodine deficiency is the commonest cause of congenital hypothyroidism in certain parts of India, while thyroid dysgenesis is the most common aetiology in non-endemic areas (75% of all cases).", "cop": 1, "opa": "Throid dysgenesis", "opb": "Transplacental maternal thyroid antibodies", "opc": "Inherited genetic disorders", "opd": "Decreased thyroglubin", "subject_name": "Pediatrics", "topic_name": null, "id": "cf7d1a56-96da-42ef-947e-b9a220e4fd24", "choice_type": "single"} {"question": "Select the most common clinical sign or symptom for diagnosis Tuberous sclerosis", "exp": "Tuberous sclerosis, an autosomal dominant condition, can result in severe mental retardation and seizures. Infantile spasms, a hypsarrhythmic EEG pattern, hypopigmented lesions (ash-leaf spots), cardiac tumors, sebaceous adenomas, a shagreen patch (a roughened, raised lesion over the sacrum), and calcifications on the CT scan are all features of this condition. No specific treatment is available.", "cop": 2, "opa": "A 6-month-old child with blindness on the same side as a large facial lesion", "opb": "An infant with infantile spasms, a hypsarrhythmic EEG pattern, and ash-leaf depigmentation on her back", "opc": "An 18-year-old patient with a history of fractures and optic gliomas, who now has developed a malignant schwannoma", "opd": "A 2-year-old child with multiple episodes of skin infection and failure to thrive", "subject_name": "Pediatrics", "topic_name": "Musculo Skeletal Disorders", "id": "922878af-0601-45df-ab41-109f33437220", "choice_type": "single"} {"question": "Amount of Protein present in 100ml of breast milk is", "exp": "Amount of Protein present in 100ml of breast milk is: 1.1 g/dl", "cop": 2, "opa": "2.2 g", "opb": "1.1 g", "opc": "0.55g", "opd": "3.3g", "subject_name": "Pediatrics", "topic_name": "JIPMER 2019", "id": "83a46f0a-6314-48e8-b6a9-846317eef4cd", "choice_type": "single"} {"question": "A five year old child presents to the emergency depament with burns. The burn area corresponding to the size of his palm is equal to", "exp": "Ans is 'a' i.e., 1% BSA Estimation of body surface area (BSA) of burn o Rough estimate of body surface area of burn can be estimated by following rules. Rule of Palm (Palmar method) o It can be used in Children as well as in adults. o It is used for small burns ( o The palm of patients hand represents I% of BSA. Palm is the area of hand from wrist crease to the finger crease. o Entire burned hand represents 2.5% of BSA: 1 /% for palm; 1% for dors um of the hand (excluding fingers);0. 5% for the fingers. Rule of 9 o It can be used in adults and children older than 14 years. o It is used for larger surface area burns. o Estimates the body surface area of an adult in multiples of 9. o An adult who has been burned, the percent of the body involved can be calculated as follows: Head and Neck = 9% Anterior chest = 9% Anterior abdomen =9% Upper back = 9% Lower back = 9% Right upper extremity (Anterior + Posterior) = 9% Left upper extremity (Anterior + Posterior) = 9% Right lower extermity, Anterior = 9% Right lower extremity, Posterior = 9% Left lower extremity, anterior = 9% Left lower extremity, Posterior = 9% Perenium = 1% o Percent of the burn in a child can be calculated as follows: Head and neck = 9% Anterior trunk = 18% Posterior trunk = 18% Upper extremity (Right) = 9% Upper extremity (Left) = 9% Lower extremity (Right) = 18% Lower extremity (Left) = 18% Perenium = 1%", "cop": 1, "opa": "1% BSA", "opb": "5% BSA", "opc": "10% BSA", "opd": "20% BSA", "subject_name": "Pediatrics", "topic_name": null, "id": "d284e1a0-686c-4742-bc38-17e9798a39b1", "choice_type": "single"} {"question": "3 months old child with typical mewing cry and congenital hea disease, chromosome abnormality", "exp": "Ans. is 'b' i.e., 5p Cri-du-chat syndrome. A high-pitched cry like a cat, giving the syndrome its name. Genetic studies have confirmed that this characteristic cry results from the deletion at 5p15 The main features are hypotonia, sho stature, characteristic cry, microcephaly with protruding metopic suture, moonlike face, hypeelorism, bilateral epicanthic folds, high arched palate, wide and flat nasal bridge, and mental retardation. Cardiac abnormalities including ventricular septal defect, atrial septal defect, patent ductus aeriosus and Fallot's tetralogy.", "cop": 2, "opa": "5g", "opb": "5p", "opc": "4p", "opd": "9p", "subject_name": "Pediatrics", "topic_name": null, "id": "33cf1750-eded-4842-98a4-36969615e126", "choice_type": "single"} {"question": "Most common cause of lower respiratory tract infection in 3 year old child is", "exp": "Ans. is 'c' i.e., Streptococcal pneumonia", "cop": 3, "opa": "Klebsella", "opb": "H-influenza", "opc": "Streptococcal pneumonia", "opd": "Staphe aureus", "subject_name": "Pediatrics", "topic_name": null, "id": "c5c85fb3-b838-4e07-b572-0fd85f5a0ebc", "choice_type": "single"} {"question": "A 5 year lod child is assessed to have developmental age of one year. His developmental patient would be", "exp": null, "cop": 3, "opa": "100", "opb": "80", "opc": "20", "opd": "60", "subject_name": "Pediatrics", "topic_name": null, "id": "b349b664-7fcb-44bb-a1ed-53c4d3f7a3a2", "choice_type": "single"} {"question": "Most common cause of ESRD in children", "exp": "This is why kidney failure is also called end-stage renal disease, or ESRD for sho. Diabetes is the most common cause of ESRD. Common causes of acute renal failure include: Hea attack. Illegal drug use and drug abuse. Not enough blood flowing to the kidneys. Urinary tract problems Reference: GHAI Essential pediatrics, 8th edition", "cop": 2, "opa": "ADPKD", "opb": "Nephronopthisis", "opc": "Medullary cystic disease", "opd": "ARPKD", "subject_name": "Pediatrics", "topic_name": "Urinary tract", "id": "cc707bf8-c1a7-43a6-be17-648375645e38", "choice_type": "single"} {"question": "Most common organism causing meningitis in a 1 year old child", "exp": "Streptococcus pneumonia is the most common organism causes meningitis even in the 1 year old child. Reference: GHAI Essential pediatrics, 8th edition", "cop": 1, "opa": "Streptococcus pneumoniae", "opb": "H. influenza", "opc": "Listeria", "opd": "Neisseria meningitidIs", "subject_name": "Pediatrics", "topic_name": "Central Nervous system", "id": "3b289823-9143-40e7-818d-93d16fcdc323", "choice_type": "single"} {"question": "Most common cause of hea failure in infancy is", "exp": "Most common cause of hea failure in infancy is CHD. Ref : Ghai essential of pediatrics, eighth edition, p.no:397", "cop": 4, "opa": "Myocarditis", "opb": "Rheumatic fever", "opc": "Cardiomyopathy", "opd": "Congenital hea disease", "subject_name": "Pediatrics", "topic_name": "C.V.S", "id": "589ef2d9-916b-48d3-831b-21565c49c717", "choice_type": "single"} {"question": "The karyotype of a patient with androgen insensitivity Syndrome is", "exp": "Karyotype-46 ,XY. Normal genitK appearance -SRY deletion ,SF1 defect,Gonadal dysgenesis ,Severe StAR defect ,Complete androgen insensitivity syndrome. Genital ambiguity -Testicular dysgenesis ,Steroidogenic defects,Paial androgen insensitivity syndrome ,Aromatase deficiency. * Chromosomal study: Karyotype is 46, XY. * In infants and children, stimulation by hCG before measurement of hormones needed. * Serum testosterone is normal (or higher) * Serum E2 level is high normal for males * Serum LH level is normal or slightly elevated * FSH is normal. * USG: --Ovaries, uterus, upper 2/3rd of vagina and tubes are absent Reference: OP Ghai ,essential.paediatrics ,8 th edition, page no- 538,table 17.32 .", "cop": 2, "opa": "46XX", "opb": "46XY", "opc": "47XXY", "opd": "45X0", "subject_name": "Pediatrics", "topic_name": "Endocrinology", "id": "ff5eb061-ca6a-4082-a70f-0b467b06a0c1", "choice_type": "single"} {"question": "Automatism is seen in", "exp": "A complex paial seizure is a type of seizure that arises in one lobe of the brain, rather than the whole brain. The seizure affects people&;s awareness and may cause them to lose consciousness . A complex paial seizure stas focally within the brain and causes impairment of consciousness. This definition is based on both clinical and electroencephalographic (EEG) data. A complex paial seizure is most commonly a manifestation of temporal lobe epilepsy, but the term is so broadly defined (le, as any focal seizure with Impairment of consciousness) that it is very nonspecific. For this reason, many clinicians make a point of distinguishing between temporal and extratemporal complex paial seizures. Complex paial seizures are paial seizures that are associated with an alteration of consciousness ( Complex-Paial Seizures with Automatisms). The term psychomotor describes a paicular semiology characterized by automatisms and impairment of consciousness, usually of temporal lobe origin. Automatisms: Automatisms are nonpurposeful, stereotyped, and repetitive behaviors that commonly accompany complex paial seizures (In the sem lologic classification, they define automotor seizures). The behavior Is Inappropriate for the situation. Verbal automatisms range from simple vocalizations, such as moaning, to more complex, comprehensible, stereotyped speech. The most common automatisms, at least In temporal lobe epilepsy, are oral (eg, lip smacking, chewing, swallowing) and manual (eg, picking. fumbling, patting). Unilateral manual automatisms accompanied by contralateral arm dystonia usually Indicates seizure onset from the cerebral hemisphere !psilateral to the manual automatisms. Automatisms can also be more elaborate, coordinated movements Involving bilateral extremities. Bizarre automatisms, such as alternating limb movements, right-to-left head rolling, or sexual automatisms, may occur with frontal lobe seizures. Reference: GHAI Essential pediatrics, 8th edition", "cop": 2, "opa": "Paial seizures", "opb": "complex paial seizures", "opc": "Infantile spasms", "opd": "Absence seizures", "subject_name": "Pediatrics", "topic_name": "Central Nervous system", "id": "5a675843-7d4a-4933-8c72-5f74e6a1b297", "choice_type": "single"} {"question": "3 months old baby with fever and respiratory rate 60, irritable, and well feed, no stridor, no chest indrawing, no convulsions. Diagnosis", "exp": "The above given history is suggestive of pneumonia as there is increased respiratory rate. New ARI classification (2014) by WHO: Diagnosis Clinical features Pneumonia Fast breathing and/or chest indrawing Severe pneumonia or Very severe disease General danger signs present If the child Is: Fast breathing Is: 2 months up to 12 months 50 breaths per minute or more 12 months up to 5 years 40 breaths per minute or more", "cop": 1, "opa": "Pneumonia", "opb": "Severe pneumonia", "opc": "Very sever pneumonia", "opd": "No Pneumonia", "subject_name": "Pediatrics", "topic_name": "FMGE 2019", "id": "626164b8-6137-4061-843b-04314e24e821", "choice_type": "single"} {"question": "A child is brought by mother with HO massive hemetemesis with HO drug intake previously with NSAIDS and on Rx.Associated with moderate splenomegaly diagnosis is", "exp": "Massive hematemesis,moderate splenomegaly are features of oesophageal varices Ref : ESSENTIAL PEDIATRICS,O.P.GHAI.,PG NO:290, 7th edition", "cop": 1, "opa": "Oesophageal varices", "opb": "Duodenal ulcer", "opc": "Drug induced gastritis", "opd": "Peptic ulcer", "subject_name": "Pediatrics", "topic_name": "Gastrointestinal tract", "id": "496d8590-38f5-486c-bd9e-07c91707a2bb", "choice_type": "single"} {"question": "Flag hair sign is seen in", "exp": "Ans. (a) KwashiorkorRef: Nelson's Pedia, 19th ed. Ch 670# kwashiorkorDisease due to decreased protein and caloric intake.Clinical Features are:# Hypoalbuminemia which leads to pedal edema# Hepatomegaly- Hallmark feature of kwashiorkor# Skin: Sun-exposed skin is relatively spared, as are the feet and dorsal aspects of the hands; Hypo pigmentation# Hair and Nail: Nails are thin and soft, and hair is sparse, thin, and depigmented, sometimes displaying a flag sign of alternating light and dark bands that reflect alternating periods of adequate and inadequate nutrition.# Apathy# Lack of appetite* PELLAGRA:This presents with edema, erythema, and burning of sun-exposed skin on the face, neck, and dorsal aspects of the hands, forearms, and feet* SCURVY (VITAMIN C OR ASCORBIC ACID DEFICIENCY): This presents initially with follicular hyperkeratosis and coiling of hair on the upper arms, back, buttocks, and lower extremities. Perifollicular erythema and hemorrhage, swollen, erythematous gums; stomatitis; and subperiosteal hematomas are also seen.", "cop": 1, "opa": "Kwashiorkor", "opb": "Marasmus", "opc": "Scurvy", "opd": "Pellagra", "subject_name": "Pediatrics", "topic_name": "Nutrition, Food Security, and Health", "id": "3b76dd11-d921-4dbb-a85f-01ca8d802e44", "choice_type": "single"} {"question": "Most common benign tumours during infancy is", "exp": "An infantile hemangioma is one of the most common benign tumors of infancy and occurs in approximately 5-10% of infants.They occur more frequently in females,premature and low bih weight infants. Reference: GHAI Essential pediatrics, 8th edition", "cop": 2, "opa": "Lymphangioma", "opb": "Hemangioma", "opc": "Cystic hygroma", "opd": "Lipoma", "subject_name": "Pediatrics", "topic_name": "Childhood tumors", "id": "508ea8fb-9159-4a66-bc01-4d8f533bf40a", "choice_type": "single"} {"question": "A new bom presented with Jaundice. Most diagnostic investigation of choice is", "exp": "(Total and Direct bilirubin) (171 - Ghair 6th)* Unconjugated Hyperbilirubinemia1. Clinical Jaundice appearing in the first 24 hours2. Increase in level of total bilirubin by more than 0.5 mg/dL /hour or 5mg/dL /24 hours3. Total bilirubin >15mg/dL (hyper bilirubinemia)4. Direct bilirubin >2.0mg /dL* Conjugated hyperbilirubinemia - is rare in newborn, conjugated hyper bilirubinemia is defined as a direct reacting fraction of >2mg/dL or >15% of the total bilirubin", "cop": 1, "opa": "Total and Direct bilirubin", "opb": "Conjugated bilirubin", "opc": "Serum. Bilirubin", "opd": "Uroporphyrin levels", "subject_name": "Pediatrics", "topic_name": "New Born Infants", "id": "b0165b96-bae0-4564-ae48-18d8c04899f4", "choice_type": "single"} {"question": "Brain abscess in Cyanotic hea disease is commonly located in", "exp": "Brain abscess in CHD is located mainly in frontal lobe. Ghai essential of pediatrics, eighth edition, p.no:411", "cop": 2, "opa": "Carebellar hemisphere", "opb": "Frontal lobe", "opc": "Temporal lobe", "opd": "Parietal lobe", "subject_name": "Pediatrics", "topic_name": "C.V.S", "id": "cffb0908-602b-48cc-afaf-11f8fc2a2b15", "choice_type": "single"} {"question": "Commonest cause of short stature in children", "exp": null, "cop": 4, "opa": "Under nutrition", "opb": "Hypothyroidism", "opc": "Familial short stature", "opd": "Constitutional delay", "subject_name": "Pediatrics", "topic_name": null, "id": "b2f4acdb-d947-4007-b8f7-43fbc2f53b79", "choice_type": "single"} {"question": "Minimal interval between two live vaccines administered by the oral route is", "exp": "Live vaccines administered by the oral route (Oral Polio vaccine-OPV; oral typhoid and Rota virus) are not believed to interfere with each other if not given simultaneously. These vaccines may be given at any time before or after each other. Live parenteral (injected) vaccines and live intranasal influenza vaccine should either be administered at the same visit or they should be seperated by atleast 4 weeks. Ref: Ghai essentials 9th edition Pgno: 190", "cop": 1, "opa": "Any time", "opb": "2 weeks", "opc": "4 weeks", "opd": "3 months", "subject_name": "Pediatrics", "topic_name": "All India exam", "id": "99be6250-8f74-4521-b450-3b4cade34103", "choice_type": "single"} {"question": "Investigation of choice in VUR", "exp": "c. Voiding cystourethrogram(Ref: Nelson's 20/e p 2562-2567, Ghai 8/e p 485-486)The most useful process for conclusive diagnosis of VUR is micturating cystourethrogram (MCU)MCU demonstrates the grade of reflux as well as urethral anatomy.Radionuclide imaging is sensitive at detecting reflux but does not demonstrate anatomic details", "cop": 3, "opa": "Ultrasound", "opb": "MRI", "opc": "Voiding cystourethrogram", "opd": "Cystoscopy", "subject_name": "Pediatrics", "topic_name": "Kidney", "id": "1d468343-c892-4f36-a8e4-e3f20347ad69", "choice_type": "single"} {"question": "Chromosomal anomalies more than 20% is associated with aEUR'", "exp": "Omphalocele \"Chromosomal abnormalities have been repoed in as many as 43% fetuses with omphalocele. The most common being 13, 18, 21 and Turner's syndrome\". Omphalocele Omphalocele is a midline defect of the anterior abdominal wall characterized by herniation of the abdominal visceral in to the base of the umbilical cord. Omphalocele is a defect of the umbilical ring which results from a failure of the two lateral abdominal wall folds to migrate and. fuse normally in the midline around the 3rd or 4th week of intrauterine life. The defect is characteristically located at the base of the umbilical cord and may contain abdominal and thoracic structures. The herniated visceral are included in a sac which is .formed internally by the peritoneum and externally by Whaon's jelly and the amnion. Omphalocele is associated with number of congenital anomalies Anomalies Gastroschisis Gastroschisis is a herniation of the abdominal viscera through a paraumibilical defect of the anterior abdominal wall. The defect is usually located at the right side of the cord inseion and comprises full thickness of the abdominal wall. The herniated organ is usually bowel and very rarely other structures are involved. In contrast to omphalocele - \"Gastrochisis is rarely associated with other congenital anomalies\". The most frequent associated anomalies are gastrointestinal in origin and are related to the same vascular embryologic problem that originates the gastrochisis. - Intestinal atresia or stenosis (7-30%) cardiac malformation seen in 8% cases. It is also associated with I.U.G.R.", "cop": 2, "opa": "Gastroschisis", "opb": "Omphalocele", "opc": "Spina Bifida", "opd": "Cleft Palate", "subject_name": "Pediatrics", "topic_name": null, "id": "8a64dfd6-6cbb-4b89-9e5f-12dd88a535cd", "choice_type": "single"} {"question": "Low bih weight is defined as", "exp": "Low bih weight (LBW) neonate- A neonate weighing less than 2500g at bih irrespective of the gestational age. Very low bihweight (VLBW) neonate- A neonate weighing leas than 1500g at bih irrespective of the gestational age. Extremely low bih weight (ELBW) neonate- A neonate weighing leas than 1000g at bih irrespective of the gestational age. Reference : page 125 Ghai Essential Pediatrics 8th edition", "cop": 1, "opa": "Weight < 2.5 kg", "opb": "Weight < 2.0 kg", "opc": "Weight < 1.5 kg", "opd": "Weight < 10 percentile for their gastational age", "subject_name": "Pediatrics", "topic_name": "New born infants", "id": "f8ab00a6-ca59-4865-ad15-3f61dac2ccca", "choice_type": "single"} {"question": "Umbilical cord becomes black in", "exp": "The cord becomes brownish black within 2 or 3 days after bihIt falls off in about 10-14 days(Refer: Nelson's Textbook of Pediatrics, SAE, 1st edition, pg no. 811)", "cop": 1, "opa": "2-3 days", "opb": "5-7 days", "opc": "7-10 days", "opd": "10-14 days", "subject_name": "Pediatrics", "topic_name": "All India exam", "id": "8ccccc44-01d8-4569-a737-dae7bfabf16d", "choice_type": "single"} {"question": "As per as latest resuscitation guidelines Umbilical cord clamping must be delayed for atleast ( in preterm )", "exp": "Umbilical cord clamping must be delayed for atleast 30 seconds (in term + preterm babies) Benefits of delayed cord clamping | Total body iron stores Expand blood volume | Incidence of anemia in neonates Note - If case of Bih asphyxia, cord should be clamped immediately NELSON states 30-60 seconds so, better response is 30 seconds as the value is common to our 2 references and the question mentions ''ATLEAST''.", "cop": 2, "opa": "20 seconds", "opb": "30 seconds", "opc": "40 seconds", "opd": "60 seconds", "subject_name": "Pediatrics", "topic_name": "Neonatal Resuscitation", "id": "cb422d25-1b68-44cc-8783-3fb86dbfcbbf", "choice_type": "single"} {"question": "Most common Enzyme Deficiency leading to childhood hypeension is", "exp": "Most common enzyme deficiency causing hypeension in pediatric child group is APPARENTMINERALOCOICOID EXCESS. Apparent mineralocoicoid excess (AME) is an autosomal recessive disorder caused by deficiency of 11 beta hydroxysteroid dehydrogenase 2 (11-beta HSD 2) enzyme which is encoded by a gene in chromosome 16q22. Congenital adrenal hyperplasia due to 11b-hydroxylase deficiency is a form of congenital adrenal hyperplasia (CAH) which produces a higher than normal amount of androgen,resulting from a defect in the gene encoding the enzyme steroid 11b-hydroxylase (11b-OH) which mediates the final step of coisol synthesis in the adrenal. 11b-OH CAH results in hypeension due to excessive mineralocoicoid effects. It also causes excessive androgen production both before and after bih and can virilize a genetically female fetus or a child of either sex. Reference: GHAI Essential pediatrics, 8th edition", "cop": 3, "opa": "17- Alpha hydroxylase", "opb": "21- Beta hydroxylase", "opc": "11- Beta hydroxylase", "opd": "3- Beta hydroxy steroid dehydrogenase", "subject_name": "Pediatrics", "topic_name": "Genetic and genetic disorders", "id": "11a63442-080e-4952-9dbd-7c17f9b04ab7", "choice_type": "single"} {"question": "One month old baby is referred for failure to thrive. On examination there are features of congestive cardiac failure. Femoral pulses are feeble compared to brachial pulses. The likely diagnosis is", "exp": "In coarctation of aoa there will be radiofemoral delay. Ghai essential of pediatrics, eighth edition, p.no:432", "cop": 1, "opa": "Coarctation of aoa", "opb": "Patent ductus aeriosus", "opc": "Congenital aoic stenosis", "opd": "Congenital aoa-iliac disease", "subject_name": "Pediatrics", "topic_name": "C.V.S", "id": "de199236-369f-4bf9-a78f-1fc0236c4a46", "choice_type": "single"} {"question": "ASD patient with murmur similar to MR and left axis detion of 40 degrees is having", "exp": "Floppy mitral valve REF: Pediatrics: A Concise Text - Text - SK Kabra, RN Srivastava Page 135 Repeat in December 2011 The murmur is the clinching point here as the patient is having ostium primum type of ASD with MR and Ostium primum defect alone cannot produce such murmur and in question it is already mentioned that the patient is having ASD, Floppy mitral valve should be the answer. \"Presence of pansystolic murmur of mitral regurgitation (MR) in a patient with ASD suggests the presence of ostium primum defect with cleft mitral laeflet or floppy mitral valve with mitral valve prolapsed\" \"ECG shows right axis detion in ostium secundum ASD and extreme left axis detion in ostium primum ASD\" \"Clinical presentation of ostium primum is similar to ostium secundum except that there may be a mitral systolic murmur due to MR and ECG shows left axis detion\"", "cop": 4, "opa": "TGA", "opb": "Ostium secondum", "opc": "Ostium primum", "opd": "Floppy mitral valve", "subject_name": "Pediatrics", "topic_name": null, "id": "a00d5ec0-a315-4ef5-a4ad-9e79c40fcb37", "choice_type": "single"} {"question": "Drug of choice for Prophylaxis of Bronchialitis in a child with heart disease", "exp": "Treatment for Bronchiolitis in a child with co-morbidities is inhaled Ribavirin.\nDrug of choice for prophylaxis of Bronchialitis in a child with co-morbidities is Palivizumab.", "cop": 3, "opa": "Ribavirin", "opb": "Doxycycline", "opc": "Palivizumab", "opd": "Penicillin V", "subject_name": "Pediatrics", "topic_name": null, "id": "2b523b65-91cd-4e21-9e91-ebfc05fce3a1", "choice_type": "single"} {"question": "Earliest sign of Rickets is", "exp": "A. i.e. (Craniotabes) (82- Ghai 7th) (128 - Ghai 6th)RICKETSSCURVY* Craniotabes* is the earliest manifestations (felt like a ping pong ball)* Anterior fontanel is large and its closure is delayed beyond 18 months* Bossing of frontal and parietal bones* Rachitic rosaiy (costochondral junction become prominent dome shaped and semicircular)* Pigeon breast - sternum projects forward* Harrison's groove (Horizontal depression)* Eruption of primary teeth delayed* Moderate degree of scoliosis kyphosis or lordosis* Epiphysis of long bones are widened and broadening of wrists* Long bones of legs get deformed when the child starts bearing weight* Anterior bowing of legs, knock knee and coxa vera*** Pot- belly (Abdomen become protuberant)* Visceroptosis and lumber lordosis* Child is listless, anorexic, fretful and cries onbeing handled* Bones are tender* Hemorrhages occur under the periosteum of long bones* Gum bleeds are common* Petechiae may be seen over the skin* Costochondral junction is prominent and appears sharp and angular* Scorbutic rosary is attributed to separation of epiphysis of ribs and backward displacement of sternum* Wound healing is delayed* Anemia may co- exist* Metaphyseal flaring of the anterior ends of the ribs is characteristically seen in rickets*** **First change to appear is \"loss of normal zone ofprovisional calcification\" adjacent to metaphysis", "cop": 1, "opa": "Craniotabes", "opb": "Harrison's groove", "opc": "Rachitic rosary", "opd": "Pigeon breast", "subject_name": "Pediatrics", "topic_name": "Nutrition", "id": "e0a5cb7f-2b28-4417-a52c-4a27b949427c", "choice_type": "single"} {"question": "Most common cause of Congenital hypothyroidism is", "exp": "Most common cause of congenital hypothyroidism is thyroid dygenesis > thyroid dysharmonogenesis.", "cop": 3, "opa": "Thyroid agenesis", "opb": "Thyroid dysharmonogenesis", "opc": "Thyroid dysgensis", "opd": "Iodine deficiency", "subject_name": "Pediatrics", "topic_name": null, "id": "fa7f6990-3cc2-4793-be8a-afda9fa5dc0b", "choice_type": "single"} {"question": "Cause of \"bloody\" vomitus in a neonate", "exp": "Meckel&;s dieiculum may cause severe hemorrhage due to peptic ulceration. The blood is usually passed per rectum and is maroon in colour. it may cause bloody vomitus. Ref : BAILEY AND LOVE&;S SHO PRACTICE OF SURGERY,24 TH EDITION PG NO:1159", "cop": 1, "opa": "Meckel's diveiculum", "opb": "Intussusception", "opc": "Malrotation", "opd": "Cholecystitis", "subject_name": "Pediatrics", "topic_name": "Gastrointestinal tract", "id": "6e9ea73b-13c6-4306-9945-0dbfcdfa071f", "choice_type": "single"} {"question": "Surfactant deficiency occurs in", "exp": "Fetal lung maturation in diabetic pregnancy. The increased incidence of the idiopathic respiratory distress syndrome (IRDS) in infants of diabetic mothers may be explained by preterm delivery and asphyxia but the metabolic derangement per se may also be responsible for the inadequate production of surfactant Reference: GHAI Essential pediatrics, 8th edition", "cop": 1, "opa": "Infant of diabetic mother", "opb": "Meconeum aspiration syndrome", "opc": "Transient tachypnea of newborn", "opd": "Bronchopulmonary dysplasia", "subject_name": "Pediatrics", "topic_name": "New born infants", "id": "aae67ca1-0bed-42ea-89a4-fbe7f8f8251c", "choice_type": "single"} {"question": "After 3 days of bih, the base of the umbilical cord is red and swollen. It indicates", "exp": "Redness or edema at the base of umbilical cord indicate inflammatory changes and infectionIt is called as omphalitis(Refer: Nelson's Textbook of Pediatrics, SAE, 1st edition, pg no. 890, 811)", "cop": 3, "opa": "Normal phenomenon", "opb": "CHF", "opc": "Infection", "opd": "Clotting factor deficiency", "subject_name": "Pediatrics", "topic_name": "All India exam", "id": "6620e14c-88bc-4f88-a928-195e1119cf62", "choice_type": "single"} {"question": "Diagnostic Criteria of JRA", "exp": "It is defined as ahritis of one or more joints with onset below 16 yr age and persisting for atleast 6 weeks Ref Ghai pediatrics eighth edition pg no 625", "cop": 1, "opa": "Disease persisting 6 weeks or longer", "opb": "Onset before age 16 years", "opc": "Ahritis involving >5 joints", "opd": "Polyaicular JRA ANA is +ve", "subject_name": "Pediatrics", "topic_name": "Musculoskeletal disorders", "id": "dd43d72c-1a78-4d51-b8c5-d376182cc99a", "choice_type": "single"} {"question": "OATP 1B1/2 gene mutation is seen in", "exp": "OATP 1B1/2 gene mutation - Rotor syndrome\nABCCA gene mutation - Dubin johnson syndrome", "cop": 2, "opa": "Gilbert syndrome", "opb": "Rotor syndrome", "opc": "Crigler Najjar syndrome", "opd": "Dubin Johnson syndrome", "subject_name": "Pediatrics", "topic_name": null, "id": "5a9d0548-367f-4baa-b501-a55ed3bcd2cd", "choice_type": "single"} {"question": "Persistence of Moro's reflex is abnormal beyond the age of", "exp": "3rd month According to Ghai 6th/e p. 146 \"Moro's reflex disappear at three month of age and persistence of this reflex beyond three months is abnormal.\" According to Ghai 7m/e p. 114 \"Moro's reflex disappear by six months in normal infant\" We have done an extensive search for this one. We are sure of one point. Persistence of Moro's reflex beyond 6 months of age indicates neurological abnormality \"In the normal infant the 17101-0' s reflex begins to fade at three months of age and gradually disappear at 4-6 months of age. When it persist beyond 6 months of age it indicates delay in CNS development such as cerebral palsy.\" (Tachdjian clinical paediatric ohopaedic p. 371) But still we are not sure what would be the answer for Moro's reflex disappear at ??", "cop": 1, "opa": "3rd month", "opb": "4th month", "opc": "5th month", "opd": "6th month", "subject_name": "Pediatrics", "topic_name": null, "id": "ea67a628-a54c-4ea3-8bf1-1d57bbe47e53", "choice_type": "single"} {"question": "Neonatal hepatitis, cholestasis, triangle shaped face, pulmonary stenosis is seen in", "exp": "The above clinical features seen in Alagille syndrome.", "cop": 3, "opa": "Apert syndrome", "opb": "Noonan syndrome", "opc": "Alagille syndrome", "opd": "William syndrome", "subject_name": "Pediatrics", "topic_name": null, "id": "0017cf50-1635-45f8-843a-7edce8448317", "choice_type": "single"} {"question": "In a neonate, Jaundice appears for the first time in the 2nd week. The following is not a cause", "exp": "In case of rh incompatibility the baby may have jaundice within 24 hrs and not late. Reference: GHAI Essential pediatrics, 8th edition", "cop": 2, "opa": "Galactosemia", "opb": "Rh Incompatibility", "opc": "Hypothyroidism", "opd": "Breast milk jaundice", "subject_name": "Pediatrics", "topic_name": "New born infants", "id": "d86bc87d-9db1-4950-b57a-206e93df3aba", "choice_type": "single"} {"question": "After premature delivery, mother's milk is low in", "exp": "Ans. a (Lactose) (Ref OP Ghai 6th/ p. 158; 7th/p. 131, 134)After premature delivery, mother's milk is low in:# Calcium # Phosphates # LactoseADVANTAGES OF BREAST MILK# Docosahexanoic acid (DHA, W3, 2:3) present in breast milk helps in brain development.# Maximum synthesis of breast milk occurs at 12 months.# Has better calcium than cow milk for baby to absorb.# Iron in breast has 70% availablity. # Prevents against infections by:- Human milk contains bacterial and viral antibodies, including secretory IgA. They probably account for the lower incidence of diarrhea, otitis media, pneumonia, bacteremia, and meningitis during the 1st yr of life.e- Macrophages in human milk may synthesize complement, lysozyme, and lactoferrin.- Lactoferrin has an inhibitory effect on the growth of Escherichia coli in the intestine.- The lower pH of the stool of breast-fed infants is thought to contribute to the favorable intestinal flora of infants fed human milk vs. formula (i.e., more bifidobacteria and lactobacilli; fewer E. coli), which also helps protect against infections caused by some species of E. coli.- Human milk also contains bile salt-stimulated lipase, which kills Giardia lamblia and Entamoeba histolytica.DISADVANTAGES OF BREAST MILK# Breast milk is balanced, will supply all the necessary nutrients except, perhaps, fluoride and, after several months, vitamin D.# The iron content of human milk is somewhat low. However, most normal infants have sufficient iron stores for the first 6 mo of life. Moreover, human milk iron is well absorbed.# The vitamin K content of human milk also is low and may contribute to hemorrhagic disease of the newborn. Parenteral administration of 1 mg of vitamin K, at birth is recommended for all infants, edpecially for those who will be breast- fed.# Cytomegalovirus (CMV), human T-cell lymphotropic virus type 1, rubella virus, hepatitis B virus, and herpes simplex virus also have been demonstrated in breast milk. Of these, the presence of CMV is the most troublesome.Nutritive value of milk compared BuffaloCowGoatHuman1Fat (g)6.54.14.53.42Protein (g)4.33.23.31.13Lactose (g)5.14.44.67.1Q4Calcium (mg)210120170285Iron (mg)0.20.20.3--6Vitamin C (mg)12137Minerals (g)0.80.80.80.18Water (g)81.08786.888", "cop": 1, "opa": "Lactose", "opb": "Fat", "opc": "Protein", "opd": "Sodium", "subject_name": "Pediatrics", "topic_name": "Nutrition", "id": "2ec73e7d-7ab4-4724-8bc5-fb2213dd5217", "choice_type": "single"} {"question": "Myelodysplasia in fetus can be best diagnosed by", "exp": "Ans. a (AFP); (Ref. Robbins, Pathologic Basis of Disease, 6th/pg.325)Maternal serum AFP (MSAFP)# It is a oncofoetal protein# Highest level of which in foetal serum and amniotic fluid, it reaches around 13 weeks and decreases thereafter.# It reaches a peak around 32 weeks.# It decreases in Down's syndrome.# Its increased levels are seen in (Mnemonic = WORMIAN)- Wrong gestational age,- Open neural tube defects,- Renal anomalies,- Multiple pregnancy,- IUD,- Anterior abdominal wall defects,- Nephrotic syndrome (Congenital).# Ach esterase has better diagnostic value than AFP in open neural tube defects. (AI 2005)# Normal levels of AFP in liquor at 16th week = 20 mg/dL.# AFP is tumor marker for HCC, Non-seminomatous GCTs and ovarian sexcord tumors like Sertoli-Leydig tumors.", "cop": 1, "opa": "AFP", "opb": "Lumbar puncture", "opc": "HCG", "opd": "Unconjugated estriol", "subject_name": "Pediatrics", "topic_name": "Miscellaneous", "id": "bca22e0b-6e5f-4a8a-bac1-4203bf2a3dd5", "choice_type": "single"} {"question": "A Full Term Baby, exclusively breast fed, at the end of 1 week was passing Golden yellow stools and was found to have adequate hydration with normal systemic examination. The weight of the baby was just same as it was at bih. The pediatrician should now advice", "exp": "Sometimes the physiological changes may vary from different children,proper reassurance to mother is very impoant. Reference: GHAI Essential pediatrics, 8th edition", "cop": 4, "opa": "Give oral solution with breast feeding", "opb": "Sta tap feeding", "opc": "Investigate for lactic acidosis", "opd": "Reassure the mother that nothing is abnormal", "subject_name": "Pediatrics", "topic_name": "New born infants", "id": "ca9a20d1-3db3-4570-a94e-66392e0fc2ff", "choice_type": "single"} {"question": "Central bronchiectasis is seen with", "exp": "The distribution . bronchiectasis mar be impoant diagnostically A central - Perihilar allergic bronchopulmonary aspergillosis. Predominant upper lobe o Middle and lower lobe - Cystic fibrosis or one of its variants. Distribution is consistent with PCD Lower lobe involvement is - Middle lobe and lingular segment of the LUL involvement is characteristic of non tuberculous mycobacteria (NTM). Idiopathic bronchiectasis REF : OP-GHAI 9th edition", "cop": 2, "opa": "Cystic Adenomatoid Malformation", "opb": "Cystic fibrosis", "opc": "Broncho carcinoma", "opd": "Tuberculosis", "subject_name": "Pediatrics", "topic_name": "All India exam", "id": "562e5228-d165-442b-b03a-0a78d9c30130", "choice_type": "single"} {"question": "ECG new born child with Down syndrome", "exp": "Ans. is 'a' i.e., Normal Neonatal ECG screening for congenital hea disease in Down syndrome. Narchi HI He studied the value of routine neonatal electrocardiography (ECG) in the 1st 48 hours of life to diagnose congenital hea disease in 37 neonates with Down syndrome. 60% infants had no clinical evidence of congenital hea disease, had normal ECGs and normal cardiac anatomy on echocardiography. About 40% children with down syndrome have congenital hea disease. Endocardial cushion defect (ASD with ostium primum) account for 40-60% of cases.", "cop": 1, "opa": "Normal", "opb": "ASD", "opc": "VSD", "opd": "TOF", "subject_name": "Pediatrics", "topic_name": null, "id": "9cb655fe-6b01-40a6-a518-97b9b9fd074d", "choice_type": "single"} {"question": "The cardiac silhouette can have an \"egg on side\" appearance in", "exp": "Transposition of great vessels (TGA) is defined as aoa arising from the right ventricle and pulmonary aery from the left ventricle. In patients with TGA, the oxygenated pulmonary venous blood recirculates in the lungs whereas the systemic venous blood recirculates in the systemic circulation. The pulmonary aery saturation is thus always higher than the aoic saturation. Survival depends on the mixing available between the two circulations. Patients of complete TGA with intact ventricular septum are cyanotic at bih. Since the interatrial communication results in poor mixing, the neonates present with rapid breathing and congestive failure secondary to hypoxemia within the first week of life. Physical examination shows severe cyanosis, congestive failure, normal first sound, single second sound and an insignificant grade one to two ejection systolic murmur. The electrocardiogram shows right axis detion and right ventricular hyperophy. The thoracic roentgenogram shows cardiomegaly with a narrow base and plethoric lung fields. The cardiac silhouette can have an \"egg on side\" appearance: The right upper lung fields appear more plethoric than other areas. The thymic shadow is often absent. Patients of TGA with VSD have increased pulmonary blood flow; mixing at the ventricular level determines the severity of cyanosis. They develop congestive failure around 4-10 weeks of age. Physical findings consist of cyanosis, cardiomegaly, congestive failure, normal first sound, single or normally split second sound and grade II-IV ejection systolic murmur. Apical third sound gallop or a mid-diastolic rumble may be present. Electrocardiogram shows right axis detion with biventricular, right ventricular or left ventricular hyperophy. Chest X-ray shows cardiomegaly, plethoric lung fields and features of pulmonary venous hypeension. The aerial switch operation is now established as the treatment of choice for TGA, Prostaglandin El can help reduce cyanosis in selected cases by keeping the PDA open. Interim palliation can be accomplished through a balloon atrial septostomy. Reference: Essential Paediatrics; O.P. Ghai; Page no: 425", "cop": 4, "opa": "VSD", "opb": "PDA", "opc": "TOF", "opd": "Transposition of Great Vessels", "subject_name": "Pediatrics", "topic_name": "C.V.S", "id": "160f1ef7-451b-4362-bd71-97c72fcc8140", "choice_type": "single"} {"question": "Confirmatory test for cushing's disease is", "exp": null, "cop": 4, "opa": "High dose dexamethasone suppression test", "opb": "24 hr urine free cortisol", "opc": "Overnight dexamethasone suppression test", "opd": "Bilateral inferior petrous sinus sampling", "subject_name": "Pediatrics", "topic_name": null, "id": "6f88efd9-6b88-464e-afde-30a089058fb9", "choice_type": "single"} {"question": "Chromosome associated with autosomal dominant PCKD", "exp": "PKD1 gene → Chromosome 16.\nPKD2 gene → Chromosome 4.", "cop": 3, "opa": "6 and 14", "opb": "12 and 14", "opc": "4 and 16", "opd": "12 and 16", "subject_name": "Pediatrics", "topic_name": null, "id": "bdce8547-481d-4aa4-8ac1-1fbf732162ac", "choice_type": "single"} {"question": "Hyperbilirubinemia in a child can be due to", "exp": "Breast milk jaundice - bilirubin in excess of 10mg/dl. Seen beyond 3rd-4th weeks of life. A diagnosis of breast milk jaundice should be considered if this is unconjugated (not staining nappies) and other causes for prolongation such as inadequate feeding, continuing hemolysis, extravasated blood, G6PD deficiency and hypothyroidism have been ruled out. Cystic fibrosis is an autosomal recessive disorder caused by mutation in the gene encoding the chloride conductance channel, CF transmembrane conductance regulator(CFTR). The failure of chloride conductance by epithelial cells leads to dehydration of secretions that are too viscid and difficult to clear. Fanconi syndrome is characterised by tubular proteinuria and aminoaciduria and variable degrees of bicarbonaturia, phosphaturia, electrolyte wasting and glucosuria. Alpha1-antitrypsin deficiency (AATD) is a genetic inherited autosomal-codominant condition. People with AATD are predisposed to obstructive pulmonary disease and liver disease (eg, cirrhosis and hepatocellular carcinoma in children and adults). Reference : page 172,499, 393 Ghai Essential Pediatrics 8th edition", "cop": 1, "opa": "Breast milk jaundice", "opb": "Cystic fibrosis", "opc": "Fanconi's syndrome", "opd": "Alpha - 1 antitrypsin deficiency", "subject_name": "Pediatrics", "topic_name": "New born infants", "id": "0c09b283-a185-4b89-825b-04c0079c3343", "choice_type": "single"} {"question": "Profuse watery diarrhoea in an immune compromised child is due to", "exp": "Giardiasis caused by Giardia lamblia is a major cause of diarrhoea in children and travellers. Individuals with malnutrition,humoral immunodefeciencies,and cystic fibrosis are particularly susceptible.\nFeatures: acute diarrhoea with sudden onset of explosive, watery foul smelling stools, along with nausea and anorexia,abdominal distension,flatulence,epigastric camps and mild fever.", "cop": 3, "opa": "Cryptococcus", "opb": "Amoeba", "opc": "Giardia", "opd": "Lactose intolerance", "subject_name": "Pediatrics", "topic_name": null, "id": "c088fbf4-f74b-405c-9f75-06120a90ff1a", "choice_type": "single"} {"question": "Early eiphyseal closure is seen with", "exp": "(Estrogens): Ref: 1859-Nelson 17thEstrogen has direct result of the known on promoting epiphyseal fusion and therapy must begin before the bone age has reached 12 years (Marfan's syndrome)* Recent studies have demonstrated that is estrogen that mediates epiphyseal fusion in both girls and boys* Androgens also accelerates skeletal maturation, presumably via aromatization to estrogens, but at the price of rapid virilization.", "cop": 3, "opa": "LH", "opb": "FSH", "opc": "Estrogens", "opd": "Progesterone", "subject_name": "Pediatrics", "topic_name": "Growth, Development, and Behavior", "id": "ffb7b407-1ace-4ccc-8726-b0b69929defc", "choice_type": "single"} {"question": "Trisomy 13 is seen in", "exp": "Trisomy 13 is seen in Patau syndrome Edward syndrome is trisomy 18 Down syndrome trisomy 21 and turner syndrome is due to 45 XO . Reference : Nelson TB of pediatrics 19th edition.", "cop": 2, "opa": "Edward Syndrome", "opb": "Patau Syndrome", "opc": "Down Syndrome", "opd": "Turnur Syndrome", "subject_name": "Pediatrics", "topic_name": "Genetic and genetic disorders", "id": "1e8a8adb-1e71-42e9-8815-85aa752e139b", "choice_type": "single"} {"question": "A 9 months old girl is presented with PR bleed, vomiting, mass & colicky pain in Rt lumbar region with masked liver dullness. She is in shocklike condition. Management should includes", "exp": ".Intussusception Mostly between 6-11 months of age Features:intermittent colicky abdominal pain,vomiting,bloody mucous stools(current jelly) History of recent change of milk formula,upper respiratory tract infection,vaccination may be present. Sausage shaped mass maybe palpable on right quadrant of abdomen...iliac fossa may appear empty it is a surgical emergency but aim is reduction of the obstructed bowel.in intussusception of sho duration hydrostatic pressure of the barium enema or saline enema usually relieve it. ESSENTIAL PEDIATRICS,O.P.GHAI,PG NO:259,7 th edition", "cop": 1, "opa": "Barium enema", "opb": "Saline reduction", "opc": "Give O2", "opd": "Nasogastric tube", "subject_name": "Pediatrics", "topic_name": "Gastrointestinal tract", "id": "73829694-f73d-4a87-a101-7b7a29a87c18", "choice_type": "single"} {"question": "Expressed breast milk can be stored at room temperature for", "exp": "Expressed Breast Milk (EBM) If a mother is not in a position to feed her baby (e.g. ill mother, preterm baby, working mother, etc.), she should express her milk in a clean wide-mouthed container and this milk should be fed to her baby. EBM can be stored at room temperature for 6-8 hr, in a refrigerator for 24 hr and a freezer at -20degC for 3 months. Ref: Paediatrics; O.P. Ghai; 8th edition; Page no: 155", "cop": 2, "opa": "1-4 hours", "opb": "6-8 hours", "opc": "12-24 hours", "opd": "> 24 hours", "subject_name": "Pediatrics", "topic_name": "New born infants", "id": "31c356ad-31a8-473e-8168-844172c8ef8f", "choice_type": "single"} {"question": "Wilson disease is caused by defect in", "exp": "ATP 7B gene on chromosome 13 is mutated in Wilson disease.", "cop": 4, "opa": "ATP 7A mutation", "opb": "ATP 6A mutation", "opc": "ATP 6B mutation", "opd": "ATP 7B mutation", "subject_name": "Pediatrics", "topic_name": null, "id": "ace998ba-39cf-4efc-8f43-a53bc1576b3f", "choice_type": "single"} {"question": "A 3 year old child comes with complaint of limp diagnosis is", "exp": ".Septic ahritis, also known as jointinfection or infectious ahritis, is the invasion of a joint by an infectious agent resulting in joint inflammation. Symptoms typically include redness, heat and pain in a single jointassociated with a decreased ability to move the joint.", "cop": 1, "opa": "Septic Arhritis", "opb": "Slipped capital femoral epiphysis", "opc": "Pehe's disease", "opd": "DDH", "subject_name": "Pediatrics", "topic_name": "Musculoskeletal disorders", "id": "41da97d2-ffff-4e11-8667-0898943e2635", "choice_type": "single"} {"question": "Mousy odor of urine is seen in", "exp": "Children with classic phenyl ketonuria have an unpleasant odour of urine due to presence of phenylacetic acid,which is often described as musty or mousey. Phenylketonuria is due to deficiency of enzyme phenylalanine hydroxylase or of its cofactor tetra hydro biopterin. Reference: Nelson TB of pediatrics pg 418 edit", "cop": 2, "opa": "Isovaleric acidemia", "opb": "Phenyl ketonuria", "opc": "MSUD", "opd": "Tyrosinemia", "subject_name": "Pediatrics", "topic_name": "Genetic and genetic disorders", "id": "52941748-3e30-4e06-8873-d1e7ce639f4b", "choice_type": "single"} {"question": "Most common GI malignancy of childhood", "exp": "Ans. is 'b' i.e., Lymphoma Over all hemangioma is most common Tumor in infant. Hemangioma is usually benign in Nature. Leukemia is most common malignancy in pediatric age gyp. 2\"' most common is CNS Tumor = Lymphoma is the most common malignancy of the gastrointestinal tract in children. About 30% of children with non-Hodgkin lymphoma present with abdominal tumors.", "cop": 2, "opa": "Adenocarcinoma", "opb": "Lymphoma", "opc": "Sarcoma", "opd": "carcinoid", "subject_name": "Pediatrics", "topic_name": null, "id": "3c6fba5c-2380-4e7c-8590-af1ada0a39ed", "choice_type": "single"} {"question": "The test is done to diagnose syphilis in a newborn if the mother is syphilitic is", "exp": "Diagnosis of congenital syphilisIt involves Dark ground Microscopy and VDRL.As IgM does not cross the placenta, its presence in neonatal serum confirms congenital syphilis and helps to differentiate it from seropositivity due to passively transferred maternal antibodies. The tests to detect IgM are-FTA-ABS (19S IgM FTA-ABS)Syphilis Capita M test(Refer: Nelson's Textbook of Pediatrics, SAE, 1st edition, pg no. 1475-1477)", "cop": 1, "opa": "Syphilis Capita M test", "opb": "Detection of IgG", "opc": "ZN staining", "opd": "Fluorescent antigen test", "subject_name": "Pediatrics", "topic_name": "All India exam", "id": "21bd180c-94a3-4a44-b512-36ab1fe09ef9", "choice_type": "single"} {"question": "A newborn baby presented with profuse bleeding from umbilical stump after birth. Probable diagnosis is", "exp": "Ans. is 'a' i.e. Factor XIII deficiency Bleeding from umbilical stump suggests Factor XIII deficiency.Neonates with a positive bleeding history, particularly with features such as delayed bleeding, umbilical stump bleeding or miscarriages and in whom the initial panel of screening test is negative should be tested for factor XIII deficiency.The normal screening tests such as PT and PTT are normal in factor XIII deficiency because factor XIII is needed to stabilize the clot after a fibrin clot has formed, so tests used for evaluating the extrinsic or intrinsic pathways are not affected. (Note, that factor XIII is used, once the clot has formed.)Test used for evaluation of factor XIIITest used in qualitative evaluation of factor XIII is evaluation of clot stability in 5M urea.Clots formed from normal individuals remain stable in 5M urea, while clots from factor XIII deficient patient dissolve in urea.This assay detects only the most severely affected patients with 1% to 2% factor XIII deficiency or tests.A quantitative chromogenic assay has been developed but is not yet readily available.In addition to hemorrhages these patients may have poor wound healing a high incidence of infertility among males and abortion among affected females and a high incidence of intracerebral hemorrhage.About other options:Von Willebrand diseasePatients with Von-Willebrand disease rarely presents in the neonatal period because plasma concentration of Von Willebrand factor is high in neonatal period.Platelet function disordersGenetic disorders that result in deficient platelet function are rare causes of bleeding in infants.Most patients with these platelet disorders do not present in neonatal period but rather later in life. They are not recognized as a problem during the neonatal period.Glanzman's thromboastheniaThis is an extremely rare disorder of the blood in which the platelets lack the glycoprotein IIb/HIa.Hence platelet aggregation cannot occur and bleeding time is significantly prolonged and there is increased bleeding.Bernard Soulier syndromeIt is an autosomal recessive disorder characterized by thrombocytopenia and large platelets (giant platelets)There is absence of the platelet membrane GpIb/IX complex.Symptoms are consistent with low or dysfunctional platelets and include easy bruising, nosebleeds, mucosal bleeding, menorrhagia and occasionally G.I. bleedingFactor XII deficiency * It is not associated with clinical bleeding.", "cop": 1, "opa": "Factor XIII deficiency", "opb": "VWF deficiency", "opc": "Factor XII deficiency", "opd": "Glanzmann thrombosthenia", "subject_name": "Pediatrics", "topic_name": "The Umbilicus", "id": "f0383f5b-0334-4e96-81d9-9471ecc5bfa4", "choice_type": "single"} {"question": "Best indicator of growth monitoring in children aEUR'", "exp": "Rate of increase in height & weight Midarm circumference, Body weight and chest circumference are one time point estimates therefore they are not adequate to provide information about long term nutritional stats of the child. A measurement which shows the time trends like rate of increase in height or weight with passage of time would be more useful. - The growth chas demonstrate progressive change in the height and weight of child as age advances therefore they are the best indicator of long term nutritional status. \"The growth cha has been described as passpo to child health care. It has won international recognition and is now a standard method of monitoring children's health and nutritional status\". Uses of growth cha - For growth monitoring - Diagnostic tool for identifying \"high risk\" children - Planning and policy making - Education tool - Tool.for action - Evaluation - Tool for teaching", "cop": 3, "opa": "Weight", "opb": "Mid-arm circumference", "opc": "Rate of increase in height & weight", "opd": "Head circumference", "subject_name": "Pediatrics", "topic_name": null, "id": "4822dcf8-790c-4b4d-8841-1d551543d50e", "choice_type": "single"} {"question": "Expressed Breast milk can be stored in refrigerator up to", "exp": "(24 hours) (151-Ghai 6th) (127-Ghai 7th)Expressed breast milk can be stored at room temperature for 6-8 hours* In refrigerator - 24 hours* In freezer at - 20deg for 3 months", "cop": 3, "opa": "4 hours", "opb": "10 hours", "opc": "24 hours", "opd": "3 months", "subject_name": "Pediatrics", "topic_name": "New Born Infants", "id": "23aa558d-99c2-4667-8286-18762c27e6cd", "choice_type": "single"} {"question": "Acquired megacolon can be due to", "exp": "Acquired megacolon may occur at any age as a result of Chagas disease (Trypanosoma cruzi), obstruction by a neoplasm or inflammatory stricture, toxic megacolon complicating ulcerative colitis, visceral myopathy, or in association with functional psychosomatic disorders (poor toilet training during infancy). Ref : Bailey and Love 26e pg: 1177", "cop": 3, "opa": "Leishmaniasis", "opb": "Typhoid", "opc": "Chagas disease", "opd": "Amoebiasis", "subject_name": "Pediatrics", "topic_name": "Gastrointestinal tract", "id": "8e9cc0a8-4ad4-4537-925e-aa7e0200ede4", "choice_type": "single"} {"question": "A3 year old girl has problems with writing and spelling. Other intelligence is normal", "exp": "Dyslexia\n\nDyslexia - problem in reading, spelling and written expression\nIntelligence is normal\nDyslexia is the most common type of learning disability.\nChildren with dyslexia have problem in reading, spelling and written expression.\nThe important clues to dyslexia in school age children include delayed language, trouble rhymic words, mispronunciation, hesitation,\nwork-finding difficulties and poor spellings with presence of letter reversal. Reading is slow, inaccurated and labored.", "cop": 3, "opa": "Specific writing disorder", "opb": "Mental retardation", "opc": "Dyslexia", "opd": "Dysphonia", "subject_name": "Pediatrics", "topic_name": null, "id": "15385aaa-b0a5-4e7f-8d56-d48ac3d00de0", "choice_type": "single"} {"question": "A 3 day child vomits everything he feeds, has a distended abdomen & diarrhoea. The urine is positive for benedicts test for reducing substance. The substance in urine is aEUR'", "exp": "Galactose Characteristic symptom i.e., vomiting, distended abdomen and diarrhea along with the presence of reducing substance in the urine suggests the diagnosis of galactosemia Benedict's Test Benedict's test is a biochemical test to detect reducing sugars. -Benedicts test detects the presence of aldehydes except (aromatic ones) and alpha hydroxyketones in a compound. All monosaccharides are reducing sugars i.e., they all have a free reactive aldehyde group and are positive on Benedicts. Some disaccharides such as fructose have alpha hydroxyl ketones and can reduce sugars. All other disaccharides are non reducing sugars and will not react with Benedict's solution Galactosemia Galactosemia is a group of inherited metabotic disorders in which an enzyme deficiency affects the normal metabolism of the sugar galactose Galactose is a monosaccharide Most of the galactose originates from the dairy products that contain lactose or milk sugar Lactose is a disaccharide that is catalyzed by the enzyme lactase into two monosaccharides i.e. lactose and glucose From the intestine the galactose is transpoed to liver where it is conveed to glucose phosphate, which can be used in several metabolic pathway. Conversion of Galactose to Gulose I phosphate occurs in several successive steps. A specific enzyme is required at each step Galactosemia occurs because of the deficiency of any of the enzymes involved in the metabolism of Galactose. The most common enzyme associated with galactosemia is the deficiency of the enzyme galactose -1- phosphate uridyl trans-ferase. The deficiency of the enzyme or the defect in the metabolism results in an accumulation of galactose -1- PO4 and galacticol that are thought to have direct toxic effect on the liver and other organs. Clinical features Infants with galactosemia appear normal at bih, however symptomsusually appear a few days to weeks after initiating galactose containing .feeds The early clinical features of severe galactosemia include liver dysfunction, manifested by jaundice and hypoglycemia, The accompanying gastrointestinal.findings are poor feeding, vomiting and diarrhoea. Due to the effect of galacticol toxicity on brain it may cause C.N.S symptoms to predominate which are irritability and seizures Diagnosis If galactosemia is suspected, urine should be tested simultaneously with Benedicts reagent and with Glucose oxidase method. - The Glucose oxidase method is specific for glucose, whereas - Benedict's reagent can detect any reducing substance -A negative glucose oxidase test with positive benedicts test suggests that a non glucose reducing substance is present. - With appropriate clinical findings this is most likely to be galactose Confirmatory diagnosis The confirmatory diagnosis is made by the enzyme assay.", "cop": 3, "opa": "Sucrose", "opb": "Glucose", "opc": "Galactose", "opd": "Fructose", "subject_name": "Pediatrics", "topic_name": null, "id": "23416b6c-93d0-4a24-8f04-b11ceb75ea3d", "choice_type": "single"} {"question": "Commonest cause of hypothyroidism is", "exp": "(Thyroid dysgenesis) (1872-73-Nelson 17th) (481-Ghai 7th)HYPOTHYROIDISM results from deficient production of thyroid hormone or a defect in thyroid hormone or a defect in thyroid hormonal receptor activity* Someform of thyroid dysgenesis (aplasia, hypoplasia or an ectopic gland) is the most common cause of congenital hypothyroidism, accounting for 85% of cases, 10% are caused by an in bom error of thyroxine synthjesis and 5% are the result of transplacental maternal thyrotropin receptor blocking antibody.* Pendred syndrome - congenital hypothyroidism with sensorineural deafness and goiter, also have a positive perchlorate discharge.* ** Sodium L-thyroxine given orally is the treatment of choices (side effects are - craniosynostosis and temperature problems pseudotumour cerebri)* Acquired hypothyroidism - is most commonly caused by auto immune destruction of thyroid as a part of Chronic lymphocytic thyroidits (Hashimoto's thyroiditis", "cop": 4, "opa": "Defective synthesis of thyroxine", "opb": "Defect of iodide transport", "opc": "Defect of the thyroglobulin synthesis", "opd": "Thyroid dysgenesis", "subject_name": "Pediatrics", "topic_name": "Endocrinology", "id": "ee998901-aeb1-4859-a627-b2475c045118", "choice_type": "single"} {"question": "Investigation of choice for confirming Henoch Schonlein Purpura is", "exp": "Biopsy of the kidney may be performed both to establish the diagnosis or to assess the severity of already suspected kidney disease. The main findings on kidney biopsy are increased cells and IgA deposition in the mesangium (pa of the glomerulus, where blood is filtered), white blood cells, and the development of crescents.(Refer: Nelson's Textbook of Pediatrics, SAE, 1st edition, pg no. 1217 - 1218)", "cop": 3, "opa": "Serum IgA levels", "opb": "CRP levels", "opc": "Renal Biopsy", "opd": "DTPA", "subject_name": "Pediatrics", "topic_name": "All India exam", "id": "9b731916-576e-492b-8e33-50d968002657", "choice_type": "single"} {"question": "The poor indicator of response during treatment of malnutrition includes", "exp": "Primary failure to respond may be characterized by:1. Failure to regain appetite by day 42. Failure to sta losing edema by day 43. Failure of the disappearance of edema by day 104. Failure to gain weight at least by 5g/kg/day by day 10 of therapyRef: Nelson Paediatrics; 20th edition", "cop": 2, "opa": "Failure to gain appetite by day 10", "opb": "Presence of edema on day 10", "opc": "Failure to gain at least 5 g/kg/day by day 4", "opd": "Failure to gain at least 2 g/kg/day by day 10", "subject_name": "Pediatrics", "topic_name": "Nutrition", "id": "fcbe1cdb-e265-4297-b43f-645404a411b8", "choice_type": "single"} {"question": "Endemic cardiomyopathy in children is linked to the deficiency of", "exp": "Deficiency of selenium has resulted in Keshan disease, a form of cardiomyopathy endemic in China. Other dietary factors such as low intake of vitamin E, protein, and methionine are also considered relevant to the causation of the disease. Selenium deficiency is usually associated with vitamin E deficiency. Ref: Page 131; Ghai essential pediatrics; 6th edition", "cop": 3, "opa": "Zinc", "opb": "Cobalt", "opc": "Selenium", "opd": "Chromium", "subject_name": "Pediatrics", "topic_name": "Nutrition", "id": "52fa6fb4-089c-44ef-af85-f51091d1a255", "choice_type": "single"} {"question": "In pediatric advanced life suppo intraosseous access for drug/fluid administration is recommended for", "exp": "Intraosseous infusion (IO) is the process of injecting directly into the marrow of a bone to provide a non-collapsible entry point into the systemic venous system. This technique is used in emergency situations to provide fluids and medication when intravenous access is not available or not feasible. A comparison of intravenous (IV), intramuscular (IM), and intraosseous (IO) routes of administration concluded that the intraosseous route is demonstrably superior to intramuscular and comparable to intravenous administration (in delivering paediatric anesthetic drugs). Due to the rapid advance and adoption of superior intraosseous access technology, IO access has now become the preferred method of establishing vascular access for patients in whom traditional access is difficult or impossible. Reference: GHAI Essential pediatrics, 8th edition", "cop": 2, "opa": "<1 year", "opb": "<6 years", "opc": "<12 years", "opd": "Any age", "subject_name": "Pediatrics", "topic_name": "All India exam", "id": "6f5a90c7-ae27-444a-bc3f-7ce2a336728d", "choice_type": "single"} {"question": "The gene for Wilson&;s disease is on", "exp": "In Wilson&;s disease,the underlying defect lies in the long arm of chromosome 13 where there is a mutation in ATP7B gene,the normal hepatic copper- excreting gene.Eventually,capacity of hepatocytes to store copper is exceeded and copper is released into circulation which then gets deposited in extrahepatic tissues such as the brain eyes, and others. Reference:Textbook of pathology-Harsh Mohan,6th edition,page no:628", "cop": 1, "opa": "Long arm of Chromosome 13", "opb": "Long arm of Chromosome 6", "opc": "Sho arm of Chromosome 13", "opd": "Sho arm of Chromosome 6", "subject_name": "Pediatrics", "topic_name": "Metabolic disorders", "id": "3646cd16-a20e-4196-8e4e-7e2f749a53b2", "choice_type": "single"} {"question": "Hematopoisis in first month of life is", "exp": "After bih, and during early childhood, hematopoiesis occurs in the red marrow of the bone. With age, hematopoiesis becomes restricted to the skull, sternum, ribs, veebrae, and pelvis. Yellow marrow, comprised of fat cells, replaces the red marrow and limits its potential for hematopoiesis. Reference: GHAI Essential pediatrics, 8th edition", "cop": 1, "opa": "Medullary", "opb": "Hepatic", "opc": "Lymphatic", "opd": "Mesoblastic", "subject_name": "Pediatrics", "topic_name": "Hematology", "id": "31dfdc88-2f02-4512-9d31-99964bb411f5", "choice_type": "single"} {"question": "Moderate Hypothermia in a neonate is defined as an axillary temperature", "exp": "Classification of Neonatal hypothermia: Cold stress: 36.0 - 36.4oC Moderate hypothermia: 32 - 35.9oC Severe hypothermia: < 32oC.", "cop": 4, "opa": "< 34 degree C", "opb": "31-34 degree C", "opc": "< 32 degree C", "opd": "32-35.9 degree C", "subject_name": "Pediatrics", "topic_name": "Neonatal hypothermia", "id": "8b92fb45-9e7f-45c3-9dec-737f7511e45e", "choice_type": "single"} {"question": "Enzyme replacement therapy is most commonly used for", "exp": "Gaucher disease is a multisystemuc lipidosis characterized by hematological abnormalities,organomegaly and,skeletal involvement.It has 3 clinical subtypes . It occurs due to the deficient activity of acid beta glucosidase . Thus glucosylceramide accumulate in reticule end I the loam cells. Treatment is mainly enzyme replacement therapy with recombinant acid beta glucosidase.Most extra skeletal symptoms are reversed by 60 IU/kg enzyme iv infusion every other week. Monthly maintenance improves skeletal manifestations. Reference: Nelsons TB of pediatrics 19th edit,pg 487", "cop": 1, "opa": "Gaucher's disease", "opb": "Fabry's disease", "opc": "Niemenn pick's disease", "opd": "Pompe disease", "subject_name": "Pediatrics", "topic_name": "Genetic and genetic disorders", "id": "82a59bbf-ca52-45a6-a50e-aede05ee61bf", "choice_type": "single"} {"question": "The Blood sugar in a neonate sholy after bih reaches the lowest level of 30 mg/dl at the age of", "exp": "Hypoglycemia is a condition in which the level of glucose in the blood is lower than normal. A baby is at risk if he or she has a mother with diabetes, is preterm, or is large. If your baby has signs of hypoglycemia, give him or her formula or glucose and water mixture. Reference: GHAI Essential pediatrics, 8th edition", "cop": 2, "opa": "1 hour", "opb": "3 hours", "opc": "6 hours", "opd": "8 hours", "subject_name": "Pediatrics", "topic_name": "New born infants", "id": "e7c06173-0ca9-4e83-a2f4-77e5b22644de", "choice_type": "single"} {"question": "Best method of estimation of amount of proteinuria in a 2 year child with nephrotic syndrome is", "exp": "c. Spot urine sample for protein/creatinine ratio(Ref: Nelson 20/e 2518)Though 24 hr urine for protein and creatinine, is more accurate, but due to difficulty in collecting 24 hour urinary sample in children, Spot urine sample for protein/creatinine ratio is preferred.", "cop": 3, "opa": "Dipstick testing", "opb": "24 hr urine protein", "opc": "Spot urine sample for protein/creatinine ratio", "opd": "Microalbuminuria", "subject_name": "Pediatrics", "topic_name": "Kidney", "id": "cee9fc3f-1d28-4832-ad61-f36d55fe76f8", "choice_type": "single"} {"question": "Most common cause of acquired hea disease in children", "exp": "Ans. is 'a' i.e., Acute Rheumatic fever Acute rheumatic fever Most common cause of acquired hea disease in children. Caused by group A f3 hemolytic streptococci Usually seen in school going children Jones criteria = very impotrant Mitral valve most common followed by aoic valve In acute phase - MR seen In RI-ID MS seen", "cop": 1, "opa": "Acute rheuamatic fever", "opb": "Kawasaki", "opc": "Takayasu", "opd": "Diabetes", "subject_name": "Pediatrics", "topic_name": null, "id": "4d7e730c-3810-4263-aed2-8442c3445770", "choice_type": "single"} {"question": "In Sickel cell disease, aplastic crisis is due to", "exp": "In Sickel cell disease, due to infection of red cell progenitors by ParvoVirus B19 infection aplastic crisis occurs.", "cop": 3, "opa": "Herpes infection", "opb": "Measles infection", "opc": "Parvo Virus B19 infection", "opd": "Rubella infection", "subject_name": "Pediatrics", "topic_name": null, "id": "46f9f6da-5c43-4fce-870e-0ae2b35bf70d", "choice_type": "single"} {"question": "In child, foreign body in lung", "exp": "The children are treated by removal of foreign body through a rigid bronchoscope.Bronchoscopy should be undeaken if the clinical and radiological picture suggests the diagnosis even when the history of foreign body aspiration is not fohcoming. Ref : Essential Paediatrics,O.P.Ghai,7th edition,Pg No:368", "cop": 1, "opa": "Rigid bronchoscopy", "opb": "Chest x-ray", "opc": "Flexible endoscopy", "opd": "Direct laryngoscopy", "subject_name": "Pediatrics", "topic_name": "Respiratory system", "id": "5da30f8c-ad44-4f29-ae5a-9f4c58911b3d", "choice_type": "single"} {"question": "A 4 year baby is having large face, large jaw, large ear and macro orchidism is", "exp": "A child with large face large ears large jaw and macro orchidism has fragile X syndrome It is due to a fragile site in one of the distal long arm of chromosome Xq27.3 Reference: Nelson TB of pediatrics pg 411 edition 19.", "cop": 4, "opa": "Mc Cuneal bright syndrome", "opb": "Down's syndrome", "opc": "Cri-du chat syndrome", "opd": "Fragile X syndrome", "subject_name": "Pediatrics", "topic_name": "Genetic and genetic disorders", "id": "944bd626-f3f8-4e4d-938e-692b21279406", "choice_type": "single"} {"question": "Calculate the silverman score for a neonate with paradoxical breathing, mild lower chest retraction, marked xiphoid retraction, minimal nasal flare and expiratory grunt audible without stethoscope", "exp": "Score for Paradoxical breathing\n2\n\n\nMild lower chest retraction\n1\n\n\nMarked xiphoid retraction\n2\n\n\nMinimal nasal flare\n1\n\n\nExpiratory grunt audible without stethoscope\n2\n\n\nTotal score\n8", "cop": 3, "opa": "6", "opb": "7", "opc": "8", "opd": "9", "subject_name": "Pediatrics", "topic_name": null, "id": "591ef9e0-76ff-470f-8219-eac8cfdd33a9", "choice_type": "single"} {"question": "Macrosomia is a", "exp": "Macrosomia:- A condition where a baby is large before bihFactors causing macrosomia:Mother having diabetesMother having gestational diabetesObese mother(Refer: Nelson's Textbook of Pediatrics, SAE, 1st edition, pg no. 777, 897 - 899)", "cop": 1, "opa": "Large size baby", "opb": "Big mouth", "opc": "Large head", "opd": "Large tongue", "subject_name": "Pediatrics", "topic_name": "All India exam", "id": "e828463d-f844-4e66-acb8-7b75e9733df4", "choice_type": "single"} {"question": "Characteristic feature of Systemic Juvenile Idiopathic Arthritis is", "exp": "Systemic Juvenile Idiopathic Arthritis\n\nIt begins at below 16 years of age\nNSAIDs are given \nRa factor is negative\nUveitus is seen in oligoarticular JIA", "cop": 4, "opa": "Uveitis is a feature", "opb": "It occurs after 16 years of age", "opc": "NSAIDs are contraindicated", "opd": "RA factor is negative", "subject_name": "Pediatrics", "topic_name": null, "id": "aee54667-ff80-4a7a-a196-550c4646a259", "choice_type": "single"} {"question": "Initial drug of choice in a child with status epilepticus", "exp": "Prompt treatment with benzodiazepines is the first-line treatment of status epilepticus, but many patients will need additional treatment with additional medications including phenytoin, valproic acid, phenobarbital, or levetiracetam. The Neurocritical Care Society's guideline states that benzodiazepines remain the \"emergent initial therapy\" of choice based both on available evidence and expe consensus. When possible, intravenous benzodiazepine administration is preferred. However, formulations exist for buccal, intranasal, intramuscular, and rectal administration, and these should be administered if intravenous access cannot be rapidly established. The American Epilepsy Society's guideline concludes that intravenous lorazepam and diazepam are efficacious at stopping seizures lasting at least five min and that rectal diazepam, intramuscular midazolam, intranasal midazolam, and buccal midazolam are probably effective at terminating seizures lasting at least five minutes . Reference: GHAI Essential pediatrics, 8th edition", "cop": 1, "opa": "Lorazepam", "opb": "Phenobarbitone", "opc": "Valproate", "opd": "Phenytoin", "subject_name": "Pediatrics", "topic_name": "Central Nervous system", "id": "915f4dec-f6e4-412d-89aa-7efa5c092fe4", "choice_type": "single"} {"question": "Exclusive breast feeding means", "exp": "(D) Breast feeding only > Exclusive Breastfeeding. Only breastmilk is given. No other food or drink, not even water, is given. Medicines, vitamins or mineral drops are permitted if indicated. An infant should be exclusively breastfed for first 6 months of life.", "cop": 4, "opa": "Breast feeding on demand only + solid foods at other times", "opb": "Breast feeding + solid foods + water", "opc": "Breast feeding + water", "opd": "Breast feeding only", "subject_name": "Pediatrics", "topic_name": "Miscellaneous", "id": "f514e5ec-8006-4ca3-8740-e05702e90bce", "choice_type": "single"} {"question": "Vision is tested in the newborn by", "exp": "Methods of assessing vision in infants:Papillary light reflexRed reflexOptokinetic nystagmus (using nystagmus drum)Visual evoked potentialsAbility to follow a target(Refer: Nelson's Textbook of Pediatrics, SAE, 1st edition, pg no. 2802)", "cop": 3, "opa": "Electrooculogram", "opb": "Electroretinogram", "opc": "Visual Evoked potentials", "opd": "Brainstem Response assessment", "subject_name": "Pediatrics", "topic_name": "All India exam", "id": "6246b106-68c8-45f7-8c84-3950a04bc779", "choice_type": "single"} {"question": "2 year old baby having testicular lump. Increased AFP diagnosis", "exp": "Yolk sac tumor is the most common characteristic testicular tumor of infants and young children up to the age of 4 years.Alphafetoprotein levels are elevated in 100% cases of yolk sac tumors. Germ cell tumours unrelated to germ cell neoplasia in situ (non-GCNIS) Spermatocytic tumour Yolk sac tumour, prepubeal type Teratoma, prepubeal type - Dermoid cyst - Epidermoid cyst - Differentiated neuroendocrine tumours Mixed teratoma and yolk sac tumour, prepubeal type Germ cell tumours derived from germ cell neoplasia in situ (GCNIS) Seminoma Embryonal carcinoma Yolk sac tumour, postpuberal type Choriocarcinoma Teratoma, postpuberal type Mixed germ cell tumours Regressed germ cell tumours Sex cord-stromal tumours Leydig cell tumour Seoli cell tumour Granulosa cell tumour Tumour containing both germ cell and sex cord-stromal elements Gonadoblastoma Haematolymphoid tumours Tumours of collecting duct and rete testis Miscellaneous Reference:Textbook of pathology-Harsh Mohan,6th edition,page no:711.", "cop": 1, "opa": "Yolk sac tumor", "opb": "Seminoma", "opc": "Choriocarcinoma", "opd": "Teratoma", "subject_name": "Pediatrics", "topic_name": "Childhood tumors", "id": "01857b92-077c-4821-b652-9fc959e430a1", "choice_type": "single"} {"question": "Drugs that can be used in Kernicterus", "exp": "Barbiturates can be used in Kernicterus . Reference: GHAI Essential pediatrics, 8th edition", "cop": 1, "opa": "Barbiturates", "opb": "Benzodiazepines", "opc": "Phenytoin", "opd": "Chlorpromazine", "subject_name": "Pediatrics", "topic_name": "New born infants", "id": "186e43a8-1100-4716-991e-81748838606c", "choice_type": "single"} {"question": "The most common manifestation of congenital toxoplasmosis", "exp": "*Congenital toxoplasmosis is caused by toxoplasma gondi .The transmissibility increases but the risk of feral disease decreases with advancing pregnancy * The classical triad of toxoplasmosis includes intracranial calcification, hydrocephalus and chorioretinitis. Diagnosis is confirmed by demonstrating IgM in serum of the baby Image : Severe, active retinochoroiditis. Reference : Ghai essential pediatrics, 9 th edition, pgno :264", "cop": 2, "opa": "Deafness", "opb": "Chorioretinitis", "opc": "Hepatosplenomegaly", "opd": "Thrombocytopenia", "subject_name": "Pediatrics", "topic_name": "Infectious disease", "id": "75f686b2-0de1-4692-b43e-e4369a67efec", "choice_type": "single"} {"question": "A 5 year old boy comes with overnight petechial spots 2 weeks back he had history of abdominal pain and no hepatosplenomegaly. Diagnosis is", "exp": "most probable diagnosis here is ITP. in ALL there will be hepatosplenomegaly of a plastic anemia infection Symptoms Immune thrombocytopenia may have no signs and symptoms. When they do occur, they may include: Easy or excessive bruising Superficial bleeding into the skin that appears as pinpoint-sized reddish-purple spots (petechiae) that look like a rash, usually on the lower legs Bleeding from the gums or nose Blood in urine or stools Unusually heavy menstrual flow Reference: GHAI Essential pediatrics, 8th edition", "cop": 3, "opa": "Aute lymphatic leukemia", "opb": "Aplastic anemia", "opc": "Idiopathic thrombocytopenis purpura", "opd": "Acute viral infection", "subject_name": "Pediatrics", "topic_name": "Hematology", "id": "6c31cc63-c5c4-4fa9-8bc2-ab56aadce29d", "choice_type": "single"} {"question": "The most common congenital cardiac abnormality associated with the maternal rubella infection during pregnancy is", "exp": "Congenital rubella syndrome characterized by PDA,PS,VSD. Ref : Ghai essential of pediatrics, eighth edition ,p.no:401", "cop": 2, "opa": "Atrial septal defect", "opb": "Patent ductus aeriosus", "opc": "Ventricular septal defect", "opd": "Coarctation of aoa", "subject_name": "Pediatrics", "topic_name": "C.V.S", "id": "751422fa-c4c2-48b6-b1db-44c7a8f07234", "choice_type": "single"} {"question": "A Child with increased Ca++ & hyperphosphatemia is seen in", "exp": "Ans. c (Renal osteodystrophy). (Ref. Harrison Medicine, 18th/Ch.44, 279, Nelson, Paediatrics, 17th/223)# The principal clinical consequences of hyperphosphatemia are hypocalcemia and systemic calcification.# Ectopic calcification occurs in untreated, chronic renal failure with severe hypercalcemia and in vitamin D intoxication.# Tumor calcinosis is rare inherited disorder in which hyperphosphatemia is associated with ectopic calcification.# Vitamin D intoxication causes excessive gastrointestinal absorption of both Ca++ and Ph++ and suppression of PTH by hypercalcemia causes decrease in renal phosphorous excretion.# Hyperphosphatemia in hyperthyroidism or acromegaly is also associated with hypercalcemia.", "cop": 3, "opa": "Vitamin D dependent rickets", "opb": "Vitamin D resistant rickets", "opc": "Renal osteodystrophy", "opd": "Hypophosphatasia", "subject_name": "Pediatrics", "topic_name": "Metabolic Disorders", "id": "8506038d-f039-4c3a-9faf-1c0f0565e91e", "choice_type": "single"} {"question": "The risk of intraventricular hemorrhage (IVH) can be reduced by using", "exp": "Ans. a (Vitamin K) (Ref. Williams Obstetrics 22nd/Ch. 29)INTRAVENTRICULAR HEMORRHAGEThere are four major categories of neonatal intracranial hemorrhage: subdural hemorrhage is usually the result of trauma. Subarachnoid hemorrhage and intracerebellar hemorrhage usually result from trauma in term infants but in preterm infants are commonly due to hypoxia. Periventricular-intraventricular hemorrhage results from either trauma or asphyxia in half of term infants but has no discernible cause in 25 percent of cases. In preterm neonates, the pathogenesis of periventricular hemorrhage is multifactorial and includes hypoxic-ischemic events, anatomical factors, coagulopathy, and many others.The severity of intraventricular hemorrhage can be assessed by ultrasound and computed tomography. Papile and colleagues (1978) devised the most widely used grading scheme to quantify the extent of the lesion and estimate prognosis.# Grade I--hemorrhage limited to the germinal matrix.# Grade II--intraventricular hemorrhage.# Grade III--hemorrhage with ventricular dilatation.# Grade IV--parenchymal extension of hemorrhage.Prevention and Treatment# Administration of corticosteroids at least 24 hours before delivery appears to prevent or reduce the incidence and severity of intraventricular hemorrhage.# The type of corticosteroid administered may be important. Although both betamethasone and dexamethasone cross the placenta in their active form, betamethasone has a longer half-life. This is because it has a larger volume of distribution and is cleared more slowly than dexamethasone# The efficacy of phenobarbital, vitamin K, vitamin E, or indomethacin in diminishing the frequency and severity of intracranial hemorrhage, when administered either to the neonate or to the mother during labor, remains controversial.# Magnesium sulfate may prevent the sequelae of periventricular hemorrhage.# It is generally agreed that avoiding significant hypoxia both before and after preterm delivery is of paramount importance. There is presently no convincing evidence, however, that routine cesarean delivery for the preterm fetus presenting cephalic will decrease the incidence of periventricular hemorrhage.", "cop": 1, "opa": "Vitamin K", "opb": "Vitamin C", "opc": "Vitamin E", "opd": "Vitamin A", "subject_name": "Pediatrics", "topic_name": "New Born Infants", "id": "037f2c36-e4f2-44e3-aa53-6ee015628094", "choice_type": "single"} {"question": "This clinical condition is seen in", "exp": null, "cop": 4, "opa": "Marasmus", "opb": "Scurvy", "opc": "Rickets", "opd": "kwashiorkor", "subject_name": "Pediatrics", "topic_name": null, "id": "e0a28062-a404-4b62-8005-125957a37c2f", "choice_type": "single"} {"question": "Treatment of choice for stage 4S neuroblastoma is", "exp": "The usual treatment for children with low-risk neuroblastoma is surgery for stages 1 and 2 Observation for stage 4S with cure rates generally >90% without fuher therapy. Treatment with chemotherapy or radiation for the rare child with local recurrence can still be curative. Reference: Nelson; Neuroblastoma; Page no: 2463", "cop": 1, "opa": "Observation", "opb": "Surgery", "opc": "Chemotherapy", "opd": "Radiotherapy", "subject_name": "Pediatrics", "topic_name": "Childhood tumors", "id": "c5e7de33-4a25-42bd-924b-5fb2bc6b3587", "choice_type": "single"} {"question": "NOT a feature of Turner's syndrome", "exp": "Cubitus valgus is seen in Turner's syndrome.", "cop": 3, "opa": "Short stature", "opb": "Normal IQ", "opc": "Cubitus varus", "opd": "Streak ovaries", "subject_name": "Pediatrics", "topic_name": null, "id": "c18beb2d-9046-4f72-8ada-9119f87bde52", "choice_type": "single"} {"question": "Most common cause of persistent hypeension in child with Intrinsic Renal disease is", "exp": "ACEIs or ARBs, appropriate diuretic therapy, and dietary salt restriction make up the foundation for the treatment of HTN in CKD. Bedtime dosing of at least one antihypeensive medication improves BP control in patients with CKD. Reference: GHAI Essential pediatrics, 8th edition", "cop": 2, "opa": "CGN", "opb": "Chronic pyelonephritis", "opc": "Obstructive uropathy", "opd": "Renal tumor", "subject_name": "Pediatrics", "topic_name": "Urinary tract", "id": "e45f585c-9c04-4deb-a1b7-8e3216307267", "choice_type": "single"} {"question": "Sign of pubey in girl is needed to change", "exp": "In girls, the first visible sign of pubey is the appearance of breast buds (Thelarche), between 8-12 years of ageIn boys the first visible sign of pubey is testicular enlargement, beginning as early as 9 1/2yr(Refer: Nelson's Textbook of Pediatrics, SAE, 1st edition, pg no. 926)", "cop": 2, "opa": "Puberche", "opb": "Thelarche", "opc": "Menarche", "opd": "Growth spu", "subject_name": "Pediatrics", "topic_name": "All India exam", "id": "70e18c36-1b4f-455b-bf15-ce03a471bd6c", "choice_type": "single"} {"question": "The most potent form of vitamin D is", "exp": "1,25 dihydroxy cholecalciferol is the metabolically active form of vitamin D. It helps in the absorption of calcium and phosphorus in the gut, and promotes bone dissolution and mineralization, thereby increasing serum calcium and alkaline phosphatase activity. Ref: Page 127; Ghai essential pediatrics; 6th edition", "cop": 4, "opa": "Ergocalciferol (Vit. D2)", "opb": "7 dihydroxy cholecalciferol", "opc": "25 dihydroxy cholecalciferol", "opd": "1,25 dihydroxy cholecalciferol", "subject_name": "Pediatrics", "topic_name": "Nutrition", "id": "1ae77d0c-2a3b-46af-ac7e-c57a3593c6d3", "choice_type": "single"} {"question": "The best indicator of nutritional status for a child is", "exp": "The best indicator of nutritional status of a child is the rate of increase in height & weight. Rest of them are age independent indices. Ref: Page 64, Ghai Essential Pediatrics; 7th edition", "cop": 3, "opa": "Mid arm circumference", "opb": "Head circumference", "opc": "Rate of increase in height and weight", "opd": "Chest circumference", "subject_name": "Pediatrics", "topic_name": "Nutrition", "id": "6ee9228b-4565-4f05-bd2e-54ae36d428c3", "choice_type": "single"} {"question": "Most common complication of mumps in children is", "exp": "(B) Aseptic meningitis > In adult males, mumps can cause bilateral orchitis and sterility, Meningitis usually follow parotitis and acute pancreatitis occurs in 2nd week, may result in diabetes mellitus.> Meningo encephalitis results 3-10 days after onset of glandular swelling.> Incubation period of mumps is 12-21 days (Mean 17 days)> COMPLICATIONS OF MUMPS: Epididymo-orchitis is a complication seen in about a third of postpubertal male patients. The testis becomes swollen and acutely painful, with accompanying fever and chills. Orchitis is usually unilateral but when it is bilateral and followed by testicular atrophy, sterility or low sperm counts may result. The central nervous system is involved in about 60 per cent of cases, as indicated by pleocytosis in the CSF but only about 10 per cent show symptoms of meningitis.> Mumps has been reported to cause about 10-15 per cent of cases of aseptic meningitis'. Mumps meningitis and meningoencephalitis usually resolve without sequelae but deafness may sometimes result. Munps meningitis may occasionally occur in the absence of parotitis, when diagnosis rests solely on laboratory evidence. The virus can be grown readily from the CSF in the early phase of meningitis.> Other less common complications are arthritis, oophoritis, nephritis, pancreatitis, thyroiditis and myocarditis. Most common complication of mumps in children is aseptic meningitis.", "cop": 2, "opa": "Orchitis", "opb": "Aseptic meningitis", "opc": "Pancreatitis", "opd": "Myocarditis", "subject_name": "Pediatrics", "topic_name": "Miscellaneous", "id": "4a46db71-d836-4482-ab14-69f49b41a2ec", "choice_type": "single"} {"question": "Most common hea lesion in down syndrome", "exp": "Ans. is 'a' i.e., ASD with ostium primum About 40% children with down syndrome have congenital hea disease. Endocardial cushion defect (ASD with ostium primum) account for 40-60% of cases.", "cop": 1, "opa": "ASD with ostium primum", "opb": "ASD with ostium secundum", "opc": "VSD", "opd": "Coaection of aoa", "subject_name": "Pediatrics", "topic_name": null, "id": "f3a6f442-442f-4365-8736-41cf3c1c1e80", "choice_type": "single"} {"question": "Felty syndrome is characterised by A/E", "exp": "Ans. is 'd' i.e., Thrombocytopenia Felty's syndrome is characterized by the combination of rheumatoid ahritis, splenomegaly and neutropenia. The condition is more common in those aged 50-70 years, and is more prevalent in females than males", "cop": 4, "opa": "Rheumatoid ahritis", "opb": "Splenomegaly", "opc": "Neutropeni", "opd": "Thrombocytopenia", "subject_name": "Pediatrics", "topic_name": null, "id": "f14ddf6b-2f16-435a-9904-ad144bb3f411", "choice_type": "single"} {"question": "Straining and Dribbling of urine in a male infant with recurrent urinary infection should lead to the suspicious of", "exp": "Posterior urethral valves are a problem with the urethra in a boy. The urethra is the tube that drains urine from the bladder to the outside of the body. These valves are small leaflets of tissue in the urethra. They paly block urine flow because not enough urine can get through them to leave the body. Reference: GHAI Essential pediatrics, 8th edition", "cop": 2, "opa": "Vesico-ureteric reflux", "opb": "Posterior urethral valve", "opc": "Pelvic ureteric junction obstruction", "opd": "Phimosis", "subject_name": "Pediatrics", "topic_name": "Urinary tract", "id": "e193341f-97d2-4656-85a1-118f4abc210a", "choice_type": "single"} {"question": "Darkening of urine on standing is associated with", "exp": "Alkaptonuria is caused by defect of the enzyme homogentisate 1,2-dioxygenase(homogentisic acid oxidase).The disorder comes to attention due to change in colour of urine to brownish black/ staining of diapers.The urine becomes dark on standing,especially if the pH of urine is alkaline,due to the presence of homogentisic acid.Excessive urine homogentisate results in positive reducing substances. Reference:Essential pediatrics-Ghai,8th edition,page no:653.", "cop": 1, "opa": "Alkaptonuria", "opb": "Cystinuria", "opc": "Fabry's disease", "opd": "Tyrosinemia", "subject_name": "Pediatrics", "topic_name": "Metabolic disorders", "id": "4e9b8c2c-3599-4b04-9655-cb891a61d598", "choice_type": "single"} {"question": "One year old male child with sparse blond hair, developmental delay and tremors, diagnosis", "exp": "Phenylketonuria is due to deficiency of enzyme phenylalanine hydroxylase or of its cofactor tetrahydrobiopterin ;leads to accumulation of phenylalanine in body fluids and brain. It is mainly characterized by profound mental retardation ,lighter complexion and neurologic symptoms like tremors,hyperreflexia and spasticity. Reference: Nelson TB of pediatrics 19th edition, pg 418", "cop": 2, "opa": "Albinism", "opb": "Phenylketonuria", "opc": "Cerebral palsy", "opd": "Infantile tremor syndrome", "subject_name": "Pediatrics", "topic_name": "Genetic and genetic disorders", "id": "dc14a38c-26c6-42e4-b3dc-d628a7781c84", "choice_type": "single"} {"question": "Most common cause of infectious croup is", "exp": "Ans. is 'b' i.e., Parainfluenza type I Infectious croup (Laryngitis & laryngotracheobronchitis)o Nearly always caused by viral infectiono Usually by parainfluenza type I.o Other-RSVParainfluenza 2,3InfluenzaAdenoviruso TreatmentHumidified oxygenIntravenous fluid for adequate hydrationAntibiotic (ampicillin ceftriaxone) if suspecting epiglottitis.", "cop": 2, "opa": "Adenovirus", "opb": "Parainfluenza type I", "opc": "RSV", "opd": "Influenza", "subject_name": "Pediatrics", "topic_name": "Disorders of the Respiratory Tract", "id": "0714e71a-d116-47b9-974e-503c6428d92e", "choice_type": "single"} {"question": "Most common cardiac anamoly associated with Noonan 's syndrome is", "exp": "Most common cardiac anamoly associated with Noonan's syndrome is pulmonary stenosis.", "cop": 3, "opa": "Aortic stenosis", "opb": "Ventricular septal defect", "opc": "Pulmonary stenosis", "opd": "Atrial septal defect", "subject_name": "Pediatrics", "topic_name": null, "id": "2467534e-db69-4826-9ebe-3f03adaad3ee", "choice_type": "single"} {"question": "Most common cause of neonatal meningitis is", "exp": "The most common bacterial causes of neonatal meningitis are GBS(S.agalactiae), E.coli, and L.monocytogenes Most common cause of neonatal sepsis in hospitals in India is - KLEBSIELLAMost common cause of neonatal sepsis in hospitals across the world is - E.COLI", "cop": 1, "opa": "S. agalactiae", "opb": "Streptococcus", "opc": "Pneumococcus", "opd": "E. coli", "subject_name": "Pediatrics", "topic_name": "All India exam", "id": "91867458-abb5-4691-8e00-da10488bb48b", "choice_type": "single"} {"question": "A 6 months old baby was brought do difficulty in feeding. '' The child was found to be hypotonic with a weak gag. The child is on breast milk and mother also gives honey to the child during periods of excessive crying. The causative agent is", "exp": "Infant botulism is associated with intake of honey, breastmilk etc. The child presents with generalized weakness and hypotonia in addition to bulbar palsies with symptoms like poor feeding, feeble cry,weak suck,drooling and obstructive apnea. Reference: Nelson textbook of pediatrics 19th edition Page 988", "cop": 3, "opa": "Gram positive aerobic COCCUS", "opb": "Gram positive anaerobic spre-suffering bacillus", "opc": "Toxin produced by gram positive anaerobic bacillus", "opd": "Echovirus", "subject_name": "Pediatrics", "topic_name": "Infectious disease", "id": "d116d4b1-ae26-4479-b577-995a16d891d9", "choice_type": "single"} {"question": "Attention deficit hyperactivity disorder is seen in", "exp": "ADHD:\n\nAttention Deficit Hyperactivity Disorder.\nDifficulty in paying attention, difficulty in controlling behavior and hyperactivity.\nEtiology- both genetic and environmental factors. Abnormality in dopamine transporter and thyroid receptor beta genes in some patients.\n\nThree subtypes are known:\n\nPredominantly hyperactive-impulsive\nPredominantly inattentive\nCombined hyperactive-impulsive and inattentive.\n\nDiagnosis is primarily clinical, using thorough clinical interview of parents and use of behavior rating scales. Physical examination includes direct observation of the child and ruling out chronic systemic illnesses that affect child's attention span.", "cop": 4, "opa": "Insulin resistance", "opb": "Thyroid hormone resistance", "opc": "Testosterone deficiency", "opd": "Calmodulin Deficiency", "subject_name": "Pediatrics", "topic_name": null, "id": "e235d36d-80c6-4d5e-b314-8b2675ebf537", "choice_type": "single"} {"question": "Sodium content in normal saline and Ringer lactate respectively are", "exp": "Sodium Content in\na) Normal saline = 154 meq / L\nb) Ringer lactate = 130 meq / L", "cop": 1, "opa": "154 meq / L; 130 meq / L", "opb": "140 meq / L; 109 meq / L", "opc": "150 meq / L; 136 meq / L", "opd": "134 meq / L; 150 meq / L", "subject_name": "Pediatrics", "topic_name": null, "id": "d75ad01e-2289-40df-84bf-7e4cfdbc984c", "choice_type": "single"} {"question": "The commonest cause of intestinal obstruction in children between 3 months to 6 years of age", "exp": "Ans. a (Intussusception) (Ref. Nelson paediatrics, p. 1569)INTUSSUSCEPTION# Intussusception occurs when a portion of the alimentary tract is telescoped into an adjacent segment. It is the most common cause of intestinal obstruction between 3 mo and 5 yr of age.# Sixty percent of patients are younger than 1 yr, and 80% of the cases occur before 24 mo: it is rare in neonates.# The incidence varies from 1 to 4/1,000 live births.# The male:female ratio is 4:1. A few intussusceptions reduce spontaneously, but if left untreated, most will lead to intestinal infarction, perforation, peritonitis, and death.Etiology and Epidemiology# The cause of most intussusceptions is unknown.# The seasonal incidence has peaks in spring and autumn.# Correlation with prior or concurrent respiratory adenovirus (type C) infection has been noted, and the condition may complicate otitis media, gastroenteritis, Henoch-Schonlein purpura.# The risk of intussusceptions in infants <1 yr of age after receiving a no longer available tetravalent rhesus-human reassortant rotavirus vaccine within 2 wk of immunization was increased.# Lymphoid nodular hyperplasia is another related risk factor. Prominent mounds of lymph tissue lead to mucosal prolapse of the ileum into the colon, thus causing an intussusception.# In 2-8% of patients, recognizable lead points for the intussusception are found, such as a Meckel diverticulum, intestinal polyp, neurofibroma, intestinal duplication, hemangioma, or malignant conditions such as lymphoma. Intussusception can complicate mucosal hemorrhage, as in Henoch-Schonlein purpura or hemophilia.# Cystic fibrosis is another risk factor.# Postoperative intussusception is ileoileal and usually occurs within 5 days of an abdominal operation.# Lead points are more common in children >2 yr of age.Pathology# Intussusceptions are most often ileocolic, less commonly cecocolic, and rarely exclusively ileal.Clinical manifestations# Spasmodic intermittent abdominal pain;# \"Red currant\" jelly stools# Palpable lump.# Dance sign, n \"Empty RIF\" signDiagnosis# Ultrasound is investigation of choice.# Ba enema is now 2nd line investigation# USG signs:- \"Doughnut or target\" appearance in transverse images.- \"Sandwich or hamburger\" sign in longitudinal images- Pseudokidney sign# Ba emema signs:- \"Claw\"sign- \"Coiled-spring\" signRx# Hydrostatic reduction# Air reduction is associated with fewer complications and lower radiation exposure than traditional contrast hydrostatic techniques.# The success rate of radiologic hydrostatic reduction under fluoroscopic or ultrasonic guidance is =50% if symptoms are present longer than 48 hr and 70-90% if reduction is done in the 1st 48 hr.# If manual operative reduction is impossible or the bowel is not viable, resection of the intussusceptions is necessary with end-to-end anastomosis.# Corticosteroids may reduce the frequency of recurrent intussusception.", "cop": 1, "opa": "Intussusception", "opb": "Meckel's diverticulum", "opc": "Intestinal polyp", "opd": "Nonspecific cause", "subject_name": "Pediatrics", "topic_name": "Gastro Intestinal System", "id": "2f396d6f-a448-41a5-9104-8ea6d148830e", "choice_type": "single"} {"question": "Baby is not at risk for hypoglycemia", "exp": "newborns are at risk for hypoglycemia : Babies born to mothers with diabetes. Babies who are small for gestational age or growth-restricted. Preterm babies, especially those with low bih weights. Babies born under significant stress. Babies with mothers treated with ceain medicines such as terbutaline. Reference: GHAI Essential pediatrics, 8th edition", "cop": 1, "opa": "Post dated pregnancy", "opb": "Mother on beta blocker", "opc": "Mother with diabetes mellitus", "opd": "IUGR", "subject_name": "Pediatrics", "topic_name": "New born infants", "id": "4dc6995c-18e8-4abe-b37c-a00bb2d5d2a2", "choice_type": "single"} {"question": "Child begins to sit with suppo, able to transfer objects from one hand to another hand and speak monosyllabic babbles at the age of", "exp": "

. Developmental milestones:- GROSS MOTOR DEVELOPMENT: 2 months: Holds head in plane of rest of the body when held in ventral suspension. In prone position in bed, the chin lifts momentarily. 3 months:lift head above the plane of the body. Head control stas by 3 months and fully developed by 5 months. 4 months:Remain on forearm suppo if put in prone position, lifting the upper pa of the body off the bed. 5 months: Rolls over. 6 months:sit in tripod fashion. 8 months: sits without suppo., crawling 9 months: Takes a few steps with one hand held. Pulls to standing and cruises holding on to furniture by 10 months. 10 months: creeps 12 months:creeps well, walk but falls, stand without suppo. 15 months: walks well, walks backward/ sideways pulling a toy. May crawl upstairs. 18 months: Runs, walks upstair with one hand held. Explores drawers 2 years: walk up and downstairs, jumps. 3 years : rides tricycle, alternate feet going upstairs. 4 years: hops on one foot, alternate feet going downstairs. 5 years:skips FINE MOTOR DEVELOPMENT:- 2 months- eyes follow objects to 180 deg. 3 months-Grasp reflex disappears and hand is open most of the time. 4 months- Bidextrous approach( reaching out for objects with both hands). 6 months- Unidextrous approach( Reach for an object with one hand).transfer object from one hand to other. 8 months- radial grasp sta to develop. Turns to sound above the level of ear. 9 months- immature pincer grasp, probes with forefinger. 12 months-Unassisted pincer grasp. Releases object on request.Uses objects predominantly for playing, not for mouthing. Holds block on each hand and bang them together. 15 months- imitate scribbling , tower of two blocks 18 months- scribbles, tower of 3 blocks.turn pages of a book, 2-3 at a time. 2 years- tower of 6 blocks, veical and circular stroke. 3 years-Tower of 9 blocks, dressing and undressing with some help, can do buttoning. 4 years- copies cross, bridge with blocks 5 years- copies triangle, gate with blocks. SOCIAL AND ADAPTIVE MILESTONES: 2 months: social smile(smile after being talked to).watches mother when spoken to and may smile. 3 months:Recognizes mother, anticipates feeds. 4 months: Holds rattle when placed in hand and regards it . Laughs aloud. Excited at the sight of food. 6 months:recognizes strangers, stranger anxiety . Enjoy watching own image in mirror, shows displeasure when toy pulled off. 9 months:waves bye bye 12 months:comes when called, plays simple ball game.kisses the parent on request. Makes postural adjustments for dressing. 15 months:jargon, stas imitating mother. 18 months: copies parents in tasking, dry by day, calls mother when he wants potty, points to three pas of body on request. 2 years: ask for food, drink, toilet, pulls people to show toys. 3 years:shares toys, know fullname and gender, dry by night. 4 years:Plays cooperatively in a group, goes to toilet alone, washes face, brushes teeth. Role play . 5 years:helps in household task , dresses and undresses. LANGUAGE MILESTONES: 1 month: Ales to sound. 2 month:respond to sound by stale or quitening to a smooth voice. 3 months: babbles when spoken to. Makes sounds (ahh,coos, ) laughs. 4 months: laughs aloud. 6 months: monosyllables 9 months: understands spoken words, bisyllables. 12 months: 1-2 words with meaning. 18 months: vocabulary of 10 words. Can name one pa of body. 2 years: 3 word simple sentences 3 years:asks questions, knows full name and gender. 4 years: says songs or poem, tells story, knows three colours. 5 years: ask meaning of words. {Reference: GHAI Essential pediatrics, eighth edition}", "cop": 2, "opa": "3 months", "opb": "6 months", "opc": "9 months", "opd": "12 months", "subject_name": "Pediatrics", "topic_name": "Growth and development", "id": "d4152b4b-88e0-456b-ae4b-ba03c0cbd641", "choice_type": "single"} {"question": "The likely diagnosis in a child with limb pain and pancytopenia is", "exp": "Child with acute lymphocytic leukemia presents with pallor, petechiae, infection(pancytopenia),. Reference: GHAI Essential pediatrics, 8th edition", "cop": 2, "opa": "Aplastic anemia", "opb": "Acute lymphocytic leukemia", "opc": "Rheumatic fever", "opd": "Rheumatoid ahritis", "subject_name": "Pediatrics", "topic_name": "Hematology", "id": "d392d7ff-0c3a-469e-906e-0934414184c6", "choice_type": "single"} {"question": "Ponds fracture is most common in", "exp": ".The Ping-Pong skull fracture or a pond skullfracture refers to a depressed skull fracture of the infant skull caused by inner buckling of the calvarium.", "cop": 1, "opa": "Children", "opb": "Adult", "opc": "Old age", "opd": "No relation with age", "subject_name": "Pediatrics", "topic_name": "Musculoskeletal disorders", "id": "fbfb8841-758d-41c2-a143-963fb20f7439", "choice_type": "single"} {"question": "DPT is contraindicated in", "exp": "C. i.e. (Progressive neurological illness) (191 - Ghai 6th) (168- 69- Ghai 7th)Following are NOT contraindication for administration of vaccine1. Mild acute illness2. Low grade fever3. Mild diarrheal4. Current antibiotic therapy5. Prematurity6. Recent exposure to an infectious disease7. History of allergies8. Allergy to penicillin9. Malnutrition* Children suffering from progressive neurological disorders or with history of convulsions to a previous dose are at a higher risk of adverse reactions following whole cell pertuessis vaccine* The vaccine is relatively contraindicated in children with progressive neurological disease, but children with stable neurological disease like developmental delay cerebral palsy and idiopathic epilepsy may be vaccinated (169 - Ghai 7th)", "cop": 3, "opa": "Family history of convulsions", "opb": "Acute respiratory tract infections", "opc": "Progressive neurological illness", "opd": "Mild diarrhea", "subject_name": "Pediatrics", "topic_name": "Immunization", "id": "150cfec7-4092-49b4-b4e2-664156528382", "choice_type": "single"} {"question": "Membraneous gap seen between fetal skull bone is known as", "exp": "Ans) a (Fontanelle) Ref D.C. Dutta p84,Nehon 18th ed p 677Flat bones of the vault are united together by nonossified membranes attached to the margin of the bone.These are called sutures and fontanelles.Wide gap in the suture line is called fontanelleCraniosynostosis is defined as premature closure of the cranial suturesDisorders Associated with a Large Anterior FontanelAchondroplisinIntrauterine growth retardationApert syndromeKenny syndromeAthyrotic hypothyroidismOsteogenesis imperfectnCleidocrininl dysostosisPrematurityCongenital rubella syndromePylenodysctslosisHallermantl-Sucill syndromeRussell-Silver syndromeHydrocephaly13-, 18-, 21-trisoniesHypophospnatasiaVitamin D deficiency ricketsPersistently small fontanels suggest microcephaly, craniosynostosis, con-genital hyperthyroidism, or wormian bones. A 3rd fontanel suggests trisomy 21 but is seen in preterm infants. Soft areas in the occipital region suggest the irregular calcification and wonnian bone formation.Important causes of wonnian bones Hypothyroidism/ cretinismHypophosphatasia RcketsCliedocraniodysostosis / pyknodysostosis Osteogenesis imperfecta", "cop": 1, "opa": "Fontanelle", "opb": "Suture", "opc": "Wormian bone", "opd": "Craniosynostosis", "subject_name": "Pediatrics", "topic_name": "Growth, Development, and Behavior", "id": "0397caba-5f9c-4cba-81ec-225677849e6c", "choice_type": "single"} {"question": "Non immune hydrops fetalis is caused by", "exp": "Primary fetal infection is associated with nonimmune fetal hydrops and intrauterine fetal demise,with the risk for fetal loss after infection estimated at <5%. The mechanism of fetal disease appears to be a viral-induced RBC aplasia at a time when the fetal erythroid fraction is rapidly expanding ,leading to profound anemia, high-output cardiac failure, and fetal hydrops. Nonimmune type * Three major causes -- parvovirus B 19 -- alfa-thalassemia -- cardiovascular defects, chromosomal anomalies, and fetal anemia Reference: Nelson textbook of pediatrics 19th edition, Page 1096", "cop": 4, "opa": "HIV", "opb": "CMV", "opc": "HSV", "opd": "Parvovirus B 19", "subject_name": "Pediatrics", "topic_name": "Infectious disease", "id": "8b16fec6-04a4-4359-ba9e-12c0f00ef2c1", "choice_type": "single"} {"question": "A 4 year child presents with a history of hoarseness, croupy cough and aphonia, the child has dyspnoea with wheezing. The most probable diagnosis is", "exp": "The given symptoms points to laryngeal foreign body. the course of illness depends on the nature of foreign body,its size,extent,and site of lodgement. Ref : Essential pediatrics,O.P.Ghai,7 th edition,pg no:367", "cop": 2, "opa": "Asthmatic bronchitis", "opb": "Laryngeal Foreign body", "opc": "Bronchopneumonia", "opd": "Retropharyngeal abscess", "subject_name": "Pediatrics", "topic_name": "Respiratory system", "id": "4388fa31-59ab-4c8e-862d-ac77f2090e0a", "choice_type": "single"} {"question": "A neonate is suspected to be suffering from necrotizing entercolitis (NEC). On fuhur examination and investigation, he is diagnosed to be Bell&;s stage INEC. The management of choice would be", "exp": ".Abdomen radiographs may show pneumatosis intestinalis,or free intraperitoneal air. management consists of agressive resuscitation and intravenous feeding.The optimal time for surgery is not in the acute phase as the baby can withstand the pressure of necrosis better than an adult,but not in the stress laprotomy. BAILEY AND LOVE&;S SHO PRACTICE OF SURGERY,PG NO:1200, 24th edition", "cop": 3, "opa": "Laporotomy and proceed", "opb": "Inseion of bilateral pelvic drains", "opc": "Conservative management with IV fluids and antibiotics", "opd": "Initial conservative management and laparotomy after 24 hours", "subject_name": "Pediatrics", "topic_name": "Gastrointestinal tract", "id": "b33aca63-309d-4307-85d4-23c53154b6c6", "choice_type": "single"} {"question": "Apnea of prematurity is defined as sudden stoppage of breathing that lasts for", "exp": "Apnea of prematurity defined as sudden stoppage of breathing that lasts for 20 sec or is associated with bradycardia or cyanosisApnea of prematurity should be different from periodic breathing which is normal phenomenon is preterm neonate.(Refer: Nelson's Textbook of Pediatrics, SAE, 1st edition, pg no. 832, 849 - 850)", "cop": 3, "opa": "10 sec", "opb": "15 sec", "opc": "20 sec", "opd": "30 sec", "subject_name": "Pediatrics", "topic_name": "All India exam", "id": "81589859-4353-42ac-87c9-bcfee4837f5a", "choice_type": "single"} {"question": "Sodium content of ReSoMal is", "exp": "Fish are naturally low in sodium and even those species with the highest sodium levels contain less than 100 milligrams per 3 ounce cooked poion. Most shellfish generally have more sodium, ranging from 100 to 500 milligrams per 3 ounce cooked serving. Reference: GHAI Essential pediatrics, 8th edition", "cop": 3, "opa": "90 mmol/lit", "opb": "60 mmol/lit", "opc": "45 mmol/lit", "opd": "30 mmol/lit", "subject_name": "Pediatrics", "topic_name": "Fluid and electrolytes", "id": "27aa38b1-1332-416a-9b83-27d6b78c6183", "choice_type": "single"} {"question": "A five year old child presents to the emergency department with burns. The burn area corresponding to the size of his palm is equal to", "exp": "Estimation of body surface area (BSA) of burn\nRough estimate of body surface area of bum can be estimated by following rules.\nRule of Palm (Palmar method)\n\nIt can be used in Children as well as in adults.\nIt is used for small burns (<10% of BSA)\nThe palm of patients hand represents I% of BSA. Palm is the area of hand from wrist crease to the finger crease.\nEntire burned hand represents 2.5% of BSA: 1/% for palm; 1% for dorsum of the hand (excluding fingers);0. 5% for the fingers\n\nRule of 9\n\nIt can be used in adults and children older than 14 years.\nIt is used for larger surface area burns.\nEstimates the body surface area of an adult in multiples of 9.\nAn adult who has been burned, the percent of the body involved can be calculated as follows:\nHead and Neck = 9%\nAnterior chest = 9%\nAnterior abdomen =9%\nUpper back = 9%\nLower back = 9%\nRight upper extremity (Anterior + Posterior) = 9%\nLeft upper extremity (Anterior + Posterior) = 9%\nRight lower extensity, Anterior = 9%\nRight lower extremity, Posterior = 9%\nLeft lower extremity, anterior = 9%\nLeft lower extremity, Posterior = 9%\nPerenium= 1%\nPercent of the burn in a child can be calculated as follows:\nHead and neck = 9%\nAnterior trunk =18%\nPosterior trunk =18%\nUpper extremity (Right) = 9%\nUpper extremity (Left) = 9%\nLower extremity (Right) = 18%\nLower extremity (Left) =18%\nPerenium = 1%", "cop": 1, "opa": "1% BSA", "opb": "5% BSA", "opc": "10% BSA", "opd": "20% BSA", "subject_name": "Pediatrics", "topic_name": null, "id": "dc57dbe3-e608-4714-aeeb-22298466e711", "choice_type": "single"} {"question": "The most common symptom of AIDS in an infant is", "exp": "Clinical features in an HIV infected infant is:Recurrent GI infection and oral thrush is commonly foundClinical features in older children:Growth failureFeverDiarrheaSecondary infection(Refer: OP Ghai's Textbook of Pediatrics, 8th edition, pg no. 202)", "cop": 1, "opa": "Gastrointestinal infection", "opb": "Persistent cough", "opc": "Failure to thrive", "opd": "Lymphadenopathy", "subject_name": "Pediatrics", "topic_name": "All India exam", "id": "d06a9b48-4fa9-44cd-8c56-12e46b1f5d2a", "choice_type": "single"} {"question": "A child presented with fever for 2 days, altered sensorium and purpuric rashes. His blood pressure is 90/60 mmHg. Treatment of choice is>", "exp": "Ans. is 'c' i.e. Penicillin .The clinical presentation of fever, altered sensorium and purpuric rashes, is highly suggestive of meningococcal meningitis.Purpuric rash in quite characteristic of meningococcemia.I. V. Penicillin is the treatment of choice for meningoccal inf. among the given options.The DOC for meningococcal infection is -IIIrd generation cephalosporin e.g. Cefotaxime, Ceftriaxone.Also RememberWaterhouse Friderichsen syndromeSeen in fulminant meningococcemiaadrenal insufficiency occurs* due Haemorrhage and necrosis in the adrenal glandsClinical presentation isprostrationhypotensionshock andUltimately coma and death.", "cop": 3, "opa": "IV Quinine", "opb": "IV Artesunate", "opc": "IV Penicillin", "opd": "Chloroquine", "subject_name": "Pediatrics", "topic_name": "CNS Infections", "id": "774b74b7-8fd5-4841-94a7-eddf0d7a94d0", "choice_type": "single"} {"question": "Pneumatoceles on chest radiograms in a child with pneumonia are seen in infection", "exp": "staphylococcal pneumonia : pneumonic process is diffuse but soon lesions suppurate and cause bronchoalveolar destruction multiple microabscesses erode the bronchial wall and discharge their contents in bronchi. several pneumatocoeles form and they fluctuate in size over time ultimately resolving and disappearing in few weeks to months in Xray pneumatocoeles persist as thin walled cysts.they are also seen in klebsiella pneumonia Ref : ESSENTIAL PEDIATRICS,O.P.GHAI, PG NO:353,7thedition", "cop": 1, "opa": "Staphylococcus", "opb": "Pneumococcus", "opc": "Streptococcus", "opd": "Haemophilus influenzae", "subject_name": "Pediatrics", "topic_name": "Respiratory system", "id": "74b2c563-d65e-4ddb-bc17-f487f8f97170", "choice_type": "single"} {"question": "The ratio of ICF volume to Extracellular fluid volume reaches adult values at the age of", "exp": "Total body water is divided between 2 main compaments: intracellular fluid (ICF) and extracellular fluid (ECF). In the fetus and newborn, the ECF volume is larger than the ICF volume. The normal postnatal diuresis causes an immediate decrease in the ECF volume. This is followed by continued expansion of the ICF volume, which results from cellular growth. By 1year of age, the ratio of the ICF volume to the ECF volume approaches adult levels.Reference: Nelson Textbook of Paediatrics; 20th edition; Chapter 55; Electrolyte and Acid-Base Disorders", "cop": 1, "opa": "1 year", "opb": "2 years", "opc": "3 years", "opd": "4 years", "subject_name": "Pediatrics", "topic_name": "Fluid and electrolytes", "id": "664f1688-1068-480d-992b-441e8ad0ce8f", "choice_type": "single"} {"question": "The pediatric disease that most closely resembles amyotrophic lateral sclerosis (ALS) is", "exp": "Werdnig-Hoffmann disease most closely resembles amyotrophic lateral sclerosis (ALS). Both of these diseases involve the degeneration of motor neurons. Presentation Symptoms symmetric flaccid paralysis most affected infants are hypotonic at bih proximal muscle involvement lower extremity greater than upper extremity sparing of upper cranial nerves e.g., normal eye movements Physical exam absent or decreased deep tendon reflexes tongue and finger fasciculatons infants have flaccid \"frog-like\" posture restrictive respiratory insufficiency ALS appears to be increasing in incidence. In 5% to 10% of cases, there is an autosomal dominant pattern with strong age-dependent penetrance. Proposed etiologies include oxidative stress, viral infection, immunologic disease, or some unknown environmental factor. Currently, the oxidative stress theory is ored because a defect in the zinc-copper binding superoxide dismutase (5001) coded on chromosome 21 was discovered. Because SO 01 is an antioxidant that conves the superoxide free radical into peroxide and oxygen, reduced activity causes apoptosis (individual cell necrosis) of spinal motor neurons. Inhibition of glutamate transpo potentiates the toxicity associated with the reduced SODllevels. ALS most commonly presents with both upper motor neuron signs (e.g., spastic paralysis) and, eventually, lower motor neuron signs (e.g., muscle atrophy, fasciculations). Atrophy of the intrinsic muscles of the hand and forearms with hand weakness and spastic changes in the lower legs are early signs. Antioxidant cocktails have now been developed, which offer some symptomatic improvement. Werdnig- Hoffmann disease is a progressive muscular atrophy noted in infants. It often presents as the floppy child syndrome. Reference: GHAI Essential pediatrics, 8th edition", "cop": 3, "opa": "Schilder disease (adrenoleukodystrophy)", "opb": "Tabes dorsalis", "opc": "Werdnig-Hoffmann disease", "opd": "Gauches disease", "subject_name": "Pediatrics", "topic_name": "Central Nervous system", "id": "7bc0db96-c370-40ce-be17-9fb4e1e110f7", "choice_type": "single"} {"question": "Most common type of Atrial septal defect is", "exp": "The atrial septal defect can occur in any poion of the atrial septum(secundum, primum or sinus venosus)OSTIUM SECUNDUM:Most common type of ASD accounts for 7% of al congenital hea disease.Defect situated at fossa ovalis.Associated with structurally normal atrioventricular (AV)valves.OSTIUM PRIMUM/ ENDOCARDIAL CUSHION DEFECT:Defect situated inferior to fossa ovalis.Associated with the cleft in AV valves.Most common cardiac lesion in Down&;s syndrome is endocardial cushion defect.", "cop": 2, "opa": "Ostium primum", "opb": "Ostium secundum", "opc": "Cushion defect", "opd": "Endocardial hyperophy", "subject_name": "Pediatrics", "topic_name": "All India exam", "id": "7ef128e6-8f19-486f-870e-d38aa437c2d4", "choice_type": "single"} {"question": "Bart's hydrops fetalis is lethal because", "exp": "Bart hydrops fetalis is a lethal complication of alpha thalassemia trait where oxygen can't release from hemoglobin", "cop": 2, "opa": "Hb Bart's cannot bind oxygen", "opb": "Hb Bart's cannot release oxygen to fetal tissues", "opc": "Microcytic red cells become trapped in the placental", "opd": "The excess a - globin form insoluble precipitates", "subject_name": "Pediatrics", "topic_name": null, "id": "a8c3cb3f-6d36-44ef-b8a2-0ce731cc975b", "choice_type": "single"} {"question": "The development of septal defects in the fetal hea occurs at", "exp": "Fetal hea septal defect develop by 6-8 weeks. Ref : Ghai essential of pediatrics, eighth edition, p.no: 402", "cop": 2, "opa": "3-5 weeks", "opb": "6-8 weeks", "opc": "9-12 weeks", "opd": "13-15 weeks", "subject_name": "Pediatrics", "topic_name": "C.V.S", "id": "266c645d-9f93-4cb5-9619-33b8113750cd", "choice_type": "single"} {"question": "NOT an indication for surgery in Ventricular septal\ndefect", "exp": "Eisenmenger's Syndrome is an absolute contraindication for Surgery in Ventricular septal defect.", "cop": 4, "opa": "Hepatomegaly", "opb": "Suck- Rest-Suck cycle while feeding", "opc": "Pulmonary: Systemic blood flow > 2:1", "opd": "Eisenmenger's Syndrome", "subject_name": "Pediatrics", "topic_name": null, "id": "ceddf62b-479e-4baf-a5d5-e5493782909a", "choice_type": "single"} {"question": "Most common lobe involved in congenital lobar emphysema", "exp": "Congenital lobar emphysema present with respiratory distress. Most common lobe involved is Left upper lobe. X-ray shows shift of mediastinum to Right side.", "cop": 3, "opa": "Right upper lobe", "opb": "Right middle lobe", "opc": "Left upper lobe", "opd": "Left lower lobe", "subject_name": "Pediatrics", "topic_name": null, "id": "9b138448-df41-43a3-8ef5-f6aca1dee3c1", "choice_type": "single"} {"question": "Most common cause of Acute epiglottis is", "exp": "Most common cause of Acute epiglottis is Hemophilius influenza B", "cop": 4, "opa": "Respiratory Syncytial Virus", "opb": "Parainfluenza virus", "opc": "Hemophilius influenza A", "opd": "Hemophilius influenza B", "subject_name": "Pediatrics", "topic_name": null, "id": "b86f609b-a355-48ca-a4e1-4d35ab33eb27", "choice_type": "single"} {"question": "Sign of pubey in boys is", "exp": "In girls, the first visible sign of pubey is the appearance of breast buds (Thelarche), between 8-12 years of ageIn boys the first visible sign of pubey is testicular enlargement, beginning as early as 9 1/2 yr.(Refer: Nelson's Textbook of Pediatrics, SAE, 1st edition, pg no. 926)", "cop": 2, "opa": "Enlargement of penis", "opb": "Enlargement of testes", "opc": "Appearance of pubic hair", "opd": "Appearance of axillary hair", "subject_name": "Pediatrics", "topic_name": "All India exam", "id": "e1f7fcc6-da4f-46ba-8367-9f8f66420766", "choice_type": "single"} {"question": "Most common delivery room emergency in newborns", "exp": "Ans. d (Failure to initiate efficient respiration) (Ref. Nelson textbook of paediatrics, p. 723)The most common delivery room emergency for neonates is secondary to failure to initiate and maintain effective respirations. Less frequently, but of major importance are shock, severe anemia, plethora, convulsions and management of life-threatening congenital malformation.Educational Points:# NEC is the most common life-threatening emergency of the gastrointestinal tract in the newborn period.# ABO incompatibility is the most common cause of hemolytic disease of the newborn.", "cop": 4, "opa": "Shock", "opb": "Life threatening congenital malformations", "opc": "Convulsions", "opd": "Failure to initiate efficient respiration", "subject_name": "Pediatrics", "topic_name": "New Born Infants", "id": "6ac2f17f-566b-419f-9f0f-62bb70ce6a54", "choice_type": "single"} {"question": "Para amino benzoic acid in breast milk prevents the infection of", "exp": "Para-amino-benzoic acid(PABA) of breast milk provides protection against malaria.Ref: Page 97; Ghai essential pediatrics; 6th edition", "cop": 1, "opa": "Plasmodium vivax", "opb": "Kleibsella-pneumonia", "opc": "Giardia", "opd": "E.coli", "subject_name": "Pediatrics", "topic_name": "Nutrition", "id": "44a325aa-8b46-49ba-9fd5-4d79cad6bb5d", "choice_type": "single"} {"question": "In neonatal cholestasis, if the serum gammaglutamyl transpeptidase is more than 600IU/L the most likely diagnosis is.", "exp": "Ans. is 'd' i.e., Biliary atresia Most of the books are of the opinion that alkaline phosphatase and gamma glutamyl transpeptidase are two enzymes that are associated with biliary tract disease.Serum levels of these enzymes are usually increased in cholestasis or bile duct obstruction.But the problem in that, all of these books also add that they may also be increased in parenchymal disease or infiltrative or mass lesions of the liverIn the question only the value of Gamma glutamyl transpeptidase is given. Nothing is mentioned about the value of other enzymes.We cannot come to a conclusive diagnosis only by the level of Gamma glutamyl transpeptidaseBut still such a considerable elevation in the serum level of Gamma glutamyl transpeptidase i.e., 10 to 12 times its normal value points towards the diagnosis of biliary tract diseaseNote:- Normal value of serum Gamma glutamyl transpeptidase is - 10-40IU/L", "cop": 4, "opa": "Neonatal Hepatitis", "opb": "Choledochal cyst", "opc": "Hypothyroidism", "opd": "Biliary atrisia", "subject_name": "Pediatrics", "topic_name": "Liver and Biliary System", "id": "1c4b55eb-fa82-41db-a2be-49d7c47557a8", "choice_type": "single"} {"question": "A single umbilical aery in newborn increases the risk of", "exp": "The umbilical cord should have 2 aeries and 1 vein. A single umbilical aery is associated with an increased risk for an occult renal anomaly.Reference: Nelson Textbook of Paediatrics; 20th edition; Chapter 94; The Newborn Infant", "cop": 2, "opa": "Meningomyelocoele", "opb": "Occult Renal anomaly", "opc": "Omphalocoele", "opd": "Gastrochisis", "subject_name": "Pediatrics", "topic_name": "New born infants", "id": "d8caff68-4d20-454f-9321-a3ced7c5ccee", "choice_type": "single"} {"question": "Hetch Giant cell pneumonia is caused by", "exp": "Hetch Giant cell pneumonia is caused by Rubeola or Measles virus.", "cop": 3, "opa": "Mumps virus", "opb": "Rubella", "opc": "Rubeola", "opd": "Varicella", "subject_name": "Pediatrics", "topic_name": null, "id": "8650b947-a5cc-4384-aaa3-4d1c88f6ff15", "choice_type": "single"} {"question": "Thrombocytopenia, macerated skin lesions, rash and periostitis in a new born are seen in", "exp": "In Syphilis, thrombocytopenia is often associated with platelet trapping in an enlarged spleen. Characteristic osteochondritis and periostitis and a mucocutaneous rash manifesting with erythematous maculopapular or vesiculobullous lesions followed by desquamation involving hands and feet are common. Reference : Nelson textbook of pediatrics 19th edition Page 1017 & 1018", "cop": 3, "opa": "Erythroblastosis fetalis", "opb": "Cytomegalovirils infection", "opc": "Syphilis", "opd": "HIV infection", "subject_name": "Pediatrics", "topic_name": "Infectious disease", "id": "77087998-31ef-40cd-a18f-a1c8f301c0e0", "choice_type": "single"} {"question": "Most common enzyme deficiency in congenital adrenal hyperplasia is", "exp": "congenital adrenal hyperplasia is a group of autosomal recessive defects in steroid synthesis characterised by deficiency of adrenocoical hormones The commonest form of congenital adrenal hyperplasia accounting for about 90% of cases is due to deficiency of 21-alpha hydroxylase It is associated with diminished synthesis of coisol and aldosterone Reference: Ghai TB of pediatrics 8th edition pg 526", "cop": 4, "opa": "3 beta hydroxylase", "opb": "11 beta hydroxylase", "opc": "17 hydroxylase", "opd": "21 alfa hydroxylase", "subject_name": "Pediatrics", "topic_name": "Genetic and genetic disorders", "id": "1681be1d-be68-465a-b2b1-41d89239b04f", "choice_type": "single"} {"question": "The most common etiological agent for acute bronchiolitis in infancy is", "exp": "Most cases of bronchiolitis are caused by the respiratory syncytial virus (RSV). RSV is a common virus that infects just about every child by the age of 2. Outbreaks of the RSV infection occur every winter. Bronchiolitis can also be caused by other viruses, including those thatcause the flu or the common cold. Bronchiolitis Lower respiratory tract (bronchioles) Microbiology : * Respiratory syncytial virus (RSV) * Rhinovirus. Parainfluenza virus, Human metapneumovirus * Influenza virus Clinical * Occurs primarily in children s 2-years-old * Often preceded by a 1 to 3 day URI prodrome * Fever. Cough * Respiratory distress (tachypnea, retractions, wheezing, crackles) Complications * Dehydration * Apnea (premature and < 2 months old) * Aspiration pneumonia, Respiratory failure Management Nonsevere bronchiolitis * Anticipatory guidance, nasal suctioning, hydration Severe bronchiolitis * Trial of inhaled bronchodilator (controversial) * Heated humidified high-flow nasal cannula * Continuous positive airway pressure * Endotracheal intubation Reference: GHAI Essential pediatrics, 8th edition", "cop": 4, "opa": "Influenza virus", "opb": "Parainfluenza virus", "opc": "Rhinovirus", "opd": "Respiratory syncytial virus", "subject_name": "Pediatrics", "topic_name": "Respiratory system", "id": "21dffd5e-d956-4a23-a5fb-922e415cb3bd", "choice_type": "single"} {"question": "Macrosomia is", "exp": "The term \"macrosomia\" is used to describe a newborn who&;s significantly larger than average ( large size baby ). A baby diagnosed with fetal macrosomia has a bih weight of more than 4,000 grams (8 pounds 13 ounces), regardless of his or her gestational age. Another definition of macrosomia is a bih weight above the 90th percentile, corrected for gestational age and sex. Factors associated with fetal macrosomia include high blood sugar levels from gestational diabetes or diabetes mellitus, ethnicity (Hispanic women are more at risk), obesity, gaining extra weight during the pregnancy, carrying baby past term, gender of fetus (males are more prone to macrosomia), history of a previous large baby. Reference : page 137 Ghai Essential Pediatrics 8th edition", "cop": 1, "opa": "Large size baby", "opb": "Big mouth", "opc": "Large head", "opd": "Large tongue", "subject_name": "Pediatrics", "topic_name": "New born infants", "id": "a7392e62-b5c1-4383-8e3f-6c09107ba633", "choice_type": "single"} {"question": "Definition of low birth weight", "exp": "Some important definitions in Neonatology Low birth weight: Birth weight of less than 2.5 kg irrespective of gestational age Very low birth weight: Birth weight of less than 1.5 kg irrespective of gestational age.\nExtremely low birth weight: Birth weight of less than 1.0 kg irrespective of gestational age Neonatal: First 28 days of postnatal life. Preterm:\nLess than 37 completed weeks of gestations Term: After 37 completed weeks of gestations Post term: After 42 completed weeks of gestations Past Dates:\nAfter 40 completed weeks of gestations Stillbirth: Delivering dead fetus after 28 weeks of gestation. (Defn changes with the country) Abortion:\nExpulsion of products of conception before 28 weeks of gestation. Small for Gestational Age (SGA):\nWeight below the I Oth percentile for the gestational age Large for Gestational Age (LGA):\nWeight above the 90th percentile for the gestational age Appropriate for Gestational Age (AGA): Weight between 10 lh & 90th percentile for the gestational age", "cop": 3, "opa": "Wt <2kg", "opb": "Wt < 1.5kg", "opc": "Wt <2.5kg", "opd": "Wt <1 k", "subject_name": "Pediatrics", "topic_name": null, "id": "5f3119bb-7c40-4a16-a295-4cbc6d14934f", "choice_type": "single"} {"question": "A 5 year old boy is suffering from mimimal change nephrotic syndrome. For bringing about a remission induction, the drug of choice is", "exp": "Because of the high prevalence of minimal-change disease (MCD) in children with nephrotic syndrome, an empiric trial of coicosteroids commonly is the first step in therapy. Coicosteroids are the treatment of choice, leading to complete remission of proteinuria in most cases. Reference: GHAI Essential pediatrics, 8th edition", "cop": 4, "opa": "Furosemide", "opb": "Levamisole", "opc": "Cyclophosphamide", "opd": "Prednisolone", "subject_name": "Pediatrics", "topic_name": "Urinary tract", "id": "29ed0cae-689a-430f-8227-50ec7ed7bdd3", "choice_type": "single"} {"question": "In &;Down&;s syndrome, the shape of the head is", "exp": "Down syndrome is due to trisomy 21. A child with down syndrome presents with brachycephaly with flat occiput. Other features include flat face,upward slanted palpebral fissure ,epicanthal folds,speckled irises and delayed fontanel closure . Image : characteristics of Down syndrome (DS) skull and brain. A: lateral views of euploid and DS skulls. Left: the face of an euploid young adult presents downward growth of the maxillae; therefore, the distance between the inferior orbital ridge, the nasal spine, and the alveolar crest is considerably increased. The mandible is angled. Right: although the DS skull grows to nearly the same size as the normal adult, it presents brachycephaly, which means \"sho headed,\" and occurs when the right and left coronal sutures close prematurely. Brachycephaly results in an abnormally broad head with a high forehead. It is often associated with other craniofacial abnormalities. In DS, the face is small, with underdeveloped maxillae, and the mandible is still relatively straight. Reference: Nelson TB of pediatrics 19th edit pg 402.", "cop": 3, "opa": "Oxycephalic", "opb": "Scaphocephalic", "opc": "Brachicephalic", "opd": "Plagiocephalic", "subject_name": "Pediatrics", "topic_name": "Genetic and genetic disorders", "id": "5a87deb5-bba7-424f-9431-e072c2282082", "choice_type": "single"} {"question": "Flaky paint appearance of skin is seen in", "exp": "There are skin lesions in kwashiorkor which consist of increased pigmentation, desquamation, and dyspigmentation. Pigmentation, when gets confluent, is known as flaky paint dermatitis. Ref: Page 100; Ghai Essential Pediatrics; 8th edition.", "cop": 4, "opa": "Dermatitis", "opb": "Pellagra", "opc": "Marasmus", "opd": "Kwashiorkor", "subject_name": "Pediatrics", "topic_name": "Nutrition", "id": "30089d3e-2a3c-4328-b3f4-6a800a5daeb0", "choice_type": "single"} {"question": "Most common organism causing neonatal meningitis is", "exp": "The most common causes of neonatal meningitis is bacterial infection of the blood, known as bacteremia (specifically group B streptococci (Streptococcus agalactiae), Escherichia coli, and Listeria monocytogenes . Reference: GHAI Essential pediatrics, 8th edition", "cop": 1, "opa": "E. coli", "opb": "Listeria", "opc": "Pseudomonas", "opd": "Staph aureus", "subject_name": "Pediatrics", "topic_name": "Central Nervous system", "id": "e0550b1a-21d3-452e-94ce-36b5910bfab2", "choice_type": "single"} {"question": "Most common cause of Extradual/Subdural Hemorrhage in children is", "exp": "(B) Skull fracture", "cop": 2, "opa": "A-V malformation", "opb": "Skull fracture", "opc": "Aneurysms", "opd": "Atherosclerosis", "subject_name": "Pediatrics", "topic_name": "Miscellaneous", "id": "aaa6c545-f413-457c-a0da-41ec956893b3", "choice_type": "single"} {"question": "Hemorrhagic disease of the newborn is attributed to the deficiency of", "exp": "• Vitamin K is produced by our gut flora. The gut of a New-born baby is sterile and hence no vitamin K is being produced.\nMoreover vitamin k is negligible in breast milk. In lieu of these factors all babies are susceptible to development of Hemorrhagic disease of New-born.\n• To prevent this from happening therefore, as standard operating procedure in all hospitals injection vitamin K 1 :\n1 mg intramuscular is given to prevent hemorrhagic disease of new born referred to as H.D.N.\n• In our country where a large number of children are born via home deliveries,\nno vitamin K would be given to the child and therefore the child may be brought to the hospital on day 5 with umbilical stump bleeding.\n• For management: the child should be given injection vitamin K lmg intravenously.\nIf bleeding is still present then FFP needs to be administered. Previous Year Questions on Topic: Bleeding Disorders in Children\n• Type of vitamin K used for treatment of H.D.N= vitamin K1\n• Umbilical stump bleeding in child on day 1 of life= factor 13 deficiency\n• Umbilical stump bleeding in child on day 5 of life= H.D.N\n• Male baby with excessive circumcision bleeding= hemophilia A\n• Vaginal bleeding in a girl child on day 5 of life= No intervention required", "cop": 1, "opa": "Vitamin K", "opb": "Vitamin A", "opc": "Vitamin E", "opd": "Vitamin C", "subject_name": "Pediatrics", "topic_name": null, "id": "b0965a2c-e952-46a7-a43a-ed340f810a00", "choice_type": "single"} {"question": "Wilm's tumour commonly presents as", "exp": "(Abdominal mass): Ref: 574-G (592-Ghai 7th)WILM'S TUMOR or nephroblastoma is the most common malignant tumour of the kidney* 80% of children under the 5 years (mean age 3years/* Congenital anomalies particularly -- Aniridia- Hemihypertrophy- Beckweith - Wiedmann syndrome- Mental retardation* Deletion in the region of 11 p 13Clinical Presentation* Most of the patients presents with a asymptomatic abdominal mass* Hematuria (10- 25%) Hypertension (about 25%)* Other frequent symptoms - abdominal pain (30%) fever (20%) anorexia and vomiting* Ultrasound scanning is the most important investigation", "cop": 2, "opa": "Pain in the abdomen", "opb": "Abdominal mass", "opc": "Fever", "opd": "Hematuria", "subject_name": "Pediatrics", "topic_name": "Kidney", "id": "776d8c50-c7e4-4ace-80c3-eda5e16aea3d", "choice_type": "single"} {"question": "Absolute C/I of Bag & Mask ventilation", "exp": "Absolute C/I of Bag and Mask ventilation is - CDH Also know Q The resuscitation bag should have capacity of 240 to 750 mL Reservoir +O2 attached - Fio2 delivered >90% Only reservoir attached - [?] 21% Only oxygen & no reservoir - [?] 40 Q Indications of PPV - 1. Apnea 2. Gasping 3. HR <100/min inspite of 100 percent oxygen Q Most successful indicator of successful resuscitation - Improvement in hea rate.", "cop": 2, "opa": "Tracheoesophageal fistula", "opb": "Congenital diaphragmatic hernia", "opc": "Choanal atresia", "opd": "Cleft palate", "subject_name": "Pediatrics", "topic_name": "Paediatrics", "id": "c5f1b094-b9cd-4496-afc9-5db1b074ea2b", "choice_type": "single"} {"question": "The natural probiotic activity in breast milk is due to", "exp": "The natural probiotic activity in the breast milk is due to lactobacillus it is the good bacteria of intestine . Reference: GHAI Essential pediatrics, 8th edition", "cop": 1, "opa": "Lactobacillus", "opb": "Sacchromyces boullardi", "opc": "Bifido bacterium", "opd": "Bacillus cereus", "subject_name": "Pediatrics", "topic_name": "New born infants", "id": "d081bd6b-0e67-4b92-9c5e-44ecf54e4e0c", "choice_type": "single"} {"question": "Most Common type of Roger's anomaly is", "exp": "Roger's anomaly is another name for ventricular septal defeat. \nMost common type is perimembranous type.", "cop": 4, "opa": "Muscular type", "opb": "Inlet type", "opc": "Outlet type", "opd": "Perimembranous type", "subject_name": "Pediatrics", "topic_name": null, "id": "98db56d0-adde-4132-b222-68aae11ee0fd", "choice_type": "single"} {"question": "A baby presents with diarrhea and perianal diaper area redness. Diagnosis is", "exp": "In Lactose intolerance, there is a deficiency of the enzyme lactaseSo there is no natural breakdown of lactose - a carbohydrate present in milkThis causes diarrheaThe stool contains reducing sugar which causes perianal excoriation(Refer: Nelson's Textbook of Pediatrics, SAE, 1st edition, pg no. 1876)", "cop": 1, "opa": "Lactose intolerance", "opb": "Shigella diarrhea", "opc": "Salmonella", "opd": "Fungal", "subject_name": "Pediatrics", "topic_name": "All India exam", "id": "594cf101-f3c9-445d-8704-725ea6f4aa27", "choice_type": "single"} {"question": "Localised Langerhans cells histiocytosis affecting head and neck is", "exp": "d. Eosinophilic granuloma(Ref: Nelson's 20/e p 2484-2489, Ghai 8/e p 620-623)Eosinophilic granuloma: Bone lesions, with no visceral involvement; Localised; affects head and neck", "cop": 4, "opa": "Letterer-Siwe disease", "opb": "Pulmonary Langerhans cell histiocytosis", "opc": "Hand-Schuller-Christian disease", "opd": "Eosinophilic granuloma", "subject_name": "Pediatrics", "topic_name": "C.V.S.", "id": "bdd33734-c747-431c-b865-4571fe94cd8e", "choice_type": "single"} {"question": "A 2 old boy with fever and rhinorrhea developed a maculopapular rash, to the face, the neck, chest, arms in that order. Diagnosis includes", "exp": "The disease is most common in preschool children; infants are protected by transplacental antibodies, which generally decay by 9 months The prodromal phase is characterized by fever, rhinorrhea, conjunctiva! congestion and a dry hacking cough. Koplik spots, considered as pathognomonic of measles, appear opposite the lower second molars on the buccal mucosa on the second or third day of the illness as grey or white lesions resembling grains of sand with surrounding erythema. The rash, usually appears on the fouh day with a rise in fever, appears as faint reddish macules behind the ears, along the hairline and on the posterior aspects of the cheeks. The rash rapidly becomes maculopapular and spreads to the face, the neck, chest, arms, trunk, thighs and legs in that order over the next 2-3 days. It then stas fading in the same order that it appeared and leaves behind branny desquamation and brownish discolouration, which fade over 10 days Refer : O.P Ghai 8e pg:213", "cop": 1, "opa": "Measles", "opb": "Meningococcal septicaemia", "opc": "Haemophilia", "opd": "Chicken pox", "subject_name": "Pediatrics", "topic_name": "Central Nervous system", "id": "1b91c0f4-fe57-44e0-880b-b1f93fbe06a0", "choice_type": "single"} {"question": "A 8 year old boy presents with recurrent pneumonia since birth. The boy had a history of delayed passage of Meconium at birth. Probable diagnosis is", "exp": "Features of Cystic fibrosis\n\nDelayed passage of Meconium\nRecurrent pneumonia\nSteatorrhea\nLate onset Diabetes\nAzoospermia", "cop": 3, "opa": "Kartagener's syndrome", "opb": "Bronchiectasis", "opc": "Cystic fibrosis", "opd": "Hirnchiprung disease", "subject_name": "Pediatrics", "topic_name": null, "id": "dccc20a4-6f29-4123-b06a-034b801eda63", "choice_type": "single"} {"question": "Normal anion gap metabolic acidosis can occur in", "exp": "In renal failure, there is retention of unmeasured anions, including phosphate, urate, and sulfate. The increase in unmeasured anions in renal failure is usually less than the decrease in the bicarbonate concentration. Renal failure is thus a mix of an increased gap and a normal gap metabolic acidosis. The normal gap metabolic acidosis is especially prominent in children with renal failure as a result of tubular damage, as occurs with renal dysplasia or obstructive uropathy, because these patients have a concurrent A.Other distractors are causes of high anion gap metabolic acidosis. Ref:- Nelson&;s textbook of paediatrics; pg num:- 574", "cop": 1, "opa": "Renal failure", "opb": "Liver failure", "opc": "Severe anemia", "opd": "Malignancy", "subject_name": "Pediatrics", "topic_name": "Metabolic disorders", "id": "91bf0367-7b8b-43ed-82ea-a05e8f9bd46f", "choice_type": "single"} {"question": "Diagnosis of beta Thalassemia is established by", "exp": "Diagnostic test for thalassemia is Hb electrophoresis. NESTROFT Test - Naked Eye Single Tube Red Cell Osmotic Fragility Test - used for screening for Thalassemia HbA1c - Used to assess long term glycemic control in diabetics (blood sugar control over past 3 months) Target cells - A feature of thalassemia, but not diagnostic Reference : Ghai essential of pediatrics, eighth edition, p.no:342", "cop": 3, "opa": "NESTROFT Test", "opb": "Hb A1c estimation", "opc": "Hb electrophoresis", "opd": "Target cells in peripheral smear", "subject_name": "Pediatrics", "topic_name": "Hematology", "id": "9fa97b8c-d9d3-4693-895c-e9b6afeb31b7", "choice_type": "single"} {"question": "Mutation in Hartnup's disease is", "exp": "SLC6A19 gene encodes neutral amino acid transporter. Its mutation is seen in Hartnup's disease.", "cop": 3, "opa": "SLC6A18", "opb": "SLC6A17", "opc": "SLC6A19", "opd": "SLC6A20", "subject_name": "Pediatrics", "topic_name": null, "id": "85a89d7b-a846-474d-b7e8-a5821b886cd4", "choice_type": "single"} {"question": "6 weeks old female baby found unconscious suddenly in the crib. She was previously healthy. Normal blood pressure, hyperpigmentation of genitals, blood glucose 30 mg/dl. Diagnosis is aEUR'", "exp": "Familial glucocoicoid deficiency Familial glucocoicoid deficiency is a rare autosomal recessive condition. It is characterized by adrenal insufficiency. The pathological examination of the adrenal gland reveals that:- This causes low coisol concentration because the zona fasciculata is primarily responsible for glucocoicoid production. Because the zona glomerulosa is well preserved mineralocoicoid action is usually unaffected. Low circulating serum coisol results in lack of feedback inhibition to the hypothalamus which results in increased ACTH secretion from pituitary. Clinical features Patients with .familial glucocoicoid deficiency generally presents with signs and symptoms of adrenal insufficiency with the impoant distinction that mineralocoicoid production is always normal. The most common initial presenting sign is \"deep hyperm,ementation\" of the skin, mucous membrane or both as a result of the action of adrenocoicotrophic hormone (ACTH) on cutaneous melanocyte stimulating hormone (MSH) receptors. The symptoms are compatible with glucocoicoid deficiency. Many patients presents 4vith recurrent hypoglycemia or severe infections. - In the neonatal period, frequent presenting signs include - feeding problems, failure to thrive, regurgitation and hypoglycemia manifesting as seizures", "cop": 2, "opa": "CAH due to 21-alpha hydroxylase deficiency", "opb": "Familial glucocoicoid deficiency", "opc": "Cushing syndrome", "opd": "Insulinoma", "subject_name": "Pediatrics", "topic_name": null, "id": "caff8829-b67a-4b5f-999c-2d77cc2b654e", "choice_type": "single"} {"question": "Infantile myocarditis and pericarditis is due to", "exp": "Ans. is 'b' i.e., Coxsackie B o The commonest cause of myocarditis is coxsackie B infection, which occurs any where from the age of a few hours to 7 weeks, with a peak around two weeks.", "cop": 2, "opa": "Coxsackie A", "opb": "Coxsackie B", "opc": "Mumps", "opd": "Pox virus", "subject_name": "Pediatrics", "topic_name": null, "id": "a78ee142-0316-4956-b077-3306e8d0e9ca", "choice_type": "single"} {"question": "Hypoplasia of limbs & scarring is caused due to aEUR'", "exp": "Varicella virus Congenital Varicella syndrome is characteristically associated with limb reduction defects (if infection occurs prior to limb bud formation) and scarring of skin The association of Limb reduction defects with scarring of skin suggests the diagnosis of congenital varicella syndrome. Limb reduction defects (Limb hypoplasia) and congenital varicella. Limb reduction defects are seen if infection occurs prior to or during limb bud formation. The virus has a tendency to select tissues that are in a rapid development stage such as the limb buds Fetus infected at 6-12 weeks of gestation appears to have maximal interruption with limb development. This may result in 1 or more shoened and malformed extremities. The remaining of the torso may be entirely normal in appearance. Scarring of Skin (Cicatrical skin lesions) and Varicella Scarring of skin is a common feature of congenital varicella syndrome. The characteristic cutaneous lesion has been called a 'Cicatrix' 'Cicatrix' represents zigzag scarring in a dermatomal distribution (and is often associated with atrophy of the affected limb) This characteristic cicatrical scaring is believed to represent the cutaneous residua of WV infection of the sensory nerves analogous to herpes zoster.", "cop": 1, "opa": "Varicella", "opb": "Herpes simplex", "opc": "Rubella", "opd": "Toxoplasma", "subject_name": "Pediatrics", "topic_name": null, "id": "b64f95a2-ab0e-45c1-a9ca-aa50b5d610a9", "choice_type": "single"} {"question": "Early strict toilet training can result in", "exp": "A condition in which a child resists having bowel movements, causing impacted stool to collect in the colon and rectum and lead to leakage. Common causes of this symptom Encopresis can have causes that aren&;t due to underlying disease. Examples include sneezing, constipation, incontinence due to lack of potty training or incontinence due to misinterpretation of body signals. Reference: GHAI Essential pediatrics, 8th edition", "cop": 2, "opa": "Nocturnal enuresis", "opb": "Encopresis", "opc": "Night terror", "opd": "Temper tantrauma", "subject_name": "Pediatrics", "topic_name": "Growth and development", "id": "a3da19ac-1e6f-4c7a-9e90-2ed8b303c71a", "choice_type": "single"} {"question": "Most common cardiac anamoly associated with Congenital Rubella Syndrome among the following is", "exp": "Most common cardiac anamoly associated with Congenital Rubella Syndrome is in the order patent ductus arteriosus > pulmonary stenosis > Ventricular septal defect.", "cop": 2, "opa": "Ventricular septal defect", "opb": "Pulmonary stenosis", "opc": "Atrial septal defect", "opd": "Truncus arteriosus", "subject_name": "Pediatrics", "topic_name": null, "id": "7f1c834c-d383-4e5d-9871-6cfa3c6925ab", "choice_type": "single"} {"question": "A new born presented with Jaundice. Most common diagnostic Investigation of choice is", "exp": "Bilirubin test : If it&;s thought your baby has jaundice, the level of bilirubin in their blood will need to be tested. This can be done using: a blood test of a sample of blood taken by pricking your baby&;s heel with a needle (the level of bilirubin in the liquid pa of the blood called the serum is then measured). Reference: GHAI Essential pediatrics, 8th edition", "cop": 1, "opa": "Total and Direct Bilirubin", "opb": "Conjugated Bilirubin", "opc": "Serum Bilirubin", "opd": "Uroporphyrin levels", "subject_name": "Pediatrics", "topic_name": "New born infants", "id": "c94be2c3-95cd-41a4-a8f5-2283ea35f562", "choice_type": "single"} {"question": "Embryonic hemoglobins among the following are", "exp": "Embryonic hemoglobins are Gower hemoglobin and Portland hemoglobin.", "cop": 4, "opa": "Fetal hemoglobin and Gower hemoglobin", "opb": "Fetal hemoglobin and Portland hemoglobin", "opc": "Portland hemoglobin and HbA2", "opd": "Gower hemoglobin and Portland hemoglobin", "subject_name": "Pediatrics", "topic_name": null, "id": "a33744dd-5e7f-4c17-8fd2-5a4f39b17f53", "choice_type": "single"} {"question": "Oral glucose tolerance test in children is done with", "exp": "Ideal weight of the glucose taken for glucose tolerance test for children are 1.75gm/kg glucose . Obtain a fasting blood sugar level, then administer an oral glucose load (2 g/kg for children aged < 3 y, 1.75 g/kg for children aged 3-10 y , or 75 g for children aged >10 y). Check the blood glucose concentration again after 2 hours. A fasting whole-blood glucose level higher than 120 mg/dL (6.7 mmol/L) or a 2-hour value higher than 200 mg/dL (11 mmol/L) indicates diabetes. However, mild elevations may not indicate diabetes when the patient has no symptoms and no diabetes-related antibodies. Reference: GHAI Essential pediatrics, 8th edition", "cop": 2, "opa": "1.5 gm/kg glucose", "opb": "1.75 gm/kg glucose", "opc": "2 gm/kg glucose", "opd": "2.5 gm/kg glucose", "subject_name": "Pediatrics", "topic_name": "Endocrinology", "id": "aa3af173-4296-4a04-84f4-6b67814329be", "choice_type": "single"} {"question": "Most sensitive indicator of intravascular volume depletion in infant aEUR'", "exp": "Hea Rate \"A significant misconception is that shock occurs only with low B.P.(hypotension). Through various compensatory mechanisms hypotension is often a late .finding. Tachycardia, with or without tachyapnea may be the 1' or only sign of early compensated shock\". Shock is an acute dramatic syndrome characterized by inadequate circulatory supply of oxygen so that the metabolic demands of vital organs and tissues are not met. Shock has been categorized into series of recognizable stage: ? Compensated Uncompensated Irreversible An initial insult triggers shock, thus disrupting blood flow to end organs leading to inadequate tissue perfusion. The body's compensatory mechanism is initiated to maintain perfusion to vital organs. If treatment is not introduced during this period, decompensated shock develops causing tissue damage that leads to multisystem organ dysfunction and death. Compensatory shock In the early phases of shock, multiple compensatory physiologic mechanisms act to maintain blood pressure and preserve tissue perfusion. - Blood pressure remains normal, initially, because of peripheral vasoconstriction and increased systemic vascular resistance. - Thus hypotension occurs late and is more a characteristic of uncompensated stage of shock. - Patients in compensated shock have relatively normal cardiac output and normal blood pressures but they have alteration in microcirculation that increase flow to some organs and reduce flow to other. - In infants compensatory increases in cardiac output are achieved primarily by \"tachycardia\" rather than by increase in stroke volume. - Hea rates of 190-210/min are common in infants with compensated shock. - On the other hand in older patients cardiac contractility (stroke volume) and (hea rate) increase to improve cardiac output.", "cop": 2, "opa": "Stroke volume", "opb": "Hea rate", "opc": "Cardiac output", "opd": "lood pressure", "subject_name": "Pediatrics", "topic_name": null, "id": "2cc16881-adee-46ab-90cf-7b871ed54b89", "choice_type": "single"} {"question": "A malignant tumor of childhood, that metastsizes to bones most often, is", "exp": "In neuroblastoma,metastasis is present in 60-70% of children,usually to the skeleton(facial bones,skull),bone marrow,also to lymph nodes. Reference:Essential pediatrics-Ghai,8th edition,page no:617.", "cop": 2, "opa": "Wilm's tumor", "opb": "Neuroblastoma", "opc": "Adrenal gland tumors", "opd": "Granulosa cell tumor of ovary", "subject_name": "Pediatrics", "topic_name": "Childhood tumors", "id": "8885f52e-c3ea-44f5-99fb-86a9f0dfec4e", "choice_type": "single"} {"question": "Precocious puberty in girls is defined by the onset of secondary sexual characteristic before the age of", "exp": "Precocious puberty :\nOnset of secondary sexual characteristic before the age of 8 years in girls and 9 years in boys.", "cop": 3, "opa": "5 years", "opb": "7 years", "opc": "8 years", "opd": "9 years", "subject_name": "Pediatrics", "topic_name": null, "id": "8e3b5309-7b30-4083-816b-c20bc268fd57", "choice_type": "single"} {"question": "The commonest cause of obstructive hydrocephalus in children is", "exp": "ETIOLOGY OF HYDROCEPHALUSCommunicating/non-obstructiveNon-communicating/obstructivePostmeningiticAqueductal stenosisposthemorrhagicDandy-walker/Chiari malformationChoroid plexus papillomaTuberculosisObstructive /non-communicating hydrocephalus develops most commonly in children due to aqueductal stenosis.Non-obstructive hydrocephalus most commonly occurs due to subarachnoid hemorrhage which is usually a result of intraventricular hemorrhage in a preterm infant.", "cop": 1, "opa": "Aqueductal stenosis", "opb": "Dandy walker syndrome", "opc": "Subarachnoid hemorrhage", "opd": "Tubercular meningitis", "subject_name": "Pediatrics", "topic_name": "All India exam", "id": "cdf23a79-cfd1-4ee8-9755-e1156f105dbb", "choice_type": "single"} {"question": "Most common organ involved by congenital TB is", "exp": "Congenital tuberculosis is rare. The fetus may be infected hematogenously through umbilical vessels. The primary focus in this case is liver. In children with congenital TB, the liver is usually affected first, which adds to the hepatoxicity of the therapeutic regimens currently available Reference : Ghai essential pediatrics, eighth edition, page no:257", "cop": 1, "opa": "Liver", "opb": "Pancreas", "opc": "Kidney", "opd": "Lung", "subject_name": "Pediatrics", "topic_name": "Infectious disease", "id": "163d1937-1e93-4caf-a8db-5372f075bbae", "choice_type": "single"} {"question": "The Most common cause of perinatal mortality is", "exp": "(B) Prematurity # Direct causes of death: About 80% of the perinatal deaths are related to perinatal hypoxia, low birth weight, infection & intracranial haemorrhage.> The undetermined group is reduced to 15%.> Thus autopsy study is essential in any perinatal; mortality study when the real causes of death can only be ascertained so that preventive measures can be taken to prevent its occurrence.> The WHO's definition \"Deaths occurring during late pregnancy (at 22 completed weeks gestation and over), during childbirh and up to seven completed days of life\" is not universally accepted. The perinatal mortality is the sum of the fetal mortality and the neonatal mortality.> Collectively, the two types of deaths are referred to as perinatal deaths. Maternal and prenatal foetal conditions leading to slow foetal growth, foetal malnutrition and immaturity are the major contributing factors.> Hypoxia, birth asphyxia and congenital anomalies are some of the major causes of perinatal deaths", "cop": 2, "opa": "Anoxia", "opb": "Prematurity", "opc": "Congenital anomalies", "opd": "Toxaemia", "subject_name": "Pediatrics", "topic_name": "Miscellaneous", "id": "9f44a29a-e1e8-4f05-89a2-e778dba5b633", "choice_type": "single"} {"question": "A 3 year old male baby presents to Casualty with complaints of sudden onset of stridor. A foreign body is visualised in the upper airway. Next line of management", "exp": "In case of a foreign body in upper airway,\nif visualised - Foreign body removal using Magill's forceps\nIf not visualised - Heimlich's maneuver.", "cop": 2, "opa": "Heimlich's maneuver", "opb": "Foreign body removal using Magill's forceps", "opc": "Foreign body removal using Rigid Bronchoscope", "opd": "Foreign body removal using Fibreoptic Bronchoscope", "subject_name": "Pediatrics", "topic_name": null, "id": "89649a10-680a-4c38-9eb0-2fa15f3757b4", "choice_type": "single"} {"question": "A new bom child presented with CHD has cyanosis, become prominanat on breathing and improved in crying. The Diagnosis is", "exp": "i.e. (Bilateral choanal atresia) : (336-Ghai 7th) (178-458-Dhingra 5th)* Affected baby cycles between spells of cyanosis and crying. Attempts at suckling immediately precipitates cyanosis in CHOANAL ATRESIACHOANAL ATRESIA* More often unilateral. More common in female (2:1), more often on the right side and more often bony than membranous (9:1)* Due to persistance of bucconasal membrane* Bilateral choanal atresia usually presents immediately after birth with respiratory distress which is due to the fact that neonates are obligate nose-breathersSometimes babies bom with choanal atresia also have other abnormalities:ColobomaHeart defectsMental retardationGrowth impairmentOthers (see also CHARGE syndrome)Also any condition that causes significant depression of the nasal bridge or midface retraction can be associated with choanal atresia. Examples include the craniosynostosis syndrome such as Crouzon syndrome, and Antley- Bixler syndrome.* Diagnosis(i) Presence of mucoid discharge in the nose(ii) Absence of air bubbles in the nasal discharge(iii) Failure to pass a catheter from nose to pharynx(iv) Putting a few drops of a dye (methylene blue) in to the nose and seeing its passage into the pharynx(v) Installing - radio - opaque dye into the nose and taking a lateral film* Bilateral choanal atresia requires - Urgent management by inserting a finger in the baby's mouth: this can be replaced by with a plastic oropharyngeal airway or a Me Govern nipple - Failure - Endotrachial tube or tracheostomy* Bilateral atresia can be associated with other syndrome head and neck abnormalities eg CHARGE association (Coloboma, hearing impairment, choanal atresia, mental retardation, genital abnormality, endocardial cushion defects) meningo-myelocoeles, craniosynostosis etc.* Most patient with CHARGE syndrome have mutation in CHD7 gene. Mitomycin C has been used to present the development of granulation tissue & stenosis (1743-Nelson 18th)Diaphragmatic hernia - occurs because offailure of closure of the pleuroperitoneal membrane - at birth presented with respiratory distress and a scaphoid abdomen and mediastinal shift", "cop": 1, "opa": "Bilateral choanal atresia", "opb": "Diaphragmatic hernia", "opc": "Genitourinary defects", "opd": "Coloboma", "subject_name": "Pediatrics", "topic_name": "C.V.S.", "id": "05838ddf-d656-4a9f-90ec-48f750977495", "choice_type": "single"} {"question": "Conjugated hyperbilirubinemia in neonates is a feature of", "exp": "The predominant causes of conjugated hyperbilirubinemia are intrahepatic cholestasis and extrahepatic obstruction of the biliary tract, with the latter preventing bilirubin from moving into the intestines. Viruses, alcohol, and autoimmune disorders are the most common causes of hepatitis. Reference: GHAI Essential pediatrics, 8th edition", "cop": 4, "opa": "Breast milk jaundice", "opb": "Gilbe syndrome", "opc": "Hypothyroidism", "opd": "Rotor syndrome", "subject_name": "Pediatrics", "topic_name": "Gastrointestinal tract", "id": "8613a7e8-5628-4700-b2de-10944e2f8608", "choice_type": "single"} {"question": "Size of tracheal tube in a newborn weighing 2.5kg is", "exp": "Guidelines for Tracheal Tube Size and Depth of InseionTUBE SIZE (MM INTERNAL DIAMETER)DEPTH OF INSEION FROM UPPER LIP (cm)WEIGHT (g)GESTATION (wk)2.56.5-7<1,000<2837-81,000-2,00028-343/3.58-92,000-3,00034-383.5/4.0>=9>3,000>38Ref: Nelson; 20th edition; Page no: 846", "cop": 2, "opa": "2", "opb": "3", "opc": "4", "opd": "5", "subject_name": "Pediatrics", "topic_name": "New born infants", "id": "b8aa83d6-44db-499d-bd17-1f65fdddbeb1", "choice_type": "single"} {"question": "A ten year old boy presents to the pediatric emergency unit with seizures. Blood pressure in the upper extremity measured as 200/140 mm Hg. Femaral pulses were not palpable. The most likely diagnosis amongst the flollowing is", "exp": "In coarctation of aoa there will be radiofemoral delay. Ref : Ghai essential of pediatrics, eighth edition,p.no:432", "cop": 4, "opa": "Takayasu aooaeritis", "opb": "Renal parenchymal disease", "opc": "Grandmal seiures", "opd": "Coarcctation of aoa", "subject_name": "Pediatrics", "topic_name": "C.V.S", "id": "57e65e72-c86f-42cc-b218-3ff77213e0ff", "choice_type": "single"} {"question": "Child changes a rattle from one hand to another at the age of", "exp": "

. Developmental milestones:- GROSS MOTOR DEVELOPMENT: 2 months: Holds head in plane of rest of the body when held in ventral suspension. In prone position in bed, the chin lifts momentarily. 3 months:lift head above the plane of the body. Head control stas by 3 months and fully developed by 5 months. 4 months:Remain on forearm suppo if put in prone position, lifting the upper pa of the body off the bed. 5 months: Rolls over. 6 months:sit in tripod fashion. 8 months: sits without suppo., crawling 9 months: Takes a few steps with one hand held. Pulls to standing and cruises holding on to furniture by 10 months. 10 months: creeps 12 months:creeps well, walk but falls, stand without suppo. 15 months: walks well, walks backward/ sideways pulling a toy. May crawl upstairs. 18 months: Runs, walks upstair with one hand held. Explores drawers 2 years: walk up and downstairs, jumps. 3 years : rides tricycle, alternate feet going upstairs. 4 years: hops on one foot, alternate feet going downstairs. 5 years:skips FINE MOTOR DEVELOPMENT:- 2 months- eyes follow objects to 180 deg. 3 months-Grasp reflex disappears and hand is open most of the time. 4 months- Bidextrous approach( reaching out for objects with both hands). 6 months- Unidextrous approach( Reach for an object with one hand).transfer objects from hand to hand. 8 months- radial grasp sta to develop. Turns to sound above the level of ear. 9 months- immature pincer grasp, probes with forefinger. 12 months-Unassisted pincer grasp. Releases object on request.Uses objects predominantly for playing, not for mouthing. Holds block on each hand and bang them together. 15 months- imitate scribbling , tower of two blocks 18 months- scribbles, tower of 3 blocks.turn pages of a book, 2-3 at a time. 2 years- tower of 6 blocks, veical and circular stroke. 3 years-Tower of 9 blocks, dressing and undressing with some help, can do buttoning. 4 years- copies cross, bridge with blocks 5 years- copies triangle, gate with blocks. SOCIAL AND ADAPTIVE MILESTONES: 2 months: social smile(smile after being talked to).watches mother when spoken to and may smile. 3 months:Recognizes mother, anticipates feeds. 4 months: Holds rattle when placed in hand and regards it . Laughs aloud. Excited at the sight of food. 6 months:recognizes strangers, stranger anxiety . Enjoy watching own image in mirror, shows displeasure when toy pulled off. 9 months:waves bye bye 12 months:comes when called, plays simple ball game.kisses the parent on request. Makes postural adjustments for dressing. 15 months:jargon, stas imitating mother. 18 months: copies parents in tasking, dry by day, calls mother when he wants potty, points to three pas of body on request. 2 years: ask for food, drink, toilet, pulls people to show toys. 3 years:shares toys, know fullname and gender, dry by night. 4 years:Plays cooperatively in a group, goes to toilet alone, washes face, brushes teeth. Role play . 5 years:helps in household task , dresses and undresses. LANGUAGE MILESTONES: 1 month: Ales to sound. 2 month:respond to sound by stale or quitening to a smooth voice. 3 months: babbles when spoken to. Makes sounds (ahh,coos, ) laughs. 4 months: laughs aloud. 6 months: monosyllables 9 months: understands spoken words, bisyllables. 12 months: 1-2 words with meaning. 18 months: vocabulary of 10 words. Can name one pa of body. 2 years: 3 word simple sentences 3 years:asks questions, knows full name and gender. 4 years: says songs or poem, tells story, knows three colours. 5 years: ask meaning of words. {Reference: GHAI Essential pediatrics, eighth edition}", "cop": 2, "opa": "3 months", "opb": "6 months", "opc": "9 months", "opd": "1 year", "subject_name": "Pediatrics", "topic_name": "Growth and development", "id": "20660ee9-47bf-4771-b4d4-82accb290120", "choice_type": "single"} {"question": "The commonest valvular manifestation of acute as well as previous rheumatic carditis is", "exp": "Mitral regurgitation (MR) is the commonest manifestation of acute as well as previous rheumatic carditis. Reference: Essential Paediatrics; O.P. Ghai; Page no: 438", "cop": 2, "opa": "Mitral stenosis", "opb": "Mitral regurgitation", "opc": "Aoic stenosis", "opd": "Aoic regurgitation", "subject_name": "Pediatrics", "topic_name": "C.V.S", "id": "f7ccc9f7-59da-4f7d-910c-01f5d2bfce70", "choice_type": "single"} {"question": "Best parameter to monitor the response to injection of a bolus dose of fluids in a dehydrated child is", "exp": "Signs of mild or moderate dehydration include: Thirst ,Dry or sticky mouth. Signs of severe dehydration include: Not peeing or having very dark yellow pee. Very dry skin. Feeling dizzy. Rapid heabeat. Rapid breathing. Sunken eyes. Sleepiness, lack of energy, confusion or irritability. Fainting. Reference: GHAI Essential pediatrics, 8th edition", "cop": 4, "opa": "Skin condition", "opb": "Difference between core and surface temperature", "opc": "Capillary refilling time", "opd": "Radial pulse", "subject_name": "Pediatrics", "topic_name": "Fluid and electrolytes", "id": "fb71ea73-e6f9-4ebd-985b-b29a6a1a3d2b", "choice_type": "single"} {"question": "Testes are not palpable in", "exp": "WT-1 and SF-1 genes stimulate genital ridge to form primitive (bipotential) gonad which then differentiates either into ovary or testis.SRY genes involved in the development of male gonads (testes) from primitive (bipotential gonads)DAX-1, WNT-4, and RSPO1 genes are involved in the development of female gonads (ovary)(Refer: Nelson's Textbook of Pediatrics, SAE, 1st edition, pg no. 2730)", "cop": 1, "opa": "SRY deletion", "opb": "DAX 1 deletion", "opc": "WNT4 gene mutation", "opd": "RSPO1 gene mutation", "subject_name": "Pediatrics", "topic_name": "All India exam", "id": "9884a244-07e6-41e4-915a-26c51efa601d", "choice_type": "single"} {"question": "Drug of choice in Absence Seizure", "exp": "Ethosuximide is the drug of choice in Absence seizures. Ethosuximide: Ethosuximide (Zarontin), a succinimide, has been considered an effective agent for the treatment of absence seizures since the mid-20th century.Although ethosuximide's mechanism of action is not fully understood, the blockade of a specific voltage-gated calcium channel (the T channel) in thalamic neurons appears integral to its antiepileptic activity. This blockade suppresses thalamic excitability, which is thought to be necessary for sustaining the characteristic spike-wave discharge of absence seizures. Serious adverse events linked to ethosuximide include agranulocytosis, aplastic anemia, pancytopenia, Stevens-Johnson syndrome (SJS), and systemic lupus erythematosus; however, these occur rarely and the medication is generally well tolerated. Reference: GHAI Essential pediatrics, 8th edition", "cop": 4, "opa": "Carbamazepine", "opb": "Phenytoin", "opc": "ACTH", "opd": "Ethosuximide", "subject_name": "Pediatrics", "topic_name": "Central Nervous system", "id": "1a4ec9f2-a71d-4e25-b01a-40ac1c978b94", "choice_type": "single"} {"question": "Blood specimen for Neonatal thyroid screening is obtained on", "exp": "GUIDELINES FOR NEONATAL THYROID SCREENING - AMERICAN ACADEMY OF PEDIATRICS When to screen: Normal hospital delivery at term - Filter paper collection ideally at 2-4 days of age or at time of discharge NICU / preterm home bih - Within 7 days of bih Maternal history of thyroid medication / family history of congenital hypothyroidism - cord blood for screening Cord blood is to be used only if there is family history of hypothyroidism. In normal delivery, blood is taken at 2-4 days of age. Reference: GHAI Essential pediatrics, 8th edition", "cop": 4, "opa": "Cord blood", "opb": "24 hours after bih", "opc": "48 hours after bih", "opd": "72 hours after bih", "subject_name": "Pediatrics", "topic_name": "Endocrinology", "id": "33eb2af0-2e65-43d6-8c6a-df604de19363", "choice_type": "single"} {"question": "In children the most commonly recognized form of Familial Hyperlipidemia is", "exp": "Familial combined hyperlipidemia is a disorder that is passed down through families. It causes high cholesterol and and early coronary aery disease. Reference: GHAI Essential pediatrics, 8th edition", "cop": 4, "opa": "Hyperiglyceridemia", "opb": "Hypercholesterolemia", "opc": "Hyperchlyomicronemia", "opd": "Combined hyperlipidemia", "subject_name": "Pediatrics", "topic_name": "Genetic and genetic disorders", "id": "c33d91a3-5a09-4201-aa1a-5010276ab4a4", "choice_type": "single"} {"question": "Chesrry red epiglottis is seen in", "exp": "On laryngoscopy, cherry red epiglottis is seen in Acute epiglottitis.", "cop": 4, "opa": "Larryngomalacia", "opb": "Croup", "opc": "Bronchiolitis", "opd": "Acute epiglottitis", "subject_name": "Pediatrics", "topic_name": null, "id": "c35df83a-c3c8-4520-9819-5ad370bfb26b", "choice_type": "single"} {"question": "Apnea in infant is", "exp": "Apnea may be defined as cessation of respiraton for 20s with or without bradycardia and cyanosis or for shoer periods if it is associated with bradycardia and cyanosis.Apnea is a central problem in preterm neonates.It could be central ,obstructive, or mixed. In preterm neonates it occurs in 2-5 th day of life and is because of the immaturity of the developing brain.Central apnea can also occur due to pathological causes like sepsis,metabolic problems,temperature instability,respiratory distress, anemia ,polycytemia. Treatment:suppoive,underlying cause corrected.Drugs used include aminophylline and caffeine. Ref : ESSENTIAL PEDIATRICS,O.P.GHAI,PG NO:146,7th edition", "cop": 2, "opa": "> 10 sec", "opb": "> 20 sec", "opc": "> 30 sec", "opd": "> 40 sec", "subject_name": "Pediatrics", "topic_name": "Respiratory system", "id": "4a16a9a7-226e-4c9c-936c-9f2b774ade8a", "choice_type": "single"} {"question": "Drug of choice for Steroid Resistant Nephrotic Syndrome is", "exp": "Drug of choice for Steroid Resistant Nephrotic Syndrome is Calcineurin Inhibitors like Cyclosporine A or Tacrolimus.", "cop": 4, "opa": "Cyclophosphamide", "opb": "Mycophenolate mofetil", "opc": "Prednisolone", "opd": "Tacrolimus", "subject_name": "Pediatrics", "topic_name": null, "id": "76c74d9c-43e7-479f-a325-7b0dcc3eb831", "choice_type": "single"} {"question": "First change of improvement noted after iron therapy is initiated", "exp": "First change of improvement noted after iron therapy is decreased irritability followed by increased reticulocyte count. Reference: GHAI Essential pediatrics, 8th edition", "cop": 1, "opa": "Decreased irritability", "opb": "Reticulcytosis", "opc": "Increase in serum iron levels", "opd": "Replenishment of iron stores", "subject_name": "Pediatrics", "topic_name": "Hematology", "id": "a937600a-0f2d-47a7-a689-e2792d77b580", "choice_type": "single"} {"question": "In infancy, atopic dermatitis spares", "exp": "Atopic dermatitis (AD), or eczema, is the most common chronic relapsing skin disease seen in infancy and childhood. Infants with the AD are predisposed to the development of allergic rhinitis and/or asthma later in childhood, a process called \"the atopic march.Major features:Pruritus Facial and extensor eczema in infants and childrenFlexural eczema in adolescentsChronic or relapsing dermatitisPersonal or family history of atopic diseaseMost patients with AD have dry, lackluster skin irrespective of their stage of illness. The AD is generally acuter in infancy and involves the face, scalp, and extensor surfaces of the extremities. The diaper area is usually spared. Older children and children with chronic AD have lichenification and localization of the rash to the flexural folds of the extremities.Reference: Nelson Textbook of Paediatrics; 20th edition; Chapter 145; Atopic Dermatitis (Atopic Eczema)", "cop": 4, "opa": "Extensor surfaces", "opb": "Forehead", "opc": "Scalp", "opd": "Diaper area", "subject_name": "Pediatrics", "topic_name": "Genetic and genetic disorders", "id": "d6d564b0-ae07-4892-b755-74ebc065231d", "choice_type": "single"} {"question": "Lactose content of breast milk per 100 ml is", "exp": "Lactose content of breast milk per 100 ml is 7.0 g. Ref: Table 5.4, Page 97; Ghai essential pediatrics; 6th edition", "cop": 4, "opa": "3.5 gm", "opb": "4.5 gm", "opc": "6 gm", "opd": "7 gm", "subject_name": "Pediatrics", "topic_name": "Nutrition", "id": "3afae527-ed37-4073-acf3-24d896cdb3ff", "choice_type": "single"} {"question": "6 year old drowsy child came in emergency with history of vomiting, loose motion for 3 days. On examination he had sunken eye, hypothermia, skin on pinching taken time to reve diagnosis", "exp": "Sunken eyes, mouth and tongue very dry , condition lethargy ,skin pinch goes back very slowly The patient has 2 / more signs there is severe dehydration Ref: Ghai pediatrics eighth edition pg no 293", "cop": 4, "opa": "No dehydration", "opb": "Mild dehydration", "opc": "Some dehydration", "opd": "Severe dehydration", "subject_name": "Pediatrics", "topic_name": "Fluid and electrolytes", "id": "af6a0acd-5f08-4a8d-b034-01706f801d62", "choice_type": "single"} {"question": "In a mother with active tuberculosis, treatment for infant is", "exp": "Prevention of perinatal TB If the mother has active disease, ATT should be staed for motherNeonate should be given INH prophylaxis till mother is sputum negative or at least 6 monthsRoutine BCG vaccinationSeparation of mother and neonate is not required and breastfeeding should be continued.(Refer: Nelson's Textbook of Pediatrics, SAE, 1st edition, pg no. 1454)", "cop": 1, "opa": "Isoniazid prophylaxis", "opb": "No treatment", "opc": "Full treatment with HRE", "opd": "Treatment with Isoniazid and rifampicin", "subject_name": "Pediatrics", "topic_name": "All India exam", "id": "c8095764-57d1-42ee-b747-a626978a633e", "choice_type": "single"} {"question": "In a newborn, Harlequins skin change is due to", "exp": "HARLEQUIN COLOR CHANGEA rare but dramatic vascular event, harlequin color change occurs in the immediate newborn period and is most common in low bihweight infants. It probably reflects an imbalance in the autonomic vascular regulatory mechanism. When the infant is placed on 1 side, the body is bisected longitudinally into a pale upper half and a deep red dependent half. The color change lasts only for a few minutes and occasionally affects only a poion of the trunk or face. Changing the infant's position may reverse the pattern. Muscular activity causes generalized flushing and obliterates the color differential. Repeated episodes may occur but do not indicate a permanent autonomic imbalance.Ref: Nelson textbook of Paediatrics; 20th edition; Chapter 647; Diseases of the Neonate", "cop": 3, "opa": "Polycythemia", "opb": "Septicemia", "opc": "Autonomic dysfunction", "opd": "Ichthyosis", "subject_name": "Pediatrics", "topic_name": "Hematology", "id": "b155f144-cf2b-4e62-bace-b8afe2b2c504", "choice_type": "single"} {"question": "Mousy odour urine is seen in", "exp": "Mousy order urine is seen in phenylketonuria due to the presence of excessive phenylalanine and its metabolites.Mousy body odour is also present due to the excretion of phenyl lactic acid,its metabolite through sweat. Reference:Essential pediatrics-Ghai,8th edition,page no:652.", "cop": 2, "opa": "Maple syrup urine", "opb": "Phenylketonuria", "opc": "Isovalericacidria", "opd": "Cystinuria", "subject_name": "Pediatrics", "topic_name": "Metabolic disorders", "id": "fab32640-c89a-430c-927e-46ab763e9678", "choice_type": "single"} {"question": "Widely split S1 is heard in", "exp": "In ASD there Will be widely fixed split S2. Ref : Ghai essential of pediatrics, eighth edition, p.no:413", "cop": 3, "opa": "Right bundle branch block (RBBB)", "opb": "Mitral regurgitation (MR)", "opc": "Atrial septal defect (ASD)", "opd": "Ventricular septal defect (VSD)", "subject_name": "Pediatrics", "topic_name": "C.V.S", "id": "e64bc315-a3b2-43aa-a4d6-41fec3098dbe", "choice_type": "single"} {"question": "Subependymal calcified nodules are seen in", "exp": "Definite Tuberous Sclerosis Complex is diagnosed when at least 2 major or one major plus 2 minor features are presentMajor featuresMinor featuresCoical tuberSubependymal noduleSubependymal giant cell astrocytomaFacial angiofibroma or forehead plaqueUngual or periungual fibroma (non-traumatic)Hypomelanotic macules (>3)Shagreen patchMultiple retinal hamaomasCardiac rhabdomyomaRenal angiomyolipomaPulmonary lymphangioleiomyomatosisCerebral white matter migration linesMultiple dental pitsGingival fibromasBone CystsRetinal achromatic patchConfetti skin lesionsNonrenal hamaomasMultiple renal cystsHamaomatous rectal polypsSubependymal nodules are lesions found along the wall of the lateral ventricles where they undergo calcification and project into the ventricular cavity, producing a candle-dripping appearance. These lesions do not cause any problems; however, in 5-10% of cases, these benign lesions can grow into subependymal giant cell astrocytomas (SEGAs).(Refer: Nelson's Textbook of Pediatrics, SAE, 1st edition, pg no. 2877 - 2878)", "cop": 2, "opa": "Sturge Weber syndrome", "opb": "Tuberous sclerosis", "opc": "Neurofibromatosis type 2", "opd": "Von Hippel Lindau syndrome", "subject_name": "Pediatrics", "topic_name": "All India exam", "id": "3085bf7f-b8de-42b1-8911-84963f2eadbd", "choice_type": "single"} {"question": "Children born to mothers with SLE are likely to have one of the following anomalies", "exp": "Systemic lupus erythematosus (SLE) is one of the most common autoimmune disorders that affect women during their childbearing years. SLE increases the risk of spontaneous aboion, intrauterine fetal death, preeclampsia, intrauterine growth retardation, and preterm bih. HEA INVOLVEMENT IN NEONATAL LUPUS Complete hea block (CHB) is the most serious manifestation of the neonatal lupus syndrome (NLS), a congenital syndrome in which maternal IgG anti-Ro/SS-A autoantibodies cross the placenta and injure an otherwise normally developing hea. CHB is often regarded as a model of passively acquired autoimmunity in which antibodies are necessary but insufficient to cause CHB, and fetal factors are likely contributory . Reference: GHAI Essential pediatrics, 8th edition", "cop": 4, "opa": "ASD", "opb": "TOF", "opc": "TGA", "opd": "Complete hea block", "subject_name": "Pediatrics", "topic_name": "C.V.S", "id": "5a8f6841-f3a6-4051-a6b8-8a0adb20d3fc", "choice_type": "single"} {"question": "Bater syndrome is mutation in", "exp": "Bater syndrome is a group of similar rare conditions that affect the kidneys. It&;s genetic, which means it&;s caused by a problem with a gene. If you have it, too much salt and calcium leave your body when you pee. It also may cause low levels of potassium and high levels of acid in the blood. Reference: GHAI Essential pediatrics, 8th edition", "cop": 1, "opa": "Na-k-cl co transpoer", "opb": "Na-Cl channel", "opc": "Renin", "opd": "ENaC protein", "subject_name": "Pediatrics", "topic_name": "Urinary tract", "id": "2ca4e357-e473-4648-8a5a-6a8014920ae0", "choice_type": "single"} {"question": "Fluid administration in an infant of 10 kg is", "exp": "For infants 3.5 to 10 kg the daily fluid requirement is 100 mL/kg. For children 11-20 kg the daily fluid requirement is 1000 mL + 50 mL/kg for every kg over 10. A guide for maintenance fluid therapy for children is as follows: 0-10 kg - 100 mL/kg/day (4 mL/kg/hr) 10-20 kg - 1000 mL/day + 50 mL/kg/day (40 mL/hr + 2 mL/kg/hr) >20 kg - 1500 mL/day + 25 mL/kg/day (60 mL/hr + 1 mL/kg/hr) Reference: GHAI Essential pediatrics, 8th edition", "cop": 4, "opa": "1 ml/kg/hr", "opb": "2 ml/kg/hr", "opc": "3 ml/kg/hr", "opd": "4 ml/kg/hr", "subject_name": "Pediatrics", "topic_name": "Fluid and electrolytes", "id": "9b1f2635-17fe-4405-b7a2-860f234b1d8f", "choice_type": "single"} {"question": "Orchidopex is done in cases of undescended testes at the age of", "exp": "Treatment is always surgical. Surgery is done between two and four years of age. The ideal timing for surgical correction of undescended testes (orchidopexy) has been considered in a number of fora, which have resulted in a range of consensus statements/guidelines stating that orchidopexy should ideally be performed between 6 and 12 months of age, or 18 at the very latest Reference: SRB&;S manual of surgery,5 th edition. page no 1080", "cop": 2, "opa": "Neonate", "opb": "1-2 yrs", "opc": "5 yrs", "opd": "Pubey", "subject_name": "Pediatrics", "topic_name": "Endocrinology", "id": "b320fddb-bec8-44a2-8563-da0e52a80e01", "choice_type": "single"} {"question": "Picket fence temperature seen in", "exp": "(Lateral sinus thrombosis): (95-Dhingra E.N.T. 5th edition)Picket-Fence type of fever - is a feature of the Lateral sinus thrombosis, due to septicaemia often coinciding with release of septic emboli into blood stream. Fever is irregular having one or more peaks a day. It is usually accompanied by chills and rigors. Profuse sweating follows fall of temperature* Clinical picture resembles malaria but lakes regularity* In between the bouts of fever, patient is alert with a sense of well being. Patient receiving antibiotics may not show the picture* Griesinger's sign - is due to thrombosis of mastoid emissary vein. Oedema appears over the posterior part of mastoid is another important sign of lateral sinus thrombosis", "cop": 2, "opa": "Otic hydrocephalus", "opb": "Lateral sinus thrombosis", "opc": "Extradural abscess", "opd": "Meningitis", "subject_name": "Pediatrics", "topic_name": "Central Nervous System", "id": "bd6ab896-9b71-40c3-b24a-b9293c72a84c", "choice_type": "single"} {"question": "\"Microsomia\" is defined as", "exp": "A neonate whose weight falls between the 10th and <90th percentile for gestational age is considered as Appropriate for gestational age (AGA). If the weight falls below the 10th percentile, the neonate is classified as Small for gestational age (SGA) or microsomic. If the weight falls at or above the 90th percentile, then the neonate is classified as Large for gestational age (LGA) or macrosomic. Reference : page 137 Ghai Essential Pediatrics 8th edition", "cop": 2, "opa": "Bih weight below 90th percentile", "opb": "Bih weight below 10th percentile", "opc": "Bih weight below 20th percentile", "opd": "Bih weight below 50th percentile", "subject_name": "Pediatrics", "topic_name": "New born infants", "id": "eaaeca46-414c-4a5c-95c6-6e1ea3208aa2", "choice_type": "single"} {"question": "NOT a feature of TAR syndrome", "exp": "TAR syndrome is characteristically associated with Atrial septal defect.", "cop": 3, "opa": "Thrombocytopenia", "opb": "Multiple skeletal anomalies", "opc": "Ventricular septal defect", "opd": "Autosomal recessive type of inheritance", "subject_name": "Pediatrics", "topic_name": null, "id": "9e5dceda-0419-4b8a-a514-78379ead77ad", "choice_type": "single"} {"question": "Most common secondary tumor with Retinoblastoma is", "exp": "Most common secondary tumor with Retinoblastoma Osteosarcoma, Soft tissue sarcoma and malignant melanoma.", "cop": 4, "opa": "Neuroblastoma", "opb": "Pineoblastoma", "opc": "Osteochondroma", "opd": "Osteosarcoma", "subject_name": "Pediatrics", "topic_name": null, "id": "8a4fa741-4898-40bb-bd11-8cedb1b2dd09", "choice_type": "single"} {"question": "Breast feeding contraindication", "exp": "Ans. is 'd' i.e., Active untreated tuberculosis Contraindication of Breast feeding Galactosemia Active untreated tuberculosis - only in initial period HIV positive mother - especially in developed country. Some medication Not contraindication of breast feeding Hbs Ag positive mother Hepatitis C infection CMV Febrile Tobacco smoking Alcohol", "cop": 4, "opa": "Hep A", "opb": "Hep B", "opc": "CMV", "opd": "Active untreated T.B", "subject_name": "Pediatrics", "topic_name": null, "id": "3fa2b578-85ab-44d0-aff7-850de20b9737", "choice_type": "single"} {"question": "A child presents with massive hepatomegaly and hypoglycemia. There is no improvement in blood glucose on administration of glucagon. The probable diagnosis is", "exp": "Clinical manifestations result, directly or indirectly, from inability to maintain an adequate blood glucose level during the post-absorptive hours of each day organ changes due to glycogen accumulation excessive lactic acid generation damage to tissue from hyperuricemia in GSD Ib, bleeding and infection risk from blood cell effects. Reference: GHAI Essential pediatrics, 8th edition", "cop": 1, "opa": "Von-Gierke disease", "opb": "McArdle disease", "opc": "Cori 's disease", "opd": "Forbe's disease", "subject_name": "Pediatrics", "topic_name": "Metabolic disorders", "id": "b8ef2939-6495-47aa-be6c-c5d45643b863", "choice_type": "single"} {"question": "A normal child will acquire complete neck control by", "exp": "GROSS MOTOR DEVELOPMENT:\n2 months: Holds head in the plane of rest of the body when held in ventral suspension. In the prone position in bed, the chin lifts momentarily.\n3 months: lift head above the plane of the body. Head control starts by 3 months and fully developed by 5 months.\n4 months: Remain on forearm support if put in prone position, lifting the upper part of the body off the bed.\n5 months: Rolls over.\n6 months: sit in a tripod fashion.\n8 months: sits without support., crawling\n9 months: Takes a few steps with one hand held. Pulls to standing and cruises holding on to furniture by 10 months. \n10 months: creeps\n12 months: creeps well, walk but falls, stand without support.\n15 months: walks well, walks backward/ sideways pulling a toy. May crawl upstairs.\n18 months: Runs, walks upstairs with one hand held. Explores drawers\n2 years: walk up and downstairs, jumps.\n3 years: rides tricycle, alternate feet going upstairs.\n4 years: hops on one foot, alternate feet going downstairs.\n5 years: skips", "cop": 2, "opa": "8 weeks", "opb": "16 weeks", "opc": "28 weeks", "opd": "24 weeks", "subject_name": "Pediatrics", "topic_name": null, "id": "8370f8aa-c1d6-4a55-98a0-a27e90ca1144", "choice_type": "single"} {"question": "NOT a feature of Classical triad of Congenital Rubella Syndrome is", "exp": "Classical triad of Congenital Rubella Syndrome is\n\nCongenital Cataract\nCongenital heart disease (most common- Patent Ductus Arteriosus)\nDeafness", "cop": 3, "opa": "Cataract", "opb": "Patent Ductus Arteriosus", "opc": "Microcephaly", "opd": "Deafness", "subject_name": "Pediatrics", "topic_name": null, "id": "632479c6-e92e-4e1c-b477-0af995715f3f", "choice_type": "single"} {"question": "Nagayama spots is seen in", "exp": "Ulcer at Uvulopalatoglossal  junction is called Nagayama spots caused by Roseola infantum.", "cop": 4, "opa": "Measles", "opb": "Mumps", "opc": "Rubella", "opd": "Roseola infantum", "subject_name": "Pediatrics", "topic_name": null, "id": "a8d42114-bf9d-46e7-bf2b-15c6ae11af19", "choice_type": "single"} {"question": "An infant with failure to thrive, hypertension, metabolic alkalosis and hyperkalemia presents to a clinician. Most probable cause is", "exp": "d. Gordon syndrome(Ref: Nelson 20/e p 2535)Out of the given causes, hyperkalemia is seen only in Gordon syndromeGordon syndrome or familial hyperkalemic hypertension, is due to gain-of-function mutations in WNK1 & ioss-of-funotion mutations in WNK4 in distal convoluted tubule, leading to excessive NCCT-mediated salt reabsorption with the clinical picture of pseudohypoaldosteronism type 2Liddle syndrome is due to gain-of-function mutations of the gene that encodes the epithelial sodium channel in the collecting duct, leading to hypertension, hypokalemia & suppressed aldosterone", "cop": 4, "opa": "Liddle's syndrome", "opb": "Bartter's syndrome", "opc": "Gittelman's syndrome", "opd": "Gordon syndrome", "subject_name": "Pediatrics", "topic_name": "Kidney", "id": "690f8f8e-3dda-4f40-a25a-74cc9482f71b", "choice_type": "single"} {"question": "Lowest recurrence in nocturnal enuresis is seen with", "exp": "Enuresis Defined as normal, complete evacuation of the bladder at a wrong place and time at least twice a month after 5 year of age. More than 85% children attain complete diurnal and nocturnal control by 5 years of age. Enuresis is primary when the child has never been dry and secondary when bed wetting stas after a minimum period of six months of dryness at night. It is said to be monosymptomatic if not accompanied by any lower urinary tract infection and nocturnal if it occurs during sleep. It has to be differentiated from continuous or intermittent incontinence or dribbling.The bed is usually soaking wet in enuresis, compared to incontinence in which there is loss of urine without normal emptying of bladder. ETIOLOGY:- 1. Maturation delay is most likely cause. 2. Anxiety producing episodes during the 2 nd to 5 th years, the time for development of nocturnal bladder control, are associated with increased risk of enuresis. 3. Lack of circadian rhythm of ADH or impaired response of kidneys to ADH. 4. Secondary enuresis can be precipitated by acute stressful condition or traumatic experience. Bladder irritability due to urinary tract infection or severe constipation with the full rectum impinging on the bladder can cause enuresis. 5. Conditions causing polyuria, spina bifida, ectopic ureter and giggle and stress incontinence are other causes. Treatment:- General advice given to all enurectic children but active treatment need not begin before 6 years of age. Caffeinated drinks like tea , coffee, sodas should be avoided in evening. Adequate fluid intake during day as 40% in morning, 40% in afternoon and 20% in evening is recommended. First line of treatment is usually non pharmacological, comprising motivational therapy and use of alarm devices. Motivational therapy:- Successful in curing enuresis upto 25%. The child is reassured and provided emotional suppo.Every attempt is made to remove any feeling of guilt. The child is encouraged to assume active responsibility, including keeping a dry night diary, voiding urine before going to bed, and changing wet clothes and bedding. Dry nights should be credited with praise.punishments and angry parenteral response should be avoided. Child is encouraged both to void frequently enough to avoid urgency and daytime incontinence and to have daily bowel movement. Alarm therapy: Involves the use of device to elicit a conditioned response of awakening to the sensation of full bladder. Best used afteemr seven years if age. Successful in two third cases without much recurrence. The alarm device consists of a small sensor attached to child&;s underwear or a mat under the bed sheet and an alarm attached to the childs collar or at bedside. When the child stas wetting, sensors are activated thus causing the alarm to ring. Pharmacotherapy:- Used if enuresis persists despite alarm therapy, regular voiding habits, exclusion or treatment of constipation, exclusion of post voidal residual urine. Imipramine works by aleing the arousal sleep mechanism. It gives satisfactory initial response at dise of 1-2.5 mg/ kg/ day. Relapse rate after discontinuation of therapy seen. Cardiac arrythmia is a serious adverse effect. Anticholinergic drugs reduce uninhibited bladder contractions and are useful in children who have significant daytime urge incontinence besides nocturnal enuresis. Usual dose is 5 mg for oxybutynin, 2 mg for tolterodine. Desmopressin works by reducing volume of urine. Reference: GHAI Essential pediatrics, 8th edition", "cop": 1, "opa": "Bed alarms", "opb": "Desmopressin", "opc": "Imipramine", "opd": "Oxybutynin", "subject_name": "Pediatrics", "topic_name": "Growth and development", "id": "7cee35ef-a87f-442a-a8d4-60e751dbc975", "choice_type": "single"} {"question": "Cardiomyopathy is not a feature of", "exp": "Lowe&;s syndrome is not associated with cardiomyopathy. Cardiomyopathy is are associated with genetic disorder like duchenne dystrophy,Friedrich ataxia ,pompe disease. Ref : Harrison&;s principles of internal medicine, 19th edition, p.no:1555", "cop": 4, "opa": "Duchenne's Muscular Dystrophy", "opb": "Friedriech's ataxia", "opc": "Pompe disease", "opd": "Lowe's syndrome", "subject_name": "Pediatrics", "topic_name": "C.V.S", "id": "90113e3a-ac77-4caf-9248-6b7ca7063c58", "choice_type": "single"} {"question": "Iron stores are best indicated by", "exp": "Serum ferritin reflects the storage of iron which is decreased in Iron deficiency anaemia.\nIt is the most sensitive marker.", "cop": 1, "opa": "Serum ferritin", "opb": "Serum iron", "opc": "Serum transferrin", "opd": "TIBC", "subject_name": "Pediatrics", "topic_name": null, "id": "0b76a025-187b-400d-8e2c-e1686a984b6b", "choice_type": "single"} {"question": "HbE is common in", "exp": "Ans. is 'c' i.e., Bengal o Besides hemoglobin S, there are other hemoglobinopathies due to point mutation in beta globin chain of hemoglobin. o These are ? 1) Hemoglobin C --> Seen in italians and black. 2) Hemoglobin E --> Prevalent in Bengal, Assam and in East India. 3) Hemoglobin D Prevalent in Punjabis and Gujaratis.", "cop": 3, "opa": "Punjab", "opb": "Kerala", "opc": "Bengal", "opd": "Maharstra", "subject_name": "Pediatrics", "topic_name": null, "id": "2c45ac50-f014-43e5-91a0-48a56d8f2fe5", "choice_type": "single"} {"question": "A child can ride a tricycle, copy a circle and knows age sex by the age of", "exp": "

. Developmental milestones:- GROSS MOTOR DEVELOPMENT: 2 months: Holds head in plane of rest of the body when held in ventral suspension. In prone position in bed, the chin lifts momentarily. 3 months:lift head above the plane of the body. Head control stas by 3 months and fully developed by 5 months. 4 months:Remain on forearm suppo if put in prone position, lifting the upper pa of the body off the bed. 5 months: Rolls over. 6 months:sit in tripod fashion. 8 months: sits without suppo., crawling 9 months: Takes a few steps with one hand held. Pulls to standing and cruises holding on to furniture by 10 months. 10 months: creeps 12 months:creeps well, walk but falls, stand without suppo. 15 months: walks well, walks backward/ sideways pulling a toy. May crawl upstairs. 18 months: Runs, walks upstair with one hand held. Explores drawers 2 years: walk up and downstairs, jumps. 3 years : rides tricycle, alternate feet going upstairs. 4 years: hops on one foot, alternate feet going downstairs. 5 years:skips FINE MOTOR DEVELOPMENT:- 2 months- eyes follow objects to 180 deg. 3 months-Grasp reflex disappears and hand is open most of the time. 4 months- Bidextrous approach( reaching out for objects with both hands). 6 months- Unidextrous approach( Reach for an object with one hand). 8 months- radial grasp sta to develop. Turns to sound above the level of ear. 9 months- immature pincer grasp, probes with forefinger. 12 months-Unassisted pincer grasp. Releases object on request.Uses objects predominantly for playing, not for mouthing. Holds block on each hand and bang them together. 15 months- imitate scribbling , tower of two blocks 18 months- scribbles, tower of 3 blocks.turn pages of a book, 2-3 at a time. 2 years- tower of 6 blocks, veical and circular stroke. 3 years-Tower of 9 blocks, dressing and undressing with some help, can do buttoning.copies circle 4 years- copies cross, bridge with blocks 5 years- copies triangle, gate with blocks. SOCIAL AND ADAPTIVE MILESTONES: 2 months: social smile(smile after being talked to).watches mother when spoken to and may smile. 3 months:Recognizes mother, anticipates feeds. 4 months: Holds rattle when placed in hand and regards it . Laughs aloud. Excited at the sight of food. 6 months:recognizes strangers, stranger anxiety . Enjoy watching own image in mirror, shows displeasure when toy pulled off. 9 months:waves bye bye 12 months:comes when called, plays simple ball game.kisses the parent on request. Makes postural adjustments for dressing. 15 months:jargon, stas imitating mother. 18 months: copies parents in tasking, dry by day, calls mother when he wants potty, points to three pas of body on request. 2 years: ask for food, drink, toilet, pulls people to show toys. 3 years:shares toys, know fullname and gender, dry by night. 4 years:Plays cooperatively in a group, goes to toilet alone, washes face, brushes teeth. Role play . 5 years:helps in household task , dresses and undresses. LANGUAGE MILESTONES: 1 month: Ales to sound. 2 month:respond to sound by stale or quitening to a smooth voice. 3 months: babbles when spoken to. Makes sounds (ahh,coos, ) laughs. 4 months: laughs aloud. 6 months: monosyllables 9 months: understands spoken words, bisyllables. 12 months: 1-2 words with meaning. 18 months: vocabulary of 10 words. Can name one pa of body. 2 years: 3 word simple sentences 3 years:asks questions, knows full name and gender. 4 years: says songs or poem, tells story, knows three colours. 5 years: ask meaning of words. {Reference: GHAI Essential pediatrics, eighth edition}", "cop": 3, "opa": "30 months", "opb": "42 months", "opc": "36 months", "opd": "48 months", "subject_name": "Pediatrics", "topic_name": "Growth and development", "id": "0d6f42e8-3711-4a19-b4bb-7f8690defe22", "choice_type": "single"} {"question": "Delayed puberty is seen in", "exp": "Delayed puberty may be related to chronic, reversible causes like systemic disease, malnutrition,eating disorders,hyperprolactinemia,and hypothyroidism.Irreversible defects include the destruction of the hypothalamic-pituitary axis by infection, surgery, tumour. All these are due to chronic disease.", "cop": 4, "opa": "Hypothyroidism", "opb": "Turnur's syndrome", "opc": "Malabsorbtion syndrome", "opd": "Chronic disease", "subject_name": "Pediatrics", "topic_name": null, "id": "b1f21f2d-5c9a-488d-9804-24ef0f4139d3", "choice_type": "single"} {"question": "Best diagnostic test in Thalassemia is", "exp": "(HbF) (310-11- Ghai 6th) (2035- Nelson 18th) (308-Ghai 7lh)Laboratory Investigations in Thalassemia* The infant is bom only with HbF or some cases HbF and HbE (heterozygosity for J3deg thalassemia)* Hb | (2-6 gm/dL), HbF T and HbA2 is nrmal* Reculocyte count is commonly <8% and is in appropriately low compared with the degree of anemia because of ineffective erythropoiesis* Unconjugated serum bilirubin level is usually elevated but the chemistry values may be normal at an early stages* Peripheral smears shows - hypochromia, anisocytosis and poikilocytosis, (Target and tear drop cell, Howell Jolly bodies & Heinz bodies) with marked basophilic stippling* Bone marrow - hypercellular with erythroid hyperplasia with erythroid hyperplasia with increased number of stippled erythroblasts and side roblasts* Osmotic fragility decreased* Serum iron, serum ferritin levels increased", "cop": 1, "opa": "HbF", "opb": "HbA2", "opc": "Serum iron levels", "opd": "Reticulocyte count", "subject_name": "Pediatrics", "topic_name": "Blood", "id": "c372e5bf-08fa-4716-8cf2-7708a0442a8d", "choice_type": "single"} {"question": "Dose of i.v adrenaline in term infant during neonatal resuscitation is", "exp": "When HR is <60/min even after 30sec of effective Positive Pressure Ventilation and chest compressions, adrenaline is given i.v through umbilical vein at a dose of 0.1 - 0.3 ml/kg in 1:10,000 concentration. Reference : page 132 Ghai Essential Pediatrics 8th edition", "cop": 3, "opa": "0.1 - 0.3 ml/kg in 1 : 1000", "opb": "0.3-0.5 ml/kg in 1:1000", "opc": "0.1-0.3 ml/kg in 1:10,000", "opd": "0.3-0.5 ml/kg in 1:10,000", "subject_name": "Pediatrics", "topic_name": "New born infants", "id": "ba6110d3-082c-43c0-b7c4-763050314afc", "choice_type": "single"} {"question": "Causes of status epilepticus in a child", "exp": "Approximately 25 % of childhood status epilepticus is idiopathic; 25% is associated with fever or meningoencephalitis; 50% of patients have a neurodevelopmental abnormality, head trauma, stroke, drug intoxication, subarachnoid bleed, pyridoxine deficiency, or metabolic abnormality ( hypoglycemia and hyponatremia ).Ref: Ghai, 8th edition; Pg no: 553", "cop": 2, "opa": "Hypernatremia", "opb": "Hyponatremia", "opc": "Hyperkalemia", "opd": "Hypokalemia", "subject_name": "Pediatrics", "topic_name": "Central Nervous system", "id": "cb7f3858-d68c-4dc8-8393-893a070df944", "choice_type": "single"} {"question": "APGAR stands for", "exp": null, "cop": 3, "opa": "Appearance, Pulse rate, Growth, Activity, Respiratory rate", "opb": "Appearance, Pulse rate, Grimace, Activity, Respiratory rate", "opc": "Appearance, Pulse rate, Grimace, Activity, Respiratory effort.", "opd": "Activity, Pulse rate, Grimace, Activity, Respiratory effort.", "subject_name": "Pediatrics", "topic_name": null, "id": "8879927c-c6be-45e0-862e-afdaaf3849f7", "choice_type": "single"} {"question": ". Pondreal index of baby with weight of 2000gm and height of 50 cm. aEUR'", "exp": "1.6 [Re.f: O.P. Ghai 7/e p. t091 Ponderal index = X 100 = X 100 = 100X2/125 = 1.6 Ponderal index less than normal indicates that growth rates of adipose tissue and skeletal muscle the principle determinants of weight are less than normal. Ponderal index < 2 ----> Asymmetrical UGR Ponderal index > 2 Symmetrical UGR", "cop": 1, "opa": "1.6", "opb": "2.2", "opc": "2.6", "opd": "3.6", "subject_name": "Pediatrics", "topic_name": null, "id": "457e6877-1dac-4059-85cc-ff559e41cbcf", "choice_type": "single"} {"question": "A premature infant is born with a patent ductus aeriosus. Its closure can be stimulated by administration of", "exp": "Before 2 weeks of gestation indomethacin 0.2mg/kg dose orally every 12-24 hr for 3 doses. Ref : Ghai essential pediatrics,eighth edition P.no: 419", "cop": 4, "opa": "Prostaglandin analogue", "opb": "Estrogen", "opc": "Anti-estrogen compounds", "opd": "Prostaglandin inhibitors", "subject_name": "Pediatrics", "topic_name": "C.V.S", "id": "c67832fc-0811-49d2-b49c-984e87ff6759", "choice_type": "single"} {"question": "Purposeful movement is staed at", "exp": "Palmer grasp goes at age of 4 monthTransfer object hand to hand 5.5 monthThe purposeful movement staed when there is an absence of Palmer group, so an above option best answer is 6 month(Refer: Nelson's Textbook of Pediatrics, SAE, 1st edition, pg no. 2798)", "cop": 1, "opa": "6 months", "opb": "8 months", "opc": "9 months", "opd": "12 months", "subject_name": "Pediatrics", "topic_name": "All India exam", "id": "592fea53-9a72-43d5-ba1b-9b3dcfdff066", "choice_type": "single"} {"question": "A one month old baby presents to the emergency room with history of vomiting, irritability and jaundice. The baby was previously healthy without any symptoms. One week ago cereals and fruit juices were introduced into the diet. USG revealed hepatomegaly and liver functions test were abnormal. The deficient enzyme is", "exp": "Answer: b) Aldolase-B (NELSON 20th ED, p-727)HEREDITARY FRUCTOSE INTOLERANCEDeficiency of fructose-1,6-bisphosphate aldolase (aldolase B)Patients are asymptomatic until fructose or sucrose (table sugar) is ingested (usually from fruit, fruit juice, or sweetened cereal)Symptoms may occur early in life, soon after birth if foods or formulas containing these sugars are introduced into the diet.Early clinical manifestations resemble galactosemia and include jaundice, hepatomegaly, vomiting, lethargy, irritability, and convulsionsLaboratory findings include prolonged clotting time, hypoalbuminuria, elevation of bilirubin and transaminase levels, and proximal tubular dysfunctionTreatment - complete elimination of all sources of sucrose, fructose, and sorbitol from the dietGalactose-1-phosphate uridyl transferase deficiency - Onset of symptoms typically by the second half of the 1st wk of lifeFructokinase deficiency - not associated with any clinical manifestations.Lysosomal acid alpha glucosidase (Pompe's disease) - affected infants present in the first few weeks to months of life with hypotonia, generalized muscle weakness with a \"floppy infant\" appearance, neuropathic bulbar weakness, feeding difficulties, macroglossia, hepatomegaly, and a hypertrophic cardiomyopathy followed by death from cardiorespiratory failure or respiratory infection usually by 1 yr of age", "cop": 2, "opa": "Fructokinase", "opb": "Aldolase-B", "opc": "Galactose-1 phosphate uridyl transferase", "opd": "Acid alpha glucosidase", "subject_name": "Pediatrics", "topic_name": "Inborn Errors of Metabolism", "id": "1ee9c404-7356-4e37-8ce2-075bee318828", "choice_type": "single"} {"question": "Hypoglycemia in neonates occurs in blood glucose less than", "exp": "B. i.e. (40%) (177 - Ghai 6th) (155. Ghai 7th)NEONATAL HYPOGLYCEMIA is defined as blood glucose of less than 40 mg/dL irrespective of the gestational age* Early transitional adaptive hypoglycemia occurs in LBW and babies of diabetic mothers* Classical transient neonatal hypoglycemia is observed in infants with small for gestational age infants* Recurrent severe hypoglycemia is caused by enzymatic or metabolic defectsClinical features* Sweating, tremors, Jitteriness and tachycardia, Diminished utilization of glucose in the cerebrum manifests as lethargy or irritability , restlessness, disturbance in sensorium and convulsions* Hypercalciuria is said to be present in children if daily calcium excretion in urine is more than 4 mg/ kg* Total body water of newborn is 80% of body weight**** Cardiac out put of a normal newborn is 350 ml/kg/ml", "cop": 2, "opa": "20 mg%", "opb": "40 mg%", "opc": "60 mg%", "opd": "10mg%", "subject_name": "Pediatrics", "topic_name": "Endocrinology", "id": "24603c3a-5c4a-41ad-877e-935aee072545", "choice_type": "single"} {"question": "Drug of choice of Strongyloides stercoralis", "exp": "Ans. c (Ivermectin) (Ref. Nelson paediatrics 18th, Table 276-1; Harrison's Internal Medicine 17th ed., Ch. el7)Nematodes (roundworms)TransmissionDiseaseRxStrongyloides stercoralisLarvae in soil penetrate the skinintestinal infection causing vomiting, diarrhea, epigastric pain (may be peptic ulcer- like)Ivermectin or albendazoleIVERMECTIN# Ivermectin is a derivative of the macrocyclic lactone avermectin produced by the soil-dwelling actinomycete Streptomyces avermitilis.# Recent data suggest that ivermectin acts by opening the neuromuscular membrane-associated, glutamate-dependent chloride channels. The influx of chloride ions results in hyperpolarization and muscle paralysis--particularly of the nematode pharynx, with consequent blockage of the oral ingestion of nutrients.# Ivermectin is available only as an oral formulation.# The drug is highly protein bound; it is almost completely excreted in feces.# The effect of food on bioavailability is unknown. Ivermectin is distributed widely throughout the body.# Ivermectin is generally administered as a single dose of 150-200 pg/kg.# Ivermectin is the drug of choice for the treatment of -- Strongyloidiasis,- Onchocerciasis,.....SOCS = Mneumonic- Cutaneous larva migrans,- Scabies.# Ivermectin is highly active against microfilariae of the lymphatic filariases but has no macrofllaricidal activity.# When ivermectin is used in combination with other agents such as DEC or albendazole for treatment of lymphatic filariasis, synergistic activity is seen.# While active against the intestinal helminths Ascaris lumbricoides and Enterobius vermicularis, ivermectin is only variably effective in trichuriasis and is ineffective against hookworms.# Adverse effects- In patients with filarial infections include fever, myalgia, malaise, lightheadednes,postural hypotension.- More severe complications of ivermectin therapy for onchocerciasis include encephalopathy in patients heavily infected with Loa loa. This reaction has led to the suspension of ivermectin distribution for this indication in regions where the two filarial infections are coendemic.- Ivermectin although is the first-line agent for the treatment of onchocerciasis, in areas of Africa co-endemic for O. volvulus and L. loa, however, ivermectin is contraindicated (as it is for pregnant or breast-feeding women) because of severe posttreatment encephalopathy.", "cop": 3, "opa": "Albendazole", "opb": "Mebendazole", "opc": "Ivermectin", "opd": "Pyrantel pamoate", "subject_name": "Pediatrics", "topic_name": "Infection", "id": "20451144-5d50-4777-ab57-f98e700c1c0a", "choice_type": "single"} {"question": "The first sign of pubey in female is", "exp": "Thelarche is the first sign of pubey in a girl around the age of 10 yearsDefinition:- beginning of secondary (Postnatal) breast development at the onset of pubey in girlsTanner stage 2 breast development.Because of rising level of estradiol.Breast development in a male is termed as gynecomastia, not thelarche(Refer: Nelson's Textbook of Pediatrics, SAE, 1st edition, pg no. 926)", "cop": 2, "opa": "Tanner stage I", "opb": "Tanner stage II", "opc": "Pubic hair", "opd": "Axillary hair", "subject_name": "Pediatrics", "topic_name": "All India exam", "id": "b1ff71eb-b907-4b72-b43c-79719fe2358c", "choice_type": "single"} {"question": "A preterm weight of 1600grams should not receive the following vaccine at bih", "exp": "Preterm infants generally can be vaccinated at the same chronologic age as full-term infants according to the recommended childhood immunization schedule. An exception is the bih dose of hepatitis B vaccine. Infants weighing >=2 kg and who are stable may receive a bih dose. However, hepatitis B vaccination should be deferred in infants weighing <2 kg at bih until 30 days of age, if born to an HBsAg negative mother. All preterm, low bihweight infants born to HBsAg positive mothers should receive hepatitis B immunoglobulin and hepatitis B vaccine within 12 hr of bih. However, such infants should receive an additional 3 doses of vaccine staing at 30 days of ageReference: Nelson Textbook of Paediatrics; 20th edition; Chapter 172; Immunization Practices", "cop": 2, "opa": "BCG", "opb": "Hepatitis", "opc": "Polio", "opd": "DPT", "subject_name": "Pediatrics", "topic_name": "New born infants", "id": "c427c4b6-23be-4c30-8d6c-213e2d379a4b", "choice_type": "single"} {"question": "For cardiopulmonary resuscitation in neonates ratio of chest compression to ventilation is", "exp": "Ans: b (3:1) Ref: Nelson, 18th ed,p. 389; 17th ed,p. 280Chest compression-ventilation relationship Neonate1-8 years> 8 yearsCompression rate120At least 100100Compression to ventilation ratio3:15:115:2Pulse checkUmbilical arteryBrachialcarotidQuestion is a direct pick from the above Table in Nelson.", "cop": 2, "opa": "2:01", "opb": "3:01", "opc": "4:01", "opd": "5:02", "subject_name": "Pediatrics", "topic_name": "New Born Infants", "id": "d333793c-8111-400f-8158-bb0389d55e14", "choice_type": "single"} {"question": "Daily water requirement in child weighing 30 kgs, height 123 m and BSA of 1 m2 is", "exp": "For calculating the minimum amount of fluid per day, a formula based on body weight is recommended: 1500 ml is the minimum water intake with 15ml fluid per kg to be added for the actual weight minus 20kg. Reference: GHAI Essential pediatrics, 8th edition", "cop": 2, "opa": "1300 ml", "opb": "1700 ml", "opc": "2000 ml", "opd": "2500 ml", "subject_name": "Pediatrics", "topic_name": "Fluid and electrolytes", "id": "302d1538-f9f9-4e63-81e1-0b985e1609c3", "choice_type": "single"} {"question": "A one year old child presented with multiple fractures seen in various stages of healing. The most probable diagnosis in this case is", "exp": ".Battered child syndrome: A disease in which children are physically abused. The battered child syndromeis a form of child abuse. Not until the 19th century were children granted the same legal status as domesticated animals in regard to protection against cruelty and/or neglect.", "cop": 3, "opa": "Scurvy", "opb": "Rickets", "opc": "Battered baby syndrome", "opd": "Sickle cell disease", "subject_name": "Pediatrics", "topic_name": "Musculoskeletal disorders", "id": "d7643279-2bde-44cd-8147-fadb84f42c39", "choice_type": "single"} {"question": "Most common mutation in Hereditary Spherocytosis is", "exp": "Most common mutation in Hereditary Spherocytosis is Ankyrin > Band 3 > Spectrin and Band 2", "cop": 2, "opa": "Band 3", "opb": "Ankyrin", "opc": "Spectrin", "opd": "Band 4.2", "subject_name": "Pediatrics", "topic_name": null, "id": "95233bd9-9d82-41cf-98f9-d603ac556aed", "choice_type": "single"} {"question": "Most common cause of abdominal swelling in newborn", "exp": "It is dysplastic , non functional kidney replaced by cysts.\nIt is usually U/L : B/L is not compatible with life.", "cop": 1, "opa": "Multicystic dysplastic kidney", "opb": "Autosomal dominant PCKD", "opc": "Autosomal recessive PCKD", "opd": "Horseshoe Kidney", "subject_name": "Pediatrics", "topic_name": null, "id": "a185ecf0-2103-4ffb-87a7-a4a0ce3c2227", "choice_type": "single"} {"question": "NARP syndrome a disorder of", "exp": "NARP syndrome is seen associated with mitochondrial disorders.NARP syndrome refers to Neurogenic weakness,Ataxia and Retinitis Pigmentosa. Mitochondrial disorders refer to defect in oxidation phosphorylation pathway,thereby interfering with energy production. Tissues such as brain,liver and kidney have high energy requirement and are susceptible to injury. Reference: Ghai TB of pediatrics 8th edition pg 659", "cop": 2, "opa": "Lysosome", "opb": "Mitochondria", "opc": "Nucleus", "opd": "Cytosol", "subject_name": "Pediatrics", "topic_name": "Genetic and genetic disorders", "id": "27e4a98f-e229-4be4-833d-d302992e6af1", "choice_type": "single"} {"question": "Duchnne's muscular dystrophy", "exp": "Duchene muscular dystrophy is a X linked recessive disorder. Usually become symptomatic before 5 yr and may have history of delayed walking", "cop": 2, "opa": "X-linked dominant", "opb": "X-linked recessive", "opc": "Autosomal dominant", "opd": "Autosomal recessive", "subject_name": "Pediatrics", "topic_name": "Musculoskeletal disorders", "id": "2e11771c-cdfb-47d0-8e20-52c762862017", "choice_type": "single"} {"question": "Most common Organism causing Bronchiolitis", "exp": "Most common Organism causing Bronchiolitis Respiratory Syncytial Virus.", "cop": 1, "opa": "Respiratory syncytial Virus", "opb": "Parainfluenza virus", "opc": "Hemophilius Influenza A", "opd": "Hemophilius Influenza B", "subject_name": "Pediatrics", "topic_name": null, "id": "9d9f1c38-2bc0-4ec5-91e3-c2af09bd88b7", "choice_type": "single"} {"question": "Zinc dose in 8 months infants", "exp": "Ans. is 'd' i.e., 20 mg/kg x 14 days", "cop": 4, "opa": "10 mg/kg x 10 days", "opb": "20 mg/kg x 10 days", "opc": "10 mg/kg x 14 days", "opd": "20 mg/kg x 14 days", "subject_name": "Pediatrics", "topic_name": null, "id": "2499af53-81c9-4589-b624-3c2b9961693d", "choice_type": "single"} {"question": "In pediatric advanced life suppo, intraosseous access for drug/fluid administration is recommended for", "exp": "** Intraosseous (IO) access was initially thought to be less applicable in patients older than 6 years, now there is growing suppo for consideration of IO techniques in patients of any age as rapid and equally effective alternatives to intravenous (IV) peripheral lines. Intraosseous infusion (IO) is the process of injecting directly into the marrow of a bone to provide a non-collapsible entry point into the systemic venous system.This technique is used in emergency situations to provide fluids and medication when intravenous access is not available or not feasible. A comparison of intravenous (IV), intramuscular (IM), and intraosseous (IO) routes of administration concluded that the intraosseous route is demonstrably superior to intramuscularand comparable to intravenous administration (in delivering paediatric anaesthetic drugs).Due to the rapid advance and adoption of superior intraosseous access technology, IO access has now become the preferred method of establishing vascular access for patients in whom traditional access is difficult or impossible.", "cop": 2, "opa": "<1yr", "opb": "<6 yrs", "opc": "<12yrs", "opd": "Any age", "subject_name": "Pediatrics", "topic_name": "Fluid and electrolytes", "id": "f6b3113f-11dc-42a7-8752-0359a722e9fc", "choice_type": "single"} {"question": "Encopresis is associated with", "exp": "(D) Severe emotional disturbances # Severe emotional disturbances> Encopresis (also known as paradoxical diarrhea) is involuntary fecal soiling in adults and children who have usually already been toilet trained. Persons with encopresis often leak stool into their undergarments.> Strong emotional reactions typically result from failed and repeated attempts to control this highly aversive bodily product.> These reactions then in turn may complicate conventional treatments using stool softeners, sitting demands, and behavioral strategies. The estimated prevalence of encopresis in four-year-olds is between one and three percent.> The disorder is thought to be more common in males than females, by a factor of 6 to 1.", "cop": 4, "opa": "Low socioeconomic class", "opb": "Less than 4 year old child", "opc": "Female sex", "opd": "Severe emotional disturbances", "subject_name": "Pediatrics", "topic_name": "Miscellaneous", "id": "edf5e447-a791-4c9c-b741-371f7a7ce295", "choice_type": "single"} {"question": "Increased ammonia level, normal pH, acidosis (High anion gap) is seen in", "exp": "C. i.e. (Organic acidemia) (610 - 13 - Ghai 6th) (632 - 33 - Ghai 7th)* Sodium benzoate usedfor hyperammonemic states in organic aciduriasAlkaptonuria* *Homogentisic acid oxidase deficiency* *Unne becomes dark on standing* *Ochronosis - Pigment deposition in the sclera, ear, nose* Renal stones and nephrosisTreatment - Ascorbic acid, Nitisirone**GalactosemiaDeficiency of Galactose 1. phosphate uridyl transferase deficiency or Galactokinase deficiency* Autosomal recessive, Physiolgical jaundice persisit longer, Hepatomegaly cataract *, mental retardation ** Vomiting, diarrhea and failure to thrive are early and common manifestationsTreatment - Galactose free diet **Urea cycle disordersDue to accumulation of ammonia, citrulline, arginosuccinic acid or arginine* Recurrent vomiting and irritability are observed early in life C.P - lethargy, coma, ataxia, and convulsion, mental retardation, muscle rigidity, opisthotonus and delayed development Blood urea levels are nearly normal * Organic Acidurias - develops a life threatening episodes of metabolic acidosis characterized by an increased anion gapCLINICAL APPROACH TO INFANTS WITH ORGANIC ACIDEMIA* Hyperammonia may suggest the presence of genetic defects in the urea cycle enzymes. Patients with defects in urea cycle enzymes are not acidotic (548 - Nelson 18th)* Calcification of intervertebral disc occurs in Alkaptonuria*** Positive urinary anion gap is seen in Renal tubular acidosis and Negative UAG seen in Diarrhea)", "cop": 3, "opa": "Urea cycle", "opb": "Galectosemia", "opc": "Organic - acidemia", "opd": "Alkaptonuria", "subject_name": "Pediatrics", "topic_name": "Metabolic Disorders", "id": "111ca9b9-3429-4ba1-99ac-22b2d0b8ae0e", "choice_type": "single"} {"question": "Subacute sclerosing panencephalitis is late neurological complication associated with infection due to", "exp": "Subacute sclerosing panencephalitis occurs 7 - 13 years after primary meastes infection. It has fatal outcome.", "cop": 2, "opa": "Mumps virus", "opb": "Measles virus", "opc": "Rubella virus", "opd": "Para influenza virus", "subject_name": "Pediatrics", "topic_name": null, "id": "95152540-8c14-42cc-8997-e233ba3df330", "choice_type": "single"} {"question": "Most common bacteria causing diarrhea in children", "exp": "- Enteropathogenic E.coli : these have been associated mainly with diarrhea in infants and infants. - It do not produce any toxin nor are they invasive. Reference : Anathanarayan & paniker&;s 9th edition, pg no:281", "cop": 4, "opa": "Enterotoxigenic E.coli", "opb": "Enteroinvasive E.coli", "opc": "Enterohemorrhagic E.coli", "opd": "Enteropathogenic E.coli", "subject_name": "Pediatrics", "topic_name": "Infectious disease", "id": "0603cfcf-1ae7-42b9-a6e2-475a236e7bb3", "choice_type": "single"} {"question": "Most common anomaly of upper urogenital tract is", "exp": "The most common obstructed lateral fusion defects is unilateral obstruction and is most commonly seen in women with complete duplication of the reproductive tract(double uterus, cervix and vagina) . Unilateral UPJ obstruction is the most common prenatally detected obstructive disease, often picked up on maternal ultrasounds during pregnancy. Reference: GHAI Essential pediatrics, 8th edition", "cop": 1, "opa": "Uretero pelvic junction stenosis", "opb": "Ectopic uretheral opening", "opc": "Ureterocele", "opd": "Ectopic ureter", "subject_name": "Pediatrics", "topic_name": "Urinary tract", "id": "b40cbd24-46f1-4117-8c7a-22dfde84ae8c", "choice_type": "single"} {"question": "Hemangioblastoma is most commonly associated with", "exp": "Von - Hippel lindau syndrome is associated with Cerebellar Hemangioblastoma, Spinal cord Hemangioblastoma, Retinal Hemangioblastoma.", "cop": 1, "opa": "Von - Hippel lindau syndrome", "opb": "Neurofibromatosis", "opc": "Tuberous Sclerosis", "opd": "Li - Fraumeni syndrome", "subject_name": "Pediatrics", "topic_name": null, "id": "77a6342d-053b-49d7-bee3-854aa3333542", "choice_type": "single"} {"question": "Oxygenated blood to the fetus is carried by", "exp": "Fetus receive oxygenated blood from placenta left and right umbilical vein. Ref : Ghai Essential pediatrics,eighth edition ,p.no:402", "cop": 2, "opa": "Umbilical aery", "opb": "Umbilical vein", "opc": "SVC", "opd": "Pulmonary aery", "subject_name": "Pediatrics", "topic_name": "C.V.S", "id": "a537d12a-ef20-48b3-8fb5-9219cdb49a38", "choice_type": "single"} {"question": "Most common cause of nephrotic syndrome in paediatric age group is", "exp": "Ans. a (Minimal change disease). Refer to Q. No.49 of Pathology section of same paper.Minimal-Change Disease (Lipoid Nephrosis)# This relatively benign disorder is the most frequent cause of the nephrotic syndrome in children.# It is characterized by glomeruli that have a normal appearance by light microscopy but show diffuse effacement of podocyte foot processes when viewed with the electron microscope.# Although it may develop at any age, this condition is most common between ages 1 and 7 years.# When the changes in the podocytes reverse (e.g., in response to corticosteroids), the proteinuria remits.", "cop": 1, "opa": "Minimal change disease", "opb": "Membranous glomerulonephritis", "opc": "Malarial infection", "opd": "Mesangioproliferative glomerulonephritis", "subject_name": "Pediatrics", "topic_name": "Kidney", "id": "0d504d21-0926-4976-8ccc-1d77482128a2", "choice_type": "single"} {"question": "Diagnosis of the following karyotyping", "exp": "Cri du chat syndrome is due to deletion of short arm of 5th Chromosome.", "cop": 2, "opa": "Down's syndrome", "opb": "Cri du chat syndrome", "opc": "Turner's syndrome", "opd": "Patau Syndrome", "subject_name": "Pediatrics", "topic_name": null, "id": "138827c8-6087-4df0-9355-5489691ac369", "choice_type": "single"} {"question": "Commonest cause of Nephritic syndrome in children is", "exp": "There are many conditions that may cause nephritic syndrome, and it can occur in people of all ages. Common causes are infections, immune system disorders and inflammation of the blood vessels. The main symptoms are passing less urine than normal, leading to a fluid buildup in the body, and having blood in the urine. Reference: GHAI Essential pediatrics, 8th edition", "cop": 1, "opa": "Minimal change disease", "opb": "Membranous GN", "opc": "IgA nephropathy", "opd": "Mesangioproliferative GN", "subject_name": "Pediatrics", "topic_name": "Urinary tract", "id": "3a2ed35c-f438-4685-86d6-7c8033dd537a", "choice_type": "single"} {"question": "Bulging Anterior fontanel is are seen in", "exp": "Rickets - bowlegs, bulging anterior fontanelle are characteristic features of rickets Reference: GHAI Essential pediatrics, 8th edition", "cop": 1, "opa": "Rickets", "opb": "CMV infection", "opc": "Scurvy", "opd": "Hypothyroidism", "subject_name": "Pediatrics", "topic_name": "Central Nervous system", "id": "8aa01de4-8480-4683-bd52-b2485e53d631", "choice_type": "single"} {"question": "Poal venous pressure in a child is", "exp": "Poal venous pressure is the blood pressure in the hepatic poal vein, and is normally between 7 mmHg. Raised poal venous pressure is termed poal hypeension,and has numerous sequelae such as ascites and hepatic encephalopathy. Reference: GHAI Essential pediatrics, 8th edition", "cop": 3, "opa": "5 mm Hg", "opb": "6 mm Hg", "opc": "7 mm Hg", "opd": "8 mm Hg", "subject_name": "Pediatrics", "topic_name": "Gastrointestinal tract", "id": "31d39f06-d621-4899-8e4b-5eedf0e21cec", "choice_type": "single"} {"question": "Triphasic Cells seen in Wilm's tumour are", "exp": "Wilm's tumour is Triphasic tumour. It has Blastemal Cells, Stromal Cells, Epithelial Cells.", "cop": 4, "opa": "Blastemal Cells, Embryonal Cells, Epithelial Cells", "opb": "Stromal Cells, Epithelial Cells, Embryonal Cells", "opc": "Mesenchymal Cells, Stromal Cells, Embryonal Cells", "opd": "Stromal Cells , Blastemal Cells, Epithelial Cells", "subject_name": "Pediatrics", "topic_name": null, "id": "1c945ffb-3574-4313-9f26-076ad7488f3e", "choice_type": "single"} {"question": "Most common CVS lesions in Down&;s Syndrome is", "exp": "Down syndrome:- Trisomy 21 Patients with down syndrome have mental and physical retardation, flat facial profile, an upward slant of eyes and epicanthic folds. Oblique palpebrak fissure seen. Nose is small with flat nasal bridge. Mouth shows narrow sho palate with small teeth and furrowed protruding tongue. There is significant hypotonia. Skull appears small and brachycephalic with flat occiput . Ears are small and dysplastic. Characteristic facial grimace on crying. Hands are sho and broad. Clinodactyly( hypoplasia of middle phalanx of fifth finger) and simian crease are usual. There is wide gap between the first and second toe( Sandle gap). Associated anomalies:- 1. Congenital hea disease- ventricular septal defect. 2. Gastrointestinal - atresia, annular pancreas and Hirschsprung disease. 3. Ophthalmic- cararact, nystagmus, squint. 4. Thyroid dysfunction 5. Conductive hearing loss Best serological marker for down syndrome is Beta HCG. Reference: GHAI Essential pediatrics, eighth edition", "cop": 1, "opa": "VSD", "opb": "ASD", "opc": "TOF", "opd": "COA", "subject_name": "Pediatrics", "topic_name": "Genetic and genetic disorders", "id": "ce786c22-08ab-4245-8dae-5ed29acd8347", "choice_type": "single"} {"question": "A two year child is classified as having pneumonia, if the respiratory rate is more than", "exp": "According to ARI control programme pneumonia must have Respiratory rate(RR)/min 60 or more in <2 months of age 50 or more in 2 -12 months of age 40 or more in 1-5 years of age Ref : ESSENTIAL PEDIATRICS,O.P.GHAI,7 th edition PG NO:356", "cop": 2, "opa": "30/min", "opb": "40/min", "opc": "50/min", "opd": "60/min", "subject_name": "Pediatrics", "topic_name": "Respiratory system", "id": "c0954cd5-e00a-4bc0-8e23-c2ed530d17ce", "choice_type": "single"} {"question": "In Unconjugated hyperbilirubinemia, the risk of kernicterus increases with the use of", "exp": "Sulfonamides (also called sulfa drugs) are a group of antibiotics that kill bacteria. Sulfonamides may unbind bilirubin from albumin, which increases blood levels of bilirubin. The unbound bilirubin can cross into the brain and cause kernicterus. Reference: GHAI Essential pediatrics, 8th edition", "cop": 4, "opa": "Ceftriaxone", "opb": "Phenobarbitone", "opc": "Ampicillin", "opd": "Sulphonamide", "subject_name": "Pediatrics", "topic_name": "New born infants", "id": "9e797df6-40cf-475e-804f-944bbe7cbb64", "choice_type": "single"} {"question": "Principle mode of heat exchange in an infant incubator is", "exp": "WORKING PRINCIPLE : light bulbs heat air in the bottom pa of the incubator,the air passes over a container with evaporating water, so that its humidity increases,the warm, humid air then flows upwards (chimney effect) into the baby compament Reference: GHAI Essential pediatrics, 8th edition.", "cop": 3, "opa": "Radiation", "opb": "Evaporation", "opc": "Convection", "opd": "Conduction", "subject_name": "Pediatrics", "topic_name": "New born infants", "id": "da1add73-2168-49f1-9db6-ebd0df3f6d11", "choice_type": "single"} {"question": "The vitamin A supplement administered in \"Prevention of nutritional Blindness in children programme\" contain", "exp": "Under the National Vitamin A Prophylaxis Programme, children between 1& 5 yrs of age were given oral dose of 2 lakh IU, every 6 months. But due to inadequate coverage, currently this vaccine is being given to children under 3 yrs under routine immunization to improve coverage i.e, (1lakh IU- 9months; 2lakh IU- 15 to 18 months) According to NPPNB, Vit A 2 lakh IU orally is given every 6 months to pre school children,(1-6yrs) And half the dose ( 1lakh IU orally) to children age between 6 months and 1 yr of age. Ref: Ghai , 9th edition chapter 8; PSM 25th edition.", "cop": 2, "opa": "25,000 IU/ml", "opb": "1 Lakh IU/ml", "opc": "3 Lakh IU/ml", "opd": "5 Lakh IU/ml", "subject_name": "Pediatrics", "topic_name": "Nutrition", "id": "cd2c2544-961d-4279-a1e3-fd71d1189605", "choice_type": "single"} {"question": "Best serogical marker of Down&;s syndrome in first trimester is", "exp": "Down syndrome:- Trisomy 21 Patients with down syndrome have mental and physical retardation, flat facial profile, an upward slant of eyes and epicanthic folds. Oblique palpebrak fissure seen. Nose is small with flat nasal bridge. Mouth shows narrow sho palate with small teeth and furrowed protruding tongue. There is significant hypotonia. Skull appears small and brachycephalic with flat occiput . Ears are small and dysplastic. Characteristic facial grimace on crying. Hands are sho and broad. Clinodactyly( hypoplasia of middle phalanx of fifth finger) and simian crease are usual. There is wide gap between the first and second toe( Sandle gap). Associated anomalies:- 1. Congenital hea disease- ventricular septal defect. 2. Gastrointestinal - atresia, annular pancreas and Hirschsprung disease. 3. Ophthalmic- cararact, nystagmus, squint. 4. Thyroid dysfunction 5. Conductive hearing loss Best serological marker for down syndrome is Beta HCG. Reference: GHAI Essential pediatrics, eighth edition", "cop": 3, "opa": "Nuchal translucency", "opb": "Skeletal abnormalities", "opc": "b-SCG", "opd": "Serum estriol", "subject_name": "Pediatrics", "topic_name": "Genetic and genetic disorders", "id": "d0dc31b0-b947-45f6-9fd0-55c7c16ce61a", "choice_type": "single"} {"question": "Moro's Reflex disappears at", "exp": "Moro's reflex disappears by 3 - 6 months in normal infants. Persistence of the reflex after 6 months and absence or diminution of the Moro reflex within 2 to 3 months of age can be regarded as abnormal. Asymmetry of the response is usually a sign of local injury.", "cop": 2, "opa": "3 months", "opb": "6 months", "opc": "5 months", "opd": "7 months", "subject_name": "Pediatrics", "topic_name": null, "id": "71317695-7403-4114-94d3-9cc8601dd99f", "choice_type": "single"} {"question": "Increased fetal hemoglobin is seen in", "exp": "Juvenile chronic myeloid leukemia is characterised by elevated fetal hemoglobin levels and low or normal leukocyte alkaline phosphatase score.Thrombocytopenia and anemia are common.Peripheral smear shows leukocytosis with the full spectrum of granulocytic precursors and increased normoblasts.Bone marrow aspirate shows an increased cellularity with predominance of granulocytic cells in all stages of maturation;megakaryocytes are normal or decreased.Philadelphia chromosome is negative,and monosomy 7 is found in 30% patients. Reference:Essential pediatrics-Ghai,8th edition,page no:608.", "cop": 1, "opa": "Juvenile CML", "opb": "Congenital red cell aplasia", "opc": "Hereditary shperocytosis", "opd": "AML", "subject_name": "Pediatrics", "topic_name": "Childhood tumors", "id": "7fd0f0f4-e0a3-4cfe-885f-1e942a22f57d", "choice_type": "single"} {"question": "A child presented with cola coloured urine, proteinuria 2+ & h/o rash 2 week ago. Probable Diagnosis is", "exp": "Henoch schonlein purpura : It is one of the most common vasculitic disorder of childhood. Characterised by the presence of nonthrombocytopenic , palpable purpura, transient ahralgia and abdominal symptoms. The illness begins with a rash more prominent over the extensor aspects of lower extremities and buttocks. It may be macular, maculopapular or even uicarial to begin with . Glomerulonephritis seen in one third of cases. And ca coloured urine Gastrointestinal manifestations usually occur in first 7-10 days if illness. Abdominal pain is intermittent, colicky and periumbilical.vomitingseen , whereas melena and hemetemesis are less common. Rare manifestations include CNS vasculitis, coma, Guillain Barre syndrome, pulmonary hemorrhage, carditis and orchitis. Criteria for childhood HSP:- Palpable purpura in the presence of at least one of the following 4 features: 1. Diffuse abdominal pain. 2. Any deposit showing IgA deposition. 3. Ahritis/Ahralgia. 4. Renal involvement. Investigation: Nonspecific rise in total serum IgA levels. Skin biopsy shows leukocytoclastic vasculitis. On indirect immunofluorescence there are mesangial deposits of IgA andC3 in skin and renal biopsy. Reference: GHAI essential Paediatrics", "cop": 2, "opa": "IgA nephropathy", "opb": "HSP", "opc": "HUS", "opd": "Wegener Granulomatosis", "subject_name": "Pediatrics", "topic_name": "Urinary tract", "id": "422e52eb-d890-4c3e-b899-dbd78e619d67", "choice_type": "single"} {"question": "Cat eye syndrome is", "exp": "Cat eye syndrome:-Schmid Fraccaro syndrome Paial trisomy of chromosome 22. It is coined cat eye because of the paicular appearance of the veical colobomas in the eyes of some patients. Other features: 1. Anal atresia 2. Unilateral or bilateral iris coloboma 3. Downward slanting palpebral fissure. 4. Preauricular pits/ tags. 5. Cardiac defects 6. Kidney problems 7. Sho stature 8. Intellectual disabilityi 9. Micrognathia 10. Cleft palate Reference: GHAI Essential pediatrics, 8th edition", "cop": 4, "opa": "Paial trisomy 18", "opb": "Paial trisomy 13", "opc": "Paial trisomy 21", "opd": "Paial trisomy 22", "subject_name": "Pediatrics", "topic_name": "Genetic and genetic disorders", "id": "c1b1957b-7bfb-4d95-b21d-b1d7ba50b471", "choice_type": "single"} {"question": "The most common cardiac defects seen in congenital rubella syndrome is", "exp": "Common cardiac defects in congenital rubella syndrome is PDA and PS. Ghai essential of pediatrics, eighth edition, p.no:401", "cop": 4, "opa": "Pulmonary aery stenosis and coarctation of aoa", "opb": "Vantricular septal defect (VSD) and patent ductus aesian (PDA)", "opc": "VSD and pulmonary aery stenosis", "opd": "Pulmonary aery stenosis and PDA", "subject_name": "Pediatrics", "topic_name": "C.V.S", "id": "9187ee54-9e08-4110-89b1-d504f6a036ec", "choice_type": "single"} {"question": "Boold volume in preterm neonate is", "exp": "At the moment of bih the newborn infant was estimated to have a blood volume of 78 ml/kg with a venous hematocrit of 48 %. The average blood volume at bih was found to be 109 ml. per kilogram, a value slightly higher than in fullterm infants. In the first 2 1/2 months of life there was a tendency for the plasma volume to increase as the erythrocyte volume decreased. This had the effect of maintaining the total blood volume at a relatively high level. Reference: GHAI Essential pediatrics, 8th edition", "cop": 1, "opa": "90 ml/kg", "opb": "80 ml/kg", "opc": "70 ml/kg", "opd": "60 ml/kg", "subject_name": "Pediatrics", "topic_name": "Fluid and electrolytes", "id": "5da06b95-7aed-42ed-8b07-21ee0373e4bd", "choice_type": "single"} {"question": "Useful in acute diarrhea is", "exp": "Most cases of diarrhea are acute, self-limited, and due to infections or medication. Fluid and electrolyte replacement is of central impoance to all forms of acute diarrhea. In moderately severe nonfebrile and non-bloody diarrhea, antimotility and antisecretory agents such as loperamide can be useful adjuncts to control symptoms. Judicious use of antibiotics is appropriate in selected instances of acute diarrhea and may reduce its severity and duration . Zinc (20 mg/d until recovery) may be an effective adjunctive therapeutic strategy for diarrheal disease and pneumonia in children >= 6 months of age. Ref: Harrison&;s Principles of Internal Medicine; 19th edition; Chapter 96e; Vitamin and Trace Mineral Deficiency and Excess", "cop": 1, "opa": "Zinc", "opb": "Magnesium", "opc": "Calcium", "opd": "Potassium", "subject_name": "Pediatrics", "topic_name": "Gastrointestinal tract", "id": "fb94df6e-dfd8-42cd-b37e-86f30fd7324a", "choice_type": "single"} {"question": "Daily iron requirement of a lOyr old boy is", "exp": "Ans) c (8mg) Ref Nelson 18th edp210", "cop": 3, "opa": "2 mg", "opb": "4mg", "opc": "8 mg", "opd": "10 mg", "subject_name": "Pediatrics", "topic_name": "Nutrition", "id": "b5152e73-1c7c-4976-aa1e-e1fbd2a151da", "choice_type": "single"} {"question": "A child comes with the complaints of bed wetting. First line of Treatment", "exp": "Nocturnal enuresis' refers to involuntary urination at night beyond the age of 5 years. Treatment- First line - Diet and lifestyle changes, Motivational therapy Second line- Bed alarm technique- gives excellent response Third line - Drugs Bed Alarm technique These are moisture sensing alarms As soon as the child passes the urine in bed, it would detect the moisture in undergarment of the child and will ring. Combination of drug and bed alarm technique has lowest relapse rates.", "cop": 3, "opa": "Desmopressin", "opb": "Oxybutynin", "opc": "Motivational therapy", "opd": "Bed alarm technique", "subject_name": "Pediatrics", "topic_name": "FMGE 2019", "id": "e0d0c241-bb21-447c-af16-be8fa66b0daf", "choice_type": "single"} {"question": "The neonatal feature of cystic fibrosis is", "exp": "Meconium is the first stool (bowel movement) that a newborn has. This stool is very thick and sticky. Meconium ileus is a bowel obstruction that occurs when the meconium in your child&;s intestine is even thicker and stickier than normal meconium, creating a blockage in a pa of the small intestine called the ileum. Most infants with meconium ileus have a disease called cystic fibrosis. Reference: GHAI Essential pediatrics, 8th edition", "cop": 3, "opa": "Bronchitis", "opb": "Bronchiolitis", "opc": "Meconeum ileus", "opd": "Bronchiectasis", "subject_name": "Pediatrics", "topic_name": "Genetic and genetic disorders", "id": "e13d791e-96d8-455f-a84f-617e8c075684", "choice_type": "single"} {"question": "Abnormal mousy/mushy odour of urine is associated with", "exp": "Ans. a (Phenylketonuria) (Ref. Harrison's Internal Medicine 17th/Ch. 358)Inborn errors of amino acid metabolism associated with abnormal odourInborn error of metabolismUrine odorGlutaric acidemia (type II)Sweaty feet, acridHawkinsinuriaSwimming poolIsovaleric acidemiaSweaty feet, acridMaple syrup urine diseaseMaple syrup/Burnt sugarHypermethioninemiaBoiled cabbageMultiple carboxylase deficiencyTomcat urineOasthouse urine diseaseHops-likePhenylketonuriaMousy or mustyTrimethylaminuriaRotting fishTyrosinemiaBoiled cabbage, rancid butterTHE HYPERPHENYLALANINEMIAS# result from impaired conversion of phenylalanine to tyrosine.# The most common and clinically important is phenylketonuria (frequency 1:10,000), which is an autosomal recessive disorder characterized by an | concentration of phenylalanine and its by-products in body fluids and by severe mental retardation if untreated in infancy.# It results from | activity of phenylalanine hydroxylase (phenylketonuria type I).# The accumulation of phenylalanine inhibits the transport of other amino acids required for protein or neurotransmitter synthesis, reduces synthesis and increases degradation of myelin, and leads to inadequate formation of norepinephrine and serotonin.# Phenylalanine is a competitive inhibitor of tyrosinase, a key enzyme in the pathway of melanin synthesis, and ac- counts for the hypopigmentation of hair and skin.# Untreated children with classic phenylketonuria are normal at birth but fail to attain early developmental milestones, develop microcephaly, and demonstrate progressive impairment of cerebral function.# Hyperactivity, seizures, and severe mental retardation are major clinical problems later in life.# EEG abnormalities; \"mousy\" odor of skin, hair, and urine (due to phenylacetate accumulation); and a tendency to hypopigmentation and eczema are devastating clinical picture.# In contrast, affected children who are detected and treated at birth show none of these abnormalities.Treatment# To prevent mental retardation, diagnosis and initiation of dietary treatment of classic phenylketonuria must occur before the child is 3 weeks of age.# Dietary phenylalanine restriction is usually instituted if blood phenylalanine levels are >250 pmol/L (4 mg/dL).# Treatment consists of a special diet low in phenylalanine and supplemented with tyrosine, since tyrosine becomes an essential amino acid in phenylalanine hydroxylase deficiency.# With therapy, plasma phenylalanine cone should be maintained between 120 and 360 pmol/L (2 and 6 mg/dL).# Dietary restriction should be continued and monitored indefinitely..# Pregnancy risks can be minimized by continuing lifelong phenylalanine-restricted diets and assuring strict phenylalanine restriction 2 months prior to conception and throughout gestation.ConditionEnzyme DefectClinical FindingsInheritance1. Phenylketonuria type IPhenylalanine hydroxylaseMental retardation, microcephaly, hypopig- mented skin and hairs, eczema, \"mousy\" odorAR2. Phenylketonuria type IIDihydropteridine reductaseMental retardation, hypotonia, spasticity, myoclonusAR3. Phenylketonuria type III6-Pyruvoyl- tetrahydropterin synthaseDystonia, neurologic deterioration, seizures, mental retardationAR4. GTP cyclohydrolase I deficiencyGTP cyclohydrolase IMental retardation, seizures, dystonia, temperature instabilityAR5. Carbinolamine dehydratase deficiencyPterin-4-carbinolamine dehydrataseTransient hyperphenylalaninemia (benign)AR", "cop": 1, "opa": "Phenylketonuria", "opb": "Tyrosinemia", "opc": "Maple syrup urine disease", "opd": "Hawkinsuria", "subject_name": "Pediatrics", "topic_name": "Inborn Errors of Metabolism", "id": "64f80edc-9494-4b09-b361-0d50d21ff9cb", "choice_type": "single"} {"question": "Low birth weight is defined as", "exp": "Low birth weight (LBW) neonate- A neonate weighing less than 2500g at birth irrespective of the gestational age.\nVery low birthweight (VLBW) neonate- A neonate weighing leas than 1500g at birth irrespective of the gestational age.\nExtremely low birth weight (ELBW) neonate- A neonate weighing leas than 1000g at birth irrespective of the gestational age.", "cop": 4, "opa": "Weight < 10 percentile for their gastational age", "opb": "Weight < 2.0 kg", "opc": "Weight < 1.5 kg", "opd": "Weight < 2.5 kg", "subject_name": "Pediatrics", "topic_name": null, "id": "4ad97ad9-6716-4351-bfbe-f1b35c411f57", "choice_type": "single"} {"question": "Commonest cause of intestinal obstruction in children is", "exp": ".Intussusception Most common in children Primary or secondary to intestinal pthology eg:polyps,appendix Ileocolic is the most common variety Can lead to ischaemic segment peak incidence at 3-9 months BAILEY AND LOVE&;S SHO PRACTICE OF SURGERY,PG NO:1195, 24 th edition", "cop": 1, "opa": "Intussusception", "opb": "Volvulus", "opc": "Hernia", "opd": "Adhesions", "subject_name": "Pediatrics", "topic_name": "Gastrointestinal tract", "id": "2cb505fc-979a-4e34-9021-622d961cc409", "choice_type": "single"} {"question": "Vertibular Schwannoma, spinal cord astrocytoma, meningioma are seen in", "exp": "Neurofibromatosis  - 2 :\nVertibular Schwannoma.\nMeningioma.\nSpinal cord ependymoma.\nSpinal cord astrocytoma.", "cop": 4, "opa": "Tuberous sclerosis", "opb": "Neurofibromatosis - 1", "opc": "Von Hippel - lindeu syndrome", "opd": "Neurofibromatosis - 2", "subject_name": "Pediatrics", "topic_name": null, "id": "08a7a3c0-35fa-4879-8457-6df463d1f6af", "choice_type": "single"} {"question": "Normal QRS duration at 1 year of age is", "exp": "Normal QRS duration at various age groups Age group QRS duration in seconds 0-6 months 0.03-0.07 (0.05) 1-5 yr 0.04-0.08 (0.06) 10-15 yr 0.04-0.09 (0.07) >15 yr 0.06-0.09 (0.08) Reference: Essential Paediatrics; O.P. Ghai; Page no: 459", "cop": 2, "opa": "0.03-0.07 seconds", "opb": "0.04-0.08 seconds", "opc": "0.04-0.09 seconds", "opd": "0.06-0.09 seconds", "subject_name": "Pediatrics", "topic_name": "C.V.S", "id": "2d86bcd6-3996-4c84-bb94-4a81b69204c2", "choice_type": "single"} {"question": "Retinopathy of Prematurity (ROP) screening is done in the following babies", "exp": "Retinopathy of Prematurity (ROP) Screening is done in\n\nPreterm babies < 32 weeks of gestation\nBabies with birth weight < 1.5 kg\nPreterm babies of  32 - 34 weeks of gestation\n\tBabies with birth weight 1.5 - 2 kg\n\nPreterm babies of 32-34 weeks and babies with birth weight 1.5-2kg should have had bad neonatal course like:\na) Ventilatory support\nb) O2 supply for > 24 hours\nc) Ionotrope support for condition like shock\nd) Blood transfusion\ne) Culture positive sepris.", "cop": 2, "opa": "Preterm < 34 weeks of gestation", "opb": "Preterm of 33 weeks of gestation who required mechanical ventilation", "opc": "Babies with birth weight < 1.8 kg", "opd": "Babies with birth weight < 2 kg", "subject_name": "Pediatrics", "topic_name": null, "id": "10285f15-d722-4df7-bf62-39d8e05e7426", "choice_type": "single"} {"question": "Respiratory rate in a 2 month old, to label it tachypnea is", "exp": "Tachypnea ( WHO criteria ) 1 . Newborn to 2 months : 60 breaths per minute 2 . Infant 2 months to 1 year : 50 breaths per minute 3 . Preschool Child 1 to 5 years : 40 breaths per minute 4 . School age Child: 20 - 30 breaths per minute 5 . Adults : 20 breaths per minute Reference : page 381 Ghai Essential Pediatrics 8th edition.", "cop": 3, "opa": "40", "opb": "50", "opc": "60", "opd": "70", "subject_name": "Pediatrics", "topic_name": "New born infants", "id": "8326d162-6376-46d9-9a77-df84d123c3fb", "choice_type": "single"} {"question": "Nocturnal Enuresis is occurrence of involuntary voiding at night in a child more than", "exp": "Ans. (d) 5yrs* OP ghai states enuresis as urinary incontinence beyond the age of 4 years for daytime and 6 years for night-time enuresis.* It is said to be present if it occurs twice per week for 3 consecutive months.* Most of children attain complete bladder control by age of 5 years* The prevalence of enuresis is 7% in boys and 3% in girls at age of 5 years and keeps on decreasing for every subsequent year.* Treatment of choice for nocturnal enuresis is alarm devices with desmopressin nasal spray.", "cop": 4, "opa": "2 1/2 yrs", "opb": "3 1/2 yrs", "opc": "4 yrs", "opd": "5 yrs", "subject_name": "Pediatrics", "topic_name": "Developmental-Behavioral Screening and Surveillance", "id": "0f8ae024-14f2-48c4-9361-621b728215e5", "choice_type": "single"} {"question": "MC cause of cholestatic jaundice in newborn is", "exp": "MC cause of cholestatic jaundice in newborn is Neonatal hepatitis. MCC of jaundice in 1st 24 hrs of life - Hemolytic disease of newborn. MCC of jaundice in 24 hrs to 72 hrs of life - Physiological jaundice.", "cop": 2, "opa": "Hypoplasia of biliary tract", "opb": "Neonatal hepatitis", "opc": "Choledochal cyst", "opd": "Physiological", "subject_name": "Pediatrics", "topic_name": "FMGE 2018", "id": "6470f5ba-04cb-4b1f-ba67-7064f165cb76", "choice_type": "single"} {"question": "Only male are affected in", "exp": "Hunter's syndrome belongs to Type II mucopolysaccharidoses in which mucopolysaccharides accumulate in the lysosomes causing disorganization of the cell structure & function.This syndrome is characterised by mild mental retardation;marked skeletal changes like thickening of the skull,marked deformity of sella tursica,broad spatula like ribs,beak shaped veebrae(around L1 veebra) and proximal tapering of metacarpals-these abnormalities are referred to as dysostosis multiplex;coarse facies & hepatosplenomegaly. Reference:Essential pediatrics-Ghai,8th edition,page no: 660,661.", "cop": 2, "opa": "Scheie's syndrome", "opb": "Hunter's syndrome", "opc": "Hurler's syndrome", "opd": "Gaucher's disease", "subject_name": "Pediatrics", "topic_name": "Metabolic disorders", "id": "3cdcb8a3-7240-430c-96f7-3838086bcb59", "choice_type": "single"} {"question": "Commonest site of extramedullary relapse of", "exp": "Ans. is 'a' i.e., CNS", "cop": 1, "opa": "CNS", "opb": "Lung", "opc": "Liver", "opd": "Testis", "subject_name": "Pediatrics", "topic_name": null, "id": "42dd5362-a07e-48dc-9c1c-c2cafed2d363", "choice_type": "single"} {"question": "Most common cause of Rapid progressive glomerulonephritis in children is", "exp": "Pauci - immune glomerulonephritis like Churg Strauss, Wegener's glomerulonephritis have highest risk for RPGN.", "cop": 4, "opa": "Good Pasteur syndrome", "opb": "Membranous glomerulonephritis", "opc": "Ig A nephropathy", "opd": "Pauci - immune glomerulonephritis", "subject_name": "Pediatrics", "topic_name": null, "id": "a91dc181-2987-47d7-bca0-6378d96fbf2f", "choice_type": "single"} {"question": "Most common mutation in hereditary elliptocytosis", "exp": "- Most common mutation in hereditory elliptocytosis - Spectrin - Inherited as Autosomal Dominant manner - In Spectrin mutations, the most common RBC abnormality is Hereditary Elliptocytosis > spherocytosis.", "cop": 1, "opa": "Spectrin", "opb": "Ankyrin", "opc": "Glycophorin A", "opd": "Band 4.2", "subject_name": "Pediatrics", "topic_name": "Hemolytic anemia", "id": "12437f60-7564-4e7b-953f-981677810b78", "choice_type": "single"} {"question": "Most common manifestation of rheumatic fever", "exp": "Ans. is 'a' i.e., Ahritis Ahritis in RF It is the most frequent major manifestation (75% of cases). o It is migratory polyahritis, i.e., several joints are involved in quick succession and each for a brief period of time. o Any joint may be affected but involvement of large joints such as knee, ankle, elbow, and wrist is common. Small joints of hands and feet are not involved. o Inflammation of any one joint subside spontaneously within a week and the entire bout of polyahritis rarely last more than 4 weeks. Resolution is complete with no residual joint damage. o A striking feature is dramatic response to salicylates.", "cop": 1, "opa": "Ahritis", "opb": "Carditis", "opc": "Chorea", "opd": "Nodules", "subject_name": "Pediatrics", "topic_name": null, "id": "d6c4e3cf-b30a-49fa-a049-4d69acd92159", "choice_type": "single"} {"question": "A 12 year old boy presents with hemetemesis, malena and mild splenomegaly. There is no obvious jaundice or ascitis. The most likely diagnosis is", "exp": ".Extra Hepatic Poal Venous Obstruction 5-6 years of age hematemesis with or without melena splenomegaly depends on duration of blockage liver usually normal in size ascites is rare variable extent of growth retardation is seen ESSENTIAL PEDIATRICS,O.P.GHAI,PG NO:289,7th edition", "cop": 1, "opa": "EHPVO", "opb": "NCPF", "opc": "Cirrhosis", "opd": "Malaria with DIC", "subject_name": "Pediatrics", "topic_name": "Gastrointestinal tract", "id": "7ccc4cdd-dcc8-43ce-90cb-1f35bbb9c96b", "choice_type": "single"} {"question": "Hypertrophic pyloric stanosis causes", "exp": "In Hypertrophic pyloric stenosis dehydration causes hypokalemic, hypochloremic metabolic alkalosis with paradoxical aciduria.", "cop": 4, "opa": "Hyperkalemic metabolic alkosis", "opb": "Hyperkalemic metabloic acidosis", "opc": "Hyperkalemic metabolic alkalosis with paradoxical aciduria", "opd": "Hypokalemic metabolic alkalosis with paradoxical aciduria", "subject_name": "Pediatrics", "topic_name": null, "id": "f2ac5132-e9b4-4251-bb60-cb02a3ec7bbd", "choice_type": "single"} {"question": "The newborn cardiac output is", "exp": "AgeCardiac output (ml/kg/min)Fetus450 Newborn3501-2 months of life150Adult75", "cop": 4, "opa": "200ml/kg/min", "opb": "250 ml/kg/min", "opc": "300 ml/kg/min", "opd": "350 ml/kg/min", "subject_name": "Pediatrics", "topic_name": "All India exam", "id": "44688fa3-27a5-4c73-9697-2f98ae6f0492", "choice_type": "single"} {"question": "Kangaroo mother care can be initiated immediately in babies weighing", "exp": "Kangaroo mother care (KMC) refers to care of preterm or low bih weight infants by placing the infant in skin-to-skin contact with the mother or any other caregiver.KMC was first suggested in 1978 by Dr Edgar Rey in Bogota, Colombia. The term kangaroo care is derived from practical similarities to marsupial caregiving, i.e. the infant is kept warm in the maternal pouch and close to the breasts for unlimited feeding.ComponentsKangaroo position. The kangaroo position consists of skin-to-skin contact between the mother and the infant in a veical position, between the mother&;s breasts and under her clothes. The provider must keep herself in a semi-reclining position to avoid the gastric reflux in the infant. The kangaroo position is maintained until the infant no longer tolerates it, as indicated by sweating in the baby or baby refusing to stay in KMC position.Kangaroo nutrition: Kangaroo nutrition is exclusive breastfeeding.Kangaroo discharge and followup: Early home discharge in the kangaroo position from the neonatal unit is one of the original components of the KMC intervention.Physiological BenefitsKMC results in keeping neonates warm and cozy. Babies get protected against cold stress and hypothermia.Physiological parameters such as hea and respiratory rates, oxygenation, sleep patterns get stabilized.Clinical BenefitsKMC significantly increases milk production in mothers and exclusive breastfeeding rates.KMC improves weight gain in the infants and improves thermal protection. It reduces incidence of respiratory tract and nosocomial infections, improves emotional bonding between the infant and mothers and results in earlier discharge from the hospital.Bih Weight > 1800grams, KMC can be initiated immediately after bih1200 to 1800grams: May take a few days before KMC can be initiated< 1200grams: May take days to weeks before KMC can be initiatedRef: Paediatrics; O.P. Ghai; 8th edition; Page no: 150", "cop": 4, "opa": "500 - 1000 grams", "opb": "1000 - 1200 grams", "opc": "1200 - 1800 grams", "opd": "> 1800 grams", "subject_name": "Pediatrics", "topic_name": "New born infants", "id": "4b2499ae-bc17-4b19-b57d-357b61eb25ea", "choice_type": "single"} {"question": "The most common malignant neoplasm of infancy is", "exp": "Neuroblastoma is the most common extracranial and intra- abdominal solid malignant tumor of infancy.It is a disease of early childhood with approximately 90% of patients presenting before 5 years of age & almost 50% within the first 2 years of life. Reference:Essential pediatrics-Ghai,8th edition,page no:616.", "cop": 2, "opa": "Malignant teratoma", "opb": "Neuroblastoma", "opc": "Wilms' tumor", "opd": "Hepatoblastoma", "subject_name": "Pediatrics", "topic_name": "Childhood tumors", "id": "2f9bdd8b-9a46-412b-be94-900e348b7973", "choice_type": "single"} {"question": "The hea lesion not found in Congenital Rubella infection is", "exp": "Classical triad of congenital rubella syndrome is cataract,deafness,cardiac defects. Cardiac defects include pulmonary stenosis,patent ductus aeriosus,ventricular septal defect. Ref : Harrison&;s principles of essential medicine P.no:1299", "cop": 1, "opa": "ASD", "opb": "VSD", "opc": "PDA", "opd": "PS", "subject_name": "Pediatrics", "topic_name": "C.V.S", "id": "84476f68-e01b-4c63-9072-f7485d9b0139", "choice_type": "single"} {"question": "Most common chromosomal anomoly in miscarriage", "exp": "Most common Chromosomal anomoly in miscarriage Trisomy 16.", "cop": 3, "opa": "Down's syndrome", "opb": "Turner's syndrome", "opc": "Trisomy 16", "opd": "Edward syndrome", "subject_name": "Pediatrics", "topic_name": null, "id": "07bf5838-1d38-4c56-ab76-4c8c2182fe49", "choice_type": "single"} {"question": "Characteristic lab findings of hemophilia A are", "exp": "In hemophilia there will be abnormal aPTT and normal PT. Symptoms People with hemophilia A often, bleed longer than other people. Bleeds can occur internally, into joints and muscles, or externally, from minor cuts, dental procedures or trauma. How frequently a person bleeds and the severity of those bleeds depends on how much FVIII is in the plasma, the straw-colored fluid poion of blood. Normal plasma levels of FVIII range from 50% to 150%. Levels below 50%, or half of what is needed to form a clot, determine a person's symptoms. Mild hemophilia A- 6% up to 49% of FVIII in the blood. People with mild hemophilia Agenerally experience bleeding only after serious injury, trauma or surgery. In many cases, mild hemophilia is not diagnosed until an injury, surgery or tooth extraction results in prolonged bleeding. The first episode may not occur until adulthood. Women with mild hemophilia often experience menorrhagia, heavy menstrual periods, and can hemorrhage after childbih. Moderate hemophilia A. 1% up to 5% of FVIII in the blood. People with moderate hemophilia A tend to have bleeding episodes after injuries. Bleeds that occur without obvious cause are called spontaneous bleeding episodes. Severe hemophilia A. <1% of FVIII in the blood. People with severe hemophilia A experience bleeding following an injury and may have frequent spontaneous bleeding episodes, often into their joints and muscles. Reference: GHAI Essential pediatrics, 8th edition", "cop": 2, "opa": "PT", "opb": "aPTT", "opc": "X-linked recessive", "opd": "Presence of 30% of factor level express the disease", "subject_name": "Pediatrics", "topic_name": "Hematology", "id": "b0238c09-abde-4a63-b686-f27941e7985b", "choice_type": "single"} {"question": "Commonest congenital lesion complicated by infective endocarditis is", "exp": "VSD is the commonest congenital lesion complicated by infective endocarditis. The incidence of infective endocarditis has been estimated as 2/100 patients in a follow up of ten years, that is 1/500 patient years. The incidence of infective endocarditis is small enough that it is not an indication for operation in small defects. However, it is impoant to emphasize good oral-dental hygiene in all patients with VSD. Reference: Essential Paediatrics; O.P. Ghai; Page no: 417", "cop": 2, "opa": "ASD", "opb": "VSD", "opc": "PDA", "opd": "TOF", "subject_name": "Pediatrics", "topic_name": "C.V.S", "id": "01bec318-bb07-45cd-89eb-e98998c67cf2", "choice_type": "single"} {"question": "Drop metastases is seen in", "exp": "Drop metastases - Dissemination through CSF giving rise to nodular masses at some distance from primary tumour. It is seen in Medulloblastoma", "cop": 1, "opa": "Medulloblastoma", "opb": "Ependymoma", "opc": "Cerebellar astrocytoma", "opd": "Neuroblastoma", "subject_name": "Pediatrics", "topic_name": null, "id": "40aeabaa-c021-4196-a644-a4134a6665c8", "choice_type": "single"} {"question": "Unilateral renal agenesis is associated with", "exp": "Ans. is 'c' Single Umbilical Artery Nelson says \"Incidence of unilateral Renal Agenesis is increased in Newborns with a single umbilical artery \"Other associated malformations with U/L renal agenesisAbsent ureterContralateral vesicoureteric refluxAbsent Ipsilateral vas deferensNow something about bilateral renal agenesisBilateral Renal agenesis is incompatible with extrauterine life, death occurs shortly after birth from pulmonary hypoplasia.Bilateral renal agenesis is also known as Potter syndrome*", "cop": 3, "opa": "Polycystic disease of pancreas", "opb": "Hiatus Hernia", "opc": "Single umbilical artery", "opd": "Hypogonadism", "subject_name": "Pediatrics", "topic_name": "Nephropathy", "id": "17c7a565-83ed-4fe8-bd71-5654012ee23f", "choice_type": "single"} {"question": "The different manifestations of hypothermia are", "exp": "Hypothermia causes peripheral vasoconstriction resulting in acrocyanosis, cool extremities and delayed peripheral capillary refill time(C). The baby becomes restless and then lethargic. Chronic or recurrent episodes cause poor weight gain. CVS manifestations include bradycardia, hypotension, raised pulmonary aery pressure with resultant hypoxemia, tachypnoea and distress. Presence of lethargy, poor reflexes, decreased oral acceptance and apnea denotes neurological depression. Abdomen distension, vomiting and feeding intolerance make enteral intake difficult. Acidosis, hypoglycemia, oliguria, azotemia and generalised bleeding can occur in severe cases. Reference : page 146 Ghai Essential Pediatrics 8th edition", "cop": 1, "opa": "Apnea", "opb": "Hypoglycemia", "opc": "Hyperglycemia", "opd": "Tachycardia", "subject_name": "Pediatrics", "topic_name": "New born infants", "id": "fce533c8-3c97-42d8-b23a-5e3f8d6d6328", "choice_type": "single"} {"question": "Very low bih weight babies are", "exp": "Low bih weight (LBW) neonate- A neonate weighing less than 2500g at bih irrespective of the gestational age. Very low bihweight (VLBW) neonate- A neonate weighing leas than 1500g at bih irrespective of the gestational age. Extremely low bih weight (ELBW) neonate- A neonate weighing leas than 1000g at bih irrespective of the gestational age. Reference : page 125 Ghai Essential Pediatrics 8th edition", "cop": 2, "opa": "< 2500 gm of bih weight", "opb": "< 1500 gm of bih weight", "opc": "<1000 gm of bih weight", "opd": "Between 2500-300 gm of bih weight", "subject_name": "Pediatrics", "topic_name": "New born infants", "id": "43019623-790f-4e23-a5c3-cef2d2181d2f", "choice_type": "single"} {"question": "Four years old child presents with mild fever, malaise, ahritis, abdominal pain and microscopic hematuria. Most probable diagnosis", "exp": "Henoch-Schonlein Purpura (HSP) : Henoch-Schonlein purpura (HSP) is a disease involving Although HSP can affect people at any age, most cases occur in children between the ages of 2 and 11. It is more common in boys than girls. Adults with HSP are more likely to have more severe disease compared to children. HSP usually ends after four to six weeks -- sometimes with recurrence of symptoms over this period, but with no long-term consequences (recurrences are fairly common). If organs such as the Causes and Risk Factors for Henoch-Schonlein Purpura The exact cause of HSP is not known. The body&;s immune system is believed to play a role in targeting the blood vessels involved. An abnormal immune response to an infection may be a factor in many cases. Approximately two-thirds of the cases of HSP occur days after symptoms of an upper respiratory tract infection develop. Some cases of HSP have been linked to , ; foods, drugs, chemicals, and , and/or related , including blood in urine. Before these symptoms begin, patients may have two to three weeks of fever, Reference: GHAI Essential pediatrics, 8th edition", "cop": 4, "opa": "Thrombasthenia", "opb": "Idiopathic thrombocytopenic purpura", "opc": "SLE", "opd": "Henoch-Schonlein purpura", "subject_name": "Pediatrics", "topic_name": "Metabolic disorders", "id": "9a62dbfd-0b28-4969-930f-615eda4036de", "choice_type": "single"} {"question": "A new born infant is diagnosed with phenylketonuria. The enzyme that is deficient is", "exp": "Phenylketonuria is a disorder of phenylalanine metabolism and occurs due to deficiency of phenylalanine hydroxylase.It is a recessive condition.Here,phenylalanine could not be conveed to tyrosine,so phenylalanine accumulates and alternate minor pathways are opened to produce and excrete phenylketone(phenyl pyruvate),phenyl lactate and phenyl acetate in urine. Phenylalanine hydroxylase is an enzyme your body uses to conve phenylalanine into tyrosine, which your body needs to create neurotransmitters such as epinephrine, norepinephrine, and dopamine. PKU is caused by a defect in the gene that helps create phenylalanine hydroxylase. When this enzyme is missing, your body can't break down phenylalanine. This causes a buildup of phenylalanine in your body. Reference:Essential pediatrics-Ghai,8th edition,page no:652;Textbook of biochemistry-DM Vasudevan,6th edition,page no:207", "cop": 3, "opa": "Phenylamine oxidase", "opb": "Tyrosine hydroxylase", "opc": "Phenylalanine hydroxylase", "opd": "Dihydropteridine reductase", "subject_name": "Pediatrics", "topic_name": "Metabolic disorders", "id": "fbebdaa8-8ba5-44dc-a07d-49e7b5189c46", "choice_type": "single"} {"question": "Association of sexual precocity, multiple cystic bone lesions and endocrinopathies are seen in", "exp": "mc cube Albright syndrome characterized by recurrent ovarian cyst, cutaneous, skeletal & endocrine abnormalities. Figure : Cafe-au-lait skin pigmentation. (a) Skin lesions in a newborn demonstrating the characteristic association with the midline of the body, and distribution reflecting patterns of embryonic cell migration (developmental lines of Blaschko). (b) A typical lesion on the chest, face, and arm demostrating the irregular \"coast of Maine\" borders, relationship with the midline of the body, and distribution following developmental lines of Blaschko. (c). Typical lesions frequently found on the nape of the neck and crease of the buttocks Reference: Ghai essential of pediatrics, eighth edition, p.no:532.", "cop": 1, "opa": "McCune-Albright's syndrome", "opb": "Granulosa cell tumor", "opc": "Androblastoma", "opd": "Hepatoblastoma", "subject_name": "Pediatrics", "topic_name": "Endocrinology", "id": "1961bb84-a353-4e26-b9c1-9a39e55adda3", "choice_type": "single"} {"question": "Down syndrome is most commonly associated with", "exp": "An ostium primum atrial septal defect (ASD) occurs when then atrial septum near the atrioventricular valves has a communication between the two atrium causing a left to right shunt. The image to the right shows the location of the different types of atrial septal defects while looking from the right atrium. 1 = Upper sinus venosus defect 2 = Lower sinus venosus defect 3 = Ostium secundum atrial septal defect 4 = Coronary sinus atrial septal defect 5 = Ostium Primum atrial septal defect Reference: GHAI Essential pediatrics, 8th edition", "cop": 2, "opa": "ASD with ostium secundum", "opb": "ASD with ostium primum", "opc": "VSD", "opd": "TOF", "subject_name": "Pediatrics", "topic_name": "Genetic and genetic disorders", "id": "c9b8011a-4e40-4826-804e-4988cc861fb9", "choice_type": "single"} {"question": "Microcephaly is defined as", "exp": "Macrocephaly : Head circumference > 2SD above the mean for age and sex", "cop": 1, "opa": "Head circumference < -3SD below the mean for age and sex", "opb": "Head circumference < -2SD below the mean for age and sex", "opc": "Head circumference below 10th percentile the mean for age and sex", "opd": "Head circumference below 15th percentile the mean for age and sex", "subject_name": "Pediatrics", "topic_name": null, "id": "9b01ee36-e217-4836-b015-22287b4a15c4", "choice_type": "single"} {"question": "Congenital Wilson's disease is characterized by", "exp": "Ans. is 'b' i.e., May present as acute hepatitis o KF ring develop after sometime due to deposition of copper in cornea. o It may present as acute or chronic liver disease. o Hepatic copper concentration and urinary excretion of copper are incerased.", "cop": 2, "opa": "KF ring is present at bih", "opb": "May present as acute hepatitis", "opc": "Decreased Urinary copper excretion", "opd": "Decreased hapatic copper concentration", "subject_name": "Pediatrics", "topic_name": null, "id": "9956ea86-9c7f-41a7-9654-4aa955822b33", "choice_type": "single"} {"question": "Bilirubin UDP glucuronyl transferase activity is absent in", "exp": "Syndrome\nBilirubin UDP glucuronyl transferase activity\n\n\n\n\nCrigler-Najjar type I\n\t\t\tCrigler-Najjar type II\n\t\t\tGilbert syndrome\nAbsent\n\t\t\tMarkedly reduced : 0-10% of Normal\n\t\t\tReduced : 10-30% of Normal", "cop": 1, "opa": "Crigler-Najjar type I", "opb": "Crigler-Najjar type II", "opc": "Rotor syndrome", "opd": "Gilbert syndrome", "subject_name": "Pediatrics", "topic_name": null, "id": "6d04cf58-0d4b-466c-8b63-c8feae372e13", "choice_type": "single"} {"question": "A child is brought by mother with HO massive hemetemesis with HO drug intake previously with NSAIDS and on Rx. Associated with moderate splenomegaly diagnosis is", "exp": "Causes of portal hypertension\n\nPre sinusoidal\nExtrahepatic (most commoon) :portal vein/splenic vein thrombosis,splenic AV fistula,massive splenomegaly.\nIntrahepatic: sarcoidosis, schistosomiasis, congenital hepatic fibrosis, myeloproliferative disorders, nodular degenerative hyperplasia, idiopathic portal fibrosis.\nSinusoidal\nCirrhosis due to any cause.\nPost sinusoidal\nBudd chiari syndrome,right heart failure,constrictive pericarditis,web in inferior venacava.", "cop": 1, "opa": "Oesophageal varices", "opb": "Duodenal ulcer", "opc": "Drug induced gastritis", "opd": "Peptic ulcer", "subject_name": "Pediatrics", "topic_name": null, "id": "7bae26db-c361-4346-b7b8-3fb0247601fb", "choice_type": "single"} {"question": "The commonest cause of vomiting in a one month old infant is", "exp": "Aerophagia (var. aerophagy) is a condition of excessive air swallowing, which goes to the stomach. Aerophagia may also refer to an unusual condition where the primary symptom is excessive flatus, belching is not present, and the actual mechanism by which air enters the gut is obscure.Aerophagia in psychiatry is sometimes attributed to nervousness or anxiety. Reference: GHAI Essential pediatrics, 8th edition", "cop": 3, "opa": "Pyloric stenosis", "opb": "Cardiac chalasia", "opc": "Aerophagy", "opd": "Gastro-esophageal reflux", "subject_name": "Pediatrics", "topic_name": "Gastrointestinal tract", "id": "46010421-742d-42ea-b7a0-8e14fe64695b", "choice_type": "single"} {"question": "The immediate treatment of 10 kg weight infants presented with tetany", "exp": "Calcium gluconate 10%, 9mg calcium per ml is the preperation of choice.care should be taken to administer the drug slowly to avoid cardiac effects and tissue necrosis. Reference: OP G hai,eseential paediatrics,8 th edition. page no 522", "cop": 2, "opa": "IV Diazepam", "opb": "IV calcium gluconate with cardiac monitoring", "opc": "IV slow phenobarbitone", "opd": "Wait and watch", "subject_name": "Pediatrics", "topic_name": "Endocrinology", "id": "0e71d41c-aba0-40a1-904d-96be242ed207", "choice_type": "single"} {"question": "A one month old baby presents with frequent vomiting and failure to thrive, There are features of moderate dehydration, Blood sodium is 122 m Eq/i and potassium is 6.1 in Eq/L The most likely diagnosis is", "exp": "21 Hydroxylase deficiency - (A) Salt losing form of CALL due to associated with aldosterone deficiency, these patients presents with severe vomiting, and vascular collapse. Hyponatreniia hypochloremia and hyperkalemia (B) A Cryptic form of CAH is asymptomatic - presents with menstrual irregularities, acne, hirsutism * Female pseudo hermaphroditism in female 11 b hydroxylase deficiency - presents with hypeension, and hypokalemia Bater's syndrome - - Presents during childhood, having growth retardation, normal to low BP, hypokalemia secondary to renal K+ wasting, metabolic alkalosis, nephrocalcinotic Gitelman's syndrome - (Variant of Bater's syndrome presents during adolescence/adult hood) * Hypokalemia, hypocalciuria, hypomagnesemia and normal prostaglandin production * Metabolic alkalosis", "cop": 3, "opa": "Gitelman syndrome", "opb": "Bater syndrome", "opc": "21 hydroxylase deficiency", "opd": "11 B-hydroxylase deficiency", "subject_name": "Pediatrics", "topic_name": null, "id": "a048fd18-4d51-4ce3-9080-b2fe2f8517f7", "choice_type": "single"} {"question": "In sickle cell trait, number of bands found in Hb", "exp": "Ans. is 'a' i.e., 2 In hemoglobin electrophoresis, various types of hemoglobin move at varying speed. In sickle cell trait (heterozygous state), there are two types of hemoglobin - HbS (40%) and HbA (60%). So, on electrophoresis two different bands are produced. In Homozygous state, almost all hemoglobin is HbS. So, single band is produced. Laboratory findings of sickle cell anemia Moderate to severe anaemia o Peripheral smear will show Sickle cells Howell - Jolly bodies because of autosplenectomy Target cells o A positive sickling test with a reducing substance like sodium metabisulfite.", "cop": 1, "opa": "2", "opb": "1", "opc": "4", "opd": "5", "subject_name": "Pediatrics", "topic_name": null, "id": "46854fd5-f700-4907-97c1-af31cda3534c", "choice_type": "single"} {"question": "Most common presentation of a child with Wilm's tumour is", "exp": "Most common initial clinical presentation for Wilm's tumour is incidental discovery of Asymptomatic abdominal mass by parents while bathing an affected child or by physician during routine physical examination.", "cop": 3, "opa": "Hematuria", "opb": "Hypertension", "opc": "Asymptomatic abdominal mass", "opd": "Abdominal pain", "subject_name": "Pediatrics", "topic_name": null, "id": "061671bd-fe8e-49b1-807a-7e071162ee1c", "choice_type": "single"} {"question": "In the neonate, an intramuscular injection is given on", "exp": "Anterolateral aspect of the thigh This is preferred site up to 12 months of age(Refer: Nelson's Textbook of Pediatrics, SAE, 1st edition, pg no. 1258)", "cop": 3, "opa": "Deltoid", "opb": "Gluteal region", "opc": "Thigh", "opd": "Abdomen", "subject_name": "Pediatrics", "topic_name": "All India exam", "id": "7f864c8a-fee2-4b7b-8a28-57ca2eeb5f88", "choice_type": "single"} {"question": "1 day old m ale baby delivered by LSCS had swelling over back in midline.", "exp": "Ans. is 'b' i.e., Folic acid o Folic acid supplementation decreases the incidence and recurrence of neural tube defects (NTD). Because the neural tube closes within the 1st 28 days ofconception, periconceptional supplementation is needed for prevention. It is recommended that women without a prior history of a NTD, ingest 400 ?g/day throughout their reproductive years. Women with a history of a prior pregnancy complicated by a NTD or a Ist-degree relative with a NTD should have preconceptual counseling and they should ingest 4 mgfday of supplemental folic acid at least 1 mo before conception. Use of some antiepileptic drugs (valproatet carbamazepine) during pregnancy is associated with an increased risk of NTD. Women taking these medications should ingest 1-5 mg of folic acid/day in the preconception period.", "cop": 2, "opa": "Iron", "opb": "Folic acid", "opc": "Thiamine", "opd": "Vit A", "subject_name": "Pediatrics", "topic_name": "New Born Infants", "id": "87c6d7dd-7ce6-47c3-9daf-4e2ad02a1652", "choice_type": "single"} {"question": "Normal child has dysfluency of speech between", "exp": "Stuttering is a defect in speech characterized by hesitation or spasmodic repetition of some syllables with pauses. There is difficulty in pronouncing the initial consonants caused by spasm of lingual and palatal muscles. It can affect up to 5% of children between 2 and 5 years of age. In this age group, parents should be reassured as most of them show resolution. If it persists beyond or appears after 5 years of age, opinion of psychologist and speech therapist should be sought", "cop": 1, "opa": "2 and 4 years", "opb": "4 and 6 years", "opc": "6 and 8 years", "opd": "8 and 10 years", "subject_name": "Pediatrics", "topic_name": "Growth and development", "id": "5d717952-fa80-4f93-b44a-5e564229f343", "choice_type": "single"} {"question": "Exclusive breastfeeding should be practiced at least until", "exp": "An infant should be exclusively breastfed until 6 months of age. During this age, additional food or fluid is not required as breast milk is nutritionally complete for the child&;s growth & development & it protects from infections and strengthens immune systemRef: Page 60; Ghai Essential Pediatrics; 7th edition", "cop": 2, "opa": "4 month", "opb": "6 month", "opc": "8 month", "opd": "10 month", "subject_name": "Pediatrics", "topic_name": "Nutrition", "id": "29ff8d4a-b1a2-438b-9c37-5a763c520ad1", "choice_type": "single"}